Você está na página 1de 204

Língua P

R
Portuguesa É

INSTITUTO
FEDERAL
PIAUÍ

Escola Municipal________________________________________________
Estudante:_______________________________________________________

2023
Sumário

LINGUAGEM ................................................................................................................................................... 2
ALTERAÇÕES SEMÂNTICAS E VARIAÇÃO LINGUÍSTICA
....................................................................................................................................................... 8
DENOTAÇÃO ................................................................................................................................................ 15
FIGURAS DE LINGUAGEM ........................................................................................................................ 20
FONÉTICA E FONOLOGIA ........................................................................................................................ 25
MORFOLOGIA-PROCESSO DE FORMAÇÃO DE PALAVRAS ......................................................... 31
MORFOSSINTAXE DA LÍNGUA PORTUGUESA – PARTE I .............................................................. 36
MORFOSSINTAXE DA LÍNGUA PORTUGUESA- PARTE II ............................................................... 42
MORFOSSINTAXE DA LÍNGUA PORTUGUESA - PARTE III ............................................................. 47
CONCORDÂNCIA......................................................................................................................................... 52
CONCORDÂNCIA NOMINAL ..................................................................................................................... 57
ORTOGRAFIA ............................................................................................................................................... 61
NOÇÕES GERAIS DE LITERATURA ....................................................................................................... 70
TEXTO LÍRICO- POEMA ............................................................................................................................. 76
GÊNEROS LITERÁRIOS............................................................................................................................. 84

1
LINGUAGEM  Descrição (identificar, qualificar,
descrever). Ex.: Cardápio, relatos, trechos
O termo linguagem é comumente empregado
de textos narrativos etc.
para referir-se a qualquer processo de
comunicação, como a linguagem dos animais,  Exposição (discutir, informar). Ex.:
linguagem corporal, linguagem das artes. Outro verbetes, seminários, enciclopédia etc.
sentido é o de linguagem como a habilidade ou  Injunção (dar ordem, apresentar regras,
capacidade que apenas os seres humanos procedimentos). Ex.: receita, propaganda,
possuem de se comunicar por meio de línguas. regulamentos etc.
Quando nos referimos a língua portuguesa,
língua espanhola ou língua inglesa, fazemos alusão  INTERTEXTUALIDADE
a uma língua como unidade ideal, reconhecida - e Em sentido restrito, todo texto faz referência,
praticada - pelos falantes nativos ou por falantes de explícita ou implícita, a outro(s) efetivamente já
outras línguas. produzido(s) e que faz(em) parte da memória
 Linguagem verbal, não-verbal e social dos leitores.
mista/híbrida. Como exemplo, leiamos o texto:

https://www.facebook.com/linguaportuguesa07/photos/a.2717
73609503284/955711114442860/?type=1&theater

LINGUAGEM LINGUAGEM NÃO


VERBAL VERBAL
 A comunicação  A comunicação ocorre Qual o “texto” que se encontra na base da
ocorre por meio por meio de sons, produção do anúncio publicitário acima? Sem
de palavras. imagens, gestos. dúvida, identificamos com facilidade o filme
 A língua pode  Pinturas. brasileiro “Tropa de elite”, do cineasta José
Padilha. Isso só é possível em razão do forte
ser falada ou  Movimentos.
apelo popular da produção, que ganhou grande
escrita.  Formas.
projeção em nossa sociedade.
 TEXTO
A linguagem codificada (código sonoro, gráfico,
imagético, gestual) de modo que construa um
sentido. Esse sentido que se procura em um
texto é a sua coerência.
 GÊNERO TEXTUAL
Textos diferenciados pelo conteúdo, estilo, forma
a fim de cumprir funções em diversas situações
comunicativas.
 TIPOLOGIA TEXTUAL
Como o texto é organizado, isto é, seus aspectos
lexicais, sintáticos, tempos verbais, relações
lógicas.
 POLISSEMIA
 Tipologias textuais:
Conjunto de significados, cada um unitário,
 Narração (relatar fatos). Ex.: contos, relacionados com uma mesma marca linguística, ou
crônicas etc. seja, a polissemia consiste em uma palavra que
apresenta vários significados.
 Argumentação (defender um ponto de
Exemplo:
vista). Ex.: editorial, resenha crítica, carta
1. Não estou com cabeça para isso.
do leitor etc.
2. Minha cabeça está doendo.
3. Você é a cabeça do projeto.
2
PROFESSOR RESPONDE: família semântica: “bullying”, “bulir” e “bolinar”. Caprichos
da língua.
Bullying e bolinar: alguma coisa a ver? Revista Língua (Setembro de 2013)
O termo “bullying” é de introdução recente em Blog do Aldo Bizzocchi
Disponível em:http://revistalingua.uol.com.br/textos/blogabizzocchi/
português. Já a prática que ele nomeia é mais velha do um-bolo-de-significados-333948-1.asp
que o mundo. Suspeito que, desde que existe essa
instituição chamada escola, a chacota e a intimidação 01. (IFPI – 2015) De acordo com o texto:
contra os alunos mais fracos ou “diferentes” faz parte a) A etimologia dos termos “bulir”, “bullying” e
dela, como o feijão e o caruncho. Talvez, antes mesmo “bolinar” são obscuras e seus significados são
de surgirem as instituições de ensino, já no tempo das definidos, portanto, a partir do linguajar popular.
cavernas se praticasse essa traumática demonstração b) A palavra “bullying” é característica do
de superioridade, como a dominância que um animal ambiente escolar e, portanto, circula apenas no
exerce sobre outro montando-lhe nas costas. Enfim, o
bullying parece ser um resquício de nossas raízes
vocabulário desse ambiente.
animais, reminiscência deixada pela evolução numa c) O termo “bullying” vem acompanhando a
espécie que, mesmo tendo deixado a selva, jamais evolução de nossa espécie que continua
deixou de ser selvagem. selvagem como em suas raízes.
Mas vamos ao que interessa: desde que “bullying” d) As palavras “bulir”, “bullying” e “bolinar” têm
passou a circular no vocabulário escolar, estudantes origens diferentes, mas se aproximam–
alegam ter sido “bolinados” ou que fulaninho é um aproximação licenciada pela língua – por
“bolinador”. Há algo em comum entre “bullying” e o verbo possuírem semelhanças de sons e sentidos.
“bolinar”? e) O dicionário Michaelis adota um significado
A aproximação das duas palavras é uma dessas preferencial para “bolinar” que é “tocar em
curiosas peças que a língua, em seus cegos meandros,
alguém com intuitos libidinosos”.
nos prega (...). Em português, o casamento de “bullying”
com “bolinar” foi facilitado pela pronúncia “bulinar”, de 02. (IFPI -2015) Na passagem “Perscrutar a origem
longe a mais corrente no Brasil. de expressões populares é um dos grandes
O fato é que “bolinar” é palavra bem mais antiga que desafios da etimologia (...)”, o termo “perscrutar”
“bullying” e veio da linguagem náutica. Bolinar é apontar
só não pode ser entendido como:
a embarcação na direção do vento, navegar à bolina, de
viés. E, segundo o dicionário Michaelis, bolina, do inglês a) Diferenciar.
“bowline”, é “cada uma de duas chapas de aço ou outras b) Investigar.
saliências longitudinais semelhantes a uma nadadeira, c) Esquadrinhar.
fixadas uma em cada lado arredondadodo fundo do d) Averiguar.
navio para evitar grandes balanços”. e) Sondar.
Da navegação, “bolinar” passou ao linguajar sexual,
designando, segundo o Houaiss, o ato de “apalpar ou 03. (IFPI – 2015) A passagem “A etimologia do termo
encostar-se a uma outra pessoa com fins libidinosos, é obscura (...)” (5º parágrafo), refere-se, no texto,
geralmente de modo furtivo”. Para o Michaelis, é “tocar a:
em alguém com intuitos libidinosos”. Já “bullying” vem do a) “bully”
inglês “bully”, definido pelo American Heritage Dictionary b) “bullying”
como uma pessoa habitualmente cruel, especialmente c) “bull”
com pessoas menores ou mais fracas. A etimologia do d) “boele”
termo é obscura, sendo que o dicionário em questão a e) “bolinar”
atribui possivelmente ao holandês medieval “boele”
(namorado/a). Outro étimo apontado é a palavra “bull” 04. (IFPI – 2015) Em qual passagem do texto fica
(touro), talvez pela semelhança entre a impetuosidade clara a diferença na origem das palavras “bolinar”
desse animale a truculência do “bully”. e“bullying”:
Mas como “bolinar” assumiu conotação sexual? E o
que isso tem a ver com chacota e intimidação? Segundo a) “Em português, o casamento de ‘bullying’
o etimólogo Antenor Nascentes, o sentido libidinoso de com‘bolinar’ foi facilitado pela pronúncia
“bolinar” data de 1892: o bolinador senta-se ao lado de ‘bulinar’, de longe a mais corrente no Brasil”.
uma mulher, como um navio à espera de vento favorável. b) “Ora, o bolinador escolar aborrece,
Mas quem sabe a explicação seja mais complexa do que
importuna,leva à loucura suas vítimas”.
isso. Perscrutar a origem de expressões populares é um
dos grandes desafios da etimologia, já que c) “No entanto, eu lançaria mão de um terceiro
documentação comprobatória na maioria dos casos não elemento, o verbo ‘bulir’, sinônimo de
existe. No entanto, eu lançaria mão de um terceiro ‘mexer’(...)”.
elemento, o verbo “bulir”, sinônimo de “mexer”, que, d) “Enfim, o bullying parece ser um resquício de
como este, também significa “incomodar, importunar, nossas raízes animais, reminiscência deixada
caçoar”. Ora, o bolinador escolar aborrece, importuna, pela evolução numa espécie que, mesmo
leva à loucura suas vítimas (já houve até casos de tendo deixado a selva, jamais deixou de ser
suicídio de crianças assediadas na escola); além disso, selvagem.”
quando se dirige às meninas, em geral seu assédio e) “O fato é que ‘bolinar’ é palavra bem mais
envolve sexo: propostas indecorosas, obscenidades
cochichadas ao ouvido, apalpadelas...
antiga que ‘bullying’ e veio da linguagem
Essa convergência de sons e sentidos pode ter náutica(...)”.
reunido três palavras de origens tão diversas numa única

3
05. (IFPI–2015) Trata-se de um texto,
predominantemente:
a) Descritivo
b) Narrativo
c) Informativo
d) Injuntivo
e) Argumentativo

ALUNO RESPONDE

01. (IFPI – 2013)


02. (IFPI – 2015) Analise as afirmações abaixo e
“Já vi pessoas chorando por assinale aalternativa correta:
filme, novela, futebol, mas não
encontro pessoas chorando I. A charge acima não constitui um texto por
pela restauração da família”. que só existe texto se for escrito;
II. Mesmo sem palavras, a charge acima
Padre ChrystianShankar
(tv.cançaonova.com) constitui um texto e expressa uma visão
crítica sobre a adolescência;
É correto afirmar sobre o texto que: III. Tanto o poema quanto a charge são
textos,porém construídos a partir de
a) Ele aborda como tema central o amor que o
códigos diferentes.
brasileiro tem pelo futebol;
b) Ele mostra em seu conteúdo a situação das a) Apenas a afirmação I está correta;
famílias brasileiras que passam fome; b) Apenas a afirmação II está correta;
c) Tem como objetivo criticar as pessoas que c) Apenas a afirmação III está correta;
gostam de assistir a filmes, novelas e d) As afirmações I e II estão corretas;
futebol; e) As afirmações II e III estão corretas.
d) Está criticando as pessoas que valorizam
03. (IFPI – 2015) Ainda comparando a charge com
mais os programas de TV do que a sua
o poema, assinale a alternativa correta:
própria família;
e) Faz uma crítica às pessoas que se deixam I. Ambos discutem o mesmo aspecto da
influenciar por programas de televisão e não adolescência: o medo de descobrir o novo;
se importam com os reais problemas que as II. O poema está mais voltado para a reflexão
famílias enfrentam, como a separação e o da vida como um todo, enquanto a charge
divórcio. volta-se à questão da aparência e
mudanças físicas durante a adolescência;
LEIA OS TEXTOS PARA AS QUESTÕES 02 E 03. III. Tanto na charge quanto no poema, o
adolescente quer ser notado, visto.
ADOLESCENTE
Mário Quintana a) Apenas I está correto.
A vida é tão bela que chega a dar medo. b) Apenas II está correto.
c) Apenas III está correto.
Não o medo que paralisa e gela, d) I e II estão corretos.
estátua súbita, e) Todos os itens acima estão corretos.
mas
04. (IFPI – 2015) Observe a tirinha.
esse medo fascinante e fremente de
curiosidade que faz
o jovem felino seguir para frente varejando o
vento ao sair, à primeira vez, da gruta.

Medo que ofusca: luz!


Cumplicemente,
as folhas contam-te um segredo
velho como o mundo:

Adolescente, olha! A vida é nova...


A vida é nova e anda nua
-Vestida apenas com o teu desejo!
Compare a charge abaixo com o poema
ADOLESCENTE, de Mário Quintana.
4
Analise as seguintes afirmações:
I. A vontade da mãe da Mafalda impediu que
esta subtraísse para si o troco da padaria.
II. O “inquilino” a que Mafalda se refere é a
consciência dela.
III. Os pronomes “isso” e “esse”, no segundo e
no último balão, respectivamente, têm valor
anafórico, uma vez que retomam um termo
anterior.
Está(ão) correta(s):
a) I, II e III.
b) II e III. De acordo com o conteúdo exposto no texto,
c) I e II. podemos afirmar que esse fato retrata:
d) Apenas a I. a) Um caso de afogamento na praia;
e) Apenas a II. b) Um caso de suicídio por afogamento;
LEIA O TEXTO PARA AS QUESTÕES 05 E 06 c) Um caso de agressão a mulheres, seguido de
morte;
d) Um caso de agressão a menores;
e) Um caso típico de assalto, comum nos
grandes centros urbanos.
08. (IFPI – 2013-Modificada) Observe a imagem.

05. (IFPI – 2013) Após a leitura da imagem,


podemos entender que ela critica um fato
corriqueiro que atinge toda sociedade mundial. É correto afirmar sobre a charge que:
Esse fato refere-se: a) Ela faz uma crítica à lei das empregadas
a) Ao desmatamento e à poluição do globo domésticas;
terrestre; b) Critica as pessoas extrovertidas que usam a
b) Ao efeito estufa causado pela emissão de conversa como um entretenimento;
gases poluentes nas cidades grandes; c) Faz referência à jornada de trabalho,
c) Ao consumismo, um problema enfrentado criticando os patrões que não obedecem a
pela sociedade nos dias atuais e que sofre esse critério;
influência direta da mídia; d) Está criticando as pessoas que desperdiçam
d) A um incentivo ao consumo exagerado; seu tempo com conversas sem conteúdos;
e) Ao preconceito racial que ainda afeta muitos e) Enfatiza a importância de não se perder
países do globo terrestre. tempo durante o trabalho.
06. (IFPI – 2013) A ausência de palavras no texto é ANALISE A TIRINHA E RESPONDA AS QUESTÕES 09 E 10.
uma característica da linguagem não verbal.
Com relação a isso, podemos afirmar que:
a) Dificulta o entendimento, pois só é possível
a compreensão de textos verbais;
b) A interpretação do texto não ficou
prejudicada pela ausência de palavras;
c) Esse tipo de linguagem é a mais utilizada
pela sociedade.
d) Podemos afirmar que o texto apresenta um
fato fora da realidade social; 09. (IFPI – 2016-Modificada) Analisando a tirinha,
e) Ao observarmos a imagem, podemos é possível perceber que procrastinação é o
concluir que não se trata de um texto, pois a mesmo que:
mensagem trazida por ele não está clara. a) Antecipar uma atividade.
b) Deixar alguma coisa para depois.
07. (IFPI – 2013-Modificada) A charge apresenta
c) Abreviar uma ação.
como tema central um problema social que
d) Acelerar algo.
afeta as grandes cidades do Brasil.
e) Realizar pontualmente uma ação.
5
10. (IFPI – 2016) Considerando a tirinha, julgue os LEIA O TEXTO PARA RESPONDER À
itens abaixo: QUESTÃO 14.
I. A substituição da palavra “definitivo” por “em Somos todos inúteis
construção” sugere que esses termos têm
significados opostos e, por isso, podem ser A sociedade é que nos prepara para a vida.
considerados antônimos. Claro que temos nossas particularidades, nossas
II. O termo “procrastinador” designa a pessoa idiossincrasias, mas nossa visão de mundo e o
que procrastina. grosso de nosso comportamento são-nos transmitidos
III. Na expressão “procrastinador ideal”, é pela família, o grupo, a escola, a igreja, enfim, pela
possível afirmar que o termo sublinhado sociedade.
qualifica o termo “procrastinador”. A história do Robinson Crusoé é uma boa
história, mas ficção. Ela seria impossível porque o
Está(ão) correto(s): homem é um ser social e não adianta espernear.
Paul Sweezi, filósofo da Universidade de
a) Todos os itens Harvard, tem um belo estudo sobre o modo como a
b) Apenas o item I troca de equivalentes, princípio fundamental do
c) Apenas o item II Capitalismo, afeta todo o comportamento humano. Se
d) Os itens I e II você vai à casa de um amigo, na despedida,
e) Os itens II e III invariavelmente, afirma, agora você me deve uma
visita. Pronto. Está propondo uma troca. Mesmo o
LEIA O TEXTO E RESPONDA ÀS QUESTÕES DE convívio com os amigos deixou de ser gratuito. A
11 A 13. troca de equivalentes foi a maior invenção da
O problema ecológico burguesia na Idade Média.
E se o mercado é o demiurgo que nos rege a
Se uma nave extraterrestre invadisse o espaço vida, que nos salva da miséria, então devemos
aéreo da Terra, com certeza seus tripulantes diriam pensar que é por sua óptica, do mercado, que
que neste planeta não habita uma civilização acontecem coisas difíceis de se aceitar. No
inteligente, tamanho é o grau de destruição dos pensamento do mercado (se ele fica nervoso, como
recursos naturais. Essas são palavras de um leio nos jornais, é porque também pensa) o ideal seria
renomado cientista americano. Apesar dos avanços uma pessoa nascer aí por volta dos dezoito anos, o
obtidos, a humanidade ainda não descobriu os que evitaria uma imensa inversão de recursos em sua
valores fundamentais da existência. O que chamamos preparação. Já nasceria em idade útil.
orgulhosamente de civilização nada mais é do que Além disso, não deveria continuar vivo depois
uma agressão às coisas naturais. A grosso modo, a dos cinquenta e cinco anos, pois seu rendimento
tal civilização significa a devastação das florestas, a como produtor já estaria em declínio. Ele já não é,
poluição dos rios, o envenenamento das terras e a com essa idade, muito interessante do ponto de vista
deterioração da qualidade do ar. O que chamamos de econômico. Pouco produz e menos consome (com
exceção de remédios, é claro).
progresso não passa de uma degradação deliberada
Que importância ele pode ter para a economia
e sistemática que o homem vem promovendo há
de um país? Já não participa mais ou participa muito
muito tempo, uma autêntica guerra contra a natureza.
pouco do progresso. Econômico, claro. Progresso
Afrânio Primo. Jornal Madhva (adaptado) social é outro assunto, não é mesmo?
Há famílias, e muito mais do que se imagina,
11. (IFPI – 2016) Assinale a única alternativa que
que internam seus idosos em hospitais e asilos,
demonstra a preocupação do cientista visitam-nos uma vez por semana nos dois primeiros
americano, segundo o texto. meses. Levam fatias do bolo que restou da festa,
a) o desequilíbrio ambiental. duas maçãs compradas no caminho ali do quitandeiro
b) a qualidade do espaço aéreo. (uma boa lembrança de alguém da família), e, depois
c) o que pensam os extraterrestres. de um ano, não se lembram mais da existência deles.
d) a vida neste planeta. Uma fotografia na parede da sala, como
e) os seres de outro planeta. homenagem póstuma, ao idoso que a família não
12. (IFPI – 2016) Para o autor, a humanidade: sabe que continua vivo. Mas também, para que
a) demonstra ser muito inteligente. perder tempo com essas inutilidades que já passaram
dos sessenta? Que não produzem nada, que, além
b) ouve as palavras do cientista.
de remédios, consomem uma ninharia, que já não
c) age contra sua própria existência. dão lucro a ninguém!
d) preserva os recursos naturais. Enfim, vivemos para o futuro, não para o
e) valoriza a existência sadia. passado.
13. (IFPI – 2016) No trecho “Essas são palavras de (...)
um renomado cientista americano”, o termo Menalton Braff Carta Capital. Novembro de 2015
Disponível em: http://www.cartacapital.com.br/
destacado sugere um cientista: cultura/somos – todos – inúteis – 2 –
a) amigo. 9917.html?utm_conten=buffer5ea49&utm_medium=s
b) velho. ocial&utm_source=twitter.com&utm_campaign=buffer
c) estranho.
d) famoso.
e) estudioso.

6
14. (IFPI – 2016) Marque a alternativa correta a d) os textos são de gêneros diferentes porque,
respeito do texto. apesar da intertextualidade, foram elaborados
a) O homem vive numa sociedade que valoriza a com finalidades distintas.
troca de equivalentes, e nesse ponto estamos Observe e analise o meme a seguir para responder
presos à Idade Média. à questão 16.
b) A óptica do mercado faz com que o homem
viva melhor, especialmente entre os dezoito e
os cinquenta e cinco anos de idade.
c) De acordo com o texto, o homem, mesmo
tendo suas particularidades, é um ser social e
sua vida é regida pelo pensamento de
mercado.
d) O texto faz alusão à teoria de Jean-Jacques
Rousseau, segundo a qual o homem possui
uma natureza boa que é corrompida pelo
processo civilizador.
Fonte:https://www.pragmatismopolitico.com.br/2018/05/memes-combustiveis-
e) O autor menciona a história do Robinson greve-caminhoneiros.html
Crusoé para ilustrar a ideia de que o homem, 16. (IFPI 2018.2) A frase “Meu combustível minha
assim como o personagem da história, é um vida” faz alusão ao nome de um Programa Social
ser social. do Governo Federal chamado “Minha Casa
15. (ENEM – 2009) Minha Vida”. Esse recurso linguístico denomina-
se:
Texto I
a) Metonímia
No meio do caminho b) Anáfora
No meio do caminho tinha c) Catacrese
uma pedra d) Intertextualidade
Tinha uma pedra no meio e) Aliteração
do caminho
Tinha uma pedra Leia o texto abaixo para responder às questões:
No meio do caminho tinha
uma pedra
[...]
ANDRADE, C. D. Antologia poética. Rio de
Janeiro/São Paulo: Record, 2000. (fragmento).

Texto II

17. (2019.2) Considerando a linguagem e o contexto do


anúncio acima, analise as afirmações abaixo e
marque a alternativa CORRETA.
a) A intenção do anúncio não é alcançada, uma
vez que há uma ambiguidade no uso do
vocábulo “baratas”
b) O propósito do anúncio é alcançado por meio
da compreensão, por parte do leitor, da
A comparação entre os recursos expressivos
polissemia da palavra “baratas”.
que constituem os dois textos revela que:
c) O termo “baratas” é utilizado em um único
a) o texto 1 perde suas características de gênero sentido, o que favorece a compreensão do
poético ao ser vulgarizado por histórias em anúncio.
quadrinho.
d) Existe uma ambiguidade no uso do termo
b) o texto 2 pertence ao gênero literário, porque “empresas”, o que compromete a interpretação
as escolhas linguísticas o tornam uma réplica
do leitor, de acordo com os objetivos do
do texto 1. anúncio.
c) a escolha do tema, desenvolvido por frases e) O duplo sentido do pronome “você” interfere na
semelhantes, caracteriza-os como interpretação correta do anúncio pelo leitor.
pertencentes ao mesmo gênero.
7
ALTERAÇÕES SEMÂNTICAS E VARIAÇÃO
LINGUÍSTICA relacionadas. São analogias fonéticas,
gramaticais e léxicas; por isso a língua
Sinônimos são palavras que apresentam
portuguesa não é um sistema único, mas um
sentidos equivalentes em determinados
conjunto de sistemas.
contextos. A relação entre dois sinônimos é
Os sistemas (variedades linguísticas) que
chamada de sinonímia.
integram a língua portuguesa apresentam
três aspectos fundamentais de diferenças
Ele faleceu às 9h do dia 14 de maio, no
entre si:
hospital Santa Cecília.
A) No espaço geográfico, constituindo os
Ele morreu às 9h do dia 14 de maio, no diferentes dialetos, que são as marcas
hospital Santa Cecília. determinantes referentes a diferentes
regiões. Por exemplo: tangerina, mexerica,
bergamota/ mandioca, macaxeira, aipim.
Diogo chega somente depois das 18h. Figurando também esta modalidade estão os
sotaques, ligados às características orais da
Diogo chega somente após as 18h. linguagem. Essa diversidade no espaço se
diz diatópica (do grego dia ‘através de’,
tópos ‘lugar’).
 ANTONÍMIA

Antônimos são palavras que apresentam


sentidos opostos. A relação entre antônimos
é chamada de antonímia.

 PARONÍMIA

Parônimos são palavras que apresentam


semelhança na escrita e pronúncia, mas têm
significados distintos. A relação entre dois
parônimos é chamada de paronímia.

B) No nível sociocultural, constituindo os


diferentes níveis de língua e estratos ou
camadas socioculturais. A norma culta, por
exemplo, é empregada pelas pessoas
escolarizadas. Pertencem a este nível
Tirinha de Adão Iturrusgarai.
também as gírias e os jargões, estes
ligados à linguagem profissional.
 VARIAÇÃO LINGUÍSTICA
A língua portuguesa encerra em si várias Essa diferença no estrato sociocultural se diz
tradições linguísticas, em parte análogas e diastrática (do latim stratum ‘estrato’, ‘camada’).
em parte divergentes, mas historicamente

8
Antigamente

“Antigamente, as moças chamavam-se


mademoiselles e eram todas mimosas e
muito prendadas. Não faziam anos:
completavam primaveras, em geral dezoito.
Os janotas, mesmo sendo rapagões,
faziam-lhes pé-de-alferes, arrastando a asa,
mas ficavam longos meses debaixo do
balaio. E se levavam tábua, o remédio era
tirar o cavalo da chuva e ir pregar em outra
freguesia. As pessoas, quando corriam,
antigamente, era para tirar o pai da forca e
não caíam de cavalo magro. Algumas
jogavam verde para colher maduro, e
sabiam com quantos paus se faz uma
canoa. O que não impedia que, nesse
entrementes, esse ou aquele embarcasse
em canoa furada.[...] Os mais idosos, depois
da janta, faziam o quilo, saindo para tomar
fresca; e também tomavam cautela de não
apanhar sereno. Os mais jovens, esses iam
ao animatógrafo, e mais tarde ao
cinematógrafo, chupando balas de alteia.
Ou sonhavam em andar de aeroplano; os
quais, de pouco siso, se metiam em camisa
de onze varas [...]
Acontecia o indivíduo apanhar constipação;
ficando perrengue, mandava o próprio
chamar o doutor e, depois, ir à botica para
aviar a receita, de cápsulas ou pílulas
fedorentas. [...]”
Carlos Drummond de Andrade

C) No estilo ou aspecto expressivo, isto é, em


relação a diferentes situações do falar e  Significado de algumas expressões:
estilos de língua. O contexto comunicativo
determina a forma de se expressar. Por 01. Fazer pé-de-alferes: namorar
exemplo: Um advogado em seu local de 02. Ficar debaixo do balaio: procurar namorado
trabalho: “Gostaria de fazer uma objeção,
03. Fazer o quilo: Período de descanso após as
meritíssimo”. O mesmo advogado no bar: refeições.
“Vou botar a boca no trombone, galera.”.
04. Animatógrafo / Cinematógrafo: Antigo
Essa diferença se diz diafásica (do grego
cinema
fásis ‘ expressão’)
05. Meter-se em camisa de onze varas: entrar-
D) Outra distinção essencial é a que se se em sérios problemas.
estabelece por meio do tempo. A língua não 06. Aviar a receita: preparar medicamento
é estática, mas sempre se renova. Essa segundo uma prescrição.
diferença no tempo se diz diacrônica(do
grego khrónos ‘tempo’)

9
PROFESSOR RESPONDE c) Para manifestar sua tendência à coloquialidade,
01. (UFT – 2012) o texto ignora completamente procedimentos do
A “netiqueta” Português padrão.
Como se comportar corretamente no mundo virtual: d) A coexistência da norma culta com a linguagem
- Maiúsculas: textos em maiúsculas (CAPS LOCK coloquial indica a diversidade dos usos do
ativado), na maioria dos casos, são a entender que você Português no Brasil.
está gritando. Se quiser destacar algo, sublinhe ou coloque
entre aspas. Se o programa utilizado na comunicação
e) No texto, encontramos exemplos do Português
permitir, use itálico ou negrito, mas sempre de forma apenas no seu uso padrão.
moderada para não poluir o texto. 03. (ENEM – 2014- Adaptado)
- Erros de grafia: em conversas informais é normal que
Só há uma saída para a escola se ela quiser ser mais
a norma culta da língua seja posta de lado. O que não quer
bem-sucedida: aceitar a mudança da língua como um
dizer que se possa escrever de qualquer jeito. Atenção
fato. Isso deve significar que a escola deve aceitar
para os erros que podem mudar o significado do que quis
qualquer forma da língua em suas atividades escritas?
dizer, como usar “mais” em vez de “mas”, “e” em vez de
Não deve mais corrigir? Não!
“é”, “de” em vez de “dê” e assim por diante.
A outra dimensão a ser considerada: de fato, no
- Pontuação: por mais informal que seja o texto, o
mundo real da escrita, não existe apenas um português
interlocutor pode não conseguir acompanhar o fluxo de
correto, que valeria para todas as ocasiões: o estilo dos
pensamento do redator. Daí a necessidade de pausas. Por
contratos não é o mesmo do dos manuais de instrução;
isso atenção à pontuação e à divisão de parágrafos.
o do juízes do Supremo não é o mesmo do dos
MURANO, Edgard. O texto na era digital. Revista Língua. cordelistas; o dos editoriais dos jornais não é o mesmo
Fev. de 2011 (texto adaptado)
do dos cadernos de cultura dos mesmos jornais. Ou do
As “netiquetas” apresentadas tratam do de seus colunistas.
POSSENTI, S. Gramática na cabeça. Língua Portuguesa, ano 5, n. 67, maio 2011
comportamento dos usuários em relação à (adaptado)

produção de textos no mundo virtual. Sobre os usos Sírio Possenti, professor, pesquisador e escritor
recomendados pelo texto, podemos afirmar que: brasileiro, considerado um dos mais conhecidos e
a) As letras maiúsculas têm um significado respeitados linguistas brasileiros da atualidade,
específico no mundo virtual e só podem ser defende a tese de que não existe “português
usadas para destacar passagens do texto, correto”. Assim sendo, o domínio da língua
quando o programa não possuir outro recurso. portuguesa implica, entre outras coisas, saber:
b) Em ambientes virtuais mais informais, a a) descartar as marcas de informalidade do texto.
exigência com a correta ortografia e com a b) reservar o emprego da norma padrão aos textos
pontuação adequada é dispensável, ainda que de circulação ampla.
algumas trocas de palavras possam acarretar c) moldar a norma padrão do português pela
mudança de significado. linguagem do discurso jornalístico.
c) A pontuação e a divisão de parágrafos são d) adequar as formas da língua a diferentes tipos
consideradas acessórias para que o leitor de textos e contexto.
acompanhe o pensamento do autor, uma vez e) desprezar as formas da língua previstas pelas
que marcam as pausas na leitura. gramáticas e manuais divulgados pela escola.
d) Um bom usuário de internet deve se preocupar
04. Considerando as diferenças entre língua oral e
com a poluição visual do texto, preferindo usar
língua escrita, assinale a opção que representa
itálico e negrito em vez de maiúsculas.
uma inadequação da linguagem usada ao
e) As palavras ‘mais’, ‘mas’, ‘e’, ‘é’, ‘de’ e ‘dê’
contexto.
estão grafadas em desacordo com as normas
a) “Só um instante, por favor. Eu gostaria de fazer
ortográficas vigentes.
uma observação” - alguém comenta em uma
02. (ENEM – 2006) reunião de trabalho.
Pronominais b) “O carro bateu e capotô, mas num deu pra vê
Dê-me um cigarro direito” - um pedestre que assistiu ao acidente
Diz a gramática comenta com o outro que vai passando.
Do professor e do aluno
c) “E aí, cara! Como vai essa força?” - um jovem
E do mulato sabido.
Mas o bom negro e o bom branco que fala para um amigo.
Da Nação Brasileira d) “Venho manifestar meu interesse em candidatar-
Dizem todos os dias me ao cargo de Secretária Executiva desta
Deixa disso camarada conceituada empresa” - alguém que escreve
Me dá um cigarro uma carta candidatando-se a um emprego.
(Oswald de Andrade. In: Poesia Pau-Brasil)
e) “Porque se a gente não resolve as coisas como
Assinale a alternativa correta a respeito desse poema: têm que ser, a gente corre o risco de termos,
a) Ao adotar a norma culta como mecanismo de num futuro próximo, muito pouca comida nos
julgamento estilístico, o texto implicitamente lares brasileiros e nós pode, com isso, passar
condena o analfabetismo. fome” - um professor universitário em um
b) O uso do Português padrão no primeiro verso, congresso internacional;
em contraste com a adoção da linguagem
coloquial no último, insinua a superioridade
daquele sobre este.
10
05. (ENEM – 2011) c) Apenas o item II.
MANDIOCA — mais um presente da Amazônia Aipim, d) Os itens I e II.
castelinha, macaxeira, maniva, maniveira. As e) Os itens II e III.
designações de Manihot utilíssima podem variar de 03. (IFPI – 2013) Entendendo que paronímia é a
região, no Brasil, mas uma delas deve ser levada em relação que se estabelece entre palavras
conta em todo o território nacional: pão-de-pobre - e semelhantes entre si na pronúncia e na escrita,
por motivos óbvios. Rica em fécula, a mandioca - uma mas com significados diferentes, assinale a
planta rústica e nativa da Amazônia disseminada no
alternativa em que a palavra destacada não
mundo inteiro, especialmente pelos colonizadores
portugueses - é a base de sustento de muitos está empregada adequadamente.
brasileiros e o único alimento disponível para mais de a) “Em entrevista à Folha, em dezembro do ano
600 milhões de pessoas em vários pontos do planeta, passado, Fux admitiu ter se encontrado com
e em particular em algumas regiões da África. Dirceu, mas negou ter dado qualquer garantia
O melhor do Globo Rural. Fev. 2005 (fragmento).
de absolvição”. (Folha de São Paulo – Poder e
Política, abril de 2013)
De acordo com o texto, há no Brasil uma variedade b) “A decisão da juíza Carmen Fernandez, que
de nomes para a Manihot utilíssima, nome científico deferiu a reintegração do prédio da USP Leste,
da mandioca. Esse fenômeno revela que: utiliza de uma argumentação completamente
a) existem variedades regionais para nomear uma contrária à empregada pelo juiz Adriano Marcos
mesma espécie de planta. Laroca”. (Rede Brasil Atual, outubro de 2013)
b) mandioca é nome específico para a espécie c) “Os deputados aprovaram ainda os Projetos de
existente na região amazônica. Lei nº 32 e 35 do Poder Executivo, que tratam
c) “pão-de-pobre” é designação específica para a sobre a dispensa de multas e juros de mora no
planta da região amazônica. pagamento de débitos em atraso do ICMS e
d) os nomes designam espécies diferentes da IPVA”. (www.detran.pi.gov.br, novembro de
planta, conforme a região. 2013).
e) a planta é nomeada conforme as d) “A Câmara dos Deputados aprovou o projeto de
particularidades que apresenta. decreto legislativo que autoriza a ratificação do
ALUNO RESPONDE acordo firmado entre o Brasil e a Organização
ANALISE A TIRINHA E RESPONDA ÀS das Nações Unidas (ONU) para a realização da
QUESTÕES 01 E 02. Conferência das Nações Unidas sobre
Desenvolvimento Sustentável (Cnuds), a
Rio+20, no Rio de Janeiro, de 13 a 22 de junho”.
(Portal Brasil, abril de 2012)
e) “As Nações Unidas estão comprometidas em
combater todas as formas de descriminação”.
(www.onu.org.br)
04. (IFPI – 2016) Leia o texto:
Somos todos inúteis
A sociedade é que nos prepara para a vida. Claro
01. (IFPI – 2016-Modificada) Analisando a tirinha, é
que temos nossas particularidades, nossas
possível perceber que procrastinação é o mesmo idiossincrasias, mas nossa visão de mundo e o
que: grosso de nosso comportamento são-nos
a) Antecipar uma atividade. transmitidos pela família, o grupo, a escola, a
b) Deixar alguma coisa para depois. igreja, enfim, pela sociedade. [...]
c) Abreviar uma ação. Analisando o contexto em que o termo foi
d) Acelerar algo. empregado no primeiro parágrafo, é possível
e) Realizar pontualmente uma ação. perceber que idiossincrasia é o mesmo que:
02. (IFPI – 2016) Considerando a tirinha, julgue os a) Estratégia de comunicação em grupos.
itens abaixo: b) Atitude de liderança em grupos sociais.
I. A substituição da palavra “definitivo” por “em c) Relações entre as instituições sociais.
construção” sugere que esses termos têm d) Traço comum entre os cidadãos burgueses.
significados opostos e, por isso, podem ser e) Característica de comportamento peculiar de
considerados antônimos. um indivíduo.
II. O termo “procrastinador” designa a pessoa que 05. (IFPI – 2014 - modificada) As línguas não são
procrastina. estanques, fixas ou imutáveis. Ao contrário, uma
III. Na expressão “procrastinador ideal”, é possível mesma língua pode se apresentar de diferentes
afirmar que o termo sublinhado qualifica o termo formas, a depender da região, dos falantes, da
“procrastinador”. situação de uso, da modalidade (escrita ou
Está(ão) correto(s): falada). Nesse sentido, a Língua Portuguesa é
a) Todos os itens. um grande e rico exemplo de variação
b) Apenas o item I. linguística.
11
a) A língua varia de acordo com cada região.
b) A língua é um sistema vivo e pode ser
modificada por seus falantes de acordo com a
situação linguística.
c) A linguagem é a mesma empregada por todos
os falantes, independente da situação
linguística.
d) A linguagem não varia de acordo com a idade.
e) A linguagem não tem qualquer relação com a
cultura, região ou contexto social.
09. (IFPI – 2013)
O texto acima retrata uma situação de uso da língua: “Já vi pessoas chorando por
a) Em que se espera a utilização da modalidade filme, novela, futebol, mas não
encontro pessoas chorando
formal da língua, já que se trata de uma
pela restauração da família”.
entrevista de emprego; Padre Chrystian Shankar
b) Na qual se espera o emprego da modalidade (tv.cançaonova.com)

informal da língua portuguesa, uma vez que a Baseando-se nas diferentes esferas sociais do
pessoa entrevistada já afirmou dominar outras uso da língua, o falante é obrigado a adaptá-la
línguas; às variadas situações de comunicação. Partindo
c) Em que há preconceito linguístico, já que um dos dessa afirmação, podemos deduzir que o texto
falantes não tem a sua fala respeitada pelo faz uso:
outro;
d) Na qual nenhum dos falantes não demostra a) Da norma culta ou padrão, pois o contexto
conhecimento da língua portuguesa; retrata uma situação de uso formal da língua;
e) Que raramente ocorre. b) Da norma culta, mas a situação formal em que
foi utilizada a língua não exige esse uso;
06. (IFPI - 2014) A palavra que descaracteriza, c) Da variante informal, já que mostra intimidade
segundo o texto, a modalidade culta da entre os interlocutores;
Língua Portuguesa é: d) De uma linguagem relaxada e descuidada,
a) Fluentemente; demonstrando que o mais importante é o
b) Inglês; conteúdo do texto;
c) Russo; e) Da norma padrão, o que dificulta o seu
d) Português; entendimento.
e) Varêia.
10. (IFPI – 2016) A situação abaixo foi retirada da
07. (IFPI - 2014) De acordo com a charge acima, página de um blog de língua portuguesa. Trata-
a) O homem demonstra consciência da variação da se da participação e opinião de um internauta
língua; sobre a língua, mais especificamente sobre:
b) A mulher demonstra consciência da variação da
língua; Segunda-feira | 22/06/2015 / Anônimo
c) Nenhum dos personagens demonstra
consciência da variação linguística;
d) A palavra “varêia” foi empregada para demostrar
a variação da língua; Faço muita confusão com a língua portuguesa, é
e) Não há qualquer problema com o emprego de muito complexa uma hora é outra não é...
nenhuma palavra da língua portuguesa. a) A variação da língua portuguesa.
b) A rigidez das regras gramaticais da língua
08. (IFPI – 2016) Relacionando a imagem abaixo
portuguesa.
à variação da linguagem, assinale a alternativa
c) A dificuldade na ortografia da língua portuguesa.
que melhor traduz essa relação.
d) A dificuldade na pronúncia de algumas
expressões da língua.
e) As regras de concordância.

12
11. (IFPI - 2015) meses antes, a estudar preparatórios. Vivia tranquilo,
naquela casa assobradada da rua do Senado, com os
Cordel adolescente, ó xente! meus livros, poucas relações, alguns passeios. A
Sou mocinha nordestina, família era pequena, o escrivão, a mulher, a sogra e
Meu nome é Doralice, duas escravas. Costumes velhos. Às dez horas da
tenho treze anos de idade, noite toda a gente estava nos quartos; às dez e meia a
conto e reconto o que disse, casa dormia. Nunca tinha ido ao teatro, e mais de uma
pois me chamo Doralice, vez, ouvindo dizer ao Meneses que ia ao teatro, pedi-
sou quem vende meu cordel lhe que me levasse consigo. Nessas ocasiões, a sogra
nas feiras lindas de longe fazia uma careta, e as escravas riam à socapa; ele não
onde a poesia se esconde respondia, vestia-se, saía e só tornava na manhã
nas sombras do meu chapéu! seguinte. Mais tarde é que eu soube que o teatro era
Eu falo tudo rimado um eufemismo em ação. Meneses trazia amores com
no adoçado da palavra uma senhora, separada do marido, e dormia fora de
do Nordeste feiticeiro; casa uma vez por semana. Conceição padecera, a
no meu jeito brasileiro, princípio, com a existência da comborça; mas, afinal,
aqui vim dizer e digo resignara-se, acostumara-se, e acabou achando que
que escrevo muito livro era muito direito.
que penduro num cordel,
todo fato acontecido
12. (IFPI – 2013) A linguagem utilizada pelas
eu coloco num papel! personagens do conto de Machado de Assis é
[...] bem diferente da usada nos dias de hoje, nas
Vim pra feira, noutro dia, relações sociais, isso se dá porque:
armei a minha poesia a) A língua sofre transformações de acordo com o
num cordel de horizonte. tempo e o contexto social em que está inserida;
Quem passava no defronte b) A língua nunca deve se modificar, pois isso
daquilo que eu vendia, dificulta a comunicação e as relações sociais;
parava e me escutava, c) Somente a língua portuguesa sofreu
pois sou mocinha falante,
modificações relacionadas ao tempo e ao
declamava o que escrevia!
contexto social;
Contei de uma garota d) As mudanças na língua se dão apenas no
que amava um cangaceiro,
âmbito social;
era um tal cabra da peste,
um valentão do Nordeste
e) As variações linguísticas ocorreram porque no
que montava a Ventania, século XXI não utilizamos os mesmos meios de
trazia susto e coragem comunicação usados no século XIX.
por cada canto que ia!
[...] 13. (IFPI- 2013) Ainda sobre a variedade da língua
Sylvia Orthof. Cordel adolescente, ó xente!. São usada no texto, é correto afirmar que:
Paulo, Quinteto, 1996.
a) O texto apresenta a variedade padrão da língua
A variação linguística é característica da escrita;
linguagem por apresentar diferentes formas de ser b) A fala das personagens apresenta
empregada, dependendo de vários fatores como características do uso coloquial da língua;
região, faixa etária, situação e modalidade. c) A palavra “comborça”, no final do segundo
Assinale os versos em que se verifica uma parágrafo, é um exemplo de gíria usada ainda
expressão típica do falar nordestino, hoje;
característica, portanto de uma variação regional. d) Se fôssemos escrever o texto hoje, usaríamos
a) Sou mocinha nordestina, Meu nome é Doralice os mesmos termos, já que a língua é estática;
b) Eu falo tudo rimado no adoçado da palavra e) O texto apresenta uma linguagem descuidada,
c) do Nordeste feiticeiro; no meu jeito brasileiro, típica dos textos informativos.
d) Contei de uma garota que amava um cangaceiro ANALISE A SITUAÇÃO ABAIXO PARA
e) era um tal cabra da peste, um valentão do RESPONDER ÀS QUESTÕES 13 E 14.
Nordeste
LEIA O TEXTO PARA AS QUESTÕES 12 E 13.
MISSA DO GALO
Machado de Assis
Nunca pude entender a conversação que tive com
uma senhora, há muitos anos, contava eu dezessete,
ela trinta. Era noite de Natal. Havendo ajustado com
um vizinho irmos à missa do galo, preferi não dormir;
combinei que eu iria acordá-lo à meia-noite.
A casa em que eu estava hospedado era a do
escrivão Meneses, que fora casado, em primeiras
núpcias, com uma de minhas primas. A segunda
mulher, Conceição, e a mãe desta acolheram me bem,
quando vim de Mangaratiba para o Rio de Janeiro,
13
14. (IFPI – 2016) “O tal de facebook está 16. Com base no texto I, analise as afirmações
acabando com o português” porque: abaixo e marque, em seguida, a alternativa
a) Nas redes sociais, os falantes de língua correta.
portuguesa não costumam empregar a I. A língua é um sistema aberto que está
modalidade escrita da língua culta; não sempre em elaboração, inclinando-se
costumam obedecer a regras da gramática conforme os moldes dos seus falantes.
normativa. II. A língua é um produto cultural diversificado no
b) No facebook, as pessoas não escrevem; vocabulário, mas não na gramática.
apenas usam emotions. III. Os livros de gramática adéquam suas regras
c) O português fala muito difícil. conforme a mudança da língua.
d) A língua portuguesa empregada em Portugal é IV. A língua é um produto cultural diversificado
diferente da linguagem empregada no Brasil. tanto no léxico quanto na gramática.
e) Foram os brasileiros que inventaram o a) As afirmações I e II estão corretas.
facebook. b) As afirmações I e III estão corretas.
15. (IFPI – 2016) A charge acima faz uma reflexão c) As afirmações I e IV estão corretas.
sobre a língua, sob o ponto de vista: d) As afirmações II e III estão corretas.
e) As afirmações III e IV estão corretas.
a) do emprego dos verbos na segunda pessoa do
singular. Analise o texto abaixo:
b) dos dialetos da língua portuguesa empregada
no Brasil e em Portugal.
c) da língua como um sistema imutável.
d) da variação da linguagem, mostrando que ela
não é igual em todas as situações de uso.
e) da fonologia da língua portuguesa.
LEIA O TEXTO PARA RESPONDER A
QUESTÃO 16.
A morte das palavras
Palavras são como as pessoas: nascem, vivem e
morrem. Umas de morte morrida, tão velhas ficaram
como as coisas que designavam. Quem hoje penteia
suas madeixas ou anda de tílburi? Quem hoje compra
rapé ou usa pince-nez?
Outras morrem de morte matada: são
substituídas por palavras mais modernas, mais 17. (IFPI 2020.1) A variedade linguística ilustrada
"antenadas" com nosso tempo. Quem hoje chamaria o na fala do personagem piauiense na charge acima
goleiro de quíper ou o médio-volante de centeralfo? decorre principalmente do seguinte aspecto:
Quem chamaria "locutor" de speaker? Quem ainda a) socioeconômico
datilografa o próprio nome ou disca um número no b) escolaridade
telefone? Evidentemente, as palavras são o espelho da c) regional
realidade e mudam com o mesmo dinamismo com que d) temporal
muda a realidade. e) contextual
Mas o espantoso é que até palavras gramaticais,
Leia o poema:
aquelas que não espelham a realidade, apenas fazem
a língua funcionar, também morram - por vezes, “Iscute o que to dizeno,
assassinadas pelos próprios falantes. É o caso de Seu dotor, seu coroné:
“cujo”, pronome relativo possessivo, muito útil no De fome tão padeceno
passado e ainda presente nos compêndios gramaticais Meus fio e minha muiér.
que, talvez por obrigar a uma inversão sintática da Sem briga, questão nem guerra,
oração, começou a causar embaraço aos usuários Meça desta grande terra
menos destros do vernáculo. Especialmente quando Umas tarefas pra eu!
está em jogo outra pedra no sapato dos falantes: a Tenha pena do agregado
concordância. E assim até falantes rotulados cultos Não me dexe deserdado
(pelo menos, portadores de diploma universitário) Daquilo que Deus me deu”.
(Patativa do Assaré)
fazem certos malabarismos verbais para evitar o
18. Esse falante, pelos elementos explícitos e
emprego de um “cujo” que, mal colocado, é uma
verdadeira casca de banana à espera do transeunte implícitos no poema, é identificável como:
incauto. E dá-lhe “a pessoa que o nome dela eu não a) Escolarizado proveniente de uma metrópole.
lembro agora” ou “o sujeito que o filho é médico”, no b) Sertanejo de uma área rural.
lugar de “ A pessoa cujo nome não lembro agora” ou “O c) Idoso que habita uma comunidade urbana.
sujeito cujo filho é médico”. d) Escolarizado que habita uma comunidade no
http://revistalingua.uol.com.br. (Adaptado) interior do país.
e).Estrangeiro que imigrou para uma
comunidade do sul do país.
.
14
DENOTAÇÃO 1) Função Referencial ou Denotativa
Uma palavra é usada no sentido denotativo Transmite uma informação objetiva sobre a
quando apresenta seu significado original, realidade. Dá prioridade aos dados concretos,
independentemente do contexto frásico em que fatos e circunstâncias.
aparece. Quando se refere ao seu significado mais Características da função referencial:
objetivo e comum, aquele imediatamente É impessoal, não apresentando a opinião do
reconhecido. A denotação tem como finalidade emissor;
informar o receptor da mensagem de forma clara e Utiliza a 3.ª pessoa do discurso.
objetiva, assumindo assim um caráter prático e Exemplos:
utilitário. É utilizada em textos informativos, como Numa cesta de vime temos um cacho
jornais, regulamentos, manuais de instrução, bulas de uvas, uma maçã, uma laranja, uma
de medicamentos, textos científicos, entre outros. banana e um morango. (Este texto
Exemplos: informa o que há dentro da cesta, logo,
O elefante é um mamífero. há função referencial).
Já li esta página do livro. 2) Função Expressiva ou Emotiva
A empregada limpou a casa. Reflete o estado de ânimo do emissor, os seus
“Uma sociedade extremamente organizada, sentimentos e emoções.
que não possui nenhum tipo de liderança. Parece Características da função emotiva:
impossível? Não no mundo das formigas. A maioria A mensagem transmitida é subjetiva, conforme a
das formigas em uma colônia é formada por visão do emissor.
fêmeas, reprodutoras (rainhas) e não reprodutoras É pessoal, sendo utilizada a 1.ª pessoa do
(operárias).” discurso.
CONOTAÇÃO Há a presença de interjeições que enfatizam o
Uma palavra é usada no sentido conotativo discurso.
(figurado) quando apresenta diferentes significados, Utiliza pontuação que acentua a sua entonação
sujeitos a diferentes interpretações, dependendo do emotiva, como os pontos de exclamação e as
contexto frásico em que aparece. Quando se refere reticências.
a sentidos, associações e ideias que vão além do Exemplos:
sentido original da palavra, ampliando sua a) Ah, que coisa boa!
significação mediante a circunstância em que a b) Tenho um pouco de medo...
mesma é utilizada, assumindo um sentido figurado c) Nós te amamos!
e simbólico. 3) Função apelativa ou conativa
A conotação tem como finalidade provocar Também chamada de conotativa, tem como
sentimentos no receptor da mensagem, através da principal objetivo influenciar e persuadir o
expressividade e afetividade que transmite. É receptor, sendo um apelo para que este faça algo.
utilizada principalmente numa linguagem poética e Características da função apelativa:
na literatura, mas também ocorre em conversas Predomina o uso de verbos no imperativo.
cotidianas, em letras de música, em anúncios Utiliza a 2.ª ou 3.ª pessoa do discurso (tu e você).
publicitários, entre outros. Há a presença de vocativos que direcionam a
Exemplos: mensagem.
Você é o meu sol! Recorre a pontos de exclamação para enfatizar o
Minha vida é um mar de tristezas. discurso.
Você tem um coração de pedra! Exemplos da função apelativa:
Tirar o cavalinho da chuva. Aproveite as melhores ofertas!
Só se vê bem com o coração. Não perca esta chance! Ligue ainda hoje!
Pulga atrás da orelha. Cidadão consciente, vote em mim!
4) Função poética
Tem como principal objetivo transmitir uma
mensagem elaborada, formalmente estruturada,
com as palavras cuidadosamente selecionadas
para produzir um resultado estético.
Características da função poética:
Utiliza uma linguagem elaborada e cuidada.
FUNÇÕES DA LINGUAGEM
Dá importância ao ritmo, melodia e sonoridade
Sabemos que a linguagem é uma das formas de
das palavras.
apreensão e de comunicação das coisas do mundo.
Procura o que é belo e inovador.
O ser humano, ao viver em conjunto, utiliza vários Exemplos da função poética:
códigos para representar o que pensa, o que sente, o "O poeta é um fingidor.
que quer, o que faz. Finge tão completamente
A multiplicidade da linguagem pode ser sintetizada Que chega a fingir que é dor
em seis funções ou finalidades básicas. Veja a A dor que deveras sente."
seguir: Fernando Pessoa

15
"Basta-me um pequeno gesto, 02. Identifique o item em que a função
feito de longe e de leve, predominante da linguagem é a
para que venhas comigo REFERENCIAL:
e eu para sempre te leve..."
Cecília Meireles a)
5) Função fática
A função fática tem como principal objetivo
estabelecer um canal de comunicação entre o
emissor e o receptor, quer para iniciar a transmissão
da mensagem, quer para assegurar a sua
continuação.
Características da função fática:
Recorre a frases interrogativas para obter resposta
do receptor.
Utiliza interjeições e onomatopeias para manter o
discurso.
Exemplos da função fática:
Alô! Alô? b) “Poesia/é brincar com palavras / como se
Bom dia! brinca/com bola, papagaio, pião”. (José Paulo
Sei... Paes)
Hum... hum... c) Dólar fecha abaixo de R$ 3,20 pela primeira
6) Função metalinguística vez em quase oito meses.
A função metalinguística tem como principal objetivo d) "Fonte de mel / nos olhos de gueixa”.
usar um determinado código para explicar esse (Caetano Veloso)
próprio código.
Características da função metalinguística:
Utiliza o código como tema da mensagem. LEIA O TEXTO PARA RESPONDER À
Tem uma função explicativa. QUESTÃO 03.
LEMBRE DE QUANDO EU IMAGINE O TRABALHO ESTE DESENHO É O AINDA ASSIM ELE VAI
NASCI? NÃO CONSEGUIA RESULTADO DE 6 ANOS
Exemplos da função metalinguística: NEM ME VIRAR NA CAMA
QUE FOI DESENVOLVER
AS HABILIDADES DE CONSTANTE LABUTA!
NÃO VOU VALORIZAR!
TE PAGAR É UM
O código linguístico é um sistema de signos usados SOZINHO! MEUS OLHOS MOTORAS PARA UMA VIDA INTEIRA DE 500 DOLARES INVESTI-
NÃO SABIAM FOCALIZAR! SEGURAR UM GIZ-DE ESFORÇOS PARA CHEGAR POR ELE MENTOI!
na construção de mensagens. EU NÃO CONSEGUIA -CERA, COLOCAR SUA A TAL RESULTADO.
FAZER NADA! PONTA SOBRE O PAPEL
Uma mensagem é uma comunicação oral ou escrita E IMPRIMIR-LHE MOVI-
que visa transmitir uma informação. MENTOS CONSTANTES E
COORDENADOS!

PROFESSOR RESPONDE
01. Assinale a alternativa que apresenta linguagem
conotativa:
a) “O ano de 1948, em Pernambuco, foi marcado
por um processo revolucionário, liderado por um
Partido Liberal radical.” https://www.normaculta.com.br/funcoes-da-linguagem/
b) “Nem mesmo o Recife que aprendi a amar 03. Para tentar convencer o pai a comprar seu
depois – Recife das revoluções libertárias – Mas desenho, Calvin empregou uma função de
o Recife sem história nem literatura – Recife sem linguagem específica. Assinale a alternativa
mais nada – Recife da minha infância” que indica a resposta correta:
c) “depois de analisar os prontuários de 964 a) função conativa.
pessoas operadas no Hospital das Clínicas da b) função fática.
Universidade Federal de Pernambuco, no Recife, c) função poética.
o médico Cláudio Moura Lacerda de Melo, 31 d) função emotiva.
anos, concluiu que seus colegas exageraram na
requisição de exames radiológicos e de 04. Foi empregada expressão com sentido
laboratório”. conotativo na frase da alternativa:
d) “Em todo triângulo, o quadrado de qualquer lado a) Naturalmente, recursos existem ...
é igual a soma dos quadrados dos outros dois, b) empobrece quem com grande dificuldade e
menos o duplo produto destes dois lados pelo pouco apoio se dedica a produzir alimento.
cosseno do ângulo que eles formam. c) As notícias dos jornais e televisão são de
cortar o coração: ...
d) Imagino que as autoridades tenham destinado
bom dinheiro...

16
LEIA TEXTO PARA RESPONDER À QUESTÃO 05. 02. Analisando as falas das personagens,
O exercício da crônica assinale a alternativa que contenha as
Escrever crônica é uma arte ingrata. Eu digo prosa expressões utilizadas em seu sentido
fiada, como faz um cronista; não a prosa de um ficcionista, conotativo:
na qual este é levado meio a tapas pelas personagens e a) “Você vai comer asfalto” e “estou morto”.
situações que, azar dele, criou porque quis. Com um
b) “No quinto período, seu babaca” e “você vai
prosador do cotidiano, a coisa fia mais fino. Senta-se ele
diante de uma máquina, olha através da janela e busca comer asfalto”.
fundo em sua imaginação um assunto qualquer, de c) “Ameaça terrorista” e “aula de educação
preferência colhido no noticiário matutino, ou da véspera, física”.
em que, com suas artimanhas peculiares, possa injetar um d) “Ameaça terrorista” e “estou morto”.
sangue novo. Se nada houver, restar-lhe o recurso de
03. (Fuvest- SP- adaptada) Todas as frases a
olhar em torno e esperar que, através de um processo
associativo, surja-lhe de repente a crônica, provinda dos seguir estão no sentido conotativo. Leia-as e
fatos e feitos de sua vida emocionalmente despertados em seguida marque a opção correta:
pela concentração. Ou então, em última instância, recorrer I. Uma andorinha só não faz verão
ao assunto da falta de assunto, já bastante gasto, mas do II. Nem tudo que reluz é ouro
qual, no ato de escrever, pode surgir o inesperado. III. Quem semeia ventos, colhe tempestades
(MORAES, V. Para viver um grande amor: crônicas e
poemas. São Paulo: Cia das Letras, 1991).
IV. Quem não tem cão caça com gato.
As ideias centrais dos provérbios acima são, na
05. Predomina nesse texto a função da linguagem
ordem:
que se constitui
a) Solidariedade–aparência–vingança-
a) nos elementos que servem de inspiração ao
dissimulação.
cronista.
b) cooperação– aparência -punição- adaptação.
b) nos assuntos que podem ser tratados em uma
c) egoísmo-ambição-vingança-falsificação.
crônica.
d) cooperação–ambição–consequência-
c) no papel da vida do cronista no processo de
dissimulação.
escrita da crônica.
d) nas dificuldades de se escrever uma crônica por 04. O item em que o termo sublinhado está
meio de uma crônica. empregado no sentido denotativo é:
a) “Além dos ganhos econômicos, a nova
ALUNO RESPONDE realidade rendeu frutos políticos.”
01. O termo (ou expressão) destacado que está b) “....com percentuais capazes de causar
empregado em seu sentido próprio, denotativo, inveja ao presidente.”
ocorre em c) “Os genéricos estão abrindo as portas do
a) “(...) É de laço e de nó mercado...”
De gibeira o jiló d) “....a indústria disparou gordos
Dessa vida, cumprida a sol(...)” investimentos.”
(Renato Teixeira. Romaria. Kuarup Discos. setembro de 1992.)
e) “Colheu uma revelação surpreendente:...”
b) “Protegendo os inocentes é que Deus, sábio
demais, põe cenários diferentes nas impressões 05. Em todas as frases a seguir há função
digitais.” (Maria N. S. Carvalho. Evangelho da Trova. /s.n.b.) emotiva, EXCETO:
c) “O dicionário-padrão da língua e os dicionários a) Ah, fiquei tão feliz com essa notícia!
unilíngues são os tipos mais comuns de b) Minha nossa, sinto-me tão triste e cansado...
dicionários. (Maria T. Camargo Biderman. O dicionário-padrão da língua. c) Estou sentindo um ódio extremo dele.
Alfa (28), 2743, 1974 Supl.)
ÀS VEZES EU TENHO A . . . DE QUE A BATERIADELE NÃO ACABA
d) A educação pública não apresenta bons
d) IMPRESSÃO . . . resultados.
LEIA O TEXTO PARA RESPONDER À QUESTÃO
06.
Às vezes a saudade deita ao meu lado na cama, e
eu sinto o seu peso no meu ombro. Às vezes ela
caminha comigo ao meu lado, sorri para mim quando
O Globo. O menino maluquinho: agosto de 2002 passo em frente de determinados lugares. Ela
LEIA O TEXTO PARA RESPONDER À QUESTÃO sempre, sempre, puxa a manga da minha camisa e
02. me aponta o outro lado da rua quando estou indo
para o trabalho, indo para o futebol, indo para o
shopping ler – há uma rua onde a saudade está
sempre na esquina, me pedindo carona. Tentando
fugir, fui à praia caminhar, final de tarde. A saudade
sentou ao meu lado, estendeu uma toalha branca, me
serviu uma taça de vinho e deitou a cabeça no meu
colo. A saudade tem caprichos, me impede de ir a
determinados lugares, não me deixa voltar a certos
locais. Às vezes a saudade embaça meus olhos, não
me deixa ver direito as cores que há no dia. Às vezes
17
ela passa o dia sem aparecer, e eu penso que tudo LEIA O TEXTO PARA RESPONDER À QUESTÃO
vai voltar ao normal. Então eu chego a casa, tomo um 09.
banho, deito e escuto seus passos. Ela vem, passa a
mão pela minha barba malfeita, toca de leve meu MEC quer rever veto a livro de Monteiro Lobato
peito e deita ao meu lado, cantando baixinho as O ministro da Educação, Fernando Haddad,
músicas que devo escutar. Eu sinto seu peso em meu pedirá que o CNE (Conselho Nacional de
ombro. Ela me embala. Mas eu demoro muito a Educação) reveja o parecer que recomendou
dormir. restrições à distribuição do livro “Caçadas de
Steller de Paula Pedrinho”, de Monteiro Lobato, em escolas públicas.
06. No texto acima, predomina a função da O Conselho de Educação quer vetar livro de
Monteiro Lobato em escolas.
linguagem
Como revelou a Folha, o conselho sugeriu que a
a) apelativa, num jogo de sentido pelo qual o obra não seja distribuída pelo governo ou, caso isso
poeta transmite uma forma idealizada de amor. seja feito, que contenha uma “nota explicativa”,
b) referencial, privilegiando-se a expressão de devido a um suposto teor racista.
forma racional. Haddad disse ter recebido diversas reclamações
c) emotiva, marcada pela contenção dos de educadores e especialistas contra a decisão do
sentimentos, dando vazão ao subjetivismo. CNE. “Foram muitas manifestações para que o MEC
d) poética, fazendo uso da personificação como afaste qualquer hipótese de censura a qualquer
base da construção do texto. obra”, afirmou.
(PINHO, Angela. In: http://www.substantivoplural.com.br/monteiro-lobato-e-a-
LEIA A ESTROFE ABAIXO: proibicao-da-cacada-de-pedrinho/. Acessado em 09/07/2017)
"Oh! ter vinte anos sem gozar de leve 09. No trecho acima predomina a função
A ventura de uma alma de donzela! referencial da linguagem, por meio da qual o
E sem na vida ter sentido nunca
emissor:
Na suave atração de um róseo corpo
Meus olhos turvos se fechar de gozo! a) imprime ao texto as marcas de sua atitude
Álvares de Azevedo pessoal, seus sentimentos.
07. A presença da interjeição, as exclamações e a 1ª b) transmite informações objetivas sobre o tema
pessoa gramatical identificam no texto a função de que trata o texto.
da linguagem: c) busca persuadir o receptor do texto a adotar
a) Conativa. certo comportamento.
b) Referencial. d) procura explicar a própria linguagem que
c) Metalinguística. utiliza para construir o texto.
d) Emotiva. 10. Identifique a função poética nas opções a
LEIA O TEXTO PARA RESPONDER À seguir:
QUESTÃO 08. a) "Economize com a HDI seguros. Peça uma
O seu santo nome cotação HDI Auto para seu corretor."
Não facilite com a palavra amor. b) "A minha alma partiu-se como um vaso
Não a jogue no espaço, bolha de sabão. vazio." (Álvaro de Campos)
Não se inebrie com o seu engalanado som. c) Preciso que você confirme alguns dados,
Não a empregue sem razão acima de toda a razão pode ser? Certo. Podemos começar?
( e é raro).
d) Nossa! Que dia cansativo! Trabalhei feito um
Não brinque, não experimente, não cometa a
loucura sem remissão condenado. Ainda bem que o fim de semana
de espalhar aos quatro ventos do mundo essa chegou e eu vou me divertir!
palavra LEIA O TEXTO PARA RESPONDER À
que é toda sigilo e nudez, perfeição e exílio na QUESTÃO 11.
Terra. Samba de uma nota só
Não a pronuncie. Eis aqui este sambinha feito numa nota só.
Carlos Drummond de Andrade
https://www.pensador.com/frase/MzQ5MTEx Outras notas vão entrar, mas a base é uma só.
08. No texto lido, predomina uma função de Esta outra é consequência do que acabo de dizer.
linguagem que também pode ser verificada: Como eu sou a consequência inevitável de você.
a) Em notícias de jornal nas quais o repórter Quanta gente existe por aí que fala tanto e não diz
registra fatos do cotidiano de interesse para o nada,
Ou quase nada.
leitor.
Já me utilizei de toda a escala e no final não sobrou
b) Em textos científicos que relatam descobertas nada,
e experiências realizadas pelos cientistas. Não deu em nada.
c) Em textos literários quando o "eu" poético ou o E voltei pra minha nota como eu volto pra você.
narrador expõem seus sentimentos e emoções. Vou contar com uma nota como eu gosto de você.
d) Em propagandas e publicidade em geral, que E quem quer todas as notas: ré, mi, fá, sol, lá, si, dó.
procuram orientar o comportamento do Fica sempre sem nenhuma, fique numa nota só.
interlocutor. Tom Jobim/Newton Mendonça

18
11. A letra desta música tem predominância de d) fática, porque o texto testa o funcionamento
qual função da linguagem? do canal de comunicação.
a) Metáfora LEIA O TEXTO PARA RESPONDER ÀS
b) Metalinguagem QUESTÕES DA 16 A 18.
c) Emotiva
A menina tonta passa metade do dia
d) Poética a namorar quem passa pela rua,
12. Assinale a alternativa em que a função apelativa que a outra metade fica
da linguagem é a que prevalece: pra namorar-se no espelho
a) Trago no meu peito um sentimento de solidão A menina tonta tem olhos de retrós preto,
sem fim… sem fim… cabelos de linha de bordar,
b) “Não discuto com o destino o que pintar eu e a boca é um pedaço de qualquer tecido
vermelho.
assino.” (Manuel de Fonseca)
c) Machado de Assis é um dos maiores escritores 16. O texto acima:
brasileiros. I- É escrito em prosa.
d) Conheça você também a obra desse grande II- É escrito em verso.
mestre. III- Emprega linguagem predominantemente
LEIA O TEXTO PARA RESPONDER ÀS conotativa.
QUESTÕES 13 E 14. IV- Emprega linguagem predominantemente
Nova Poética denotativa.
Vou lançar a teoria do poeta sórdido. Está(ão) correto(s) o(s) item(ns):
Poeta sórdido: a) I.
Aquele em cuja poesia há a marca suja da vida. b) II.
Vai um sujeito, c) I e III.
Saí um sujeito de casa com a roupa de brim branco d) II e IV.
muito bem engomada, e na primeira esquina passa e) II e III.
um caminhão, salpica-lhe o paletó ou a calça de uma
nódoa de lama: 17. No texto, o que sugere a expressão “linha de
É a vida bordar”, referindo-se aos cabelos da menina?
O poema deve ser como a nódoa no brim: a) cabelo liso e longo.
Fazer o leitor satisfeito de si dar o desespero [...] b) cabelo cacheado.
Manuel Bandeira letrasearmas.blogspot.com.br/2013/02/exercicios-sobre-
funcoes-da-linguagem.html c) cabelo sedoso.
d) cabelo fino e longo.
13. As funções de linguagem predominantes no e) cabelos curtos.
texto são:
a) Poética e metalinguística. 18. Quanto à função da linguagem, pode-se afirmar
b) Conativa e referencial. que o texto acima:
c) Referencial e emotiva. a) tem função referencial.
d) Metalinguística e conativa. b) é metalinguístico.
c) tem função poética.
14. As funções de linguagem predominantes no d) narra um episódio específico.
texto acima se justificam pelos seguintes fatores. e) é emotivo, expressando sentimentos do eu
a) metadiscursividade e interpelação ao leitor. lírico.
b) informatividade e sentimentalismo.
c) interpelação ao leitor e informatividade. Leia o texto abaixo para responder às questões:
d) criatividade linguística e metadiscursividade.
LEIA O TEXTO PARA RESPONDER À
QUESTÃO 15.
A biosfera, que reúne todos os ambientes onde se
desenvolvem os seres vivos, se divide em unidades
menores chamadas ecossistemas, que podem ser
uma floresta, um deserto e até um lago. Um
ecossistema tem múltiplos mecanismos que regulam
o número de organismos dentro dele, controlando sua
reprodução, crescimento e migrações.
DUARTE, M. O guia dos curiosos. São Paulo: Companhia das Letras, 1995. 19. (IFPI 2019.2) A função da linguagem
15. Predomina no texto a função da linguagem predominante no texto, característica do discurso
a) referencial, porque o texto trata de noções e publicitário, é:
informações conceituais. a) Fática
b) conativa, porque o texto procura orientar b) Poética
comportamentos do leitor. c) Referencial
c) poética, porque o texto chama a atenção d) Emotiva
para os recursos de linguagem. e) Apelativa

19
FIGURAS DE LINGUAGEM 4 - Continente pelo conteúdo: Bebeu o cálice
METÁFORA E COMPARAÇÃO (Figuras de palavra) todo. (= Bebeu todo o líquido que estava no
Metáfora é a figura que transporta a palavra (ou cálice.)
expressão) do seu sentido literal para o sentido 5 - Instrumento pela pessoa que utiliza: Os
figurado. Trata-se de uma comparação que é microfones foram atrás dos jogadores. (= Os
expressa sem os termos que caracterizam uma repórteres foram atrás dos jogadores.
comparação (COMO). 6 - Parte pelo todo: Várias pernas passavam
Exemplos: apressadamente. (= Várias pessoas
 Seus olhos são luzes brilhantes. passavam apressadamente.)
 Esse problema é só a ponta do iceberg. “Os meus braços precisam dos teus”
 A guerra é um monstro devorador. 7 - Singular pelo plural: A mulher foi chamada
 Meu pensamento é um rio subterrâneo. para ir às ruas na luta por seus direitos. (= As
A comparação acontece quando é estabelecida mulheres foram chamadas, não apenas uma
entre palavras ou expressões uma relação mulher.)
comparativa explícita, marcada pela presença de 8 - Marca pelo produto: Minha filha adora
termos como “como, assim como, tal como, igual a, danone. (= Minha filha adora o iogurte que é da
que nem”, entre muitos outros. marca Danone.)
Exemplos: ANTÍTESE(figuras de pensamento)
 Seus olhos são como luzes brilhantes. Consiste na utilização de dois termos que
 A paixão é como um fogo. contrastam entre si. Ocorre quando há uma
aproximação de palavras ou expressões de
CATACRESE
sentidos opostos.
Costuma ocorrer quando, por falta de um termo
Observe os exemplos:
específico para designar um conceito, toma-se outro
 "O mito é o nada que é tudo." (Fernando
"emprestado".
Pessoa)
Exemplos de catacrese usados no dia a dia:
 A cabeça do alfinete
 O corpo é grande e a alma é pequena.
 O dente de alho  "Quando um muro separa, uma ponte une."
 O pé da mesa  A sina dos médicos é conviver com a
 A maçã do rosto doença e a saúde.
 O braço da cadeira  Ele estava entre a vida e a morte.
 A batata da perna  A vida é mesmo assim, um dia a gente ri e
 O pé da cama no outro a gente chora.
 O céu da boca  Alegrias e tristezas são constantes da vida.
 Coroado abacaxi  O Renato estava dormindo acordado na
 O fio de azeite aula.
 A árvore genealógica EUFEMISMO
 A pele do tomate Consiste em empregar uma expressão mais
 A boca do túnel
suave, mais nobre ou menos agressiva, para
 A raiz do problema comunicar alguma coisa áspera, desagradável
SINESTESIA ou chocante.
Consiste em mesclar, numa mesma expressão, as Exemplos:
sensações percebidas por diferentes órgãos do  Depois de muito sofrimento, entregou a
sentido. alma ao Senhor. (morreu)
Exemplos:  A pobre mulher passou desta para melhor.
 Um grito áspero revelava tudo o que sentia. (morreu).
(grito = auditivo; áspero = tátil)  O prefeito ficou rico por meios ilícitos.
 No silêncio negro do seu quarto, aguardava (roubou).
os acontecimentos. (silêncio = auditivo; negro  Fernando faltou com a verdade. (mentiu).
= visual)
IRONIA
 Ela sentiu o sabor frio da derrota.
Consiste em dizer o contrário do que se pretende
METONÍMIA ou em satirizar, questionar certo tipo de
1 - Autor pela obra: Gosto de ler Machado de pensamento com a intenção de ridicularizá-lo, ou
Assis. (= Gosto de ler a obra literária de Machado ainda em ressaltar algum aspecto passível de
de Assis.) crítica.
“Eu adoro ler Maurício de Souza”. Veja os exemplos abaixo:
2 - Símbolo pelo objeto simbolizado: Não te  Como você foi bem na última prova, não
afastes da cruz. (= Não te afastes da religião.) tirou nem a nota mínima!
3 - Efeito pela causa: Sócrates bebeu a morte. (=  Parece um anjinho aquele menino, briga
Sócrates tomou veneno.) com todos que estão por perto.

20
HIPÉRBOLE 04. Identifique a opção que NÃO apresenta uma
É a expressão intencionalmente exagerada com o sinestesia:
intuito de realçar uma ideia. a) Ela tinha um sorriso muito doce.
Exemplos: b) Gosto de quem canta com voz macia.
 Faria isso milhões de vezes se fosse preciso. c) Andressa estava em cima da hora.
 "Rios te correrão dos olhos, se chorares." d) As cores quentes estão em alta nesta
(Olavo Bilac) estação.
 “Eu quero ter um milhão de amigos e bem mais 05. Na frase "Não tenho mais Maizena em casa",
forte poder cantar…” (Roberto Carlos) qual figura de linguagem é empregada?
 As crianças estavam mortas de sede. a) Metáfora
 Que calor infernal! b) Metonímia
c) Comparação
PROSOPOPEIA OU PERSONIFICAÇÃO
d) Catacrese
Consiste em atribuir ações ou qualidades de seres
animados a seres inanimados, ou características ALUNO RESPONDE
humanas a seres não humanos. 01. Indique a frase que NÃO traz uma antítese:
Observe os exemplos:
a) Tristeza não tem fim, felicidade sim"
 As pedras andam vagarosamente.
(Vinícius de Moraes)
 A floresta gesticulava nervosamente diante da b) Durante a vida, acreditamos em muitas
serra. verdades e mentiras.
 Chora, violão. c) A relação deles era de amor e ódio.
 Essa música tem um violino tão melancólico. d) O vento fazia promessas suaves a quem o
PROFESSOR RESPONDE escutasse.
02. O eufemismo está presente nos seguintes
01. (IFPI-2016) Assinale a figura de linguagem fragmentos, EXCETO:
predominantemente no trecho. a)
“E entra a Saudade… Fiquei QUE TAL “ NARRAR
OK! VAMOS COMEÇAR O POETICAMENTE O
Como assombrado e sem voz!” BRAINSTORM PARA O NOSSO
PRIMEIRO QUAL
SERÁ NOSSO DESAFIO DOS SERES
DOCUMENÁRIO SOBRE OS OBJETIVO COM RACIONAIS MAIS FRÁGEIS
HUMANOS! O FILME? DO PLANETA DIANTE DO
(Teixeira de Pascoaes) APOCALIPSE ”?
DENUN-
CIAR A
QUANTA DECADÊNCIA
a) Metáfora. DIVERSÃO DE UMA É, EU FALEI
RAÇA. QUASE
b) Metonímia. ISSO!
c) Antítese.
d) Comparação.
02. Assinale a opção onde há uma comparação:
a) “O pavão é um arco-íris de plumas”
b) “Minha vida é uma colcha de retalhos, todos da b) Era uma moça de inteligência bastante
mesma cor.” (Mário Quintana) limitada.
c) “Não serei o poeta de um mundo caduco.” c) O jovem rapaz partiu desta para melhor.
d) "O olhar dela é como a lua, brilha d) Essa menina parece que dorme acordada.
maravilhosamente". 03. "Muito bom aquele encanador. Colocou em
03. Relacione corretamente as figuras de linguagem nossa casa vários canos furados.". Esta frase
e as frases a seguir e assinale a opção correta: trata-se de qual tipo de figura de linguagem?
a) Metonímia
(1) metáfora
b) Ironia
(2) comparação
(3) catacrese c) Eufemismo
d) Metáfora
( ) A minha filha é como um anjo.
( ) Seu avô é forte como um touro. 04. Indique a seguir a opção onde NÃO há
( ) De qualquer jeito seu sorriso vai ser meu hipérbole:
raio de sol. (Charlie Brown Jr.) a) .
( ) “O bonde passa cheio de pernas.”
( ) Depois que terminou de costurar, percebeu
que não tinha feito a casa do botão.
( ) Ela me encarou e seu olhar era “pedra”.
Shampoo para cabelos fortes.
a) 221331
b) 223131 b)
c) 321231
d) 332211

Faca afiada.

21
c) “Por você eu dançaria tango no teto, LEIA O TEXTO E RESPONDA À QUESTÃO 08.
Eu limparia os trilhos do metrô, Eu nasci há dez mil anos atrás
Eu iria a pé do Rio a Salvador...” (Frejat) E não tem nada nesse mundo que eu não saiba
d) “…Mas, oh, não se esqueçam da rosa da rosa demais
Da rosa de Hiroshima (...)
A rosa hereditária Eu vi a arca de Noé cruzar os mares
A rosa radioativa…”(Vinícius de Moraes) Vi Salomão cantar seus salmos pelos ares
Eu vi Zumbi fugir com os negros pra floresta
05. Quais figuras de linguagem temos neste texto: Pro Quilombo dos Palmares, eu vi
"Às sete horas da manhã, a rua acordava. Era possível (...)
ouvir os gritos irritantes daquelas lindas crianças que Eu fui testemunha do amor de Rapunzel
choravam rios de lágrimas enquanto suas mães Eu vi a estrela de Davi brilhar no céu
terminavam de preparar o café da manhã." E pr’aquele que provar que eu tô mentindo
Eu tiro o meu chapéu.
a) Prosopopeia - Ironia – Hipérbole. (Eu nasci há dez mil anos atrás, Paulo Coelho e Raul Seixas.
LP, Há dez mil anos atrás, Philips, 1976)
b) Metáfora - Sinestesia – Hipérbole. 09. É possível observar, no trecho sublinhado, a
c) Metonímia - Ironia – Hipérbole. seguinte figura de linguagem:
d) Prosopopeia - Sinestesia - Hipérbole a) Metonímia.
LEIA O TEXTO A SEGUIR E RESPONDA À b) Hipérbole.
QUESTÃO 06. c) Catacrese.
d) Ironia.
Leia o texto a seguir e responda à questão 09.
AH? POIS VOU BOM, ENTÃO ...
MEU PAI FICA MEU
PROCESSÁ-LO POR INDIVÍDUO SUSPEITO,
PEDALANDO ESSA ADVOGADO
INJÚRIA, CALÚNIA DE MORAL
BICICLETA, MAS ESTÁ NA
E DIFAMAÇÃO!. DISCUTÍVEL
LINHA!
NUNCA VAI A E POUCO CONFIÁVEL.
.
LUGAR NENHUM.
DEVE SER UMA
METÁFORA DA
EXISTÊNCIA DELE.

VAMOS, ... HOMEM DO SEU


VOCÊPODE TEMPO, QUE AINDA NÃO
ESPERA: RESPONSÁVEL
FAZER TRABALHA SEI SE
https://deposito-de-tirinhas.tumblr.com/ POR GRANDES VERBAS,
MELHOR DO PARA CORRESPONDER GOSTO DO
CONDUTA POLÊMICA,
06. Nessa tirinha, a personagem faz referência a E AFINAL QUEM PODE QUE ISSO. À EXPECTATIVA TOM...
uma das mais conhecidas figuras de ATIRAR A PRIMEIRA GERAL DA NAÇÃO...?
PEDRA?
linguagem para
a) condenar a prática de exercícios físicos.
b) valorizar aspectos da vida moderna.
c) caracterizar o diálogo entre gerações.
d) criticar a falta de perspectiva do pai. Disponível em:exercicios.mundoeducacao.bol.uol.com.br/exercicios-
gramatica/exercicios-sobre-figuras-linguagem.htm
07. A figura de linguagem que ocorre no trecho 10. O discurso de uma das personagens da tirinha
"A voz áspera daquele cantor nos fazia ter sofreu algumas alterações, provocadas pelo
vontade de morrer" ocorre em qual outro uso de uma figura de linguagem muito
destes? empregada no discurso político. Laerte
a) Aquela melodia era música nos meus construiu o efeito de humor baseado em:
ouvidos. a) hipérboles
b) Sentia o cheiro bom das flores. b) metáforas
c) Cada vez que ela chegava perto, sentia o c) personificações
cheiro doce daquele perfume horrível. d) eufemismos
d) Todos podiam ver como ela era parecida 11. Nas alternativas a seguir, todas as expressões
com sua vizinha. destacadas são prosopopeias, EXCETO em:
08. Identifique a figura de linguagem no a) “Os sinos chamam para o amo”. (Mário Quintana)
fragmento a seguir: b) “A natureza me assusta. /Com seus matos
Moça linda, bem tratada,
Três séculos de família, sombrios e suas águas”. (Ferreira Gullar)
Burra como uma porta: c) “Choram as rosas/seu perfume agora se
Um amor! transforma em lágrimas/ e eu me sinto tão
Mário de Andrade perdido/ choram as rosas”. (Alfredo Matheus)
a) Metonímia d) “Os carinhos de Godofredo não tinham mais
b) Ironia gosto dos primeiros tempos”. (Autran Machado)
c) Eufemismo
d) Metáfora

22
12. Nos trechos: na pele, flor
que se dissolve na boca. Mas este açúcar
"...nem um dos autores nacionais ou não foi feito por mim.
nacionalizados de oitenta pra lá faltava nas Este açúcar veio
estantes do major". da mercearia da esquina e tampouco o fez o Oliveira,
"...o essencial é achar-se as palavras que o violão dono da mercearia.
pede e deseja". Este açúcar veio
Encontramos, respectivamente, as seguintes de uma usina de açúcar em Pernambuco
figuras de linguagem: ou no Estado do Rio
a) prosopopeia e hipérbole; e tampouco o fez o dono da usina.
b) hipérbole e metonímia; Este açúcar era cana
c) metonímia e eufemismo; e veio dos canaviais extensos
d) metonímia e prosopopeia. que não nascem por acaso
13. Em qual das opções há erro de identificação no regaço do vale.
das figuras? (…)
Em usinas escuras,
a) "Um dia hei de ir embora / Adormecer no homens de vida amarga
derradeiro sono." (eufemismo) e dura
b) "A neblina, roçando o chão, cicia, em prece. produziram este açúcar
(prosopopeia) branco e puro
c) “Ao olhar para o Universo, o homem é com que adoço meu café esta manhã em Ipanema.
nada. Ao olhar para o Universo, o homem é Ferreira Gullar. Toda Poesia. Rio
tudo”. ( metonímia) de Janeiro: Civilização Brasileira, 1980,p. 227-8

d) "Oh sonora audição colorida do aroma." 16. A antítese que configura uma imagem da
(sinestesia) divisão social do trabalho na sociedade
14. Relacione as colunas e depois assinale a brasileira é expressa poeticamente na
alternativa correta: oposição entre a doçura do branco açúcar e
(1) Metáfora a) o trabalho do dono da mercearia de onde veio
(2) Metonímia o açúcar.
(3) Catacrese b) o beijo de moça, a água na pele e a flor que
(4) Comparação se dissolve na boca.
(5) Prosopopeia c) o trabalho do dono do engenho em
( ) Gosto de ouvir Titãs. Pernambuco, onde se produz o açúcar.
( ) A doçura do teu olhar é minha vida. d) o trabalho dos homens de vida amarga em
( ) O rio engasgou num barraco. usinas escuras.
( ) Usarei no tempero um dente de alho.
( ) Você é venenosa como uma cobra. LEIA O TEXTO PARA RESPONDER À
a) 2 1 5 3 4 QUESTÃO 17.
b) 1 2 5 3 4
c) 4 2 1 5 3
d) 5 2 1 3 4
15. Leia este trecho do poema “Outro noturno”, de
Murilo Mendes.
“A Baía de Guanabara, diferente das outras baías, é
camarada, /recebe na sala de visitas todos os
navios do mundo /e não fecha a cara. /tudo perde o
equilíbrio desta noite. /as estrelas não são mais
constelações célebres, /são lamparinas com ares
domingueiros.”
Neles, é possível identificar:
a) Metonímias.
b) Hipérboles.
c) Catacreses.
d) Prosopopeia.
QUESTÕES EXTRAS
O açúcar 17. A figura de linguagem presente no texto
O branco açúcar que adoçará meu café
acima é
nesta manhã de Ipanema
não foi produzido por mim a) Catacrese
nem surgiu dentro do açucareiro por milagre. b) Hipérbole
Vejo-o puro c) Eufemismo
e afável ao paladar d) Personificação
como beijo de moça, água

23
LEIA O TEXTO PARA RESPONDER À QUESTÃO Flores enfeitando
18. Os caminhos, os vestidos
Os destinos e essa canção
A Felicidade
Tem um verdadeiro amor
Tristeza não tem fim felicidade sim.
Para quando você for
A felicidade é como a pluma
que o vento vai levando pelo ar, 19. (IFPI 2020.1) No verso “Vê o horizonte deitar
voa tão leve, mas tem a vida breve no chão” identifica-se a seguinte figura de
precisa que haja vento sem parar. linguagem:
A felicidade do pobre
parece a grande ilusão do carnaval
a) Prosopopeia
a gente trabalha o ano inteiro
por um momento de sonho
b) Metonímia
pra fazer a fantasia c) Antítese
de rei ou de pirata ou jardineira d) Pleonasmo
pra tudo se acabar na quarta-feira. e) Eufemismo
A felicidade é como a gota de orvalho Com base na canção seguinte, responda à
numa pétala de flor, questão 20.
brilha tranquila
depois de leve oscila Vida No Campo
e cai como uma lágrima de amor. Pena Branca e Xavantinho

A minha felicidade O galo cantou, é de manhã


está sonhando nos olhos A barra do dia dourada vem surgindo
da minha namorada Clareou, a passarada acorda fazendo festa
É como esta noite, passando, E a natureza sorrindo
passando em busca da madrugada
Fale baixo por favor A vida no campo é fruta madura
pra que ela acorde Amizade é coisa pura, é mel no coração
alegre com o dia Gado no curral, cuscuz com leite
oferecendo beijos de amor. Café com queijo, eu gosto de um requeijão
Vou lhe falar
MORAES, Vinicius e JOBIM, Tom. As mais belas
serestas brasileiras. 9ª ed. Belo Horizonte: Barvalle Não troco essa vida por nada desse mundo
Indústria Gráfica Ltda, 1989. Não saio desse lugar
18. Nas duas primeiras estrofes, há uma tentativa de
se definir a felicidade, para isso o eu lírico vale- Quando é meio-dia
se de A cigarra enche o mundo de som na maior alegria
a) comparações.
b) sinestesias. Anoiteceu
c) metonímias. A prosa do compadre: O bezerro
Foi a onça quem comeu
d) hipérboles.
20. (IFPI 2020.1) O verso “E a natureza sorrindo”
A cantora Marisa gravou a canção “Vilarejo”,
ilustra a seguinte figura de linguagem:
composição sua com Arnaldo Antunes, Carlinhos
a) prosopopeia
Brawn e Pedro Baby. Leia-a a seguir.
b) metonímia
Há um vilarejo ali c) antítese
Onde areja um vento bom d) sinestesia
Na varanda quem descansa e) onomatopeia
Vê o horizonte deitar no chão
Pra acalmar o coração 21. (IFPI 2020.1) Na frase “A Lei Maria da Penha
Lá o mundo tem razão é uma Ferrari, mas nossas instituições estão
Terra de heróis, lares de mãe cheias de buracos e lombadas”, encontra-se a
Paraíso se mudou para lá predominância da seguinte figura de linguagem:
Por cima das casas cal
Frutas em qualquer quintal a) metonímia
Peitos fartos, filhos fortes b) prosopopeia
Sonhos semeando o mundo real c) catacrese
Toda a gente cabe lá Palestina, d) metáfora
Shangri-lá e) gradação
Vem andar e voa
Vem andar e voa
Vem andar e voa
Lá o tempo espera
Lá é primavera
Portas e janelas ficam sempre abertas
Pra sorte entrar
Em todas as mesas pão
24
FONÉTICA E FONOLOGIA Ex.: Paraguai, arguiu.
Atenção:
1- Sílaba tônica- A sílaba proferida com mais
Não se esqueça que só as vogais /i/ e /u/ podem
intensidade que as outras.
funcionar como semivogais. Quando
Exemplos:
semivogais, serão representadas por /y/ e /w/,
cajá, caderno, lâmpada
respectivamente.
2- Sílaba subtônica- É sílaba que na palavra
Encontros consonantais
primitiva era a sílaba tônica.
Quando existe uma sequência de duas ou mais
Exemplos:
consoantes em uma mesma palavra, denominamos
TERRA/SÓ
essa sequência de encontro consonantal.
terrinha, sozinho
O encontro pode ocorrer:
3- Sílaba átona- As sílabas que não são tônicas
- na mesma sílaba: cla-ri-da-de, fri-tu-ra, am-plo.
nem subtônicas chamam-se átonas.
- em sílabas diferentes: af-ta, com-pul-só-rio.
Podem ser pretônicas (antes da tônica) ou
Prosódia
postônicas (depois da tônica).
A prosódia está relacionada com a correta
Exemplos:
acentuação das palavras, tomando como padrão a
barata (átona pretônica, tônica, átona postônica)
língua considerada culta.
máquina (tônica, átona postônica, átona postônica)
Abaixo estão relacionados alguns exemplos de
Atenção:
vocábulos que frequentemente geram dúvidas
Não confunda acento tônico com acento
quanto à prosódia:
gráfico. O acento tônico está relacionado com
1) oxítonas:
intensidade de som e existe em todas as
cateter, Cister, condor, hangar, mister, negus,
palavras com duas ou mais sílabas. O acento
Nobel, novel, recém, refém, ruim, sutil, ureter.
gráfico existirá em apenas algumas palavras e
2) paroxítonas:
será usado de acordo com regras de
avaro, avito, barbárie, caracteres, cartomancia,
acentuação.
ciclope, erudito, ibero, gratuito, ônix, poliglota,
Os fonemas da Língua Portuguesa classificam-
pudico, rubrica, tulipa.
se em vogais, semivogais e consoantes.
3) proparoxítonas:
Vogais: são fonemas pronunciados sem obstáculo
aeródromo, alcoólatra, álibi, âmago, antídoto,
à passagem de ar, chegando livremente ao exterior.
elétrodo, lêvedo, protótipo, quadrúmano, vermífugo,
Exemplos: pato, bota.
zéfiro.
Semivogais: são os fonemas que se juntam a uma
Há algumas palavras cujo acento prosódico é
vogal, formando com esta uma só sílaba.
incerto, oscilante, mesmo na língua culta.
Exemplos: couro, baile.
Exemplos:
Observe que só os fonemas /i/ e /u/ átonos
acrobata e acróbata / crisântemo e crisantemo/
funcionam como semivogais. Para que não sejam
Oceânia e Oceania/ réptil e reptil/ xerox e xérox e
confundidos com as vogais i e u serão
outras.
representados por [y] e [w] e chamados,
Outras assumem significados diferentes, de acordo
respectivamente, de iode e vau.
a acentuação:
Consoantes: são fonemas produzidos mediante a
Exemplos:
resistência que os órgãos bucais (língua, dentes,
valido/ válido
lábios) opõem à passagem de ar.
Vivido /Vívido
Exemplos: caderno, lâmpada.
Sílaba é a unidade ou grupo de fonemas emitidos
Dicas:
num só impulso da voz.
Em nossa língua, a vogal é o elemento básico,
Divisão silábica
suficiente e indispensável para a formação da
A fala é o primeiro e mais importante recurso usado
sílaba. Você encontrará sílabas constituídas só
para a divisão silábica na escrita.
de vogais, mas nunca formadas somente com
Regra geral:
consoantes.
Toda sílaba, obrigatoriamente, possui uma vogal.
Exemplos: viúva, abelha.
Regras práticas:
Há três tipos de encontros vocálicos: ditongo,
Não se separam ditongos e tritongos.
hiato e tritongo.
Exemplos: mau, averiguei.
Ditongo: é a junção de uma vogal + uma semivogal Separam-se as letras que representam os hiatos.
(ditongo decrescente), ou vice-versa (ditongo Exemplos: sa-í-da, vo-o...
crescente), na mesma sílaba. Separam-se somente os dígrafos rr, ss, sc, sç, xc.
Ex.: noite (ditongo decrescente), quase (ditongo Exemplos: pas-se-a-ta, car-ro, ex-ce-to...
crescente). Separam-se os encontros consonantais
Hiato: é a junção de duas vogais pronunciadas pronunciados separadamente.
separadamente, formando sílabas distintas. Exemplo: car-ta.
Ex.: saída, coelho Os elementos mórficos das palavras (prefixos,
Tritongo: é a junção de semivogal + vogal + radicais, sufixos), quando incorporados à palavra,
semivogal, formando uma só sílaba. obedecem às regras gerais.
25
Exemplos:de-sa-ten-to, bi-sa-vô, tran-sa-tlân-ti- fixo = 4 letras
co... /f/ /i/ /k/ /s/ /o/ = 5 fonemas
Consoante não seguida de vogal permanece na
Dígrafos
sílaba anterior. Quando isso ocorrer em início de
Além das letras acima, há ainda os dígrafos.
palavra, a consoante será anexa à sílaba seguinte.
Exemplo:
Exemplos: ad-je-ti-vo, tungs-tê-nio, psi-có-lo-go,
ch chuva /x/ /u/ /v/ /a/
gno-mo...
nh arranhar /a/ /rr/ /a/ /nh/ /a/ /r/
FONEMA E LETRA qu quindim /k/ /ĩ/ /d/ /ĩ/
Fonema e Letra representam respetivamente sons rr aborrecer /a/ /b/ /o/ /rr/ /e/ /c/ /e/ /r/
(fala) e sinais gráficos (escrita). sc nascer /n/ /a/ /c/ /e/ /r/
Os fonemas são as unidades sonoras que
compõem o discurso ou a fala e são representados
entre barras oblíquas.
As letras, por sua vez, são os sinais gráficos que
tornam possível a escrita. Juntas de forma
ordenada, as letras constituem o alfabeto.
Exemplos:
coçar = 5 letras
/k/ /o/ /s/ /a/ /r/ = 5 fonemas
máximo = 6 letras
/m/ /á/ /s/ /i/ /m/ /o/ = 6 fonemas
acesso = 6 letras
/a/ /c/ /e/ /s/ /o/ = 5 fonemas
chute = 5 letras
/x/ /u/ /t/ /e/ = 4 fonemas
Diferença entre Fonema e Letra
Muito embora o número de fonemas e letras
coincidam em muitas palavras, nem sempre essa
equivalência existe.
Letra G (fonemas /g/ e /j/).
Exemplos:
gole = 4 letras
/g/ /o/ /l/ /e/ = 4 fonemas
singelo = 7 letras
/s/ /ĩ/ /j/ /e/ /l/ /o/ = 6 fonemas
Letra H.
No início de palavras, a letra H não é fonema.
Exemplos:
harpa = 5 letras
/a/ /r/ /p/ /a/ = 4 fonemas
hoje = 4 letras
/o/ /j/ /e/ = 3 fonemas
Letras M e N
Quando tem função de nasalização, as letras M e N
não são fonemas.
Exemplos:
campo = 5 letras
/k/ /ã/ /p/ /o/ = 4 fonemas
atento = 6 letras
/a/ /t/ /ẽ/ /t/ /o/ = 5 fonemas
navio = 5 letras
/n/ /a/ /v/ /i/ /o/ = 5 fonemas
Letra X (fonemas /s/, /z/, /ks/).
Exemplos:
sexto = 5 letras
/s/ /e/ s/ /t/ /o/ = 5 fonemas
exalar = 6 letras
/e/ /z/ /a/ /l/ /a/ /r/ = 6 fonemas

26
PROFESSOR RESPONDE c) é de se esperar que o futebol faça-nos
esquecer dos problemas do Brasil.
LEIA O TEXTO PARA RESPONDER ÀS d) jogar futebol, ou assisti-lo, compromete as
QUESTÕES DE 01 A 05. obrigações serias dos brasileiros.
FUTEBOL - O ÓPIO DO POVO e) se não fosse o futebol seriamos mais evoluídos.
José Rodrigues Alves Bomfim 02. Segundo o texto, o principal problema envolvendo
o evento futebolístico é:
O futebol lamentavelmente, se tornou o ópio do
povo. É a "política do pão e circo" para poder esconder
a) que os brasileiros vão se endividar mais.
os profundos problemas que a sociedade enfrenta, b) que os brasileiros vão parar de pensar em
esquecendo por um momento os defeitos sociais e coisas importantes.
econômicos durante esse período. c) não temos uma boa seleção.
A Copa do Mundo não deixa de ser um importante d) o presidente vai parar pra assistir as partidas
evento futebolístico, entretanto será que um país como de futebol.
o Brasil, com tantos problemas sociais, com tantos e) que assim como o ópio, o futebol pode viciar.
desníveis socioeconômicos, com uma economia tão
frágil, pode se dar o luxo de parar em dias em que há 03. Releia o título do texto: O ópio do povo. Na
partidas da seleção brasileira? Será que nós palavra em destaque há um ditongo crescente.
brasileiros, que acompanhamos aos jogos da Copa, O mesmo caso ocorre em:
temos o direito de esquecer que existe pobreza no a) defeitos.
Brasil durante os jogos de futebol? b) quatro.
O evento acompanhado por bilhões de pessoas em c) pessoas.
todo o mundo, consegue uma façanha que nenhuma d) período.
propaganda de governo consegue fazer: o
e) hospitais.
mascaramento dos problemas sociais e econômicos.
Infelizmente é isso o que ocorre a cada quatro 04. No trecho É a "política do pão e circo", a palavra
anos. Parece que todos os problemas sociais acabam, em destaque tem como sílaba tônica a
a miséria, a segurança, a roubalheira política...etc. É antepenúltima. Trata-se, portanto, de uma
por isso que o governo comemora, quem sabe, uma proparoxítona. A alternativa em que todas as
melhora nas pesquisas. Além disso, transformar dias
palavras, quanto à sílaba tônica, estão na
de jogos da seleção em feriados é certo? Só falta isso
acontecer. Tem também o recesso, concedido a
mesma classificação é:
políticos durante as partidas. No dia do primeiro jogo a) econômico, desníveis, futebolístico.
da seleção, a Câmara e o Senado não terão b) infelizmente, econômicos, pobreza.
expediente e será interrompido por mais tempo. O c) Câmara, próprio, momentânea.
próprio Presidente vai parar para acompanhar o d) irônica, econômico, Câmara.
evento. E nesses dias, os hospitais, as delegacias, os e) período, futebolístico, pobreza.
bancos, supermercados, farmácias, as conduções, as
escolas. Tudo funciona precariamente. Do povo 05. Releia o seguinte o trecho:
faminto, infelizmente, nada pode ser exigido, pois o “Pode surgir sim, alguma notícia sobre o quebra-
Mundial é uma das poucas alegrias, apesar de falsa e quebra no Congresso Nacional, em Brasília [...].
momentânea.
A televisão, que é o mais importante meio Ao manter a estrutura lexical da palavra, modificando
informativo do país (ao menos é o mais visto), somente a acentuação gráfica, o vocábulo ficaria,
manipula as notícias ainda mais, causando um "efeito pois, assim: noticia. Com isso, analise as seguintes
Copa", em que as pessoas priorizam os jogos de afirmações, em seguida assinale a alternativa correta.
futebol. Pode surgir sim, alguma notícia sobre o I - Houve uma mudança de classe de palavra (notícia
quebra-quebra no Congresso Nacional, em Brasília, = substantivo; noticia = verbo).
causado pelo MLST (Movimento Libertação dos Sem II - Não alteraria o sentido da informação se o trecho
Terra), que logo é sufocada por alguma notícia da fosse modificado por: Pode surgir sim, algo que
concentração da seleção brasileira. noticia o quebra-quebra no Congresso Nacional, em
O resultado da Seleção no Mundial pode influir nas Brasília [...].
pessoas. Se houver um resultado negativo, o país III - Quanto à classificação da sílaba tônica,
volta à realidade mais cedo. Se, pelo contrário, vencer ambas são paroxítonas.
até o final, o resultado pode ser prolongado por mais IV - Quanto ao encontro vocálico, nas duas
tempo. A situação é, no mínimo, irônica, o povo está palavras ocorreu um caso de hiato.
com fome, mas ao menos está feliz. Não há V - Quanto ao número de sílabas, ambas são
problemas se as pessoas torcem para que o time trissílabas.
jogue bem e para que ganhe. A felicidade reinará no a) Todas estão corretas.
território brasileiro, mas não pode substituir condições b) Apenas a IV está incorreta.
dignas de vida. A prioridade deve ser tais condições.
c) Apenas I, II e III estão corretas.
Publicação: www.paralerepensar.com.br 20/06/2006
d) Apenas I e II estão corretas.
01. Segundo o autor: e) Apenas a V está incorreta.
a) o futebol pode ser visto como um atraso
monumental para os brasileiros.
b) o futebol ajuda no desenvolvimento brasileiro.

27
ALUNO RESPONDE 03. No texto, o uso de dois pontos, no segundo e
no quinto parágrafos, contribui para
LEIA O TEXTO PARA RESPONDER ÀS a) causar uma ruptura na sequência das ideias.
QUESTÕES DE 01 A 09. b) vincular elementos promotores da relação de
Apaixonou-se pela voz saída do rádio
causa e consequência.
A costureira e dona de casa Maria Delfino Todaro c) apresentar informações caracterizadoras das
viveu boa parte de sua vida com o rádio ligado, personagens.
ouvindo notícias e, antigamente, novelas. Aos 28, ela d) introduzir textualmente a voz do locutor.
escutou pelo aparelho a voz do futuro marido. e) informar que ainda há a algo a dizer
Na época, ela se apaixonou por uma personagem 04. Considerando-se o contexto, o trecho
de uma novela da Rádio São Paulo: um pianista que
sublinhado na frase “Aos 28, ela escutou pelo
tocava Chopin nos momentos de melancolia. Um dia,
foi à emissora conhecer o dono da voz que tanto a
aparelho a voz do futuro marido” pode ser
encantava. parafraseado por:
O ator, que também dirigia novelas e era oficial de a) “a voz daquele com quem ela vai se casar.”
Justiça quando não estava no ar, chamava-se Alfredo b) “a voz daquele com quem viria a se casar.”
Todaro. Apaixonaram-se ali, e apesar da diferença de c) “a voz do homem com quem provavelmente
20 anos entre eles, começaram um relacionamento se casaria.”
que durou 52 anos. d) “a voz da pessoa a quem está prometida para
Filha de um administrador de uma fazenda, Maria se casar.”
nasceu em Indaiatuba (SP). Com 20 e poucos anos, e) “a voz daquele com jamais casaria.”
veio a São Paulo atrás de um emprego.
Foi costureira, apesar de o marido não gostar que 05. Ainda quanto ao trecho ““Aos 28, ela escutou
ela trabalhasse. Na cozinha, era excelente, como pelo aparelho a voz do futuro marido”, pode-se
lembra a filha, Cleo: o macarrão e o arroz com afirmar que na expressão sublinhada há
lentilhas que fazia eram um sucesso em casa. a) 5 palavras oxítonas.
Muito séria, era chamada de “general” pelo marido. b) 3 oxítonas e duas paroxítonas.
A filha conta que a mãe, extremamente ativa, andava c) 2 palavras paroxítonas.
rapidinho e não se sentava nem para tomar café – d) 2 palavras proparoxítonas.
fazia-o em pé.
e) 1 oxítona e duas paroxítonas.
Maria nunca deixou de acompanhar a carreira do
marido. A filha do casal chegou até a virar nome de 06. Releia o título: Apaixonou-se pela voz saída do
uma personagem numa novela. rádio. As palavras destacadas têm como
Em 1999, Alfredo morreu. No ano seguinte, Maria encontro vocálico, respectivamente, hiato e
teve o primeiro derrame. Ficou dez anos na cadeira de ditongo crescente. O mesmo ocorre
rodas. Na segunda, não resistiu a outro derrame. respectivamente na alternativa:
Tinha 92 anos. Deixa filha, duas netas e bisneto. a) pianista e costureira.
A missa de sétimo dia será hoje, às 19h, na igreja
b) dia e fazia.
São Domingos, em São Paulo.
BERTONI, Estêvão. Folha de S. Paulo.
c) oficial e séria.
d) cadeira e séria.
01. O texto integra o obituário do jornal Folha de e) cadeira e mãe.
S. Paulo, uma seção referente a notícias 07. Há encontro de consoantes em sílabas
fúnebres. Esse texto, no entanto, distancia-se diferentes. A sequência na qual em todas as
dos obituários padrões e se aproxima do palavras ocorre esse caso é:
gênero a) costureira, ouvindo, escutou.
a) manchete. b) pianista, administrador, excelente.
b) artigo de opinião. c) costureira, extremamente, bisneto.
c) crônica. d) igreja, primeiro, nasceu.
d) romance. e) Alfredo, Igreja, administrador.
e) notícia. 08. Assinale a sequência de palavras cujas sílabas
02. Que aspecto relativo ao conteúdo do texto estão separadas corretamente.
favorece sua inclusão na seção de obituários? a) a-pa-i-xo-nou, a-dmi-nis-tra-dor, me-lan-co-lia.
a) A descrição do tempo em que Maria esteve b) In-dai-a-tu-ba, ex-ce-len-te, ad-mi-nis-tra-dor,
casada com Alfredo. c) nas-ceu, tra-ba-lha-sse, fa-zi-a.
b) As informações a respeito da missa de d) di-ri-gia, sé-ria, Ma-ria.
sétimo dia, no último parágrafo. e) a-pai-xo-nou, me-lan-co-li-a, Ma-ria.
c) A análise das circunstâncias em que Maria 09. Assinale a alternativa em que todas as
conheceu Alfredo. palavras, quanto à sílaba tônica, são oxítonas:
d) A notícia do primeiro derrame sofrido por a) conhecer, também, ali.
Maria depois da morte de Alfredo. b) administrador, casal, Maria.
e) Um fato digno de documentário ou c) oficial, rapidinho, acompanhar.
reportagem jornalística. d) será, futuro, Maria.
e) rapidinho, conhecer, oficial.
28
OBSERVE A CHARGE PARA A QUESTÃO 10 e 11. c) sugerem que há um problema social a ser
resolvido: o trabalho infantil.
d) revelam que as famílias ricas não permitem
às pobres o acesso a programas infantis.
e) conseguem revelar um só problema social: o
trabalho infantil.
13. Quanto aos vocábulos “trabalho” e
“trabalhando” podemos afirmar que
a) possuem a mesma quantidade de sílabas.
b) ambas são oxítonas.
c) possuem ditongo.
d) ambas possuem dígrafo.
10. Assinale a alternativa que não retrata uma e) ambas são proparoxítonas.
interpretação correta sobre os quadrinhos:
a) Analisando o conteúdo da fala das duas 14. Em “trabalhando” temos presente
personagens, pode-se afirmar que o a) 11 letras e 10 fonemas.
crescimento dos cursos superiores implica b) 11 letras e 9 fonemas.
consequências práticas diferentes para cada c) 11 letras e 11 fonemas.
uma delas. d) 9 letras e 11 fonemas.
b) Os detalhes expostos nos cenários em que as e) 11 letras e 8 fonemas.
personagens se inserem reforçam o que está 15. Em qual das palavras abaixo há mais letras do
afirmado nos respectivos textos verbais. que fonemas?
c) No quadrinho da direita, está pressuposto que a) achado
a necessidade de trabalhar da personagem b) táxi
reflete uma situação de desagregação familiar. c) cadeira
d) Embora seja evidente a diferença entre a d) surpresa
classe social das duas personagens, a variante e) redigir
linguística utilizada por elas não reforça essa 16. Nas palavras ALMA, PINTO e PORQUE temos,
oposição. respectivamente:
e) Ao iniciar os dois textos da mesma forma, o a) 4 fonemas - 5 fonemas - 6 fonemas.
autor dos quadrinhos deixa entrever certo b) 5 fonemas - 5 fonemas - 5 fonemas.
envolvimento dos dois jovens com a família. c) 4 fonemas - 4 fonemas - 5 fonemas.
11. O termo sublinhado no segundo balão é exemplo d) 5 fonemas - 4 fonemas - 6 fonemas.
de palavra e) 4 fonemas - 5 fonemas - 5 fonemas.
a) com 3 sílabas e oxítona. 17. A alternativa que apresenta uma incorreção é:
b) com 2 sílabas e proparoxítona. a) O fonema está diretamente ligado ao som da
c) com 3 sílabas e paroxítona. fala.
d) com 4 sílabas e oxítona. b) As letras são representações gráficas dos
e) com o “LH” em sílabas diferentes. fonemas.
OBSERVE A IMAGEM PARA RESPONDER À QUESTÃO 12. c) A palavra "tosse" possui quatro fonemas.
d) Uma única letra pode representar fonemas
diferentes.
e) A letra "h" sempre representa um fonema.
18. Identifique o vocábulo cuja pronúncia do “x” é
equivalente à pronúncia dessa letra em
“proparoxítonas”
a) léxico
b) máximo
c) exigem
d) exóticas
12. Os autores de charges costumam atingir seu e) esdrúxulas
público por meio da ironia, com a finalidade de 19. Analise as afirmações acerca das palavras
fazer importantes denúncias e desencadear proparoxítonas e marque a alternativa
reflexões. Assim ocorreu durante a charge a CORRETA.
seguir, na qual, referindo-se a um famoso I - Toda palavra proparoxítona deve ser
programa de televisão, as falas: acentuada.
a) visam a desencadear uma desconfiança em II - Proparoxítonas são palavras cuja sílaba tônica
relação ao programa em questão. é a antepenúltima.
b) fazem críticas destinadas a fins diferentes, III - Todas as palavras com mais de três sílabas
embora oriundas de um só fato. são proparoxítonas.
29
a) Somente I está correta. até no coração das coisas menos percebidas
b) Somente I e II estão corretas. (a começar pelo seu interior)
c) Somente I e III estão corretas. novo, espontâneo, que de tão perfeito nem se
d) Somente II e III estão corretas. nota,
mas com ele se come, se passeia,
e) Todas estão corretas.
se ama, se compreende, se trabalha,
20. Aponte a alternativa em que o fenômeno você não precisa beber champanha ou qualquer
fonético grifado está associado outra birita,
INCORRETAMENTE. não precisa expedir nem receber mensagens
a) Coqueiro – ditongo (planta recebe mensagens?
b) Quereres - dígrafo passa telegramas?)
c) Queria– hiato (...)
d) Querendo – ditongo (Carlos Drummond de Andrade)
e) adequação – ditongo 24. Releia a expressão “Para você ganhar
21. Assinale a alternativa que contém, belíssimo Ano Novo”. Os termos destacados
respectivamente, os números de fonemas das possuem, respectivamente:
palavras “financiamento”, “minha” e a) 5 e 8 fonemas
“combustível”. b) 5 e 9 fonemas
a) 13, 4, 11 b) 10, 5, 10 c) 6 e 8 fonemas
c) 12, 4, 11 d) 11, 5, 11 d) 6 e 9 fonemas
e) 11, 4, 10 e) 6 e 10 fonemas
Leia o texto abaixo para responder às 22 e 23 25. Em “Ah quanto tempo passou!”, as palavras
O mínimo do máximo grifadas apresentam, respectivamente:
Paulo Leminski a) ditongo – dígrafo
Tempo lento, b) encontro consonantal – dígrafo
espaço rápido, c) dígrafo – encontro consonantal
quanto mais penso, d) dígrafo – dígrafo
menos capto.
e) encontro consonantal – encontro consonantal
Se não pego isso
que me passa no íntimo, 26. Em “Ah quanto tempo passou!”, as palavras
importa muito? grifadas apresentam, respectivamente:
Rapto o ritmo. a) ditongo - dígrafo
Espaçotempo ávido, b) encontro consonantal - dígrafo
Lento espaçodentro,
c) encontro consonantal – encontro consonantal
quando me aproximo,
simplesmente me desfaço, d) dígrafo - dígrafo
apenas o mínimo e) dígrafo - encontro consonantal
em matéria de máximo Leia o texto abaixo:
22. Assinale a alternativa em que as duas palavras Equipe do IFPI desenvolve aplicativo para
NÃO possuem o mesmo número de fonemas. serviços de beleza
a) Quando; íntimo
O Núcleo de Desenvolvimento de Software do
b) Capto; rapto Campus Teresina Central do IFPI desenvolveu o
c) Muito, ritmo aplicativo Luki, que visa conectar centros de beleza
d) Tempo; lento; aos seus clientes.
e) Espaçotempo; espaçodentro A plataforma foi criada pelos estudantes João
23. Identifique o vocábulo no qual a letra “x” é Neto e Gildásio Filho, do curso de Análise e
pronunciada da mesma maneira que na palavra Desenvolvimento de Sistemas, orientados pelo
“máximo”. professor Rogério Silva, a partir de observações
a) mexilhão sobre fluxo de clientes, promoções e divulgações dos
b) exame salões de beleza da cidade, além de pesquisas com o
público.
c) táxi
d) auxílio (...)
e) reflexão Disponível em: www.ifpi.edu.br/teresinacentral/noticias/
professor-e-alunos-do-ifpi-desenvolvem-aplicativo
Receita de Ano Novo
Para você ganhar belíssimo Ano Novo 31. (IFPI 2018.1- Adaptada) A palavra “conectar”,
cor do arco-íris, ou da cor da sua paz, no primeiro parágrafo, tem a mesma quantidade
Ano Novo sem comparação com todo o tempo de fonemas da palavra:
já vivido a) público
(mal vivido talvez ou sem sentido) b) beleza
para você ganhar um ano
c) usuário
não apenas pintado de novo, remendado às carreiras,
mas novo nas sementinhas do vir-a-ser;
d) automatizar
novo e) serviços

30
MORFOLOGIA-PROCESSO DE FORMAÇÃO Ex. beber, correndo, partido
DE PALAVRAS Neologismo
Beijo pouco, falo menos ainda.
As palavras são constituídas de morfemas. Mas invento palavras
São eles: Que traduzem a ternura mais funda
Radical E mais cotidiana.
É o elemento comum de palavras cognatas, também Inventei, por exemplo, o verbo teadorar.
chamadas de palavras da mesma família. É Intransitivo:
responsável pelo significado básico da palavra. Teadoro, Teodora.
(BANDEIRA, Manuel. Estrela da vida inteira.
Ex.: terra, terreno, terreiro, terrinha, enterrar, Rio de Janeiro: José Olympio, 1970)
terrestre... Os principais processos de formação são:
Atenção: Derivação
Às vezes, ele sofre pequenas alterações. Processo de formar palavras no qual a nova
Ex.: dormir- durmo; querer- quis palavra é derivada de outra, chamada de primitiva.
As palavras que possuem mais de um radical são Os processos de derivação são:
chamadas de compostas. Derivação Prefixal
Ex.: passatempo A derivação prefixal é um processo de formar
Afixos palavras no qual um prefixo ou mais são
São partículas que se anexam ao radical para formar acrescentados à palavra primitiva.
outras palavras. Existem dois tipos de afixos: Ex.: re/com/por (dois prefixos), desfazer, impaciente.
Prefixos: colocados antes do radical. Derivação Sufixal
Ex.: desleal, ilegal A derivação sufixal é um processo de formar
Sufixos: colocados depois do radical. palavras no qual um sufixo ou mais são
Ex.: folhagem, legalmente acrescentados à palavra primitiva.
Infixos Ex.: realmente, folhagem.
São vogais ou consoantes de ligação que entram na Derivação Prefixal e Sufixal
formação das palavras para facilitar a pronúncia. A derivação prefixal e sufixal existe quando um
Existem em algumas palavras por necessidade prefixo e um sufixo são acrescentados à palavra
fonética. Os infixos não são significativos, portanto, primitiva de forma independente, ou seja, mesmo
não sendo considerados morfemas. sem a presença de um dos afixos a palavra
Ex.: café-cafeteira, capim-capinzal, gás-gasômetro continua tendo significado.
Vogal Temática Ex.: deslealmente (des- prefixo e -mente sufixo).
Vogal Temática (VT) se junta ao radical para receber Você pode observar que os dois afixos são
outros elementos. Fica entre dois morfemas. Existe independentes: existem as palavras, desleal e
vogal temática em verbos e nomes. lealmente.
Ex.: beber, rosa, sala. Derivação Parassintética
Nos verbos, a VT indica a conjugação a que A derivação parassintética ocorre quando um
pertencem (1ª, 2ª ou 3ª). prefixo e um sufixo são acrescentados à palavra
Ex.: partir- verbo de 3ª conjugação primitiva de forma dependente, ou seja, os dois
Há formas verbais e nomes sem VT. afixos não podem se separar, devendo ser usados
Ex.: rapaz, mato (verbo) ao mesmo tempo, pois sem um deles a palavra não
Dicas: se reveste de nenhum significado.
A VT não marca nenhuma flexão, portanto é Ex.: anoitecer (a- prefixo e -ecer - sufixo), neste
diferente de desinência. caso, não existem as palavras anoite e noitecer,
Tema pois os afixos não podem se separar.
Tema = radical + vogal temática Derivação Regressiva
Ex.: cantar = cant + a, mala = mal + a, rosa = ros+a A derivação regressiva existe quando morfemas da
Desinências palavra primitiva desaparecem.
São morfemas colocados no final das palavras para Ex.: mengo (flamengo), dança (dançar), portuga
indicar flexões verbais ou nominais. (português).
Podem ser: Derivação Imprópria
Nominais: indicam gênero e número de nomes A derivação imprópria, mudança de classe ou
(substantivos, adjetivos, pronomes, numerais). conversão ocorre quando a palavra, pertencente a
Ex.: casa – casas; gato - gata uma classe, é usada como fazendo parte de outra.
Verbais: indicam número, pessoa, tempo e modo Exemplos:
dos verbos. Existem dois tipos de desinências coelho - substantivo comum, usado como
verbais: desinências modo-temporal (DMT) e substantivo próprio - Daniel Coelho da Silva.
desinências número-pessoal (DNP). verde, geralmente usado como adjetivo - Comprei
Ex.: Nós corremos, se eles corressem (DNP); se uma camisa verde-, é usado como substantivo: O
nós corrêssemos, tu correras (DMT) verde do parque comoveu a todos.
Verbo-nominais: indicam as formas nominais dos
verbos (infinitivo, gerúndio e particípio).
31
PROFESSOR RESPONDE Monitoramento
A velocidade com que conseguem difundir bordões
LEIA O TEXTO PARA RESPONDER ÀS na mídia faz dos comentaristas e locutores uma fonte
QUESTÕES DE 01 A 04. de expressões. Há pressão para que sejam originais e
imprimam uma espécie de copyright na linguagem oral,
o que funciona como marketing próprio e fortalece a
(IFPI-2015) O FUTEBOL NAPONTADALÍNGUA
imagem da emissora. Mas até técnicos podem
A paixão pelo futebol no Brasil vem esgarçando os
disseminar termos e neologismos. Alguns têm
limites da língua portuguesa há coisa de um século. A
consciência disso, avalia Sílvio Lancellotti, crítico de
espontaneidade brasileira desencadeou um vocabulário
gastronomia e comentarista esportivo, colunista do
futebolístico rico e bem-humorado, usado até pelos que
portal R7.
odeiam o esporte – a vingança dos fanáticos contra as
– Os jogadores não criam quase nada, mas há
exceções
treinadores com idiomas próprios, como o Cláudio
A rua torna-se um dicionário desse léxico matreiro,
Coutinho, que inventou a expressão “ponto futuro”, ou
inventado por jogadores, torcedores, locutores e
Sebastião Lazaroni, que tentava falar bonito, mas era
comentaristas esportivos, pouco a pouco absorvido
quase sempre incompreensível – opina. (...)
pela língua geral. Assim, é possível ouvir frases do tipo:
– Um nó cego, entretanto, emaranha o debate: os
“pisou na bola”, “joga nas onze” sem que o vínculo com
jogadores monitoram-se tanto em suas declarações
o futebol sequer seja lembrado.
públicas que perdem a sintonia com a linguagem das
O próprio verbo “driblar” é exemplo: embora
ruas.
signifique o movimento específico do jogador com a
– Como são entrevistados após o jogo, com a
bola, seu uso fora de campo enfatiza a subversão de
adrenalina ainda a mil, sabem que não podem correr o
situações. (...)
risco de dizer exatamente o que estão sentindo. Por
Tanto carinho pelo tema origina mil e uma formas
isso, decoram discursos. E dizem coisas como “almejar
de dizer a mesmíssima coisa. A bola, por exemplo,
a vitória”. Quem mais fala “almejar” na linguagem
inspira incontáveis sinônimos: pelota, criança,
cotidiana? Ninguém. Um jogador vê um outro na TV e
perseguida, margarida, maricota, caroço, pipoca,
copia o discurso. E isso faz com que palavras e termos,
redonda, gorduchinha. (...)
como “ter atitude”, “o grupo” e “em torno do nosso
Não raro, é possível identificar a gestação lexical do
objetivo maior”, virem marcas registradas de
futebol.
futebolistas – diz Gehringer.
– As expressões são uma criação dos jogadores e da
Para além da verborragia da mídia e da empolgação no
imprensa. Romário criou, por exemplo, a expressão
campo, o brasileiro fez o futebol ter impacto em sua
“peixe” para chamar algum companheiro, mas foram os
linguagem. Com o futebol, e sem perceber, o Brasil faz
jornalistas esportivos que, influenciados pela Liga
sua língua ser tecida na alegria e na dificuldade de um
Profissional de Basquete dos Estados Unidos (NBA),
jogo cotidiano, em que o craque é o próprio povo.
inventaram o termo “assistência” para o jogador que dá
o passe para outro marcar o gol – diz Antonio Luciana Hidalgo Revista Língua (Edição
103, maio 2014. Adaptado)
Nascimento, ex-editor chefe de esporte do jornal O
Globo e atual secretário nacional de futebol e direitos 01. Analise as afirmações que seguem, com base
do torcedor, do Ministério do Esporte. na seguinte passagem do texto (2º parágrafo):
Criatividade I. Na palavra “matreiro”, o morfema -o é
Foi dessa forma que grandes nomes da locução e chamado de vogal temática, e sua função
da crônica esportiva deixaram seu legado à língua. é ligar-se ao radical, constituindo o tema .
João Saldanha, por exemplo, criou a divertida
II. “Matreiro” pertence à classe dos
expressão “macaquinho namorado de girafa” (jogador
que sobe e desce no campo, como um símio que adjetivos, visto que é uma palavra que
namorasse uma girafa e se movimentasse para cima e caracteriza o substantivo “léxico”.
para baixo, tentando beijá-la) – muitas destas, III. No nível semântico, “matreiro” pode
entretanto, saem de moda e pairam só nos arquivos significar: que foi repetido ou insistido;
imaginários de torcedores de todos os tempos. experimentado.
Nelson Rodrigues cunhou expressões tornadas tão Está(ão) correto(s):
populares quanto “sandálias da PORTUGUÊS a) I e II.
humildade” e divertiu-se lançando modismos como b) I e III.
“pose de Sarah Bernhardt em noite de estreia” (jogador c) II e III.
confiante demais). Inúmeras gírias não chegam ao
d) Apenas I.
dicionário, ou entram numa edição e saem na seguinte,
por caírem em desuso. Dos 228.500 verbetes do e) Apenas III.
Houaiss, os que possuem a rubrica “futebol” são cerca 02. A sílaba tônica está destacada em cada
de 0,22%, ou seja, 502. Não é muito, mas a força do palavra abaixo. Uma delas, porém, está
futebol na cultura fornece amplo conteúdo para destacada incorretamente. Assinale-a:
invencionices.
a) Verborragia
Para o consultor de carreiras Max Gehringer, essa é
a diferença entre a “pátria de chuteiras” e outros países
b) Esgarçando
ligados ao esporte. c) Rubrica
– No Brasil, as expressões aparecem e somem mais d) Invencionices
rapidamente. Nós somos um povo que, historicamente, e) Historicamente
adota e descarta palavras, de qualquer idioma, sem
muito pudor linguístico.

32
03. No que se refere à formação das palavras, a
palavra “incompreensível” é formada por
derivação:
a) Prefixal
b) Sufixal
c) Prefixal e sufixal
d) Imprópria
e) Regressiva
04. Das alternativas abaixo marque aquela que
possui derivação sufixal
a) Futebol
b) Vocabulário
c) Léxico Vai aí a Kombi.
d) Sinônimos
e) Específico Em breve, em nenhuma
concessionária perto de você.
LEIA O TEXTO PARA RESPONDER À
QUESTÃO 05. Todo carro merece um anúncio de lançamento.
Mas só um ícone como a Kombi merece também um
REMÉDIOS PARA EMAGRECER: A NOVA anúncio de deslançamento.
GERAÇÃO Isso mesmo, a última Kombi do mundo será
O lançamento de uma medicação para eliminar fabricada no final deste ano. E, como toda Kombi, já
os quilos extras sem tantos e feito colaterais anima vai sair sem computador de bordo, sem airbag, sem
os especialistas, ao mesmo tempo em que revive a freios ABS, sem painel de touchscreen. Mas com
questão: quando obesidade deve ser tratada na estilo retro e charme de fábrica.
farmácia? [...] O carro que fez diferença na vida de tanta gente
Saúde é vital, set. 2016, p. 50
está se aposentando, mas vai deixar muitas
05. (IFMA- 2017) Ao radical dos termos destacados lembranças. Conter a sua no site vw.com.br/kombi.
emagrecer, lançamento, revive e obesidade Vem ai, ou melhor, vai ai a Kombi. O
deslançamento menos esperado da indústria
foram acrescentados elementos mórficos, o automobilística mundial.
que originou a palavra em destaque. O
processo de formação de cada uma dessas
palavras é, respectivamente:
a) derivação parassintética – derivação sufixal – 02. Para anunciar o fim da fabricação de um
derivação prefixal – derivação sufixal. modelo de automóvel, a montadora
b) derivação prefixal e sufixal – derivação Volkswagen optou por uma campanha
regressiva – derivação prefixal – derivação publicitária inusitada: o “deslançamento” da
sufixal. Kombi. Assinale a alternativa que melhor
c) derivação prefixal – derivação regressiva – explica o processo de formação do neologismo
derivação prefixal – derivação sufixal. “deslançamento” e seus efeitos de sentido.
d) derivação parassintética – derivação sufixal – a) Associado ao prefixo “-des”, indicador de
derivação imprópria – derivação sufixal. negação, esse neologismo estabelece uma
e) derivação sufixal – derivação imprópria – referência pejorativa ao automóvel,
derivação prefixal – derivação sufixal. enunciando a obsolescência do produto.
b) No processo de criação, o termo ressalta o
valor de afetividade do veículo devido à
ALUNO RESPONDE presença do sufixo “-mento”, que expressa a
noção de resultado de uma ação.
01. Em: “Os denunciados tinham plena ciência do c) O efeito expressivo do termo resulta da dupla
cinema, do esquema criminoso e da origem possibilidade do processo de formação desse
espúria (ilegítimo) dos recursos que neologismo: composição por justaposição ou
receberam.” Isto É 2443 5/10/2016O termo derivação prefixal e sufixal.
em destaque sofreu alteração de classificação d) O tom irreverente, resultante de um texto que
morfológica. Nesse contexto, ele é parece uma propaganda às avessas, é
substantivo. O processo pelo qual esse termo enfatizado pelo uso do recurso de derivação
sofreu alteração é denominado: prefixal e sufixal do termo.
a) derivação prefixal. e) O termo apresenta um efeito irônico baseado
b) derivação parassintética. no processo de derivação imprópria que
c) derivação imprópria. sugere, nesse contexto, a ideia de um
d) derivação regressiva. automóvel que não vai deixar saudade.
e) derivação sufixal.

33
06. O processo de formação das palavras em
TRANSPLANTE DE AMOR “enterrado” e “papelada” presentes no
Gastrite é uma inflamação do estômago. Apendicite segundo quadrinho da tira é, respectivamente
é uma inflamação do apêndice. Otite é uma inflamação a) Derivação sufixa e parassintética.
dos ouvidos. Paixonite é uma inflamação do quê? Do b) Derivação parassintética e sufixal.
coração.
c) Composição e derivação.
Cada órgão do nosso corpo tem uma função vital e
precisa estar 100% em condições. Ao coração, coube a d) Aglutinação e justaposição.
função de bombear sangue para o resto do corpo, mas e) Derivação prefixal e derivação sufixal.
é nele que se depositam também nossos mais nobres
sentimentos. Qual é o órgão responsável pela saudade,
pela adoração? Quem palpita, quem sofre, quem
dispara? O próprio.
Foi pensando nisso que me ocorreu o seguinte: se
alguém está com o coração dilacerado nos dois
sentidos, biológico e emocional, e por ordens médicas
precisa de um novo, o paciente irá se curar da dor de
amor ao receber o órgão transplantado?
Façamos de conta que sim. Você entrou no hospital
com o coração em frangalhos, literalmente. Além de
apaixonado por alguém que não lhe dá a mínima, você 07. A palavra "embora" formou-se por:
está com as artérias obstruídas e os batimentos a) hibridismo
devagar quase parando. A vida se esvai, mas
b) parassíntese
localizaram um doador compatível: já para a mesa de
cirurgia. c) aglutinação
MEDEIROS, Martha. Non-Stop. Porto Alegre: L & d) justaposição
PM, 2001. p. 43
e) regressiva
03. A palavra apaixonado formou-se pelo mesmo 08. A palavra resgate é formada por derivação:
processo que: a) prefixal
a) impaciente b) Sufixal
b) infelizmente c) regressiva
c) honestamente d) parassintética
d) esfriado e) imprópria
e) melancia
04. No primeiro parágrafo do texto, as palavras:
gastrite, apendicite, e paixonite são formadas
pelo processo de derivação:
a) Prefixal.
b) Parassintética.
c) Regressiva.
d) Sufixal.
e) Derivação imprópria. 09. Pelo contexto da charge o termo
Observe as orações a seguir: “CAMELÁSTICA” é uma palavra cujo
processo formativo se deu por
I.Com o susto, o menino enrubesceu. a) Hibridismo.
II. Com o susto, o menino emudeceu. b) Derivação imprópria.
05. Sobre o processo de formação das palavras em c) Neologismo.
destaque estão incorretos todos os itens, exceto: d) Derivação sufixal.
a) São formadas por derivação prefixal e sufixal. e) Derivação regressiva.
b) São formadas por parassíntese.
c) São formadas por derivação prefixal. [...] Há neologismos problemáticos.
d) São compostas por aglutinação. O “famoso” imexível é um deles. Você deve estar
e) São compostas por justaposição. lembrado do seu nascimento: filho de Antônio Magri
no lançamento do Plano Collor. Observe que o
OBSERVE A TIRA: adjetivo imexível foi inventado graças ao prefixo i-
(negação) e ao sufixo -vel (possibilidade), que nós
encontramos, por exemplo, em imutável (que não
pode ser mudado). Sem dúvida, portanto, o maior
pecado do imexível está na “fonte criadora”. Devido à
sua “origem”, a palavra carrega consigo uma carga
pejorativa. Só é usada por brincadeira ou para fazer
piada.
DUARTE, Sérgio Nogueira. Língua viva: uma
análise simples e bem-humorada da linguagem do
brasileiro.Rio de Janeiro: Rocco, 1998. (Fragmento).

34
10. Sérgio Nogueira Duarte, em seu texto, explica 14. Assinale a alternativa que corresponda ao
a formação do neologismo “imexível”. Que elemento que contém o significado básico da
processo de formação de palavra dá origem a palavra:
esse vocábulo é a) desinência
a) derivação prefixal e sufixal. b) Consoante de ligação
b) derivação parassintética. c) tema
c) derivação sufixal. d) afixo
d) derivação prefixal. e) radical
e) composição. 15. Marque a alternativa incorreta com relação à
LEIA A TIRA E RESPONDA À QUESTÃO 11. classificação dos elementos mórficos
destacados nas palavras:
a) Planejam – M – desinência número-pessoal.
b) Vejo – O – desinência modo-temporal.
c) Bonita – A – desinência de gênero.
d) Embelezar – EM – prefixo.
e) Lealdade – DADE – prefixo.
Considere o texto seguinte para responder à
questão 16.
O mínimo do máximo
BROWNE, Dik. Hagar. Folha de S.Paulo, São Paulo, 12 jan. 1999.

11. A palavra “desaproximemos”, utilizada por Tempo lento,


Eddie Sortudo, é um neologismo formado pelo espaço rápido,
acréscimo do prefixo des-. O sentido desse quanto mais penso,
prefixo neste contexto seria menos capto.
a) deixar à solta. Se não pego isso
b) aproximar-se. que me passa no íntimo,
c) ausentar-se. importa muito?
d) avançar. Rapto o ritmo.
e) afastar-se. Espaçotempo ávido,
Lento espaçodentro,
12. Com relação à estrutura das palavras da Língua quando me aproximo,
Portuguesa, assinale a alternativa INCORRETA: simplesmente me desfaço,
a) Os morfemas que indicam as flexões das apenas o mínimo
palavras variáveis da língua são chamados de em matéria de máximo
desinências nominais ou verbais.
b) A vogal ou consoante de ligação é um morfema 16. Na formação dos vocábulos destacados na
incapaz de facilitar a emissão do som das questão anterior, o autor utilizou o seguinte
palavras. processo de formação de palavras:
c) O radical é um morfema comum às palavras a) Derivação prefixal
que pertencem ao mesmo campo semântico. b) Derivação parassintética
d) O elemento que liga o radical às desinências é c) Composição por justaposição
chamado de vogal temática. d) Derivação imprópria
e) Afixos são morfemas que se colocam antes ou e) Composição por aglutinação
depois do radical, alterando sua significação 17. (IFPI 2018.1- Adaptada ) Em “O Piauí saiu
básica. vencedor da disputa do Miss Brasil de 2017”, a
13. Relacione as listas e assinale a alternativa palavra destacada é um exemplo de derivação
correta: sufixal. Identifique a alternativa em que os dois
1. Consoante de ligação termos também são formados por esse mesmo
2. Variação de vogal temática processo de criação de palavras.
3. Vogal de ligação a) desigualdade - preconceito
4. Vogal temática b) racismo - discriminação
( ) dancei c) beleza - negritude
( ) sorveteria d) internacionalmente - nordestina
( ) geladeira e) competição - história
( ) bares
a) 4, 3, 1, 2
b) 2, 1, 4, 3
c) 2, 1, 3, 4
d) 2, 3, 1, 4
e) 1, 2, 3, 4

35
MORFOSSINTAXE DA LÍNGUA PORTUGUESA # Com verbos que denotam fenômenos da
– PARTE I natureza.
Ex.: Ventava forte naquela tarde. / Esfriou muito
1.Morfologia: estudo das palavras, restrito à
nesta noite.
classificação.
Obs.: Isso não ocorre se os verbos forem
2.Sintaxe: estudo das funções e relações das
usados em sentido figurado.
palavras nas frases; das frases nas orações e das
Ex.: Os papéis choveram dos prédios enfeitando
orações nos períodos.
as ruas. (o sujeito é os papéis).
3.Morfossintaxe: estudo das palavras em seus
aspectos morfológicos e sintáticos ao mesmo TIPOS DE PREDICADO
tempo.
Predicado verbal: apresenta um verbo
1. MORFOLOGIA – CLASSES DE PALAVRAS
significativo (transitivo ou intransitivo) como
a) Substantivo; b) Adjetivo e Locução adjetiva;
núcleo.
c) Pronome; d) Artigo; e) Numeral; f) Verbo e
Ex.: Os índios perderam grande parte de sua
locução verbal; g) Advérbio e Locução
cultura.
adverbial; h) Preposição e Locução prepositiva;
Predicado nominal: apresenta um verbo de
i) Conjunções.
ligação que une a qualidade (núcleo nominal) ao
2. SINTAXE
sujeito.
Frase: enunciado que estabelece uma
Ex.: O conteúdo da carta ainda continuava um
comunicação de sentido completo.
mistério. (sujeito: o conteúdo da carta / mistério:
Ex.: Cuidado! / Atenção! Ai, tristeza...
núcleo do predicado)
Oração: enunciado que contém um verbo ou
Predicado verbo-nominal: Apresenta dois
uma locução verbal.
núcleos. O núcleo verbal transmite informações
Ex.: Maria chegou tarde.
sobre o sujeito, e o núcleo nominal caracteriza o
Período: frase formada por uma ou mais
objeto.
orações.
Ex.: A diretoria da empresa considerou o relatório
Período simples: constituído de uma só
incompleto. (núcleos: considerou e incompleto).
oração.
Período composto: constituído por duas ou TERMOS RELACIONADOS AO VERBO
mais orações. #Objeto direto: complemento que se liga ao
verbo transitivo direto, sem preposição.
TERMOS ESSENCIAIS DA ORAÇÃO
Ex.: Os testes de DNA constituem um valioso
Sujeito e predicado
recurso. (constituem – VTD)
Tipos de sujeito:
#Objeto indireto: complemento que se liga ao
#Sujeito simples: O marido de Ana pensou bem
verbo transitivo indireto por meio de preposição.
sobre o assunto. Ex.: O inquilino optou pela renovação do
#Sujeito composto: As palavras finais dele e
contrato. (optou – VTI)
sua atitude agradaram-na.
#Adjunto adverbial: termo modificador do verbo
#Sujeito oculto (implícito ou desinencial):
que exprime circunstância ou intensifica um verbo,
Estamos preocupados com o resultado dos
adjetivo ou advérbio.
exames. Ex.: Ao longe avista-se a ponte sob o pôr-do-
#Sujeito indeterminado:
sol, com os arranhas céus ao fundo.
Verbo na 3ª pessoa do plural: Dizem que o
# Agente da passiva: termo da oração que
problema será resolvido.
exprime quem pratica a ação verbal quando o
Verbo flexionado na 3ª pessoa do singular e
verbo está na voz passiva.
seguido do pronome “se”, que funciona como
Ex.: Os pacientes foram atendidos pelo médico
índice de indeterminação do sujeito. Esse modo
de plantão.
só se aplica com verbos transitivo indireto,
intransitivo ou de ligação. TERMOS RELACIONADOS AO NOME
Exs.:Investe-se pouco em saúde. (VTI) / Viaja-se #Adjunto adnominal: É o termo que determina,
com maior economia e segurança. (VI) / Aqui se especifica ou explica um substantivo. O adjunto
fica bem instalado. (VL) adnominal possui função adjetiva na oração, a
qual pode ser desempenhada por adjetivos,
ORAÇÃO SEM SUJEITO
locuções adjetivas, artigos, pronomes adjetivos e
# Com o verbo haver no sentido de existir.
numerais adjetivos.
Ex.: Há bons cinemas nesse shopping.
Ex.: O poeta inovador enviou dois longos
# Com verbo fazer, haver, e ir referindo-se a
trabalhos ao seu amigo de infância.
tempo decorrido.
#Complemento nominal: É o termo que
Ex.: Faz muito tempo que não o vejo. / Morei em
completa o sentido de uma palavra que não seja
Teresina há cerca de dez anos. / Ia o tempo da
verbo. Assim, pode referir-se a substantivos,
Primeira república quando se conheceram.
adjetivos ou advérbios, sempre por meio de
# Com o verbo “ser” indicando tempo em geral.
preposição.
Ex.: Era um período de inverno rigoroso.

36
Ex.: Cecília tem orgulho da filha. / Ricardo estava Do indo-europeu, o pronome tu se manteve
consciente de tudo./ A professora agiu nos dialetos itálicos, dentre os quais o que viria a ser o
favoravelmente aos alunos. latim. Deste, o pronome foi herdado por todas as
#Predicativo do sujeito: É o termo que atribui línguas românicas ou neolatinas, inclusive o
português, e sempre na mesma forma tu (as
características ao sujeito por meio de um verbo de
diferenças, quando há, são só de pronúncia, como no
ligação. francês). Até hoje, “tu” é a principal forma de
Ex.: o resto era a noite, a lembrança. (Cecília tratamento entre pessoas íntimas em Portugal. E, no
Meireles) Brasil, terra em que “você” se generalizou, o pronome
#Predicativo do objeto: termo que atribui “tu” ainda tem grande vitalidade na região Sul, em
característica ao objeto direto ou ao objeto partes do Norte e Nordeste e entre as camadas
indireto. populares do Rio de Janeiro. Quem diria que tanta
Ex.: A música tornou Mozart famoso. gente usa a toda hora uma palavra que é falada há
#Aposto: termo da oração que explica, esclarece, pelo menos 6 mil anos!
resume ou identifica o nome ao qual ele se refere. Blog do Aldo Bizzocchi (junho de 2015)Disponível em:
http://revistalingua.com.br/textos/blog-abizzocchi/o-pronome-milenar-352287-
Ex.: Vi um filme sobre Jack, o Estripador. 1.asp
#Vocativo: termo da oração que se refere a um
interlocutor a quem se dirige a palavra. 01. (IFPI 2015-2) Assinale a alternativa cujo termo
Ex.: de vós me aparto, ó vida!(Luís de Camões) destacado pertence à classe dos adjetivos:
a) “O pronome milenar”
VOZES DO VERBO b) “dialetos itálicos”
#Voz ativa: quando o sujeito é o agente, pratica c) “forma de tratamento”
a ação do verbo. d) “preservação de vocábulo”
Ex.: Miguel fez o trabalho. e) “poucas alterações”
#Voz passiva: quando o sujeito é paciente, 02. (IFPI 2015-2)Transpondo para a voz ativa a
sofrendo a ação do verbo. oração “O pronome foi herdado por todas as
Ex.: O trabalho foi feito por Miguel. línguas românicas ou neolatinas (...)”, obtém-
Voz passiva analítica: O trabalho foi feito por se a forma verbal:
Miguel. a) herdou-se
Voz passiva sintética: A voz passiva sintética b) herdou
ou pronominal constrói-se com o verbo na 3ª c) herdaram
pessoa, seguido do pronome apassivador SE. d) herdavam
Ex.: Abriram-se as inscrições para o concurso. e) herdaram-se
/ Destruiu-se o velho prédio da escola.
#Voz reflexiva: quando o sujeito é ao mesmo LEIA O TRECHO PARA RESPONDER ÀS
tempo agente e paciente, isto é, pratica e QUESTÕES 03 E 04.
recebe a ação.
Ex.: O menino feriu-se. Congonhas (MG) – Os profetas de Aleijadinho
Do alto da colina, os 12 Profetas de
REFERÊNCIAS Aleijadinho derramam bênçãos à Congonhas (MG),
SARMENTO, Leila Lauar. Gramática em textos.2.ed.São Paulo: mas há quem diga que eles conspiram.
Moderna, 2005. Após a conclusão das obras dos Passos da Paixão,
CUNHA, Celso; CINTRA, Lindley. Gramática do português. Antônio Francisco Lisboa, o Aleijadinho, e sua equipe
6.ed.Rio de Janeiro: Lexikon,2013. começaram a construção dos Profetas no adro da
http://www.soportugues.com.br/
Igreja do Senhor Bom Jesus. De 1800 a 1805,
PROFESSOR RESPONDE Aleijadinho deixou aqui, nas imagens esculpidas em
LEIA O TEXTO PARA RESPONDER ÀS pedra-sabão, a marca de seu gênio. [...]
QUESTÕES 01 E 02. Disponível em: https://ultimaparada.wordpress. com/2009/01/27/congonhas-
mg-os-profetas-de-aleijadinho/
O pronome milenar
Algumas palavras costumam permanecer de 03. (IFPI 2015-2) Em “Após a conclusão das obras
modo mais estável na língua do que outras por fazerem dos Passos da Paixão, Antônio Francisco
parte de seu próprio arcabouço gramatical, como é o Lisboa, o Aleijadinho, e sua equipe começaram
caso de pronomes, preposições e numerais. a construção dos Profetas no adro da Igreja do
[...] Senhor Bom Jesus”, o sujeito da oração é:
Um exemplo particularmente valioso de a) O Aleijadinho.
preservação de vocábulo por milênios a fio é o pronome
b) Antônio Francisco Lisboa e sua equipe.
pessoal “tu”: conforme reconstruções realizadas a partir
de línguas documentadas, o pronome pessoal de c) Os Profetas no adro da Igreja do Senhor
segunda pessoa no idioma indo-europeu, cerca de 4 mil Bom Jesus.
anos a.C., já era tu. Essa forma passou à maioria das d) A conclusão das obras dos Passos da
línguas indo-europeias, às vezes com poucas alterações Paixão.
(grego týou sý, latim tu, antigo irlandês tu, gótico thu, e) A Igreja do Senhor Bom Jesus.
antigo alto alemão du, lituano tù, antigo prussiano tou,
antigo eslavo ty, sânscrito t(u)vamou tum, hindi tu).

37
04. (IFPI 2015-2) Na passagem “(...) mas há quem 01. (IFPI 2015-2) Considerando a tirinha, analise
diga que eles conspiram”, no 1º parágrafo, o os itens abaixo:
verbo destacado, quanto à predicação, pode ser I. O termo “Miguelito”, no primeiro quadrinho,
classificado, no contexto como: pertence à classe dos substantivos.
a) Verbo intransitivo. II. “Miguelito”, no primeiro quadrinho, exerce a
b) Verbo transitivo direto. função sintática de vocativo.
c) Verbo transitivo indireto. III. A maneira como Mafalda emprega
d) Verbo transitivo direto e indireto. “Miguelitos”, no último quadrinho, atribui a
e) Verbo de ligação. esse termo um valor de adjetivo.
Está(ão) correta(s):
LEIA O TEXTO PARA RESPONDER À
a) Apenas a I.
QUESTÃO 05.
Exagerado b) I e II.
(Cazuza / Ezequiel Neves / Leoni) c) I e III.
Amor da minha vida d) II e III.
Daqui até a eternidade e) I, II e III.
Nossos destinos foram traçados
LEIA O TEXTO PARA RESPONDER À
Na maternidade
Paixão cruel, desenfreada QUESTÃO 02.
Te trago mil rosas roubadas Epitáfio
Pra desculpar minhas mentiras Titãs
Minhas mancadas. (...) (Sérgio Britto)
Devia ter amado mais
05. (IFPI 2015-02) Observando a oração “Te trago
Ter chorado mais
mil rosas roubadas”, julgue: Ter visto o sol nascer
I. O sujeito da oração é “mil rosas”. Devia ter arriscado mais e até errado mais
II. O verbo “trago”, quanto à predicação, pode Ter feito o que eu queria fazer
ser considerado um verbo transitivo. Queria ter aceitado as pessoas como elas são
III. “Te” é identificado, sintaticamente, como Cada um sabe a alegria e a dor que traz no
complemento nominal. coração
IV. “Mil rosas roubadas” é objeto direto da O acaso vai me proteger
oração. Enquanto eu andar distraído
Marque: O acaso vai me proteger
Enquanto eu andar...
a) I, II e IV estão corretas.
Devia ter complicado menos, trabalhado menos
b) I e III estão corretas. Ter visto o sol se pôr
c) II e IV estão corretas. Devia ter me importado menos com problemas
d) III e IV estão corretas. pequenos
e) II, III e IV estão corretas. Ter morrido de amor
Queria ter aceitado a vida como ela é
ALUNO RESPONDE
A cada um cabe alegrias e a tristeza que vier (...)
LEIA A TIRINHA.
02. (IFPI 2015-2) Predomina na letra da música o
pretérito imperfeito (do indicativo) como tempo
verbal, como em “devia”, “queria”. Observando
os versos em que esses tempos verbais
aparecem, marque a alternativa correta sobre
esse tempo verbal:
a) expressa a ideia de uma ação que ocorrerá
num tempo futuro em relação ao tempo atual.
b) expressa a ideia de uma ação ocorrida no
passado, mas que é anterior a outra ação,
também passada.
c) exprime ações que dão a ideia de não estarem
totalmente concluídas no passado.
d) transmite a ideia de uma ação completamente
concluída.
e) expressa uma ação que está ocorrendo no
momento em que se fala ou uma ação que se
repete ou se perdura.

MAFALDA. Autor: Quino.


Fonte: Google Imagens.

38
03. (IFPI 2014-2) Leia uma estrofe do poema “A 05. (IFPI 2014-2) Uma das características marcantes
solidão e sua porta”. do poema acima é a inversão sintática; ou seja,
o sujeito costuma aparecer no final do verso e
A Solidão e sua porta
não no começo; os complementos verbais
Carlos Pena Filho
podem aparecer antes dos verbos, e não depois,
[...]
como em uma construção direta. Assinale o
Arquitetar na sombra a despedida
único verso que NÃO apresenta uma construção
Deste mundo que te foi contraditório sintática inversa.
Lembra-te que afinal te resta a vida a) “Nasce o Sol, e não dura mais que um dia”.
[...] b) “Em tristes sombras morre a formosura”.
c) “Como a beleza se transfigura?”.
Sobre a construção gramatical do verso d) “Mas no Sol, e na Luz falte a firmeza”.
“Lembra-te que afinal te resta a vida”, assinale e) “Começa o mundo enfim pela ignorância” da
a alternativa INCORRETA: tuberculose.
a) A forma verbal “lembra” está no imperativo. 06. (IFPI-2016) Considerando a classificação
b) O sujeito da forma verbal “lembra” é tradicional do sujeito, é possível afirmar que na
indeterminado. oração “Nas últimas semanas, tem circulado pela
c) Trata-se de um período composto. internet a seguinte imagem” o sujeito é
d) O sujeito da oração principal está implícito; a) Indeterminado.
trata-se do pronome TU. b) Determinado Simples.
e) “Que” é uma conjunção. c) Determinado Oculto.
LEIA O TRECHO PARA RESPONDER À d) Determinado Composto.
QUESTÃO 04. e) A oração é sem sujeito.
(IFPI-2016) Responda as questões 07 e 08,
“Não me prendo a nada que me defina. Sou considerando o seguinte título de um artigo
companhia, mas posso ser solidão. Tranquilidade e
publicado na Folha de São Paulo online:
inconstância, pedra e coração. Sou abraços, sorrisos,
ânimo, bom humor, sarcasmo, preguiça e sono.
Música alta e silêncio. Serei o que você quiser, mas só
quando eu quiser. Não me limito, não sou cruel
comigo! Serei sempre apego pelo que vale a pena e
desapego pelo que não quer valer… Suponho que me
entender não é uma questão de inteligência e sim de
sentir, de entrar em contato. Ou toca, ou não toca.
(Clarice Lispector)

04. (IFPI 2014-2) No trecho “Não me prendo a


nada que me defina”, o sujeito não aparece
explicitamente, mas podemos identificá-lo:
a) Por meio da desinência do verbo; Fonte: Folha de São Paulo UOL. Ano: 2015.
b) Uma vez que ele será citado ao longo de todo
o trecho; 07. Observando a expressão “Machistas não
c) Marcando-o no próprio texto; passarão”, julgue os itens:
d) Apenas no final do trecho; I. A expressão denota que uma certa classe é
e) Não podemos classificá-lo. impedida de continuar, de avançar.
LEIA O TEXTO PARA RESPONDER À II. Os termos “passarão” e “passarinho” estão
empregados nos graus aumentativo e
QUESTÃO 05.
diminutivo, respectivamente.
À Instabilidade das Cousas do Mundo III. O verbo “passarão” pode assumir um
Gregório de Matos significado específico em outras orações,
“Nasce o Sol, e não dura mais que um dia, como em “Machistas não passarão na prova”
Depois da Luz se segue a noite escura, Está(ão) correto(s):
Em tristes sombras morre a formosura, a) Apenas o item I.
Em contínuas tristezas a alegria. b) Apenas o item II.
Porém se acaba o Sol, por que nascia? c) Apenas o item III.
Se é tão formosa a Luz, por que não dura? d) Os itens I e III.
Como a beleza assim se transfigura? e) Os itens II e III.
Como o gosto da pena assim se fia? 08. Nesse título, o verbo “passarão”, quanto à
Mas no Sol, e na Luz falte a firmeza, predicação, pode ser classificado como:
Na formosura não se dê constância, a) Verbo de ligação
E na alegria sinta-se tristeza. b) Verbo intransitivo
Começa o mundo enfim pela ignorância, c) Verbo transitivo direto
E tem qualquer dos bens por natureza d) Verbo transitivo indireto
A firmeza somente na inconstância.” e) Verbo transitivo direto e indireto
39
09. (IFPI-2013) Observe as seguintes orações: 15. Assinale a alternativa em que o sujeito está
I. O verde dos teus olhos me deixou incorretamente classificado:
encantado. a) chegaram, de manhã, o mensageiro e o guia
II. O garoto ganhou uma bicicleta verde. (sujeito composto).
III. A tinta verde nem sempre combina com b) fala-se muito neste assunto (sujeito
ambientes internos. indeterminado).
A palavra “verde” pertence à classe dos c) vai fazer frio à noite (sujeito inexistente).
substantivos em: d) haverá oportunidade para todos (sujeito
a) Todas as orações. inexistente).
b) Nas orações I e III. e) não existem flores no vaso (sujeito
c) Nas orações II e III. inexistente).
d) Apenas na oração I.
LEIA O TEXO PARA RESPONDER À
e) Apenas na oração III.
QUESTÃO 16.
10. (CEFET-PR) Assinale a alternativa em que há Oh retrato da Morte, oh Noite amiga
oração sem sujeito. Manuel Maria Barbosa Du Bocage
a) Esperanças haverá sempre. “Oh retrato da Morte, oh Noite amiga,
b) Ninguém trovejou de tanta raiva quanto eu. Por cuja escuridão suspiro há tanto!
c) Haveria desejado ele tudo isso? Calada testemunha de meu pranto,
d) Alguém havia aberto a porta. De meus desgostos secretária antiga!”
e) Choveu papel picado nas ruas de Curitiba. [...]
11. (UFMG) As expressões sublinhadas 16. (IFPI-2014) “Oh retrato da Morte, oh Noite
correspondem a um adjetivo, exceto em: amiga,” é um verso formado por:
a) João Fanhoso anda amanhecendo sem a) Aposto, que caracteriza o tema do poema.
entusiasmo. b) Vocativo, a quem o eu-lírico se dirige.
b) Demorava-se de propósito naquele c) Objeto direto, já que completa o sentido do
complicado banho. verbo no verso seguinte.
c) Os bichos da terra fugiam em desabalada d) Adjunto adnominal, caracterizando a morte e
carreira. a noite.
d) Noite fechada sobre aqueles ermos perdidos e) Sujeito, uma vez que é a da morte e da noite
da caatinga sem fim. que o eu-lírico fala.
e) E ainda me vem com essa conversa de 17. Comparando o título do poema “Oh retrato da
homem da roça. Morte, oh Noite amiga” com o título do livro
abaixo, temos:
12. Marque a alternativa em que haja um artigo
definido e um artigo indefinido,
respectivamente:
a) Roberta é a melhor aluna dessa classe.
b) Gostaria de comprar um celular novo e a
sandália daquela loja.
c) Uma boa noite de sono é a melhor maneira
de evitar o estresse.
d) A casa que comprei é um pouco antiga.
e) Nenhuma das alternativas anteriores.
13. “O toque dos sinos ao cair da noite era trazido a) O mesmo valor semântico.
lá da cidade pelo vento”. O termo grifado é: b) Diferença na escrita de “oh” para “o”,
a) sujeito. marcada pelo tempo: “oh” é uma escrita
b) objeto direto. antiga, enquanto “o” é mais atual.
c) objeto indireto. c) Diferença na escrita, no sentido e na função
d) complemento nominal. morfossintática.
e) agente da passiva. d) Diferença apenas de entonação.
e) Diferenças regionais e dialetais quanto ao
14. Em: “Bebe que é doce, papai” – a palavra uso do “oh” e “o”.
grifada funciona como:
a) sujeito. A quadrinha seguinte, de autoria incerta, circula
b) aposto. pelas redes sociais. Analise-a para responder às
c) vocativo. questões de 01 a 02.
d) adjunto adverbial. O livro
e) adjunto adnominal. Eu do livro não me livro
e nem quero me livrar
se do livro eu me livro
como livre vou ficar?

40
18. Identifique a alternativa em que o elemento Leia com atenção o poema seguinte, de autoria de
destacado é INADEQUADAMENTE classificado, Bráulio Bessa, no qual se baseia a questão 05.
de acordo com sua função morfológica Sobre a vida
a) Eu do livro não me livro (verbo) “Só eu sei cada passo por mim dado
b) e nem quero me livrar (verbo) nessa estrada esburacada que é a vida,
c) Eu do livro não me livro (substantivo) passei coisas que até mesmo Deus duvida,
d) se do livro eu me livro (substantivo) fiquei triste, capiongo, aperreado,
e) como livre vou ficar? (adjetivo) porém nunca me senti desmotivado,
me agarrava sempre numa mão amiga,
19. Assinale a alternativa em que a partícula “se” e de forças minha alma era munida
desempenha a mesma função que em “se do pois do céu a voz de Deus dizia assim:
livro eu me livro”. – Suba o queixo, meta os pés, confie em mim,
a) Não se passou muito tempo desde que nos vá pra luta que eu cuido das feridas.”
vimos.
21. Analise o contexto do poema e marque a
b) Se tivéssemos nos apressado, aproveitaríamos
alternativa que indica a função morfológica e o
mais o passeio.
sentido da palavra “capiongo” (4º verso)
c) Vendem-se casas.
d) Marina livrou-se de uma bela cilada. a) Substantivo; “cabisbaixo”
e) Precisa-se de vendedores. b) Adjetivo; “melancólico”
c) Advérbio; “desconfiado”
d) Substantivo; “eufórico”
e) Adjetivo; “confuso”
22. Apenas um dos vocábulos seguintes é formado
por derivação prefixal. Identifique-o.
a) íngreme
b) irregular
c) inventivo
d) impacto
e) íntimo
23. Aponte a alternativa que contém,
respectivamente, o tempo em que as forma
verbais “arribou”, “vivia”, “seguem” estão
conjugadas.
a) Pretérito imperfeito do indicativo; pretérito
mais-que-perfeito do indicativo; presente do
indicativo
b) Pretérito imperfeito do subjuntivo; futuro do
subjuntivo; presente do subjuntivo.
c) Futuro do pretérito do indicativo; pretérito
imperfeito do indicativo; presente do
indicativo d) Pretérito perfeito do indicativo;
pretérito imperfeito do indicativo; presente do
indicativo
e) Presente do indicativo; pretérito imperfeito do
subjuntivo; pretérito perfeito do indicativo.

Considere o texto seguinte para responder à


questão 20.
20. Indique a alternativa que apresenta a função
morfológica do vocábulo “porquê” no título do
texto.
a) Substantivo
b) Pronome
c) Conjunção
d) Substantivo
e) conjunção
41
MORFOSSINTAXE DA LÍNGUA PORTUGUESA- Ex: Não tenho preparo físico; portanto, não posso
PARTE II participar do campeonato de futebol de salão.
São conjunções conclusivas: portanto, logo, por
PERÍODO COMPOSTO POR COORDENAÇÃO
isso, pois, assim.
ORAÇÃO COORDENADA 5. EXPLICATIVAS – estabelecem explicação, ou
seja, explicam o motivo da declaração feita na
LEIA A TIRIINHA: oração anterior.
Ex: Senti frio, porque estava sem agasalho.
São conjunções explicativas: que, porque, pois,
porquanto
PERÍODO COMPOSTO POR SUBORDINAÇÃO
As orações subordinadas dividem-se em três
grupos, de acordo com a função sintática que
desempenham e a classe de palavras a que
equivalem. Podem ser substantivas, adjetivas ou
adverbiais.
ORAÇÕES SUBORDINADAS SUBSTANTIVAS.
O enunciado: São classificadas em:
Você está gordo ou é minha 1. Subjetiva: exerce a função sintática de sujeito do
imaginação? verbo da oração principal. Observe:
Oração Principal O. S. S. Subjetiva
1ª oração 2ª oração você se comportar na reunião.
É fundamental
Constitui um período composto. Nele as (reduzida de infinitivo)
orações não funcionam como termos de outra É fundamental que você compareça na reunião.
oração. Elas são autônomas. Observe algumas estruturas típicas que
A ORAÇÃO COORDENADA caracteriza-se ocorrem na oração principal:
como independente do ponto de vista sintático, ou 1. Verbos de ligação + predicativo, em
seja, coloca-se ao lado da outra, mas não construções do tipo: É bom - É útil - É
desempenha função sintática. Ela pode apresentar conveniente - É certo - Parece certo - É claro -
ou não conjunção. Sendo, pois, denominada de Está evidente - Está comprovado.
SINDÉTICA ou ASSINDÉTICA. Ex.: É bom que você compareça à minha festa.
a) SINDÉTICA – é introduzida por conjunção Oração Principal O. S. S. Objetiva Direta
Ex: “OU” é minha imaginação? A garota afirmou
precisar de ajuda. (reduzida
b) ASSINDÉTICA – não é introduzida por de infinitivo)
conjunção A garota afirmou que precisa de ajuda.
Ex: Você está mais gordo 2. Expressões na voz passiva, como: Sabe-se -
Soube-se - Conta-se - Diz-se - Comenta-se -
VALORES SEMÂNTICOS DAS ORAÇÕES
É sabido - Foi anunciado - Ficou provado.
COORDENADAS SINDÉTICAS
As orações coordenadas sindéticas Ex.: Sabe-se que Aline não gosta de Pedro.
classificam-se de acordo com as ideias que 3. Verbos como: convir - cumprir - constar -
expressam. Podem ser de cinco tipos: admirar - importar - ocorrer – acontecer.
1. ADITIVAS – estabelecem ideia de adição. Ex.: Convém que não se atrase na entrevista.
Ex: “Viemos, vimos e vencemos” (Lewis Hamilton) 2. Objetiva Direta: a oração subordinada exerce a
São conjunções aditivas: e, nem, mas, também. função de objeto direto do verbo da oração
2. ADVERSATIVAS – estabelecem oposição, principal.
adversidade. 3. Objetiva Indireta: a oração subordinada exerce
Ex: Estuda mas não aprende. a função de objeto indireto do verbo da oração
São conjunções adversativas: mas, porém, todavia, principal. Vem precedida de preposição.
contudo, entretanto, no entanto. Oração Principal O. S. S. Objetiva Indireta
3. ALTERNATIVAS – estabelecem alternância. Meu pai insiste
em eu estudar. (reduzida de
Ex:Ora os alunos entram na sala de aula, ora se infinitivo)
atrasam no pátio após o intervalo. Meu pai insiste Em que eu estude.
São conjunções alternativas: ou, ou...ou, ora... ora, Obs.: em alguns casos, a preposição pode
já, quer... quer. estar elíptica na oração.
4. CONCLUSIVAS – estabelecem conclusão ou Ex.: Marta não gosta (de) que a chamem de
consequência lógica. senhora.
Oração Principal O. S. S. Completiva Nominal 4. Completiva Nominal: a oração subordinada
completa um nome que pertence à oração
Maria ficou receosa de perder o emprego principal e também vem marcada por
(reduzida de infinitivo) preposição.
Maria ficou receosa de que perdesse o emprego.

42
Observe que as orações subordinadas integrante, iniciando uma oração que completa
substantivas objetivas indiretas integram o sentido o sentido da oração anterior. Assinale a
de um verbo, enquanto que orações subordinadas alternativa em que a palavra QUE seja uma
substantivas completivas nominais integram o conjunção integrante.
sentido de um nome. Para distinguir uma da outra, a) “Não me prendo a nada que me defina”.
é necessário levar em conta o termo b) “Serei o que você quiser”.
complementado. Essa é, aliás, a diferença entre o c) “Serei sempre apego pelo que vale a pena”.
objeto indireto e o complemento nominal: o primeiro d) “...desapego pelo que não quer valer”.
complementa um verbo, o segundo, um nome. e) “Suponho que me entender não é uma
questão de inteligência”.
5. Predicativa: a oração subordinada substantiva
02. Sobre o trecho “Ou toca, ou não toca”,
predicativa exerce papel de predicativo do sujeito
podemos afirmar que:
do verbo da oração principal e vem sempre depois
a) Trata-se de um período simples.
do verbo ser.
b) Trata-se de um período composto por
Oração Principal O. S. S. Predicativa coordenação, porém sem conjunções.
O fato era que antigamente os pais liam c) Trata-se de um período composto por
para os filhos à noite. coordenação, com uma conjunção explícita.
A verdade é Que o espetáculo ficou em d) Trata-se de um período composto por
cartaz mais de um ano.
subordinação, constituído de oração
6. Apositiva: a oração subordinada substantiva adjetiva.
apositiva exerce função de aposto de algum termo e) Trata-se de um período composto por
da oração principal. subordinação, constituído de oração
Oração Principal O. S. S. Apositiva adverbial.

Antecipou-me que voltaria para o interior.


LEIA O ANÚNCIO PARA RESPONDER ÀS
esta informação: QUESTÕES DE 03 A 05.
A equipe tomou que o trabalho seria entregue
uma decisão: em uma semana.

Lembretes:
1.Oração desenvolvida é a que apresenta o
verbo flexionado no modo indicativo, subjuntivo
ou imperativo e é introduzida por uma
conjunção (ou outro conectivo). / Oração
reduzida é a que apresenta o verbo numa das
formas nominais, infinitivo, gerúndio ou
particípio, e não é introduzida por uma
conjunção (ou outro conectivo).
2. Nas orações subordinadas substantivas, as
conjunções que as introduzem recebem o
nome de conjunção integrante. 03. Sobre este anúncio escreva V ou F conforme
sejam verdadeiras ou falsas as seguintes
PROFESSOR RESPONDE
afirmações:
TRECHO PARA AS QUESTÕES 01 E 02. ( ) Os argumentos utilizados para convencer
“Não me prendo a nada que me defina. Sou o consumidor são a imagem de um lugar
companhia, mas posso ser solidão. bonito onde é possível descansar ou
Tranquilidade e inconstância, pedra e coração. pular o carnaval.
Sou abraços, sorrisos, ânimo, bom humor, ( ) De acordo com o contexto, foi veiculado
sarcasmo, preguiça e sono. Música alta e antes do carnaval.
silêncio. Serei o que você quiser, mas só quando ( ) Tem como finalidade sensibilizar e
eu quiser. Não me limito, não sou cruel comigo! convencer as pessoas sobre determinado
Serei sempre apego pelo que vale a pena e produto.
desapego pelo que não quer valer… Suponho ( ) É destinado para todas as pessoas,
que me entender não é uma questão de
principalmente para quem não gosta de
inteligência e sim de sentir, de entrar em contato.
Ou toca, ou não toca. viajar.
(Clarice Lispector) A sequência correta é:
a) V V F F
01. (IFPI 2014-2) Em um dos trechos abaixo, a
b) V VV F
palavra destacada diferencia-se das demais
c) F V V F
pela função morfossintática. Em quatro delas
d) V F F V
trata-se de um pronome relativo, retomando a
e) F V F V
palavra anterior e em apenas uma é conjunção

43
04. No balão maior, há um período composto por
RELEIA O TRECHO E RESPONDA:
coordenação.
Sobre esse período assinale a alternativa “beijo ou não beijo
CORRETA. eis a questão.”
a) Apresenta duas orações. 02. A conjunção destacada expressa a ideia de:
b) As orações são coordenadas assindéticas. a) conclusão.
c) Apresenta linguagem completamente formal. b) explicação.
d) É um período simples c) oposição.
e) Apresenta três orações. d) alternância.
05. O tipo de relação que a conjugação “ou” e) adição.
estabelece entre as orações é de: LEIA OS VERSOS
a) adição. “Sufoco de vergonha
b) alternância. e sufoco de falta de jeito”.
c) conclusão.
03. A conjunção “e” nesses versos expressa ideia
d) explicação.
de:
e) oposição.
a) oposição.
ALUNO RESPONDE b) conclusão.
c) adição.
LEIA O TEXTO QUE SEGUE PARA
d) explicação.
RESPONDER ÀS QUESTÕES DE 01 A 03.
DÚVIDAS e) alternância.
Às vezes LEIA A TIRINHA ABAIXOE RESPONDA À
eu sinto que ela quer
Outras vezes acho que não.
QUESTÃO 04.
Ah, como ela grita
o meu peito...
Cala a boca,
coração!
Ela não pode
desconfiar
que este vai ser
o meu primeiro...
Sufoco de vergonha
e de falta de jeito.
E agora, meu Deus?
O que é que eu faço
Com as mãos?
Às vezes
Eu sinto que ela quer.
Outras vezes
Eu acho que não.
Beijo ou não beijo...
Eis a questão.
01. Em relação ao texto escreva “V” ou “F”
conforme sejam verdadeiras ou falsas as
seguintes afirmações:
( ) O título do texto está relacionado com a (Glauco, Folha de S. Paulo, 25/04/2004)
dúvida que o poeta demonstra ter de beijar
ou não uma garota.
( ) A repetição da 1ª e 5ª estrofes reforça a 04. No segundo quadrinho há uma conjunção
dúvida que o poeta sente e sua coordenativa subentendida (pois). Que valor
insegurança. semântico ela apresenta no texto?
( ) O estado emocional do poeta mostra que ele a) Explicação.
é uma pessoa muito experiente. b) Oposição.
( ) O poeta revela seu amor ao ser amado, c) Alternância.
porém não é correspondido.
d) Conclusão.
A sequência correta é:
a) V F V V e) Adição.
b) F V VV
c) V VV F
d) V VVV
e) V V F F
44
05. (UFSM –RS) Assinale a sequência de 09. (FCE-SP) "Os homens sempre se esquecem de
conjunções que estabelecem, entre as orações que somos todos mortais." A oração destacada
de cada item, uma correta relação de sentido. é:
1. Correu demais ... caiu a) substantiva completiva nominal.
2. Dormiu mal ... os sonhos não deixaram em paz. b) substantiva objetiva indireta.
3. A matéria perece... a alma é imortal. c) substantiva predicativa.
4. Leu o livro... é capaz de descrever as d) substantiva objetiva direta.
personagens em detalhe. e) substantiva subjetiva.
5. Guarde seus pertences... podem servir mais 10. (FEI-SP) "Estou seguro de que a sabedoria dos
tarde. legisladores saberá encontrar meios para
a) porque, todavia, portanto, logo, entretanto. realizar semelhante medida." A oração em
b) por isso, porque, mas, portanto, que. destaque é substantiva:
c) logo, porém, pois, porque, mas. a) objetiva indireta.
d) porém, pois, logo, todavia, porque. b) completiva nominal.
e) entretanto, que, porque, pois, portanto. c) objetiva direta.
LEIA O TEXTO PARA RESPONDER Á d) subjetiva.
QUESTÃO 06. e) apositiva.
“Já vi pessoas chorando por filme, novela, futebol, 11. (UF-PA) Qual o período em que há oração
mas não encontro pessoas chorando pela restauração subordinada substantiva predicativa?
da família.” a) Meu desejo é que você passe nos exames
(Padre Chrystian Shankar: TV Canção Nova.com)
vestibulares.
06. (IFPI-2013) Em relação ao uso da conjunção b) Sou favorável a que o aprovem.
“mas”, empregada no texto, podemos afirmar c) Desejo-te isto: que sejas feliz.
que ela: d) O aluno que estuda consegue superar as
a) liga duas ideias, atribuindo a elas um sentido dificuldades do vestibular.
de contrariedade. e) Lembre-se de que tudo passa nesse mundo.
b) apenas prolonga o sentido da primeira ideia.
TEXTO PARA A QUESTÃO 12.
c) foi utilizada para explicar a primeira ideia.
d) não desempenha valor significativo no texto.
e) pode ser substituída, sem prejuízo de sentido,
pela conjunção “por”.
LEIA O TRECHO DA MÚSICA “DIAS DE LUTA,
DIAS DE GLÓRIA” DA BANDA CHARLIE
BROWN JR., E RESPONDA:
Eu sou feliz e rodo pelo mundo
Sou correria mas também sou vagabundo
Mas hoje dou valor de verdade pra minha saúde
pra minha liberdade
Que bom te encontrar nessa cidade
Esse brilho intenso me lembra você
História, nossas histórias
Dias de luta, dias de glória
07. Em relação à musica da Banda Charlie Brown
Jr., na frase “Mas hoje dou valor de verdade pra 12. A oração destacada em “Acho que exagerei”,
minha saúde” a palavra sublinhada da ideia de: é uma oração subordinada substantiva:
a) alternância. a) subjetiva.
b) finalidade. b) predicativa.
c) oposição. c) objetiva direta.
d) conclusão. d) objetiva indireta.
e) comparação. e) completiva nominal.

08. (2016.1-IFPI) Em “É bem provável que [o aluno]


ainda não saiba a diferença entre português
formal e português coloquial”, a oração
sublinhada exerce a função sintática de:
a) sujeito do verbo da oração principal.
b) objeto direto do verbo da oração principal.
c) objeto indireto do verbo da oração principal.
d) predicativo do sujeito do verbo da oração
principal.
e) aposto de algum termo da oração principal.

45
LEIA O TEXTO PARA RESPONDER À LEIA O TEXTO
QUESTÃO 13. VELHA CHÁCARA
A casa era por aqui...
Onde? Procuro-a e não acho.
Ouço uma voz que esqueci:
É a voz deste mesmo riacho.
Ah quanto tempo passou!
(Foram mais de cinquenta anos)
Tantos que a morte levou!
(E a vida... nos desenganos...)
A usura fez tábua rasa
Da velha chácara triste:
Não existe mais a casa...
13. No último quadrinho a oração “que a sobremesa — Mas o menino ainda existe.
é pêssego com creme” exerce a função sintática MANUEL BANDEIRA, In Lira dos cinquent’anos,1940
de
a) sujeito. 17. (IFPI 2018) Releia o terceiro verso do
b) predicativo. poema: “Ouço uma voz que esqueci”. A
c) aposto. oração destacada deve ser classificada
d) objeto direto. como
e) complemento nominal.
a) subordinada substantiva objetiva direta
14. (Colégio Naval) b) subordinada substantiva subjetiva
“Saí, afastando-me dos grupos, e fingido ler os c) subordinada adjetiva restritiva
epitáfios. E, aliás, gosto dos epitáfios; eles são, d) subordinada adjetiva explicativa
entre a gente civilizada, uma expressão daquele e) subordinada adverbial consecutiva
pio e secreto egoísmo que induz o homem a
arrancar à morte um farrapo ao menos da sombra
que passou. Daí vem, talvez, a tristeza
inconsolável dos que sabem os seus mortos na
vala comum; parece-lhes que a podridão anônima
os alcança a eles mesmos.”
(Quincas Borba - M. de Assis)

“(...) que a podridão anônima os alcança a eles


mesmos.” uma oração:
a) adjetiva restritiva.
b) adjetiva explicativa.
c) adverbial condicional.
d) substantiva subjetiva.
e) substantiva objetiva direta.
15. “Diga ao povo que fico” é um período:
a) simples.
b) composto por coordenação.
c) composto por subordinação.
d) composto por coordenação e subordinação.
e) composto de três orações.
16. (PUC-SP) Pode-se dizer que a tarefa crítica é
puramente formal.
No texto, temos uma oração destacada que é
______________________ e um “se” que é
____________________.
a) substantiva objetiva direta, partícula
apassivadora.
b) substantiva predicativa, índice de
indeterminação do sujeito.
c) relativa, pronome reflexivo.
d) substantiva subjetiva, partícula apassivadora.
e) adverbial consecutiva, índice de
indeterminação do sujeito.

46
MORFOSSINTAXE DA LÍNGUA PORTUGUESA - apresentam o verbo numa das formas nominais
PARTE III (infinitivo, gerúndio ou particípio).
Por exemplo:
ORAÇÕES SUBORDINADAS ADJETIVAS: são
a) Ele foi o primeiro aluno que se apresentou.
introduzidas por pronomes relativos e estabelecem
b) Ele foi o primeiro aluno a se apresentar.
uma relação de dependência ou de subordinação
No primeiro período, há uma oração subordinada
em relação à oração principal. Os pronomes
adjetiva desenvolvida, já que é introduzida pelo
relativos (com exceção de cujo e suas flexões)
pronome relativo "que" e apresenta verbo
podem ser substituídos por o (a) qual, os (as) quais,
conjugado no pretérito perfeito do indicativo. No
diferentemente da conjunção subordinativa que.
segundo, há uma oração subordinada adjetiva
Veja os exemplos:
reduzida de infinitivo: não há pronome relativo e
Exs.: a) Tudo isso com a solidez de uma empresa
seu verbo está no infinitivo.
que (= a qual) opera 7 mil MW de capacidade... (que
Exemplos de orações adjetivas reduzidas:
é pronome relativo)
1. Oração reduzida de gerúndio: Meu avô,
b) Ele disse que faria conforme o combinado.
trabalhando como caminhoneiro, viajou por todo o
(que é conjunção integrante).
Brasil. (Meu avô, que trabalhou como
Os pronomes relativos são: que, quem, onde,
caminhoneiro, viajou por todo o Brasil.)
como, o (a) qual, os (as) quais, cujo (a), cujos (as),
2. Oração reduzida de particípio: Assisti a dois
quanto.
filmes exibidos no último festival de Sundance.
Oração subordinada adjetiva é aquela que exerce a
(Assisti a dois filmes que foram exibidos no último
função de adjunto adnominal de uma palavra
antecedente (substantivo ou pronome) e é introduzida festival de Sundance.)
por um pronome relativo. 3. Oração reduzida de infinitivo: Pelé foi o
primeiro jogador a fazer mais de mil gols. (Pelé foi
CLASSIFICAÇÃO DAS ORAÇÕES o primeiro jogador que fez mais de mil gols.)
SUBURDINADAS ADJETIVAS:
1) Oração subordinada restritiva: é a oração ORAÇÕES SUBORDINADAS ADVERBIAIS
subordinada adjetiva que restringe, particulariza o Leia o anúncio:
sentido de um substantivo ou pronome antecedente.
Não é separada por pontuação.
Exs.:
Os jogadores que residem no exterior não se
apresentarão no prazo previsto.
Os alunos que estudam em São Paulo têm diversas
opções de cursos superiores.

No anúncio, temos um período composto por


subordinação. Veja:
Na tirinha ao lado ocorre uma oração adjetiva
Uma empresa só atinge a maturidade /
restritiva: "Vejo diante dos meus olhos a criatura que
(Oração principal)
me pôs no mundo".
Quando trabalha o seu lado infantil.
2) Oração subordinada explicativa: é a oração
(Oração subordinada)
subordinada adjetiva que acrescenta uma
Esse mesmo enunciado poderia ser expresso
explicação acessória à palavra antecedente. Na
através de um período simples.
escrita, é separada por vírgula (s).
Uma empresa só atinge maturidade numa fase
Exs.:O problema, que era de fácil resolução, deixou
adulta
os alunos apreensivos.
(Adj. Adv. de tempo)
Os jogadores de futebol, que são iniciantes, não
Note que:
recebem salários.
 A oração “quando trabalha o seu lado infantil” é
FORMAS DAS ORAÇÕES SUBORDINADAS dependente sintaticamente da oração principal
ADJETIVAS porque equivale a um adjunto adverbial da forma
Quando são introduzidas por um pronome verbal atinge; por isso, é uma oração
relativo e apresentam verbo no modo indicativo ou subordinada adverbial.
subjuntivo, as orações subordinadas adjetivas Assim: Oração subordinada adverbial é a que
são chamadas desenvolvidas. Além delas, tem um valor de advérbio (ou locução adverbial) e
existem as orações subordinadas adjetivas exerce a função de adjunto adverbial.
reduzidas, que não são introduzidas por pronome
relativo (podem ser introduzidas por preposição) e
47
VALORES SEMÂNTICOS DAS ORAÇÕES Ex: Houve protestos depois que o diretor saiu
ADVERBIAIS (são 9) da reunião.
As orações subordinadas adverbiais classificam- se, Conjunções temporais: quando, enquanto,
conforme os valores semânticos que expressam. logo que, assim que, etc.
1. Causais– indicam a causa da ação expressa na ORAÇÕES ADVERBIAIS REDUZIDAS
oração principal. Algumas orações subordinadas adverbiais
Ex: As ruas ficaram alagadas porque a chuva foi podem aparecer na forma reduzida, com o
muito forte. verbo no infinitivo, no gerúndio ou no particípio.
Como ninguém se interessou pelo projeto, não Veja:
houve alternativa a não ser cancelá-lo.  A professora ministrou aula, mesmo estando
Conjunções subordinativas causais: porque, que, doente.
uma vez que, visto que, etc. (Oração subord. Adv. Concessiva
2. Consecutivas– indicam uma consequência do reduzida de gerúndio)
fato referido na oração principal.  Não podia ouvir a música sem chorar
Ex: Sua fome era tanta que comeu com casca e (Oração subord. Adv. Consecutiva reduzida
tudo. de infinitivo)
Nunca abandonou seus ideais, de sorte que acabou  Terminando o filme, cada aluno fez seu
concretizando-os. resumo. (Oração subord. Adv. temporal
Conjunções consecutivas: que (precedido de tal, reduzida de particípio)
tão, tanto), de sorte que, de modo que, etc. PROFESSOR RESPONDE
3. Condicionais – expressam uma hipótese ou LEIA O TEXTO PARA RESPONDER ÀS
condição para que ocorra o fato referido na oração QUESTÕES DE 01 A 04.
principal. Crônica Engraçada
Ex: Se os alunos do preparatório pré-IFPI se (Luís Fernando Veríssimo)Minha mulher e eu
temos o segredo para fazer um casamento
dedicarem, serão vitoriosos. durar:
Conjunções condicionais: se, caso, desejo que, Duas vezes por semana, vamos a um ótimo
salvo se, a menos que, contanto que, etc. restaurante, com uma comida gostosa, uma boa
4. Concessivas – indicam um fato contrário ao bebida e um bom companheirismo. Ela vai as
referido na oração principal. terças- feiras e eu, às quintas. Nós também
dormimos em camas separadas: a dela é em
Ex: Embora Teresina seja quente, não queremos
Fortaleza e a minha, em SP. Eu levo minha
deixá-la. mulher a todos os lugares, mas ela sempre acha o
Conjunções concessivas: a menos que, se bem caminho de volta. Perguntei a ela onde ela
que, ainda que, conquanto que, etc. gostaria de ir no nosso aniversário de casamento,
5. Conformativas – Indicam conformidade em “em algum lugar que eu não tenha ido há muito
relação à ação expressa pelo verbo da oração tempo!” ela disse. Então, sugeri a cozinha. Nós
principal. sempre andamos de mãos dadas…Se eu soltar,
ela vai às compras! Ela tem um liquidificador, uma
Ex: Conforme prometeu, trouxe o presente.
torradeira e uma máquina de fazer pão, tudo
Conjunções conformativas: conforme, como, elétrico. Então, ela disse: “nós temos muitos
segundo, consoante, etc. aparelhos, mas não temos lugar pra sentar”. Daí,
6. Comparativas – Expressam uma comparação comprei pra ela uma cadeira elétrica. Lembrem-
com um dos termos da oração principal. se: o casamento é a causa número 1 para o
Ex: Os alunos do preparatório para o IFPI têm divórcio. Estatisticamente, 100 % dos divórcios
estudado como uns obstinados (estudam). começam com o casamento. Eu me casei com a
“senhora certa”. Só não sabia que o primeiro
Conjunções comparativas: como, que, do que, etc. nome dela era “sempre”. Já faz 18 meses que não
7. Finais – Expressam a intenção, o objetivo, a falo com minha esposa. É que não gosto de
finalidade do que se declara na oração principal. interrompê-la. Mas, tenho que admitir: a nossa
Ex: O aluno sentou-se na primeira fila a fim de que última briga foi culpa minha. Ela perguntou: “O que
pudesse ouvir melhor. tem na TV?” E eu disse: “Poeira”.
Conjunções finais: para que, a fim de que, que, 01. (IFPI-2017- adaptada) Sobre a crônica, é correto
que. dizer que o autor pretende:
8. Proporcionais – Indicam uma proporção relativa a) entreter os leitores e levá-los a refletir sobre
ao fato expresso na oração principal. comportamentos humanos.
Ex: Á medida que se aproxima a hora do exame, a b) tratar do tema casamento de forma científica.
tensão aumentava. c) argumentar sobre a importância de manter-se
Conjunções proporcionais: à proporção que, à casado.
medida que, etc. d) estimular os leitores a se divorciarem.
9. Temporais – Indicam o momento, a época, o e) apresentar dados estatísticos sobre
tempo de ocorrência do fato expresso na oração casamentos.
principal.

48
02. (IFPI-2017) Na frase “Se eu soltar, ela vai às a) causa.
compras”, a palavra “SE’’ expressa a ideia de: b) concessão.
a) causa c) finalidade.
b) lugar d) consequência.
c) consequência e) condição.
d) condição 02. A oração “vendo os meninos bonitos do
e) finalidade colégio” é classificada como:
03. Releia: “Minha mulher e eu temos o segredo para a) subordinada adverbial concessiva.
fazer um casamento durar:” b) subordinada adverbial reduzida de infinitivo.
O termo destacado pode ser substituído, sem c) coordenada sindética explicativa.
prejuízo de sentido, por: d) subordinada adverbial final reduzida de
a) quando. gerúndio.
b) a fim de e) coordenada assindética.
c) como LEIA O TEXTO PARA RESPONDER À
d) porque QUESTÃO03.
e) onde QUANDO O APAIXONADO É ELE
04. Em “nós sempre andamos de mãos dadas... Se Não dê esperanças se não estiver a fim dele,
ainda que você não queira magoá-lo, por outro lado,
eu soltar, ela vai às compras”. Os termos
também não precisa tratá-lo mal, só para que ele
destacados expressam, respectivamente, ideia desencane.
de: Isso vai machucar o coraçãozinho dele.
a) tempo – modo -lugar (Revista Atrevidinha, abril. 2006)
b) tempo – finalidade -modo 03. As orações subordinadas adverbiais
c) afirmação- causa -lugar destacadas são classificadas, respectivamente,
d) modo - lugar -tempo como:
e) condição - modo -lugar a) concessiva -final
05. (PUC – SP) Considere a palavra destacada neste b) consecutiva -causal.
período: “E há poetas míopes que pensam que c) temporal -final
é o arrebol”. Ela introduz, respectivamente, d) concessiva -proporcional
orações: e) causal –consecutiva
a) subordinada substantiva completiva nominal e 04. Assinale a alternativa que há erro na
subordinada substantiva objetiva direta. classificação da oração subordinada adverbial.
b) subordinada substantiva objetiva direta e a) O sono aumentava, à medida que as horas
subordinada substantiva predicativa. avançavam.(proporcional)
c) subordinada adjetiva restritiva e subordinada b) A situação agravou-se porque aumentou o
adjetiva explicativa. desemprego.(causal)
d) subordinada substantiva predicativa e c) A passarela foi feita para que os pedestres
subordinada substantiva objetiva direta. tivessem segurança.(concessiva)
e) subordinada adjetiva restritiva e subordinada d) Os professores planejaram a reunião
substantiva objetiva direta. segundo combinaram (conformativa)
e) “Quando você foi embora, fez-se noite em
ALUNO RESPONDE meu viver” (Milton Nascimento)(temporal)
LEIA A TIRINHA PARA RESPONDER AS LEIA A TIRINHA PARA RESPONDER AS
QUESTÕES DE 01 E 02. QUESTÕES 05 E 06.
PAPAI, VOCÊ TAMBÉM
POR QUE SE A MINHA
BRINCAVA COM IOIÓ MAFALDINHA! O QUE
GERAÇÃO PUXAR A SUA
QUANDO ERA PEQUENO? FOI, POR QUE VOCÊ
ESTAREMOS FRITOS!
ESTA CHORANDO
CLARO! E NINGUEM
GANHAVA DE
MIM!

05. A conjunção “porque” exprime, na oração


“porque estamos fritos”, em relação ao verbo
da oração principal, ideia de:
01. Releia “É só a gente ficar na nossa que alguma a) proporção
coisa pro nosso lado rola!” Nessa fala de Luke b) causa
há um período composto e duas orações. O tipo c) finalidade.
de relação semântica existente entre elas é: d) consequência.
e) tempo.
49
06. Em “se a minha geração puxar a sua” a 10. Assinale a alternativa em que aparece uma
conjunção “se” indica: oração subordinada adjetiva explicativa:
a) tempo. a) Há, em nossas vidas, momentos que
b) causa. parecem eternos.
c) consequência. b) Ela se formou no mês em que ele
d) finalidade. aniversariou.
e) condição. c) Ganhou um cavalo que se chamava Sun
Set.
LEIA O TEXTO PARA RESPONDER À
d) Moro na casa em que meus avós moravam.
QUESTÃO 07.
e) A perseverança, que é a marca dos fortes,
[...] leva a sucessos na vida.
“Cauto marujo, levo em segredo
Para a cama uma fatia 11. (PUC-SP) LEIA O TEXTO
de bolo e também algum brinquedo “João amava Teresa que amava Raimundo
Pois é longa a travessia” que amava Maria que amava Joaquim que
[...] amava Lili
07. A expressão destacada possui o valor de adjunto que não amava ninguém.
adverbial, expressando a circunstância de: João foi para os Estados Unidos, Teresa
a) tempo. para o convento,
b) lugar. Raimundo morreu de desastre, Maria ficou
c) afirmação. pra tia,
Joaquim suicidou-se e Lili casou com J.
d) modo.
Pinto Fernandes
e) instrumento. que não tinha entrado na história”.
LEIA O TEXTO PARA RESPONDER ÀS Carlos Drummond de Andrade
QUESTÕES 08 E 09.
A primeira parte do poema (versos de 1 a 3) é
CANÇÃO AMIGA marcada, sintaticamente, pela presença de
Eu preparo uma canção orações ____________, cujos termos
Em que minha mãe se reconheça, introdutórios atuam como _______________.
Todas as mães se reconheçam,
E que fale como dois olhos a) subordinadas adjetivas restritivas –
Caminho por uma rua conectivos – sujeitos.
Que passa em muitos países. b) coordenadas sindéticas explicativas –
Se não me veem, eu vejo simples conectivos.
E saúdo velhos amigos. c) subordinadas adverbiais comparativas –
Eu distribuo um segredo simples conectivos.
Como quem ama ou sorri. d) subordinadas adjetivas explicativas –
No jeito mais natural conectivos – sujeitos.
Dois carinhos se procuram e) coordenadas sindéticas aditivas – simples
Minha vida, nossas vidas conectivos.
Formam um só diamante.
12. (PUC) Assinale a alternativa que apresenta um
Aprendi novas palavras
E tornei outras mais belas. período composto onde uma das orações é
subordinada adjetiva
Eu preparo uma canção a) "...a nenhuma pedi ainda que me desse fé:
Que faça acordar os homens pelo contrário, digo a todas como sou"'.
E adormecer as crianças.
b) "Todavia, eu a ninguém escondo os
ANDRADE, Carlos Drumond de. sentimentos que ainda há pouco mostrei".
Poesia completa e prosa. Rio de Janeiro: Aguilar: 1973.
c) "... em toda a parte confesso que sou
08. O termo destacado no último verso é um
volúvel, inconstante e incapaz de amar três
pronome relativo que faz referência aotermo:
dias um mesmo objeto".
a) Canção.
d) "Mas entre nós há sempre uma grande
b) Faço.
diferença; vós enganais e eu desengano."
c) Homens
e) " - Está romântico!... está romântico... -
d) Crianças.
exclamaram os três..."
e) Adormecer
13. (ITA 2002) Tem gente que junta os trapos,
09. Nos versos “Eu distribuo um segredo / como
outros juntam os pedaços. O QUE,
quem ama ou sorri”, a segunda oração
empregado como conectivo, introduz uma
estabelece com a primeira uma relação de:
oração:
a) causa.
a) substantiva.
b) condição.
b) adverbial causal.
c) finalidade.
c) adverbial consecutiva.
d) tempo.
d) adjetiva explicativa.
e) comparação.
50
e) adjetiva restritiva.

14. (UF-PA) Há no período uma oração subordinada


adjetiva:
a) Ele falou que compraria a casa.
b) Não fale alto, que ela pode ouvir.
c) Vamos embora, que o dia está amanhecendo.
d) Em time que ganha não se mexe.
e) Parece que a prova não está difícil.
15. Assinale a alternativa que contém uma oração
subordinada adjetiva restritiva:
a) O professor, que facilita, ajuda os alunos nas
provas.
b) Há coisas que nos emocionam.
c) Ele, que era esperto, entendeu-me.
d) Vitor, que era bom aluno, está ganhando
muito dinheiro.
e) O vulcão, que parecia extinto, voltou a vida.
16. (UNIRIO) Assinale o item em que há uma
oração adjetiva.
a) Perdão, por Deus, perdão - respondeu o
pombo.
b) A pombinha, que era branca sem exagero,
arrulhava, humilhada e ofendida com o atraso.
c) Perdeste a noção do tempo?
d) A tarde era tão bonita que eu tinha de vir
andando.
e) O pombo caminhava pelo beiral mais alto, do
outro lado. Um pouco além, gritavam as
gaivotas

51
CONCORDÂNCIA Os Estados Unidos possuem grandes
universidades.
Concordância é o mecanismo através do qual as
Alagoas impressiona pela beleza das praias.
palavras alteram suas terminações para se
As Minas Gerais são inesquecíveis.
adequarem harmonicamente umas às outras na
Minas Gerais produz queijo e poesia de primeira.
frase.
Há dois tipos de concordância: 4) Quando o sujeito é formado por uma
 Concordância verbal: se estabelece entre o verbo expressão que indica porcentagem seguida de
(em suas flexões de número e pessoa) e o sujeito substantivo, o verbo deve concordar com o
da oração com o qual se relaciona. substantivo.
 Concordância nominal: os determinantes Exemplos:
(adjetivos, artigos, numerais adjetivos e pronomes 25% do orçamento do país deve destinar-se à
adjetivos) alteram suas desinências para se Educação.
ajustarem em número e gênero ao determinado 85% dos entrevistados não aprovam a
(substantivo) a que se referem. administração do prefeito.
1- CONCORDÂNCIA VERBAL Quando a expressão que indica porcentagem
não é seguida de substantivo, o verbo deve
REGRA GERAL: o verbo e o núcleo sujeito devem
concordar com o número. Veja:
concordar em número e pessoa.
25% querem a mudança.
Sujeito Verbo 1% conhece o assunto.
O aluno chegou.
5) Quando o sujeito é o pronome relativo
Os alunos chegaram.
"que", a concordância em número e pessoa é
Casos particulares de concordância verbal
feita com o antecedente do pronome.
a) SUJEITO SIMPLES
Exemplos:
1) Quando o sujeito é formado por
Fui eu que paguei a conta.
uma expressão partitiva (parte de, uma porção
Fomos nós que pintamos o muro.
de, o grosso de, metade de, a maioria de, a maior
parte de, grande parte de...) seguida de um 6) Quando o sujeito é pronome relativo “quem”,
substantivo ou pronome no plural, o verbo pode o verbo pode concordar com o antecedente do
ficar no singular ou no plural. pronome ou com o próprio pronome. (3ª
Por Exemplo: pessoa do singular)
A maioria dos jornalistas aprovou / aprovaram a Exemplos:
ideia. Fui eu quem pintei esse quadro.
Metade dos candidatos não apresentou / Fui eu quem pintou esse quadro.
apresentaram nenhuma proposta interessante. 7) Verbos Impessoais: por não se referirem a
Esse mesmo procedimento pode se aplicar aos nenhum sujeito, são usados sempre na 3ª
casos dos coletivos, quando especificados: pessoa do singular. São verbos impessoais:
Por Exemplo: Haver no sentido de existir;
Um bando de vândalos destruiu / destruíram o Fazer indicando tempo;
monumento. Aqueles que indicam fenômenos da natureza.
2)Quando o sujeito é formado por expressão que Exemplos:
indica quantidade aproximada(cerca de, mais de, Havia muitas garotas na festa.
menos de, perto de...) seguida de numeral e Faz dois meses que não vejo meu pai.
substantivo, o verbo concorda com o Chovia ontem à tarde.
substantivo. Observe: 8) A concordância dos verbos bater,
Cerca de mil pessoas participaram da manifestação. dar e soar se dá de acordo com o numeral.
Perto de quinhentos alunos compareceram à Por Exemplo:
solenidade. Deu uma hora no relógio da sala.
Mais de um atleta estabeleceu novo recorde nas Deram cinco horas no relógio da sala.
últimas Olimpíadas. Obs.: caso o sujeito da oração seja a palavra
Obs.: quando a expressão "mais de um" se relógio, sino, torre, etc., o verbo concordará
associar a verbos que exprimem reciprocidade, com esse sujeito.
o plural é obrigatório: Por Exemplo:
Por Exemplo: O tradicional relógio da praça matriz dá nove
Mais de um colega se ofenderam na tumultuada horas.
discussão de ontem. (ofenderam um ao outro) b) SUJEITO COMPOSTO
3)Quando se trata de nomes que só existem no 1) Quando o sujeito é composto e anteposto ao
plural, a concordância deve ser feita levando-se verbo, a concordância se faz no plural:
em conta a ausência ou presença de artigo. Sem Exemplos:
artigo, o verbo deve ficar no singular. Quando Pai e filho conversavam longamente.
há artigo no plural, o verbo deve ficar o plural. Sujeito
Exemplos:

52
2) No caso do sujeito composto posposto ao verbo, Construiu-se um posto de saúde.
passa a existir uma nova possibilidade de concordância: Construíram-se novos postos de saúde.
em vez de concordar no plural com a totalidade do Não se pouparam esforços para despoluir o rio.
sujeito, o verbo pode estabelecer concordância com O Verbo "Ser"
o núcleo do sujeito mais próximo. Convém insistir
A concordância verbal se dá sempre entre o
que isso é uma opção, e não uma obrigação.
verbo e o sujeito da oração. No caso do verbo
Por Exemplo:
ser, essa concordância pode ocorrer também
Faltaram coragem e competência.
entre o verbo e o predicativo do sujeito.
Faltou coragem e competência.
O verbo ser concordará com o predicativo do
3) Quando ocorre ideia de reciprocidade, no entanto, a sujeito:
concordância é feita obrigatoriamente
a) Quando o sujeito for representado pelos
no plural. Observe:
pronomes -isto, isso, aquilo, tudo, o - e o
Abraçaram-se vencedor e vencido. predicativo estiver no plural.
Casos Particulares Exemplos:
1) Quando os núcleos do sujeito composto são unidos Isso são lembranças inesquecíveis.
por "ou" ou "nem", o verbo deverá ficar no plural se a Aquilo eram problemas gravíssimos.
declaração contida no predicado puder ser atribuída a
O que eu admiro em você são os seus cabelos
todos os núcleos.
compridos.
Por Exemplo:
b) Quando o sujeito estiver no singular e se
Nem o professor nem o aluno acertaram a resposta.
referir a coisas, e o predicativo for um
Quando a declaração contida no predicado só puder substantivo no plural.
ser atribuída a um dos núcleos do sujeito, ou seja, se Exemplos:
os núcleos forem excludentes, o verbo deverá ficar Nosso piquenique foram só guloseimas.
no singular.
Sujeito Predicativo do Sujeito
Por Exemplo:
Você ou ele será escolhido. (Só será escolhido um) Sua rotina eram só alegrias.
2) Com as expressões "um ou outro" e "nem um nem Sujeito Predicativo do Sujeito
outro", a concordância costuma ser feita no singular, Se o sujeito indicar pessoa, o verbo concorda
embora o plural também seja praticado. com esse sujeito.
Por Exemplo: Por Exemplo:
Um e outro compareceu / compareceram à festa. Gustavo era só decepções.
Nem um nem outro saiu / saíram do colégio. Minha filha ´só alegria e orgulho.
3)Quando os elementos de um sujeito composto são c)Como impessoal na indicação de horas, dias e
resumidos por um aposto recapitulativo, a concordância distâncias, o verbo ser concorda com o numeral.
é feita com esse termo resumidor. Exemplos:
Por Exemplo: É uma hora.
Filmes, novelas, boas conversas, nada o tirava São três da manhã.
da apatia. Trabalho, diversão, descanso, tudo é Eram 25 de julho quando partimos.
muito importante na vida das pessoas. Daqui até a padaria são dois quarteirões.
O Verbo e a Palavra "SE" Saiba que:
Dentre as diversas funções exercidas pelo "se", d)Na indicação de dia, o verbo ser admite as
há duas de particular interesse para a seguintes concordâncias:
concordância verbal: 1) No singular: Concorda com a palavra explícita
a) quando é índice de indeterminação do sujeito; dia.
b) b)quando é partícula apassivadora. Por Exemplo:
Quando índice de indeterminação do sujeito, o Hoje é dia quatro de março.
"se" acompanha os verbos intransitivos, 2) No plural: Concorda com o numeral, sem a
transitivos indiretos e de ligação, que palavra explícita dia.
obrigatoriamente são conjugados na terceira Por Exemplo:
pessoa do singular. Hoje são quatro de março.
Exemplos: 3) No singular: Concorda com a ideia implícita
Vive-se no Piauí. de dia.
Precisa-se de governantes interessados em civilizar Por Exemplo:
o país. Hoje é quatro de março.
Confia-se em teses absurdas. e)Quando o sujeito indicar peso, medida,
Era-se feliz naquele tempo. quantidade e for seguido de palavras ou
Quando pronome apassivador, o "se" expressões como pouco, muito, menos de, mais
acompanha verbos transitivos diretos (VTD) e de, etc., o verbo ser fica no singular.
transitivos diretos e indiretos (VTDI) na formação Exemplos:
da voz passiva sintética. Nesse caso, o verbo Cinco quilos de açúcar é mais do que preciso.
deve concordar com o sujeito da oração. Três metros de tecido é pouco para fazer seu
Exemplos: vestido.
53
PROFESSOR RESPONDE b) Há muitos casos em que o sujeito simples é
constituído de formas que fazem o falante
01. (IFMA – 2017) Observe o texto das figuras
hesitar no momento de estabelecer a
abaixo, analise o que se afirma sobre os
concordância com o verbo. Às vezes, a
textos 1,2 e 3, em seguida marque a opção
concordância puramente gramatical é
cuja afirmação esteja CORRETA:
contaminada pelo significado de expressões
que nos transmitem noção de plural, apesar
de terem forma de singular ou vice-versa.
c) Quando o sujeito é formado por expressão
que indica quantidade aproximada (cerca de,
mais de, menos de, perto de...) seguida de
numeral e substantivo, o verbo concorda com
o substantivo. Exemplo: Cerca de cem
pessoas foram detidas na manifestação
contra o aumento da tarifa do ônibus.
d) Quando a expressão "mais de um” associar-
se a verbos que exprimem reciprocidade, o
plural não é obrigatório. Exemplo: Mais de um
manifestante ofendeu-se na tumultuada
discussão de ontem. (ofenderam um ao
outro).
e) A partícula “SE”, como índice de
indeterminação do sujeito, faz com que os
verbos associados a ela sejam conjugados na
3ª pessoa do singular. Exemplo: Precisa-se
de vendedores.
LEIA A TIRINHA PARA RESPONDER À
QUESTÃO 03.

a) Apenas o texto 1 apresenta sentença em


desacordo com a norma padrão, pois o verbo
“gerar” deveria estar no singular, concordando
com o sujeito exportação.
b) Apenas o texto 2 apresenta sentença de acordo
com a norma padrão, pois a forma verbal
“fazem”, acompanhada do pronome se
(apassivador), concorda com o sujeito
armaduras.
c) Apenas o texto 3 apresenta sentença de acordo
com a norma padrão, pois a forma verbal
“fazem”, na 3ª pessoa o plural, indica
indeterminação do sujeito e espelhos é objeto
03. Considerando as regras de concordância
direto do referido verbo.
nominal e verbal, de acordo com a norma-
d) Apenas o texto 1 apresenta sentença de acordo
padrão da língua portuguesa, assinale a
com a norma padrão, pois a forma verbal
alternativa correta.
“geram” concorda com as formas nominais
a) A comunicação e a confiança dos filhos serão
minérios e grãos.
aumentadas se os pais responderem às
e) Todos os textos das figuras apresentam
perguntas feitas por eles com clareza e
sentenças em desacordo com a norma padrão.
simplicidade.
02. Sobre as características da concordância verbal, b) A comunicação e a confiança dos filhos será
é incorreto afirmar: aumentadas se os pais responderem às
a) Concordância é a correspondência de flexão perguntas feitas por eles com clareza e
entre dois termos. Ocorre quando o verbo simplicidade.
flexiona-se para concordar com o seu sujeito.

54
c) A comunicação e a confiança dos filhos será
02. Já .......... anos, ............ neste local árvores e
aumentada se os pais responderem às perguntas
flores. Hoje, só ... ervas daninhas.
feitas por eles com clareza e simplicidade.
A alternativa que preenche corretamente os
d) A comunicação e a confiança dos filhos serão
espaços é:
aumentada se os pais responderem às perguntas
a) fazem/havia/existe
feitas por eles com clareza e simplicidade.
b) fazem/havia/existe
e) A comunicação e a confiança dos filhos serão
c) fazem/haviam/existem
aumentadas se os pais responderem às perguntas
d) faz/havia/existem
feita por eles com clareza e simplicidade.
e) faz/havia/existe
04. (IFPI – 2014). Assinale a única alternativa cuja
concordância verbal FERE as normas da 03. (FCC) Assinale a alternativa em que a
modalidade escrita- formal da língua. concordância verbal está de acordo com a
a) Eu sou abraços. norma padrão da língua.
b) Eu sou sorrisos. a) Haviam cooperativas de catadores na cidade
c) Nós somos um só de São Paulo.
d) O português são dois. b) O lixo de casas e condomínios vão para
e) É duas pessoas inteligentes. aterros.
05. (IFMA – 2017) Considerando o trecho destacado c) O tratamento e a destinação corretos do lixo
a seguir e tomando como referência a norma evitaria que 35% deles fosse despejado em
padrão, identifique as afirmações CORRETAS: aterros.
Minimizando sua importância, grande d) Fazem dois anos que a prefeitura adia a
parte dos trabalhadores usa os questão do lixo.
equipamentos de segurança obrigada e) Somos nós quem paga a conta pelo descaso
pelas leis e pelas empresas. com a coleta de lixo.
I. A forma usa está corretamente empregada no 04. Quanto à concordância do verbo SER, a
singular, pois concorda com a expressão língua padrão desaprova a seguinte
partitiva grande parte de, que se apresenta no construção:
singular. a) Ontem foram sete de janeiro.
II. A forma obrigada, no feminino, está correta, b) Todos fomos muito distraídos.
pois concorda com a expressão feminina c) Era duas e meia da manhã e chovia.
grande parte de. d) Tuas alegrias somos nós, teus filhos.
III. A concordância verbal está incorreta, pois o e) Na infância, tudo são mimos.
sujeito grande parte dos trabalhadores exige
05. LEIA:
que o verbo seja, necessariamente, flexionado
No século 19, a elite das cidades .............
no plural.
organizar saraus em suas casas, momentos
IV. A forma obrigada está incorreta, pois, como
em que as moças, em sua maioria, ..............
se refere a trabalhadores (plural), deveria ser
suas habilidades no piano, já que ainda não
flexionada no masculino e no plural: obrigados.
................. aparelhos eletrônicos.
V. A forma verbal poderia ser usam (plural),
Preenchem corretamente as lacunas da
concordando com trabalhadores, mas, nesse
frase acima, na ordem dada:
caso, seria recomendável substituir obrigada
a) costumava - demonstravam – havia.
por obrigados.
b) costumavam - demonstrava – haviam.
a) As afirmações II, III e V estão corretas.
c) costumava - demonstrava – haviam.
b) As afirmações I, III e V estão corretas.
d) costumavam - demonstravam - havia.
c) As afirmações III e IV estão corretas.
e) costumava - demonstrava – havia.
d) As afirmações I, II e V estão corretas.
e) Todas as afirmações estão corretas. 06. OBSERVE:
I. Os Estados Unidos não divulgou a notícia.
ALUNO RESPONDE II. Não fomos nós que atrapalhamos a
reunião.
01. (IFMA – 2017) Marque a ÚNICA frase em que a III. A maioria das questões apresentava
concordância verbal foge da norma padrão da dificuldades.
Língua Portuguesa. IV. O aluno, o professor, funcionário, ninguém
a) Há dois anos os Estados Unidos invadiram a viu o diretor.
Líbia. Em relação à concordância verbal, está(ão) de
b) Promove-se aulas de revisão para alunos do acordo com o padrão culto da língua:
ensino médio. a) apenas I
c) Fui eu quem resolveu todas as atividades. b) I, II
d) A maioria dos jornalistas aprovou a ideia. c) I, III, IV
e) João considerou bastante difíceis as questões d) II, III, IV
do simulado. e) todas
55
12. Assinale a alternativa que apresenta
07. (UFPR) - Qual a alternativa em que as formas dos
concordância em conformidade com a norma
verbos bater, consertar e haver nas frases abaixo,
padrão.
são usadas na concordância correta?
a) Soava seis horas no relógio da matriz
- As aulas começam quando ... oito horas.
quando eles chegaram.
- Nessa loja ... relógios de parede.
b) Apesar da greve, diretores, professores,
- Ontem ... ótimos programas na televisão.
funcionários, ninguém foram demitidos.
a) batem - consertam-se - houve
c) José chegou ileso a seu destino, embora
b) bate - consertam-se - havia
houvessem muitas ciladas em seu
c) bateram - conserta-se - houveram
caminho.
d) batiam - conserta-se-ão - haverá
d) Fomos nós quem resolvemos aquela
e) batem - consertarei – haviam
questão.
08. OBSERVE A CONCORDÂNCIA VERBAL. e) O impetrante referiu-se aos artigos 37 e 38
I. O público aprovou as feiras que se que ampara sua petição.
promoveram este ano.
13. (FUVEST) Num dos períodos seguintes não se
II. O público aprovou as feiras que ocorreram
observa a concordância prescrita pela
este ano.
gramática. Indique-o:
III. O público aprovou as feiras que houveram
a) Não se apanham moscas com vinagre.
este ano.
b) Casamento e mortalha no céu se talha.
Quais frases apresentam concordância em
c) Quem ama o feio, bonito lhe parece.
conformidade com a norma padrão da língua?
d) De boas ceias, as sepulturas estão cheias.
a) Apenas I.
e) Quem cabras não tem e cabritos vende, de
b) Apenas II.
algum lugar lhe vêm
c) Apenas III.
d) Apenas I e II. 14. Levando em consideração a norma padrão
e) I, II e III. da língua, marque a alternativa correta:
a) Sou eu que primeiro saio.
09. (UFPE) Marque a alternativa em que a
b) É cinco horas da tarde.
concordância verbal contraria a norma culta:
c) Da cidade à praia é dois quilômetros.
a) Ouviram-se as notícias mais desencontradas.
d) Dois metros de tecido são pouco para o
b) Trata-se de questões muito sérias.
terno.
c) Faziam anos que o país não escolhia
e) Fui eu que resolveu o problema.
democraticamente o presidente.
d) Poderá haver comentários positivos quanto à 15. Apenas uma alternativa não contraria o padrão
eleição. culto da língua. Marque-a.
e) Deveriam existir situações menos a) Nesta casa, consertam-se televisores e
constrangedoras. precisa-se de técnicos em eletrônica.
b) Nesta casa, conserta-se televisores e
10. (PUC) Levando-se em conta a NORMA CULTA da
precisam-se de técnicos em eletrônica.
língua, assinale a opção na qual uma das frases
c) Nesta casa, conserta-se televisores e
apresenta uma concordância verbal
precisa-se de técnicos em eletrônica.
INACEITÁVEL.
d) Nesta casa, consertam-se televisores e
a) Já é uma hora da tarde. - Já são duas horas da
precisam-se de técnicos em eletrônica
tarde.
e) Nesta casa, é consertado televisores e
b) Deve haver alguma solução para o caso. - Deve
precisa-se de técnicos em eletrônica
haver várias soluções para o caso.
c) Falta um dia para o início das aulas. - Faltam 16. Neste período “Kelly afirma que há uma
dez dias para o início das aulas. resistência de mulheres na ciência.”, O verbo
d) A maioria dos alunos não reclamou do calor. haver é impessoal e está usado de acordo com
- A maioria dos alunos não reclamaram do as regras de concordância verbal da língua
calor. portuguesa.
e) Apareceu ao longe um vulto assustador. - Identifique a alternativa em que a mesma
Apareceu ao longe vultos assustadores. situação acontece.
a)Já houveram várias pesquisadoras
11. A alternativa que contraria as normas de importantes na história da computação.
concordância prescritas pela gramática é: b) Deve haver várias pesquisadoras
a) A maioria dos estudiosos acha difícil uma importantes na história da computação.
solução para o problema. c) Haviam várias pesquisadoras importantes na
b) A maioria os conflitos foram resolvidos. história da computação.
c) Deve haver bons motivos para a sua recusa. d) Haverão várias pesquisadoras importantes
d) De casa à escola é três quilômetros. na história da computação.
e) Nem uma nem outra questão é difícil. e) Poderão haver várias pesquisadoras
importantes na história da computação.
56
CONCORDÂNCIA NOMINAL Obs.: quando a palavra "só" equivale a
"somente" ou "apenas", tem função
REGRA GERAL: os determinantes (adjetivos,
adverbial, ficando, portanto, invariável.
artigos, numerais adjetivos e pronomes adjetivos)
Por Exemplo:
alteram suas desinências para se ajustarem em
Eles só desejam ganhar presentes.
número e gênero ao determinado (substantivo) a
Casos Particulares
que se referem.
1) O adjetivo concorda em gênero e número quando se É proibido - É necessário - É bom - É preciso
refere a um único substantivo. - É permitido
Por Exemplo: a)Essas expressões, formadas por um verbo
As mãos trêmulas denunciavam o que sentia. mais um adjetivo, ficam invariáveis se o
2) Quando o adjetivo se refere a vários substantivos, a substantivo a que se referem possuir sentido
concordância pode variar. Podemos sistematizar essa genérico (não vier precedido de artigo).
flexão nos seguintes casos: Exemplos:
a)Adjetivo anteposto aos substantivos: É proibido entrada de crianças.
- O adjetivo concorda em gênero e número com o Em certos momentos, é necessário atenção.
substantivo mais próximo. No verão, melancia é bom.
Por Exemplo: b)Quando o sujeito dessas expressões estiver
Encontramos caídas as roupas e os prendedores. determinado por artigos, pronomes ou adjetivos,
Encontramos caída a roupa e os prendedores. tanto o verbo como o adjetivo concordam com
Encontramos caído o prendedor e a roupa. ele.
b)Adjetivo posposto aos substantivos: Exemplos:
- O adjetivo concorda com o substantivo mais É proibida a entrada de crianças.
próximo ou com todos eles (assumindo forma Esta salada é ótima.
masculino plural se houver substantivo feminino e A educação é necessária.
masculino). Anexo - Obrigado - Mesmo - Próprio - Incluso -
Exemplos: A indústria oferece localização e Quite
atendimento perfeito.
Essas palavras adjetivas concordam em gênero
A indústria oferece atendimento e localização
e número com o substantivo ou pronome a que
perfeita.
se referem. Observe:
A indústria oferece localização e atendimento
Seguem anexas as documentações requeridas.
perfeitos.
A menina agradeceu: - Muito obrigada.
A indústria oferece atendimento e localização
Muito obrigadas, disseram as senhoras, nós
perfeitos.
mesmas faremos isso.
Obs.: os dois últimos exemplos apresentam
Seguem inclusos os papéis solicitados.
maior clareza, pois indicam que o adjetivo
Já lhe paguei o que estava devendo: estamos
efetivamente se refere aos dois substantivos.
quites.
Nesses casos, o adjetivo foi flexionado no plural
masculino, que é o gênero predominante Bastante - Caro - Barato - Longe
quando há substantivos de gêneros diferentes. Essas palavras são invariáveis quando
- Se os substantivos possuírem o mesmo gênero, o funcionam como advérbios. Concordam com o
adjetivo fica no singular ou plural. nome a que se referem quando funcionam como
Exemplos: adjetivos, pronomes adjetivos, ou numerais.
A beleza e a inteligência feminina(s). Exemplos:
3) Quando um único substantivo é modificado por dois As jogadoras estavam bastante cansadas.
ou mais adjetivos no singular, podem ser usadas as (advérbio)
construções: Há bastantes pessoas insatisfeitas com o
a) O substantivo permanece no singular e coloca-se trabalho. (pronome adjetivo)
o artigo antes do último adjetivo. Nunca pensei que o estudo fosse tão caro.
Por Exemplo: (advérbio)
Admiro a cultura espanhola e a portuguesa. As casas estão caras. (adjetivo)
b) O substantivo vai para o plural e omite-se o artigo Achei barato este casaco.(advérbio)
antes do adjetivo. Hoje as frutas estão baratas. (adjetivo)
Por Exemplo: "Vais ficando longe de mim como o sono, nas
Admiro as culturas espanhola e portuguesa. alvoradas." (Cecília Meireles) (advérbio)
"Levai-me a esses longes verdes, cavalos de
4) A palavra "só", quando equivale a "sozinho", tem vento!" (Cecília Meireles). (adjetivo)
função adjetiva e concorda normalmente com o nome a
que se refere Meio - Meia
Por Exemplo: a)A palavra "meio", quando empregada como
Cristina saiu só. adjetivo, concorda normalmente com o nome a
Cristina e Débora saíram sós. que se refere.

57
Por Exemplo: sintaxe entre os interlocutores é chamado de coesão
Pedi meia cerveja e meia porção de polentas. textual. Ela não só está comprometida com a
b)Quando empregada como advérbio (modificando estrutura do texto, isto é, a ligação entre os termos e
um adjetivo) permanece invariável. as frases, como também com a semântica, ou seja, o
sentido que advém dessa estrutura e que é atribuído
Por Exemplo:
pelos interlocutores.
A noiva está meio nervosa.
MELO, Iran Ferreira de. Incoerente, eu? Uma reflexão sobre
Alerta - Menos coerência e coesão textuais. Revista Conhecimento Prático: Língua
portuguesa. São Paulo: Escala, n. 16,jan. 2009. p. 8-11. (Adaptado).
Essas palavras são advérbios, portanto,
permanecem sempre invariáveis. 01. De acordo com o texto, para que a coerência
Por Exemplo: textual se estabeleça, é necessário,
Os escoteiros estão sempre alerta. sobretudo, o(a)
Carolina tem menos bonecas que sua amiga. a) conhecimento individual
b) diálogo entre os interlocutores
PROFESSOR RESPONDE
c) aspecto linguístico
(CESGRANRIO) d) fator intuitivo
INCOERENTE, EU? e) construção sintática
Uma reflexão sobre coerência e coesão textuais 02. O elemento coesivo Isso (5°parágrago) tem
Você já escreveu ou falou alguma coisa que foi como referente a ideia de que
considerada incoerente por outra pessoa? Não? Então,
a) a coerência independe da compreensão de
vamos reformular a pergunta: você já escreveu ou falou
alguma coisa que foi entendida de maneira diferente da certos aspectos linguísticos.
que você gostaria que entendessem? b) o conhecimento sobre o assunto é
E aí? Mudou de opinião? fundamental à construção da coerência.
Pois é, que atire o primeiro dicionário quem nunca foi c) o puro conhecimento sociocognitivo constitui
interpretado de maneira diferente daquilo que quis os sentidos do texto.
veicular. Seja por causa da falta de informação ou do seu d) os sentidos de um texto são construídos por
excesso; seja pelo fato de a mensagem não possuir um processo de troca.
elementos contextualizadores suficientes, como título, e) os sentidos não se constroem caso não se
autoria, assinatura (no caso do escrito) ou gestos, olhares, compreenda o código linguístico.
entoação (no caso do falado); ou, ainda, seja porque o
03. No que se refere ao fenômeno da
conhecimento do conteúdo veiculado não era partilhado
suficientemente com o interlocutor (leitor ou ouvinte). concordância nominal, no subtítulo do texto, o
Todas essas razões nos fazem pensar que, quando termo textuais também admite a forma
chamamos um texto de incoerente, estamos nos referindo singular. O período em que, conforme a
à não ativação de elementos necessários para que tanto o norma-padrão, o termo destacado pode
falante/ escritor como o ouvinte/leitor atribuam sentido. A assumir tanto a forma singular quanto a plural
escola nos ajudou a pensar assim? é:
Vários pedagogos e estudiosos da educação têm a) Bastantes poemas foram lidos na aula.
relatado que o ensino de Língua portuguesa, por muito b) Custam caro os jornais de domingo.
tempo, se posicionou sobre o assunto de modo bastante c) Vendem-se quadros e esculturas usados.
negligente, não abordando os motivos empíricos que
d) Compramos livro e jornal velhos.
fazem com que os textos possam ser considerados
incoerentes. Quem não se recorda de algum professor que e) Na estante, dicionário e livros jogados.
tenha devolvido ao aluno seu texto escrito com uma cruz 04. Observe o emprego das palavras destacadas
enorme em vermelho acompanhada da frase “Seu texto nas frases a seguir.
está incoerente”? Muitas vezes, nessas situações, o aluno  Quando elas dirigem, ficam meio nervosas.
recebe a correção, mas não chegam a ele as orientações  As crianças estavam sós em casa.
para entender o que pode melhorar no texto e o que faz  Ela mesma se dirigiu ao centro.
dele incoerente. [...]  Os carros custam caro.
A coerência de um texto depende majoritariamente Acerca das regras de concordância que justificam
da troca de informações entre os interlocutores, muito o emprego dos termos anteriores, analise.
mais do que a construção sintática que possui, assim
I. A palavra “meio” é um advérbio, razão pela
como a atribuição de coerência está ligada diretamente
aos nossos conhecimentos sobre o assunto. No entanto, o qual não se flexionou.
puro conhecimento sociocognitivo não é suficiente se não II. A palavra “sós” é um adjetivo, por isso
apreendemos os aspectos estritamente linguísticos. Caso concorda com o sujeito.
o leitor não compreenda o código ali colocado, a coerência III. A palavra “mesma” sempre concorda com o
não se constituirá. Isso pode ocorrer quando há alguma substantivo e o pronome a que se refere.
expressão no texto de uma língua diferente daquela usada IV. A palavra “caro” é um advérbio, razão pela
pelo leitor, como o latim (ad hoc), o francês (déjà vu), ou o qual não se flexionou.
inglês (mainstream). Ou, ainda, quando o registro é Estão corretas:
extremamente específico de uma área, como os famosos a) I, II, III e IV.
jargões técnicos: vocabulários jurídico, médico etc.
b) I, II e IV, apenas.
Além do conhecimento das palavras, a relação
sintática também é de suma importância. O c) I, II e III, apenas.
estabelecimento da mútua compreensão sobre a d) I, III e IV, apenas.

58
e) II, III e IV, apenas. Levando-se em consideração a norma padrão
da língua:
05. (IFPI – 2014.1) Observe os avisos:
a) somente a frase 1 está correta.
AVISO 1: É PROIBIDO entrada de pessoas b) somente a frase 2 está correta.
estranhas. c) as frases 1 e 2 estão corretas
d) as frases 2 e 3 estão corretas.
AVISO 2: É expressamente PROIBIDA a e) as frases 1, 2 e 3 estão corretas.
entrada de animais domésticos.
04. Indique a alternativa que respeita a
concordância nominal prescrita pela norma
AVISO 3: A água é BOM para a saúde.
padrão da língua
a) Não é permitida a presença de menores
A concordância nominal está adequada à neste recinto.
gramática normativa: b) Muito obrigado, disse-me ela sorrindo.
a) Nos avisos I e II. c) Infelizmente temos motivos bastante para
b) Nos avisos I e III. duvidar de você.
c) Nos avisos II e III. d) Limonada é muito boa para refrescar.
d) Apenas no aviso II. e) Remetemos anexo ao recibo uma fotocópia
e) Apenas no aviso III. da conta de luz.
05. Considerando-se que há palavras variáveis e
ALUNO RESPONDE palavras invariáveis na língua portuguesa, qual
é a frase que está em DESACORDO com a
Observe a tirinha e responda à questão norma-padrão, no que diz respeito à
concordância?
a) Estamos todos alerta em relação ao
problema dos menores de rua.
b) A população está meio descrente em
relação a soluções de curto prazo.
c) As organizações que cuidam das crianças
receberam bastantes recursos este ano.
d) A partir de hoje, é proibido a adoção de
crianças que tenham pais biológicos vivos.
e) No caso de crianças sob maus tratos, muitas
vezes, elas próprias fogem para as ruas
06. Considerando a concordância nominal da
01. (IFMA – 2017) O cartaz colado por Garfield norma padrão, assinale a frase correta:
apresenta concordância correta de acordo com a) Ela mesmo confirmou a realização do
a norma padrão da Língua Portuguesa. Em qual encontro.
das alternativas abaixo a concordância NÃO b) Foi muito criticado pelos jornais a reedição
ocorre? da obra.
a) É proibido a entrada neste recinto em trajes de c) Ela ficou meia preocupada com a notícia.
banho. d) Muito obrigada, querido, falou-me
b) É proibido entrada de animais durante a vistoria emocionada.
do local. e) Anexo, remeto-lhes nossas últimas
c) É permitido o acesso ao salão até as 22h. fotografias.
d) É restrita a circulação nos corredores durante a
07. A frase em que a concordância nominal
madrugada.
contraria a norma culta é:
e) Sensibilidade é bom!
a) O poeta considera ingrata a terra e o filho.
02. CESGRANRIO - adaptada) – Há concordância
b) O poeta considera ingrato o filho e a terra.
inadequada, no que diz respeito ao padrão culto
c) O poeta fala de um filho e uma terra
da língua, em:
ingratas.
a) clima e terras desconhecidas.
d) O poeta fala de uma terra e um filho
b) clima e terra desconhecidos.
ingratos.
c) terras e clima desconhecidas.
e) O poeta fala de um filho e uma terra ingrata.
d) terras e clima desconhecido.
e) terras e clima desconhecidos.

03. Observe a concordância nestas frases:


1. A sala está meio escura.
2. Eles estavam alerta.
3. Minha farda está menas suja que a sua.

59
08. Assinale a alternativa que completa as lacunas da 13. Assinale a sequência que completa
frase abaixo, levando-se em consideração a adequadamente, de acordo com a norma padrão
concordância nominal prescrita pela gramática da língua, os espaços abaixo.
normativa: I. Segue a documentação___________.
“É ________ discussão entre homens e II. Pedro está__________com a tesouraria.
mulheres ________ ao mesmo ideal, pois já III. Os vigias estão sempre_________.
se disse ________ vezes que da discussão, IV. Maria estava_________encabulada.
ainda que ________ acalorada, nasce a luz”. a) anexo, quites, alerta, meio
a) bom - voltados - bastantes - meio. b) anexo, quites, alertas, meia
b) bom - voltadas - bastante - meia. c) anexa, quite, alerta, meio
c) boa - voltadas - bastantes - meio. d) anexa, quite, alertas, meio
d) boa - voltados - bastante - meia. e) anexo, quite, alerta, meia
e) bom - voltadas - bastantes – meia 14. Coloque A ou D nos parênteses, conforme a
09. A opção que atende às regras impostas pela concordância nominal esteja em acordo ou
gramática no que diz respeito à concordância desacordo com o padrão formal da língua.
nominal é
a) Seguem anexo os formulários pedidos. ( ) Barcaça e veleiro novos.
b) Não vou comprar esta camisa. Ela está muito ( ) Barcaças e veleiro novos.
caro. ( ) Veleiro e barcaça novo.
c) Estas questões são bastantes difíceis. Marque a sequência correta.
d) Eu lhes peço que as deixem sós. a) A – A – A.
e) Estando pronto os preparativos para o início da b) A – A – D.
corrida, foi dada a largada. c) A – D – D
10. Assinale a alternativa em que, pluralizando-se a d) D – A – D.
frase, as palavras destacadas permanecem e) D – D –A.
invariáveis: 15. Na frase: “A madrugada era escura nas
a) Este é o meio mais exato para você resolver o moitas de mangue, baixas, meio trêmulas do
problema: estude só. ventinho frio”, a palavra meio apresenta-se
b) Meia palavra, meio tom – índice de sua sob essa forma flexional porque:
sensatez. a) é um caso de adjetivo que vem antes de
c) Estava só naquela ocasião; acreditei, pois em vários substantivos, concordando com o
sua meia promessa. mais próximo.
d) Passei muito inverno só. b) concorda com ventinho frio.
e) Só estudei o elementar, o que me deixa meio c) funciona como advérbio, com valor de um
apreensivo. pouco, sendo, portanto, invariável.
11. Assinale a alternativa que indique a ordem que d) a concordância se dá com a ideia que a
preenche corretamente as lacunas: palavra moita encerra -grupo de planta
I. Justiça entre os homens é ______. e) se refere a mangue.
II. __________ entrada de estranhos.
III. Água é ____________. 16. Marque a frase inaceitável, do ponto de vista
a) necessário – proibido – gostoso. da concordância:
b) necessária – proibida – gostosa.
c) necessário – proibida – gostoso. a) É necessária paciência.
d) necessária – proibido – gostosa. b) Não é bonito defendermos aos outros.
e) necessários – proibida – gostoso. c) Cerveja é bom para desestressar.
12. (PUC - adaptada) Assinale a sequência que d) Não é permitido presença de estranhos.
completa estes períodos: e) Água de Melissa é ótimo para os nervos.
I. Ela _________ disse que não iria.
II. Vão ________ os livros.
III. A moça estava _________ aborrecida. REFERÊNCIAS
IV. É _________ atenção para atravessar a rua. TRAVALHA, Márcia. Português: trilhas e tramas. 2.ed. -
a) mesmo - anexos - meia - necessário São Paulo: Leya, 2016
b) mesma - anexos - meio - necessária TERRA, Ernani. Português: de olho no mundo do
trabalho 2 ed. – São Paulo: Scipione, 2008.
c) mesmo - anexo - meio - necessário AMARAL, Emília. Novas Palavras 2 ed. – São Paulo: FTD,
d)mesma - anexos - meio - necessário 2005
e) mesma - anexos - meia – necessário

60
ORTOGRAFIA d) após a sílaba me: mexicana, mexer.
Obs.: em palavras que derivam de primitivas
Alfabeto
grafadas com ch conserva-se o dígrafo, p.ex.:
O alfabeto da língua portuguesa é composto por
cheio – enchente.
vinte e seis letras, a saber: a, b, c, d, e, f, g, h, i, j, k,
O vocábulo “mecha” é uma exceção da regra da
l, m, n, o, p, q, r, s, t, u, v, w, x, y, z.
letra “d”.
Uso de K, W, Y
Letra “g”
a) abreviaturas e símbolos do Sistema Internacional:
a) vocábulos formados pelo sufixo - gem: viagem,
K= Potássio; Km= Quilômetro; W= Oeste; w= Watts;
miragem, ferrugem. “viadagem”, molecagem.
yd= Jarda.
b) palavras terminadas em -ágio, -égio, -ígio, -
b) Nomes próprios estrangeiros e seus derivados
ógio, -úgio: estágio, privilégio, prestígio, relógio,
portugueses: Kafka – kafkiano; Wagner –
refúgio.
wagneriano; Byron – byroniano.
c) palavras derivadas de formas primitivas
Obs.: a letra k pode ser substituída por c e qu (antes
grafadas com -g: gesso – engessar; ungir –
de -e e -i), p.ex.: Whisky – Uísque; ou uma
ungido.
consoante como /v/ em palavras de origem alemã,
Obs.: As palavras “pajem”, “lajem” e
p.ex.: Walter.
“lambujem” são exceções que não se aplicam
Uso do H
às exigências da primeira regra.
a) início de palavras que conservaram-se
Emprego do por que, porque, por quê e porquê
graficamente ao longo do tempo.
a) por que: é a ocorrência da preposição “por”
Ex.: hidrogênio, homem.
mais
b) no início e no fim de algumas interjeições.
1- que (advérbio interrogativo): usa-se em
Ex.: hã!, hem!, ah!, ih!.
perguntas diretas e indiretas e é similar à “por
c) na posição medial, encontra-se o -h em alguns
qual motivo” e “por qual razão”.
dígrafos.
Ex.: “Por que a galinha atravessou a rua?”
Ex.: cachorro, chá. 2- que (pronome relativo): equivale à forma “pelo
Letras dobradas
qual” e suas flexões. Ex.: a razão por que o
Escrevem-se -rr e -ss quando, entre vogais,
aluno faltou à aula não foi explicada.
representam os fonemas /r/ e /s/.
3- que (conjunção subordinativa integrante):
Ex.: carro, morro, curral, massa, missa, mousse.
inicia oração subordinada substantiva.
Já -cc e -cç quando o primeiro é pronunciado
Ex.: revelou-se irrequieta por que
distintamente do segundo.
começássemos a festa.
Ex.: ficção, occipício.
b) porque:
Obs.: conforme o Novo Acordo, o -r e o -s são
1- Conjunção coordenativa: liga duas orações
dobrados em compostos, sem hífen, cujo vocábulo
coordenadas para justificar ou explicar a
inicial termine em vogal e o segundo inicie por uma
proposição contida na outra oração.
destas letras.
Ex.: Entre, porque (pois) está tarde.
Ex.: anti + social = antissocial; arqui + rival =
2- Conjunção subordinativa: inicia orações
arquirrival.
subordinadas adverbiais indicando
EMPREGO DAS LETRAS
circunstância de casualidade ou finalidade.
Letra “s”
Ex.: o chão está molhado porque (já que, visto
a) em sufixos -ês, -esa, -isa, -osa, -oso, -ense:
que, dado que) choveu muito.
camponês, japonesa, sacerdotisa, bondosa,
c) por quê: emprega-se esta forma somente no
carinhoso, piauiense.
final de enunciados. interrogativos.
b) em palavras derivadas de outras palavras que têm
Ex.: Janjão não viajou por quê?
-s no radical: análise – analisar; casa – casebre.
d) porquê: substantivo masculino, com similares
c) nas formas conjugadas do verbos pôr e querer:
razão e motivo.
quis, quiseram, pôs, pus, puseram. Ex.: “Os 13 porquês”.
Letra “z”
Antecedido, muitas vezes, pelo artigo “o(s)”.
a) em palavras derivadas de outras palavras que têm
Ex.: Eles não falaram o porquê da
-zno radical: juiz – juizado; fuzil – fuzilamento.
consumação do casamento.
b) em substantivos abstratos femininos derivados de
adjetivos: lúcido – lucidez; rude – rudeza. ACENTUAÇÃO GRÁFICA
c) nas terminações -izar (para formar verbos) e - A acentuação gráfica é baseada na acentuação
ização (para formar substantivos): mental – de intensidade de acordo com as regras de
mentalizar – mentalização. ortografia.
Letra “x” Portanto, recebem acento agudo (´) ou acento
a) após ditongos: caixa, peixe. circunflexo (^):
b) após a sílaba en: enxada, enxame. a) monossílabos terminados em -a(s), -e(s), -
c) em palavras de origem indígena ou africana e o(s), ei(s): chá(s), pé(s), pó(s), géis.
em palavras inglesas aportuguesadas: abacaxi, b) oxítonos terminados em -a(s), -e(s), -o(s), -
xavante, orixá, xerife, xampu. em, -ens: babá(s), lilás, chalé(s), bebê(s),
61
avó(s), avô(s), alguém, ninguém, amém, PROFESSOR RESPONDE
parabéns, armazéns. Leia o texto para responder às questões 01 e 02.
c) paroxítonos terminados em -l, -n, -r, -x, -i(s), - Epitáfio Titãs
us, -ão(s), -ã(s), -um, -uns, -on(s), -ps: fácil, (Sérgio Britto)
têxtil, hífen, mártir, tórax, ônix, júri, lápis, bônus, Devia ter amado mais Ter chorado mais
vênus, órfão(s), órfã(s), fórum, fóruns, próton(s), Ter visto o sol nascer
tríceps. Devia ter arriscado mais e até errado mais
Ter feito o que eu queria fazer
d) todos os proparoxítonos: líquido, cérebro,
Queria ter aceitado as pessoas como elas são
lâmpada, paralelepípedo, linguística. Cada um sabe a alegria e a dor que traz no
#Dica: as oxítonas e paroxítonas são coração
diametralmente diferentes, portanto, basta lembrar O acaso vai me proteger
que, para uma paroxítona receber acento gráfico, Enquanto eu andar distraído
ela não pode possuir terminação igual à regra de O acaso vai me proteger
acentuação das oxítonas. Enquanto eu andar...
Devia ter complicado menos, trabalhado menos
CASOS ESPECIAIS DE ACENTUAÇÃO GRÁFICA Ter visto o sol se pôr
ACENTUAM-SE: Devia ter me importado menos com problemas
a) oxítonos terminados em ditongos abertos - pequenos
éis, -éu(s), -ói(s): anéis, chapéu(s), herói(s). Ter morrido de amor
b) hiato em -i e -u quando não formam sílaba Queria ter aceitado a vida como ela é
com -r, -l, -m, -n, -z ou seguidos de -nh: os A cada um cabe alegrias e a tristeza que vier
casos são exceções: saída, faísca, país, saúde, (...)
ataúde, baú(s).
01. (IFPI - 2015) 26. Epitáfio não é o mesmo que
c) paroxítonos terminados em ditongos oral
a) Frase colocada sobre o túmulo.
átono crescente ou decrescente: água, língua,
b) Elogio fúnebre.
ciência, rodoviária, início, série.
c) Inscrição sepulcral.
d) oxítonas terminadas em -i e -u quando estão
d) Em Literatura, um tipo de poesia que expressa
em posição final da sílaba sozinhas ou
um lamento pela morte de alguém.
acompanhadas de -s, mesmo precedidas de
e) Causa que leva à morte de alguém.
ditongo decrescente: Piauí, tuiuiú(s)
e) A letra -i em oxítonas das formas verbais 02. (IFPI - 2015) 29. Julgue as afirmações acerca
terminadas em -air e -uir quando seguidas de das regras de acentuação gráfica:
lo(s) e la(s): traí-lo(s), atraí-la(s), anuí-la(s). I. Até - acentuamos as oxítonas terminadas
f) as formas verbais terminadas em -a, -e, -o em e(s).
tônicas seguida dos clíticos -lo(s) e -la(s): II. Distraído - a letra i receberá acento se
amá-lo(s), vivê-los, movê-la(s). estiver sozinha na sílaba, na segunda
g)Caso das terceiras pessoas do singular e vogal do hiato.
plural dos verbos terminados em -ê, -em, -ém III. Pôr - o acento diferencial será obrigatório
Ex.: Ter: tem/têm; Conter: contém/contêm; na palavra pôr (verbo) para que não seja
Ler: lê/leem. confundida com por (preposição).
h)Acento diferencial IV. Epitáfio - acentuam-se as paroxítonas
Ex.: pôr (verbo)/ por (preposição); pôde terminadas em ditongos orais (seguidos ou
(pretérito do verbo poder)/ pode (presente do não de s).
verbo poder); Estão corretas:
fôrma (substantivo = modelo)/ forma a) I, II , III e IV.
(substantivo = aspecto; 3ª pessoa do singular b) I, II e III.
do presente do indicativo). c) I, III e IV.
i)Uso do trema d) II e III.
Depois do Acordo Ortográfico (2009), o trema e) Apenas I.
passou a ser utilizado apenas em palavras
03. (IFPI - 2014) Segundo o Novo Acordo
derivadas de nomes próprios estrangeiros que o
Ortográfico, não está acentuada corretamente
possuem.
a palavra:
Ex.: Führer, mülleriano (de Müller).
a) Hífen.
b) Próton.
BIBLIOGRAFIA CONSULTADA
BECHARA, E. Moderna Gramática Portuguesa. 37. ed., rev., ampl., c) Prótons.
conforme o novo acordo ortográfico. Rio de Janeiro: Nova Fronteira, d) Assembléia
2009. e) Vírus.
CEGALLA, D.P. Novíssima Gramática da Língua Portuguesa. 46. ed.
São Paulo: Companhia Editora Nacional, 2005.
MEDEIROS, J. B. Português Instrumental. 8. ed. São Paulo: Atlas,
2009.
SARMENTO, L. L. Gramática em Textos. 1.ed. São Paulo:
Moderna, 2009.

62
04. (IFPI - 2016) Considerando as regras de d) Essa linguagem usada no texto aplica-se a
acentuação gráfica, julgue os itens a seguir: qualquer região do Brasil, pois é padronizada
(observe as palavras sublinhadas): a todos os falantes do português brasileiro.
I. Acentua-se com circunflexo a 3ª pessoa do e) Não devemos usar esse tipo de fala, pois ela
plural do presente do indicativo dos verbos ter, não é compreendida.
como em “(...) outras línguas têm possessivos 02. (IFPI - 2014) Através da leitura do texto,
distintos para singular e plural”. podemos observar que
II. Acentuam-se todas as palavras proparoxítonas, a) Ele é uma comprovação de que a língua é
como em “português padrão inatacável”. estática e não sofre variações.
III. Acentuam-se as palavras paroxítonas b) O eu-lírico é uma pessoa que mora em zona
terminadas em ditongos crescentes, como em urbana.
“competência pragmática intuitiva”. c) Pelo modo de falar, o eu-lírico demonstra ser
Está(ão) correto(s): da zona rural, pois em sua fala existem
a) Todos os itens. elementos que retratam esse tipo de
b) Apenas o item II. linguagem.
c) Os itens I e II. d) Podemos perceber, através de elementos
d) Os itens II e III. presentes no texto, que o eu-lírico é um
e) Os itens I e III. senhor de idade avançada.
e) Não há nada no texto comprovando que ele é
05. (IFPI - 2014) Não ocorre dígrafo na palavra: mais usado na zona rural.
a) Detecção.
03. (IFPI - 2016) Observe a palavra destacada em
b) Também.
“Ela se apoia comodamente na cerca (...)”.
c) Importância.
Agora, considerando as regras de acentuação
d) Ferramentas.
gráfica, julgue os itens a seguir:
e) Conhecida.
I. A palavra “apoia”, assim como a palavra
ALUNO RESPONDE “heróico”, deveria ser acentuada, uma vez
que, segundo as regras, recebem acento os
Leia o texto para responder às questões 01 e
ditongos abertos tônicos (éu, éi, ói) das
02.
palavras paroxítonas.
Declaração aos amigos de uma forma caipira II. Com a reforma ortográfica, perdem o acento
1 Ces são o colírio do meu ôiu. os ditongos abertos (ei, oi) que não estão na
2 São o chiclete garrado na minha carça dins. sílaba final da palavra. O ditongo aberto das
3 São a maionese do meu pão.
oxítonas continua existindo.
4 São o cisco no meu ôiu (o ôtro oiu - eu ten dois).
5 O limão da minha caipirinha. III. Assim como “apoia”, pela regra, também não
6 O rechei do meu biscoito. devem receber acento as palavras “heroi” e
7 A masstumate do meu macarrão. “heroico”.
8 A pincumel do meu buteco. Está(ão) correto(s):
9 Nossinhora! a) Todos os itens.
10 Gosto dimais da conta docêis, uai. b) Apenas o item I.
11 Ces são tamém: c) Apenas o item II.
12 O videperfume da minha pintiadêra. d) Apenas o item III.
13 O dentifriço da minha iscovdidente. e) Os itens I e II.
14 Óiproceisvê,
15 Quem tem amigos assim, tem um tisôru! 04. (IFPI - 2014) Não é acentuada pela mesma regra
16 Eu guárdêsse tisouro, com todo carin, de “século” a palavra
17 Do Lado Esquerdupeito!!! a) Futebolístico.
18 Dentro do Meu Coração!!! b) Vocabulário.
- Paulo Master
c) Léxico.
01. (IFPI - 2014) Sabendo que a língua sofre d) Sinônimos.
variações por estar relacionada e condicionada a e) Específico.
fatores sociais, históricos e/ou regionais. É
05. (IFPI - 2014) Assinale a opção em que a palavra
possível compreender do texto que:
é acentuada pela mesma regra de “plágio”.
a) Ele traduz uma modalidade dialetal regional,
a) Mérito.
pois mostra o jeito caipira de falar, específico
b) Disponível.
de uma determinada região.
c) Acadêmica.
b) No texto, revela-se uma variação histórica,
d) Retuítes.
pois fica claro que os termos usados são
e) Consciência.
arcaicos e fora de uso pela sociedade atual.
c) O termo “tisouro”, linha 16, é um exemplo da
modalidade escrita da língua, comum também
em revistas científicas.

63
Texto para a questão 06.
“Os sete jovens desta reportagem têm todas as 11. Assinale o item em que todas as palavras são
características comuns aos empreendedores, mas acentuadas pela mesma regra de: também,
eles abraçaram projetos nada convencionais. É uma incrível e caráter.
nova geração, com uma nova mentalidade. Eles não a)alguém, inverossímil, tórax
tocam nem empresas normais nem ONGs, e sim os b)hífen, ninguém, possível
chamados negócios sociais. Todos acreditam que é c)têm, anéis, éter
possível, sim, obter lucro combatendo a pobreza.” d)há, impossível, crítico
Revista Pequenas Empresas & Grandes Negócios (2013)
http://revistapegn.globo.com e)pólen, magnólias, nós
06. (IFPI - 2014) Marque a alternativa que justifica a 12. Assinale a alternativa correta
obrigatoriedade (ou não) do acento na forma a) Não se deve colocar acento circunflexo em
verbal “têm”, na seguinte passagem: “Os sete palavra como avo, bisavo, porque há
jovens desta reportagem têm todas as palavras homógrafas com pronúncia aberta.
características comuns aos empreendedores”. b) Não se deve colocar acento grave no a do
a) O verbo está acentuado porque concorda com contexto: Fui a cidade.
o termo “características comuns”, que está no c) Não se deve colocar trema em palavras
plural. como tranquilo,linguiça,sequência.
b) O verbo está acentuado porque concorda com d) Não se deve colocar trema em palavras
o termo “aos empreendedores”, que está no derivadas como avozinho, vovozinho.
plural. e) O emprego do trema é facultativo.
c) O verbo está acentuado porque concorda com 13. Assinale a opção em que as palavras, quanto
o sujeito, “os sete jovens”, que está no plural. à acentuação gráfica, estejam agrupadas pelo
d) O verbo não deve ser acentuado porque mesmo motivo gramatical.
concorda com “reportagem”, que está no a) problemáticos, fácil, álcool
singular. b) já, até, só
e) Em nenhuma hipótese o verbo em questão c) também, último, análises
deve ser acentuado. d) porém, detêm, experiência
07. (IFPI – 2014) Com o novo acordo ortográfico, o e) país, atribuíram, cocaína
acento agudo desaparece nos ditongos abertos 14. Marque item em que necessariamente o
“oi” e “ei” das paroxítonas, como é o caso da vocábulo deve receber acento gráfico:
palavra “ideia”, que passou a ser escrita sem o a) historia.
acento. Não deve também ser acentuada por b) ciume.
essa razão: c) amem.
a) Herói. d) numero.
b) Papéis. e) ate.
c) Heróico.
d) Chapéu. 15. Todas as palavras devem ser acentuadas na
e) Fiéis. alternativa:
a)pudico, pegada, rubrica
08. (IFGO - 2017) As palavras que apresentam a b)gratuito, avaro, policromo
mesma regra de acentuação gráfica são: c)abdomen, itens, harem
a) econômica, será, verídica. d)magoo, perdoe, ecoa
b) públicas, filósofo, país. e) contribuia, atribuimos, caíste
c) últimos, presídios, aliás.
d) área, notícia, sábio. 16. Por serem proparoxítonos, deveriam estar
e) corrimão, situação, armazém. acentuados os vocábulos da opção:
a) refrega, ibero, decano
09. (UF-PR) Assinale a alternativa em que todos os b) aziago, pegada, avaro
vocábulos são acentuados por serem oxítonos: c) leucocito, alcoolatra, ínterim
a)paletó, avô, pajé, café, jiló d) inaudito, batavo, erudito
b)parabéns, vêm, hífen, saí, oásis e) rubrica, maquinaria, pudico
c)você, capilé, Paraná, lápis, régua
d)amém, amável, filó, porém, além 17. O acento gráfico desempenha a mesma função
e) caí, aí, ímã, ipê, abricó em:
a) carnaúba e história.
10. (PUC-RJ) Aponte a opção em que as duas b) petróleo e paciência.
palavras são acentuadas devido à mesma regra: c) jacarandá e lápis.
a) saí - dói d) glória e está.
b) relógio - própria e) mausoléu e líquido.
c) só - sóis
d) dá - custará
e) até – pé

64
18. Todas as palavras abaixo admitem dupla 23. Assinale a alternativa em que todas as
prosódia, exceto: palavras estão grafadas corretamente:
a) acróbata. a) analizar-economizar-civilizar.
b) sóror. b) receoso-prazeirosamente-silvícola.
c) íbero. c) tábua-previlégio-marquês.
d) hieróglifo. d) pretencioso-hérnia-majestade.
e) xérox. e) flecha-jeito-ojeriza.
19. Assinale a alternativa incorreta sobre o emprego 24. Marque a alternativa na qual o uso do
da letra “x”: “porquê” está adequado à gramática
normativa:
a) Deve-se empregar o “x” após os ditongos
(encontros vocálicos = vogal + semivogal em a) Porquê você parece tão despreocupada?
uma mesma sílaba). Exemplos: ameixa, b) Estou feliz depois do drama porque passei.
feixe, caixa, frouxo, etc. c) Continuo não entendendo o por quê desse
b) Emprega-se o “x” após as sílabas “en” e comportamento.
“me”, como enxame, enxame, enxurrada, d) Você não me compreende porque nunca
enxaqueca, mexicano, mexerica, mexilhão, vivenciou essa situação.
etc. e) Eu deveria viver os seus dramas? Porquê?
c) Emprega-se nas palavras de origem indígena
25. GIRASSÓIS é uma palavra grafada com SS.
ou africana, assim como nas palavras
Também devem ser grafadas, dessa mesma
inglesas aportuguesadas, como xavante,
forma, todas as palavras da seguinte opção:
xingar, xique-xique, xará, xerife, xampu, etc.
d) Palavras como enxente, enxarcar, enxapelar, a) mussulmano – concessão
enxumaçar e enxiqueirar seguem a regra do b) excessão – transgressão
emprego do “x” após a sílaba “en”. c) massiço – discussão
e) A letra “x” pode representar os seguintes d) obsessão – impressão
fonemas: /ch/, /cs/, /z/, /ss/ e /s/. e) missanga – submissão
20. São exemplos de palavras homófonas, isto é, 26. Os termos “promoções” e “divulgações” estão
palavras que possuem a mesma pronúncia, grafados de acordo com os padrões da
mas a grafia diferente: norma culta da língua portuguesa. Da mesma
forma, também estão corretamente grafadas
I. chácara e xácara.
as seguintes palavras:
II. chalé e xale.
III. cheque e xeque. a) espreguiçar – mussulmano
IV. viagem e viajem. b) excessão – dentuço
V. chá e xá. c) compreenção – crença
a) Todas as alternativas estão corretas. d) extinção – calabouço
b) Apenas I e II estão corretas. e) disfunção – distorsão
c) I, III, IV e V estão corretas.
d) Apenas V está correta.
e) I, II, III e V estão corretas.
21. Assinale a alternativa cujas palavras estejam
grafadas corretamente:
a) Ouviu-se na multidão um muchocho
indelicado.
b) As paixões incontidas enchem a pessoa
de vexame.
c) Nunca deixou de puchar a brasa para sua
sardinha.
d) Bruchuleavam as luzes do pátio.
e) Fora tida na conta de brucha.
22. (FMU) Assinale a alternativa em que todas
as palavras estão grafadas corretamente:
a) paralisar,pesquisar, ironizar, deslizar.
b) alteza,empreza, francesa, miudeza.
c) cuscus, chimpanzé, encharcar, encher.
d) incenso,abcesso, obsessão, Luís.
e) chineza, marquês, garrucha, meretriz.

65
PONTUAÇÃO h) Zeugma (supressão do verbo constante da
oração anterior)
A VÍRGULA
 O pensamento é triste; o amor, insuficiente,
É o sinal que indica pequena pausa na leitura.
i) Depois do "sim" e do "não", usados nas
Além disso, ela separa termos de uma oração e
respostas.
certas orações no período.
 Não, porque fui embora mais cedo.
A VÍRGULA SEPARANDO TERMOS DA ORAÇÃO  Sim, passaremos no concurso.
a) Termos coordenados, isto é, de mesma A VÍRGULA SEPARANDO ORAÇÕES NO
função sintática. PERIODO
 Era um rapagão corado, forte, risonho. a) Orações coordenadas assindéticas.
 A terra, o mar, o céu, tudo glorifica Deus.  O tempo não pára, não apita na curva, não
Observação: espera ninguém.
Normalmente não se separam termos unidos b) Orações coordenadas sindéticas
por e, nem e ou.  Você já sabe bastante, porém deve estudar
 Possuía lavouras de trigo, arroz e linho. mais.
 Não aprecia cinema, teatro nem circo.  Não solte balões, porque causam incêndio.
 Os mendigos pediam dinheiro ou comida.  O mal é irremediável, portanto conforma-te.
b) Vocativo, aposto, predicativo, palavras Exceção: As aditivas com a conjunção "e".
repetidas.  O agricultor colheu o trigo e vendeu-o ao
 Brasília, Capital da República, foi fundada Banco do Brasil.
em 1960. Observação:
 Senhor, eu queria saber quem foi o poeta Usa-se vírgula com a conjunção "e":
que inventou o beijo. (1) Orações coordenadas aditivas com sujeitos
 Lentos e tristes, os retirantes iam passando diferentes:
pela caatinga.  Afinal vieram outros cuidados, e não pensei
 As paredes do hospital eram brancas, mais nisso.
brancas.  O concurso foi difícil, e a prova não
c) Termos explicativos, retificativos, correspondeu ao programa.
conclusivos, enfáticos... (2) Orações coordenadas adversativas (e =
 Quer dizer que você, então, não voltou mas)
mais.  Morava no Brasil, e votava na Espanha.
 Elas, aliás, não saíam de casa. (3) Quando se quiser enfatizar o último termo
 Pois sim, faça como quiser. de uma série coordenada
 Em suma, a pontuação é um problema.  Deitou-se tarde, custou-lhe dormir, pensou
 Portanto, usa-se a vírgula nas expressões muito nela, e sonhou.
denotativas. (4) No polissíndeto (facultativa)
d) Termos antepostos (e repetidos  Os dias passavam, e as águas, e os versos,
pleonasticamente). e com eles ia passando a vida.
 Essas palavras, eu não as disse jamais. c) Orações subordinadas adverbiais
 Aos poderosos, nada lhes devo. antepostas ou intercaladas.
e) Conjunções adversativas e conclusivas  Embora estivesse muito cansado, compareci
deslocadas. à reunião.
 O sinal estava fechado; os carros, porém,  Quando chegar o verão, iremos ao Sul.
não pararam.  As viúvas inconsoláveis, quando são jovens,
 Já lhe comprei balas, sorvete; convém, pois, sempre são consoladas.
ficar calado agora. Observações:
f) Adjunto adverbial anteposto ao verbo. Com orações adverbiais pospostas, só é
 Com mais de setenta anos, andava a pé. recomendável usar vírgula:
 Os convidados, depois de algum tempo, (1) Se a oração principal for muito extensa;
chegaram ao clube.  O ar poluído corrói a saúde do povo,
Observação: embora não se perceba a curto prazo.
Adjunto adverbial de pequeno corpo costuma (2) Se a oração principal vier seguida de outra
dispensar a vírgula. qualquer.
 Amanhã(,) o Presidente viajará.  Os alunos declararam ao diretor que
Quando usada, serve para dar ênfase. estavam satisfeitos, quando o curso
g) Datas (Local e data - número e data, em acabou.
documentos) d) Orações substantivas antepostas.
 Brasília, 5 de junho de 1994.  Que venham todos, é preciso: estou
 O Decreto n° 5.765, de 18 de dezembro saudoso.
de 1971.
66
e) Orações interferentes. RETICÊNCIAS
 A História, disse Cícero, é a grande mestra
da vida. As reticências têm a função básica de indicar
f) Orações adjetivas explicativas. que a frase foi interrompida ou truncada.
 O Sol, que é uma estrela, aquece a Terra. Geralmente, são empregadas para:
g) Orações reduzidas equivalentes a adverbiais.
 Terminada a aula, todos saíram felizes.  Indicar partes suprimidas de um texto
h) Ideias paralelas dos provérbios. que não interessam à citação:
 Casa de ferreiro, espeto de pau.
 Mocidade ociosa, velhice vergonhosa. Para atender aos reclamos de progresso e de
liberdade do nosso povo (...) não hesitei pôr em
O PONTO-E-VÍRGULA risco até o meu próprio mandato.

 Indicar surpresa, dúvida, hesitação:


Assinala pausa maior que a vírgula e menor
que o ponto. Usa-se o ponto-e-vírgula nos — Eu acho que meu pai é o seu Vicente...
seguintes casos: disse, sem convicção. As crianças sorriram.
a. Separando os itens de uma enumeração; (Aníbal Machado)
A gramática normativa trata dos seguintes
assuntos: Tenho medo... não sei... tenho medo. (N.
1) fonética; Rodrigues)
2) morfologia;
3) sintaxe;  Indicar interrupção da fala do narrador
4) estilística. ou da personagem:
b. Separando as partes principais de um
período, cujas secundárias já foram separadas — Mas eu... — tentei informar. Ela não me
por vírgula; deixou concluir a frase. (C. D. Andrade)
 Na volta da escola, alguns brincavam; outros,
no entanto, vinham sérios; quando chegamos. — Fizemos uma montanha de areia, mamãe,
Todos riam. que só vendo...
c. Separando orações coordenadas com a
conjunção deslocada; TRAVESSAO
 A aula já terminou; vocês, porém, não devem
sair. O travessão e empregado basicamente em
d. Separando orações coordenadas dois casos:
(adversativas) assindéticas.  Para indicar a mudança de interlocutor
 Há muitos modos de acertar, há um só de nos diálogos:
errar.
— Como vai, Ricardo?
OS DOIS-PONTOS — Bem. E o senhor, major?
Assinalam uma pausa para indicar que a  Para destacar palavras, expressões ou
frase não foi concluída, isto é, há algo a se frases intercaladas. Nesse caso,
acrescentar. Usam-se dois-pontos nos emprega-se o duplo travessão:
seguintes casos: Ele decidiu que as joias — que não estavam
seguradas — não voltariam a sua casa. (Veja)
1. Introduzindo citação ou transcrição;
 Diz um provérbio árabe: "A agulha veste Um número pequeno — embora muito
os outros, e anda nua". estimulante — de prefeituras vem assumindo,
2. Introduzindo enumeração; com sucesso, a tarefa de melhorar a saúde de
 Os meios legítimos de adquirir fortuna seus cidadãos. (Istoé)
são três: ordem, trabalho e sorte.
3. Em oração explicativa com a conjunção
subentendida;
 Você fez tudo errado: gritou quando não
devia e calou quando não podia.

4. Com oração apositiva.


 Disse-me algo horrível: que ia casar.

67
PROFESSOR RESOLVE b) No Brasil, a diferença social é motivo de
constante preocupação.
01. Assinale o texto corretamente pontuado.
c) O candidato que chegou atrasado fez um
a) Enquanto eu fazia comigo mesmo aquela
ótimo teste no IBGE.
reflexão, entrou na loja um sujeito baixo sem
d) Tenho esperanças, pois a situação
chapéu trazendo pela mão, uma menina de
econômica não demora a mudar.
quatro anos.
e) Ainda não houve tempo, mas, em breve, as
b) Enquanto eu fazia comigo mesmo aquela
providências serão tomadas.
reflexão, entrou na loja, um sujeito, baixo, sem
chapéu, trazendo pela mão, uma menina de 2. (ABC-SP) Assinale a alternativa cuja frase está
quatro anos. corretamente pontuada:
c) Enquanto eu fazia comigo mesmo aquela a) O sol que é uma estrela, é o centro do nosso
reflexão, entrou na loja um sujeito baixo, sem sistema planetário.
chapéu, trazendo pela mão uma menina de b) Ele, modestamente se retirou.
quatro anos. c) Você pretende cursar Medicina; ela,
d) Enquanto eu, fazia comigo mesmo, aquela Odontologia.
reflexão, entrou na loja um sujeito baixo sem d) Confessou-lhe tudo; ciúme, ódio, inveja.
chapéu, trazendo pela mão uma menina de e) Estas cidades se constituem, na maior parte
quatro anos. de imigrantes alemães.
e) Enquanto eu fazia comigo mesmo, aquela 3. (SANTA CASA) Os períodos abaixo
reflexão, entrou na loja, um sujeito, baixo, sem apresentam diferenças de pontuação. Assinale
chapéu trazendo, pela mão, uma menina, de a letra que corresponde ao período de
quatro anos. pontuação correta:
02. Considere a frase abaixo (retirada do J. B. de a) José dos Santos paulista, 23 anos vive no Rio.
13/10/95, sem pontuação) b) José dos Santos paulista 23 anos, vive no Rio.
Ela tem, de acordo com as regras de uso da c) José dos Santos, paulista 23 anos, vive no Rio.
vírgula, a seguinte pontuação correta. d) José dos Santos, paulista 23 anos vive, no Rio.
a) O presidente descobriu, que tinha aliados, virou e) José dos Santos, paulista, 23 anos, vive no
a agenda de cabeça para baixo e partiu para a Rio.
reforma administrativa. 4. Assinale a opção em que está corretamente
b) O presidente, descobriu que tinha aliados, virou indicada a ordem dos sinais de pontuação que
a agenda de cabeça para baixo e partiu para a devem preencher as lacunas da frase abaixo:
reforma administrativa.
c) O presidente descobriu que tinha aliados, virou “Quando se trata de trabalho científico ___ duas
a agenda de cabeça para baixo e partiu para a coisas devem ser consideradas ____ uma é a
reforma administrativa. contribuição teórica que o trabalho oferece ___ a
d) O presidente descobriu que tinha aliados virou outra é o valor prático que possa ter”.
a agenda de cabaça para baixo, e partiu para a a) dois pontos, ponto e vírgula, ponto e vírgula
reforma administrativa. b) dois pontos, vírgula, ponto e vírgula;
e) O presidente descobriu que tinha aliados, virou c) vírgula, dois pontos, ponto e vírgula;
a agenda, de cabaça para baixo e partiu para a d) pontos vírgula, dois pontos, ponto e vírgula;
reforma administrativa. e) ponto e vírgula, vírgula, vírgula.
03. Assinale o segmento pontuado com 5. (CARLOS CHAGAS-BA) Os períodos abaixo
correção. apresentam diferenças de pontuação, assinale
a) Para solucionar os problemas, é preciso, antes, a letra que corresponde ao período de
ter vontade de fazê-lo. pontuação correta:
b) Para solucionar os problemas é preciso antes,
ter vontade de fazê-lo. a) Pouco depois, quando chegaram, outras
c) Para solucionar os problemas é preciso antes pessoas a reunião ficou mais animada.
ter vontade de fazê-lo. b) Pouco depois quando chegaram outras
d) Para solucionar os problemas, é preciso, antes pessoas a reunião ficou mais animada.
ter vontade de fazê-lo. c) Pouco depois, quando chegaram outras
e) Para solucionar os problemas, é preciso antes, pessoas, a reunião ficou mais animada.
ter vontade de fazê-lo. d) Pouco depois quando chegaram outras
pessoas a reunião, ficou mais animada.
O ALUNO RESPONDE e) Pouco depois quando chegaram outras
1. Assinale a opção que apresenta erro de pessoas a reunião ficou, mais animada.
pontuação:
a) Sem reforma, social, as desigualdades entre as
cidades brasileiras, crescerão sempre...

68
6. Assinale a letra que corresponde ao período de 7. (IFPI 2020) Releia a seguinte frase: “Não está
pontuação correta certo tirar a mulher de dentro de casa, quem
tem que sair é o agressor”. Agora, assinale a
a) Era um homem de quarenta e cinco anos,
alternativa em que a reescrita mantém o
baixo, meio gordo, fisionomia insinuante,
sentido original do período destacado.
destas que mesmo sérias, trazem impresso
constante sorriso. a) Mesmo que a mulher saia de dentro de casa,
b) Era um homem de quarenta e cinco anos, quem tem que sair é o agressor.
baixo, meio gordo, fisionomia insinuante, b) Embora a mulher saia de dentro de casa,
destas que mesmo sérias trazem, impresso quem tem que sair é o agressor.
constante sorriso. c) É com a presença da mulher que o agressor
c) Era um homem de quarenta e cinco anos, irá sair de casa.
baixo, meio gordo, fisionomia insinuante, d) Não é a mulher que tem que sair de dentro
destas que, mesmo sérias, trazem impresso, de casa, o agressor é quem tem que sair.
constante sorriso. e) Se a mulher for mantida dentro de casa,
d) Era um homem de quarenta e cinco anos, quem tem que sair é o agressor.
baixo, meio gordo, fisionomia insinuante, A LEI MARIA DA PENHA EM CORDEL
destas que, mesmo sérias trazem impresso (Tião Simpatia)
constante sorriso. A lei Maria da Penha
e) Era um homem de quarenta e cinco anos, Está em pleno vigor
baixo, meio gordo, fisionomia insinuante, Não veio pra prender homem
destas que, mesmo sérias, trazem impresso Mas pra punir agressor
constante sorriso. Pois em “mulher não se bate
Nem mesmo com uma flor”.
LEIA O TEXTO ABAIXO
A violência doméstica
Treze anos após Lei Maria da Penha, só 2,4% das cidades
Tem sido uma grande vilã
têm casas-abrigo para mulheres (25/09/2019)
E por ser contra a violência
RIO — Casada por sete anos, G. não podia sair de Desta lei me tornei fã
casa. Nem trabalhar, falar com vizinhos, usar o celular. Pra que a mulher de hoje
Não tinha ninguém com quem dividir sua rotina de Não seja uma vítima amanhã.
violência doméstica, que incluía chutes e agressões
com corda. O marido às vezes sumia por duas Toda mulher tem direito
semanas, deixando a mulher e as duas filhas sem A viver sem violência
dinheiro. Faltava comida, a água e a energia eram É verdade, está na lei.
cortadas. Que tem muita eficiência
Ela suportou até o dia em que ele ameaçou as Pra punir o agressor
meninas com uma faca. Foi até um orelhão e chamou E à vítima, dar assistência.
a polícia. Não tinha ninguém no mundo, nem dinheiro.
Elas foram acolhidas em uma casa-abrigo onde Tá no artigo primeiro
ficaram por seis meses. (...) Que a lei visa coibir;
Mesmo previsto na Lei Maria da Penha , A violência doméstica
criada há 13 anos, o acolhimento em casas-abrigo de Como também, prevenir;
mulheres ameaçadas de morte como G. só é realidade Com medidas protetivas
em 2,4% das cidades brasileiras. Ao todo, são 153. E ao agressor, punir.
Nos últimos cinco anos, não houve avanço algum. Os (...)
dados são de 2018 e constam da mais recente
pesquisa de Informações Básicas Municipais do IBGE, 8. (IFPI 2020.1) No final da primeira estrofe, há
divulgada nesta quarta-feira. um trecho entre aspas: “em mulher não se
A situação preocupa ainda se comparada à bate nem com uma flor”. Sobre as aspas, é
realidade apontada pelo levantamento anterior, correto dizer que o autor do cordel as utiliza
realizado em 2013. À época, 2,5% dos municípios
para fazer
contavam com 155 desses equipamentos sob a gestão
das prefeituras.
a) uma ironia com a situação atual.
— A Lei Maria da Penha é uma Ferrari, mas b) uma citação direta de um provérbio, de um
nossas instituições estão cheias de buracos e dito popular.
lombadas. Há uma cultura organizacional que não c) uma exemplificação de uso da linguagem
permite que ela avance. A casa-abrigo é uma política coloquial d) uma crítica à sabedoria popular
excelente, mas é um modelo emergencial, precisamos d) um elogio aos que defendem e respeitam as
pensar em outros modelos. Não está certo tirar a mulheres.
mulher de dentro de casa, quem tem que sair é o
agressor — afirma a delegada Eugênia Villa, que atua
em Teresina. (...)
Disponível em: https://oglobo.globo.com/sociedade/celina/
treze-anos-apos-lei-maria-da-penha-so-24-das-cidades-tem-casas-
abrigo-para-mulheres-23972179

69
NOÇÕES GERAIS DE LITERATURA 2. LITERATURA E O PROCESSO HISTÓRICO
A literatura propõe uma associação com
- O texto literário elementos que integram o processo histórico no
A palavra “literário” traz o significado de pertencer sentido de recontá-los de uma forma estilística,
à “literatura”, termo que vem do latim littera, ou questionadora e crítica. Trata-se da função
“letra”. No seu sentido primário, portanto, texto engajada (aliciada, atuante, participativa) pois a
literário é o que manifesta uma ideia escrita em Literatura é a expressão da sociedade, como a
linguagem elaborada de forma a causar emoções no palavra é a expressão do homem.
leitor. Em relação aos processos históricos pelos
1. FUNDAMENTOS DA LITERATURA quais os homens passam, a literatura engajada
A Literatura é uma manifestação artística. Trata- assume as mais diversas tarefas, as quais estão
se, portanto, de uma representação da realidade, resumidas nos verbos: contestar, criticar,
inventada por seu autor. Essa invenção, por sua vez, denunciar, desmitificar, modificar, questionar,
acaba se constituindo como um novo universo, do reavaliar, recriar, redimensionar, reinterpretar,
qual tomam parte os leitores. rever, transformar... O papel social não basta, há
A essência da arte literária está na palavra. de cumprir, no texto artístico, as funções
Observe: emotiva, poética e metalinguística.
1- Os homens normalmente sentem
saudades de sua infância. 3. FUNÇÕES DO TEXTO LITERÁRIO
2- Eu tenho saudades da minha A palavra “função” aqui se refere ao papel que
infância. a literatura desempenha nas sociedades, um
3- Oh! Que saudades que tenho papel que se configurou, em grande parte,
Da aurora de minha vida, porque:
Da minha infância querida  A literatura provoca reflexão
Que os anos não trazem mais! A literatura não tem o poder de modificar a
Os três textos abordam o mesmo tema, mas o realidade, mas certamente é capaz de fazer
fazem de formas diferentes. O primeiro texto, ao com que as pessoas reavaliem a própria vida
contrário dos demais, generaliza um sentimento e mudem de comportamento. Se esse efeito é
comum. O segundo, ainda que confira um caráter alcançado, o texto literário desempenha um
pessoal a esse sentimento, apenas se limita a fazer importante papel transformador, ainda que de
uma constatação. Já o autor do terceiro texto modo indireto.
(Casimiro de Abreu) utiliza todas as possibilidades  Literatura como denúncia social
expressivas da linguagem (ênfase, metáfora, Através dela é dada a descrição das
repetições, ritmo) para provocar o mesmo sentimento personagens, apresentando as dificuldades
no leitor. pelas que passam esses grupos sociais. Nela
Percebemos então o seguinte: nos textos são tratados assuntos marginalizados da
literários há uma conexão interdependente entre "o sociedade e analisa a estrutura social das
que se diz" (o assunto/tema do texto) e o "como se diferentes épocas.
diz" (a forma como o texto é dito).  Literatura como investigação psicológica
Através do texto literário podemos analisar a
TEXTO NÃO- conduta humana. Ela nos revela as
TEXTO LITERÁRIO
LITERÁRIO motivações, sentimentos, anseios e dúvidas
Texto com função Texto com mera das personagens, quer dizer o mundo interior.
estética, que explora função utilitária  A literatura como entretenimento
diferentes recursos (informativa, As obras literárias oportunizam o
linguísticos e argumentativa, envolvimento do leitor nas ações, na procura
estilísticos (figuras de científica, entre de novas pistas ou caminhos. A uma
linguagem) para outras). Aqui temos interação com o leitor no trabalho de
produzir um efeito o predomínio do desvendar os mistérios, participar nas
artístico. Predomina, sentido denotativo aventuras, despertando assim o prazer pela
portanto, o sentido (literal), em que a leitura.
conotativo (figurado), palavra é tomada  A literatura ajuda a construir identidade
aquele que permite em seu significado Nos textos literários, de certo modo entramos em
interpretações básico. contato com a nossa história, o que nos dá a
diferentes do sentido Exemplos: chance de compreender melhor nosso tempo,
literal da palavra. O manual de nossa trajetória como nação. O interessante,
Exemplos: instruções de um porém, é que essa “história” coletiva é recriada
a letra de uma canção, eletrodoméstico, por meio das histórias individuais, das inúmeras
um poema, uma uma notícia de personagens presentes nos textos que lemos, ou
narrativa ficcional, jornal, entre outros. pelos poemas que nos tocam de alguma forma.
entre outros.

70
PROFESSOR RESPONDE 02. Os dois textos, com enfoque diferentes,
01. Com base no conceito de literatura como abordam um mesmo problema, que se refere,
arte poética e ficcional, alguns textos são simultaneamente, ao campo literário e ao
literários, outros, não. Observe as social. Considerando-se a relação entre os dois
declarações que se fazem sobre eles. textos, verifica-se que eles têm em comum o
fato de que:
I. A linguagem utilizada pelos autores de
a) Tratam do mesmo tema, embora com
textos não literários é a denotativa.
opiniões divergentes, expressas no primeiro
II. A função principal dos textos não literários
texto por meio da ficção e, no segundo, por
é informar, esclarecer, convencer.
meio de uma análise sociológica.
III. O texto literário tem uma função estética,
b) Foi usada, em ambos, linguagem de caráter
enquanto o não literário não tem função
moralista em defesa de uma mesma tese: a
literária.
literatura, muitas vezes, é nociva à formação
IV. A função principal dos textos literários é
do jovem estudante.
comover, despertar sentimentos, provocar
c) São utilizadas linguagens diferentes nos dois
a reflexão.
textos, que apresentam um mesmo ponto de
V. O autor, no texto literário, procura recriar a
vista: a literatura deixa ver o que se
realidade, utilizando, principalmente, a
pretende esconder.
linguagem conotativa.
d) A linguagem figurada é predominante em
Assinale a alternativa mais adequada a respeito ambos, embora o primeiro seja uma fábula e
das declarações anteriores. o segundo um texto científico.
a) São todas falsas. e) O tom humorístico caracteriza a linguagem
b) São todas verdadeiras. de ambos os textos, em que se defende o
c) São verdadeiras apenas a II e a IV. caráter pedagógico da literatura.
d) São falsas a I e a II. 03. Marque a alternativa que apresenta um
e) São falsas a IV e a V. equívoco com relação à arte literária:
a) A Literatura permite-nos entrar em contato
Texto I com nossa história para compreendermos
melhor o presente, o passado e o futuro.
Principiei a leitura de má vontade. E logo b) A arte literária está relacionada à produção e
emperrei na história de um menino vadio que, dirigindo- à leitura de textos verbais escritos.
se à escola, se retardava a conversar com os
c) Os textos ficcionais têm o poder de provocar
passarinhos e recebia deles opiniões sisudas e bons
conselhos. Em seguida vinham outros irracionais, diferentes efeitos de sentido nos
igualmente bem-intencionados e bem falantes. Havia a leitores/ouvintes: alegria, tristeza, diversão,
moscazinha, que morava na parede de uma chaminé e emoção etc. Isso acontece porque a
voava à toa, desobedecendo às ordens maternas: tanto Literatura nos permite sair do mundo real e
voou que, afinal, caiu no fogo. Esses contos me chegar ao mundo da fantasia.
intrigaram com o (livro) Barão de Macaúbas. Infelizmente d) Enquanto arte, a Literatura é capaz de
um doutor, utilizando bichinhos, impunha-nos a registrar a realidade e fazer com que os
linguagem dos doutores. – Queres tu brincar comigo? leitores/ouvintes reavaliem a própria vida e
O passarinho, no galho, usava adjetivos colhidos seus comportamentos.
no dicionário. A figura do barão manchava o frontispício
e) A literatura é uma forma de arte que provoca
do livro, e a gente percebia que era dele o pedantismo
atribuído à mosca e ao passarinho. Ridículo um indivíduo a reflexão por meio de construções
hirsuto e grave, doutor e barão, pipilar conselhos, zumbir simbólicas. O trabalho com as palavras pode
admoestações. ser realizado com sentido denotativo ou
RAMOS, G. Infância. Rio de Janeiro: Record, 1986 (adaptado). conotativo/figurado.
04. Sobre as características da linguagem não
Texto II literária, estão corretas as alternativas:
Dado que a literatura ensina na medida em que atua I. Diferentemente do que acontece com os
com toda a sua gama, é artificial querer que ela textos literários, nos quais há uma
funcione como os manuais de virtude e boa conduta. E preocupação com o objeto linguístico e
a sociedade não pode senão escolher o que em cada também com o estilo, os textos não
momento lhe parece adaptado aos seus fins, literários apresentam características bem
enfrentando ainda assim os mais curiosos paradoxos, delimitadas para que possam cumprir sua
pois, mesmo as obras consideradas indispensáveis principal missão, que é, na maioria das
para a formação do moço, trazem frequentemente o
vezes, a de informar.
que as convenções desejariam banir. Aliás, essa
espécie de inevitável contrabando é um dos meios por II. Apresenta características como a
que o jovem em contato com realidades que se variabilidade, a complexidade, a
tenciona escamotear-lhe. conotação, a multissignificação e a
Adaptado de: Candido, Antônio. liberdade de criação.
A literatura e a formação do homem.

71
III. A linguagem não literária faz da linguagem um LEIA OS TEXTOS 1 E 2 PARA RESPONDER À
objeto estético, e não meramente linguístico, ao QUESTÃO 03.
qual podemos inferir significados de acordo com
TEXTO 1
nossas singularidades e perspectivas. É comum “Leia a posteridade, ó pátrio rio,
na linguagem não literária o emprego da Em meus versos teu nome celebrado;
conotação, de figuras de linguagem e figuras de Por que vejas uma hora despertado
construção, além da subversão à gramática O sono vil do esquecimento frio;
normativa. Não vês nas tuas margens o sombrio,
IV. Na linguagem não literária, a informação será Fresco acento de um álamo copado;
repassada de maneira a evitar possíveis entraves Não vês ninfa cantar, pastar o gado
para a compreensão da mensagem. No discurso Na tarde clara do calmoso estio.
não literário, as convenções prescritas na Turvo banhando as pálidas areias
Nas porções do riquíssimo tesouro
gramática normativa são adotadas.
O vasto campo da ambição recreias.
V. A linguagem não literária pode ser encontrada na Que de seus raios o planeta louro
prosa, em narrativas de ficção, na crônica, no Enriquecendo o influxo em tuas veias
conto, na novela, no romance e também em Quanto em chamas fecunda, brota em ouro”
verso, no caso dos poemas. Claudio Manuel da Costa

a) I e IV. TEXTO 2
b) II, III e V. Eu, Marília, não sou algum vaqueiro,
c) I, III e IV. Que viva de guardar alheio gado;
d) I e V. De tosco trato, d'expressões grosseiro,
e) Todas as alternativas estão corretas. Dos frios gelos, e dos sóis queimado.
05. Sobre os textos literários, é incorreto afirmar: Tenho próprio casal, e nele assisto;
Dá-me vinho, legume, fruta, azeite;
a) Possui grande compromisso com a clareza e a Das brancas ovelhinhas tiro o leite,
objetividade, podendo ser encontrada em E mais as finas lãs, de que me visto.
reportagens, notícias, manuais de instrução e Graças, Marília bela,
outros textos cuja principal característica seja a Graças à minha Estrela!
Tomás Antônio Gonzaga
informatividade.
b) O discurso literário, diferentemente do discurso 03. (IFPI 2013- CONCOMITANTE / SUBSEQUENTE)
adotado em nosso dia a dia, pode apresentar Podemos afirmar que por serem textos
diversos recursos estilísticos capazes de literários, em ambos o sentido predominante
oferecer múltiplas leituras e interpretações. das palavras é:
c) Uma das principais características da linguagem a) Denotativo, pois apresenta sentido único,
literária é a liberdade criativa, permitindo que o literal;
artista desvincule-se dos padrões convencionais b) Conotativo, pois as palavras ganham um
da língua, bem como da gramática normativa novo contorno e levam à reflexão:
que a rege. c) Denotativo nos três primeiros versos e
d) A complexidade da linguagem literária é notada conotativo nos demais;
no uso de conotações e metáforas, nas quais as d) Somente denotativo, porque se apresenta
palavras extrapolam seu nível semântico. com vários significados;
ALUNO RESPONDE e) Somente conotativo, por apresentar
01. A linguagem literária pode ser encontrada nos palavras em seu sentido real.
seguintes gêneros: A pátria
a) poemas, reportagens, manuais de instrução e Ama, com fé e orgulho, a terra em que nasceste!
textos injuntivos. Criança! não verás nenhum pais como este!
b) crônica, conto, poemas e narrativas de ficção. Olha que céu! que mar! que rios! que floresta!
c) textos prescritivos, notícias, novelas e romance. A Natureza, aqui, perpetuamente em festa,
d) textos jornalísticos, textos didáticos, verbetes É um seio de mãe a transbordar carinhos.
de dicionários e propagandas publicitárias. Vê que vida há no chão! vê que vida há nos ninhos,
Que se balançam no ar, entre os ramos inquietos!
02. Sobre a linguagem não literária é correto afirmar, Vê que luz, que calor, que multidão de insetos!
exceto: Vê que grande extensão de matas, onde impera,
a) É utilizada, sobretudo, em textos cujo caráter Fecunda e luminosa, a eterna primavera!
seja essencialmente informativo. Boa terra! jamais negou a quem trabalha
b) Sua principal característica é a objetividade. O pão que mata a fome, o teto que agasalha...
c) Utiliza recursos como a conotação para conferir Quem com o seu suor a fecunda e umedece,
às palavras sentidos mais amplos do que elas Vê pago o seu esforço, e é feliz, e enriquece!
realmente possuem. Criança! não verás pais nenhum como este:
Imita na grandeza a terra em que nasceste!
d) Utiliza a linguagem denotativa para expressar o
real significado das palavras, sem metáforas ou BILAC, O. Poesias infantis. Rio de Janeiro: Francisco Alves, 1929.
preocupações artísticas.
72
04. (ENEM/2015) Publicado em 1904, o poema A e te estilhaces, suicida.
pátria harmoniza-se com um projeto ideológico numa explosão
em construção na Primeira República. O de diamantes.
PAES. J. P. Prosas seguidas do
discurso poético de Olavo Bilac ecoa esse odes mínimos. São Pauto: Cia.
projeto, na medida em que das Letras, 1992.
a) a paisagem natural ganha contornos surreais, 06. (ENEM/2015) A reflexão acerca do fazer
como o projeto brasileiro de grandeza. poético é um dos mais marcantes atributos da
b) a prosperidade individual, como a exuberância produção literária contemporânea, que, no
da terra, independe de políticas de governo. poema de José Paulo Paes, se expressa por
c) os valores afetivos atribuídos à família devem um(a)
ser aplicados também aos ícones nacionais. a) reconhecimento, pelo eu lírico, de suas
d) a capacidade produtiva da terra garante ao país limitações no processo criativo, manifesto na
a riqueza que se verifica naquele momento. expressão “Por translúcida pões”.
e) a valorização do trabalhador passa a integrar o b) subserviência aos princípios do rigor formal
conceito de bem-estar social experimentado. e dos cuidados com a precisão metafórica,
Yaô como se observa em “prisão da forma”.
Aqui có no terreiro c) visão progressivamente pessimista, em face
Pelú adié da impossibilidade da criação poética,
Faz inveja pra gente conforme expressa o verso “e te estilhaces,
Que não tem mulher suicida”.
No jacutá de preto velho
d) processo de contenção, amadurecimento e
Há uma festa de yaô
Ôi tem nêga de Ogum transformação da palavra, representado
De Oxalá, de lemanjá pelos versos “numa explosão / de
Mucama de Oxossi é caçador diamantes”.
Ora viva Nanã e) necessidade premente de libertação da
Nanã Buruku prisão representa da pela poesia,
Yô yôo simbolicamente comparada à “garrafa” a ser
Yô yôoo “estilhaçada”.
No terreiro de preto velho Iaiá
Vamos saravá (a quem meu pai?) 07. É possível afirmar que a essência da arte
Xangô! literária encontra-se nas:
VIANA, G. Agó, Pixinguinha! 100 Anos. Som Livre, 1997. a) letras
05. (ENEM/2015)A canção Yaô foi composta na b) rimas
década de 1930 por Pixinguinha, em parceria com c) livros
Gastão Viana, que escreveu a letra. O texto d) histórias
mistura o português com o iorubá, língua usada e) palavras
por africanos escravizados trazidos para o Brasil.
Poesia quentinha
Ao fazer uso do iorubá nessa composição, o autor
Projeto literário publica poemas em sacos de pão na
a) promove uma crítica bem-humorada às religiões capital mineira. Se a literatura é mesmo o alimento
afrobrasileiras, destacando diversos orixás. da alma, então os mineiros estão diante de um
b) ressalta uma mostra da marca da cultura verdadeiro banquete. Mais do que um pãozinho com
africana, que se mantém viva na produção manteiga, os moradores do bairro de Barreiro, em
musical brasileira. Belo Horizonte (MG), estão consumindo poesia
c) evidencia a superioridade da cultura africana e brasileira no café da manhã. Graças ao projeto “Pão
seu caráter de resistência à dominação do e Poesia”, que faz do saquinho de pão um espaço
branco. para veiculação de poemas, escritores como
d) deixa à mostra a separação racial e cultural que Affonso Romano de Sant’Anna e Fernando Brant
dividem espaço com estudantes que passaram por
caracteriza a constituição do povo brasileiro.
oficinas de escrita poética. São ao todo 250 mil
e) expressa os rituais africanos com maior embalagens, distribuídas em padarias da região de
autenticidade, respeitando as referências Belo Horizonte, que trazem a boa literatura para o
originais. cotidiano de pessoas, além de dar uma chance a
À garrafa escritores novatos de verem seus textos impressos.
Contigo adquiro a astúcia Criado em 2008 por um analista de sistemas
de conter e de conter-me. apaixonado por literatura, o “Pão e Poesia” já
Teu estreito gargalo recebeu dois prêmios do Ministério da Cultura.
é uma lição de angústia. Língua Portuguesa, n. 71, set. 2011.
Por translúcida pões
o dentro fora e o fora dentro
para que a forma se cumpra
e o espaço ressoe.
Até que, farta da constante
prisão da forma, saltes
da mão para o chão
73
08. (ENEM/2015)A proposta de um projeto como
o “Pão e Poesia” objetiva inovar em sua área (Texto 2)
de atuação, pois O bicho
a) privilegia novos escritores em detrimento Vi ontem um bicho
daqueles já consagrados. Na imundície do pátio
b) resgata poetas que haviam perdido espaços de Catando comida entre os detritos.
Quando achava alguma coisa,
publicação impressa.
Não examinava nem cheirava:
c) prescinde de critérios de seleção em prol da Engolia com voracidade.
popularização da literatura. O bicho não era um cão,
d) propõe acesso à literatura a públicos diversos. Não era um gato,
e) alavanca projetos de premiações antes Não era um rato.
esquecidos. O bicho, meu Deus, era um homem.
09. São características da linguagem literária, (Manuel Bandeira. Em Seleta em prosa e verso. Rio de
Janeiro: J. Olympio/MEC, 1971, p.145).
EXCETO:
a) Variabilidade. I. No primeiro texto, publicado por uma revista,
b) Multissignificação. a linguagem predominante é a literária, pois
c) Denotação. sua principal função é informar o leitor sobre
d) Liberdade na criação. os transtornos causados pelos detritos.
e) Complexidade. II. No segundo texto, do escritor Manuel
Bandeira, a linguagem não literária é
Canção do vento e da minha vida
O vento varria as folhas, predominante, pois o poeta faz uso de uma
O vento varria os frutos, linguagem objetiva para informar o leitor.
O vento varria as flores... III. No texto “Descuidar do lixo é sujeira”, a
E a minha vida ficava intenção é informar sobre o lixo que
Cada vez mais cheia diariamente é depositado nas calçadas
De frutos, de flores, de folhas. através de uma linguagem objetiva e
[...] concisa, marca dos textos não literários.
O vento varria os sonhos IV. O texto “O bicho” é construído em versos e
E varria as amizades... estrofes e apresenta uma linguagem
O vento varria as mulheres...
plurissignificativa, isto é, permeada por
E a minha vida ficava
Cada vez mais cheia metáforas e simbologias, traços
De afetos e de mulheres. determinantes da linguagem literária.
O vento varria os meses 11. Estão corretas as proposições:
E varria os teus sorrisos... a) I, III e IV.
O vento varria tudo! b) III e IV.
E a minha vida ficava c) I, II, III e IV.
Cada vez mais cheia d) I e IV.
De tudo. e) II, III e IV.
BANDEIRA, M. Poesia completa e prosa. Rio 12. Sobre a linguagem não literária é correto
de Janeiro: José Aguilar, 1967. afirmar, exceto:
10. Na estruturação desse texto considerado literário, a)É utilizada, sobretudo, em textos cujo caráter
destaca-se seja essencialmente informativo.
a) a construção de oposições semânticas. b) Sua principal característica é a objetividade.
b) a apresentação de ideias de forma objetiva. c) Utiliza recursos como a conotação para
c) o emprego recorrente de figuras de linguagem, conferir às palavras sentidos mais amplos do
como o eufemismo. que elas realmente possuem.
d) a repetição de sons e de construções sintáticas d) Utiliza a linguagem denotativa para expressar
semelhantes. o real significado das palavras, sem
e) a inversão da ordem sintática das palavras. metáforas ou preocupações artísticas.
e) Explora a denotação.
LEIA OS TEXTOS ABAIXO PARA RESPONDER
À QUESTÃO 11.
(Texto 1)
Descuidar do lixo é sujeira
Diariamente, duas horas antes da chegada
do caminhão da prefeitura, a gerência de uma das
filiais do McDonald’s deposita na calçada dezenas
de sacos plásticos recheados de papelão, isopor,
restos de sanduíches. Isso acaba propiciando um
lamentável banquete de mendigos. Dezenas deles
vão ali revirar o material e acabam deixando os
restos espalhados pelo calçadão.
(Veja São Paulo, 23-29/12/92).
74
LEIA O POEMA ABAIXO E RESPONDA A 15. (IFMA-2016-INTEGRADO) O efeito de humor
QUESTÃO 13. da tira se dá em decorrência de:
POÇAS D’ÁGUA a) o sábio, apesar da pouca idade, está
As poças d´água são um mundo mágico presente na arca de Noé.
Um céu quebrado no chão b) a dúvida do legionário em relação à
Onde em vez de tristes estrelas presença do sábio na arca se confirmar,
Brilham os letreiros de gás Néon. pois, na época do dilúvio, não havia serviço
(Mario Quintana, Preparativos de viagem, de meteorologia.
São Paulo, Globo, 1994.)
c) o sábio e o legionário estarem presentes na
13. Levando-se em conta o texto como um todo, é arca de Noé.
correto afirmar que a metáfora presente no d) haver uma multidão junto com o sábio na
primeiro verso se justifica, porque as poças: arca.
a) Estimulam a imaginação. e) o legionário acreditar que o sábio estava na
b) Permitem ver as estrelas. arca junto com uma multidão.
c) São iluminadas pelo Néon.
d) Se opõem à tristeza das estrelas. LEIA A TIRINHA ABAIXO PARA RESPONDER
e) Revelam a realidade como espelhos. À QUESTÃO 16.
LEIA O TEXTO ABAIXO PARA RESPONDER À
QUESTÃO 14.
O BARULHO DA SURDEZ
Cada vez mais adolescentes convivem com
o zumbido. Só que de normal esse ruído constante
não tem nada. Ele indica, na verdade, que a audição
da molecada está em perigo.
Pode reparar: atualmente, os fones de
ouvido são quase uma extensão do corpo dos
jovens. Só que não desgrudar do aparelho cobra
consequências. Ainda mais quando o barulho que 16. (IFMA-2016-INTEGRADO) Sobre o texto dos
sai dele é similar ao de uma casa de show – algo quadrinhos é CORRETO afirmar que:
recorrente hoje
a) houve falha na comunicação, porque o
[...]
Saúde é vital, set. 2016, p. 56 cozinheiro não é filósofo.
b) houve falha na comunicação quando o
14. (IFMA-2017-INTEGRADO) Do trecho acima, infere- recruta, ao fazer a pergunta, usou termos
se que o zumbido constante: desconhecidos pelo cozinheiro.
a) indica o funcionamento saudável da audição de c) não houve falha na comunicação, pois ambos
adolescentes. empregaram a língua portuguesa para
b) compromete a audição de adolescentes. expressar suas ideias.
c) faz parte, há tempos, do cotidiano de crianças e d) não houve falha na comunicação, pois o
adolescentes. rapaz do terceiro quadrinho explicou ao
d) não é motivo de preocupação, pois os cozinheiro o significado das palavras que ele
adolescentes convivem há tempos com ele. não entendeu.
e) é um barulho normal, porém poucos adolescentes e) houve falha na comunicação, porque o falante
ouvem-no, atualmente. desconhece a culinária brasileira.
LEIA A TIRINHA ABAIXO PARA RESPONDER
À QUESTÃO 15.

75
TEXTO LÍRICO- POEMA  Verso pentassílabo: também conhecidos
como redondilha menor, de numeração
É certo tipo de texto no qual um eu-lírico (a
ímpar, com cinco sílabas;
voz que fala no poema, que nem sempre  Verso heptassílabo: também conhecidos
corresponde à do autor) exprime suas emoções,
como redondilha maior, de numeração
ideias e impressões em face do mundo exterior.
ímpar, com sete sílabas;
Normalmente, os pronomes e os verbos estão em
 Verso eneassílabo: de numeração ímpar,
primeira pessoa e há o predomínio da função
com nove sílabas.
emotiva da linguagem. Observe o lirismo destes
Estrofes
versos:
Este grupo de versos também pode ser dividido
"Ardo em desejo na tarde que arde! em:
Oh, como é belo dentro de mim  Quadra: forma com quatro versos com, no
Teu corpo de ouro no fim de tarde: máximo, sete sílabas;
Teu corpo que arde dentro de mim  Quarteto: forma com quatro versos de oito
Que ardo contigo no fim da tarde"
(Manuel Bandeira) a dez sílabas;
 Quintilha: forma com cinco versos;
ATENÇÃO  Sextilha: forma com seis versos;
Não confunda o eu-lírico com o próprio  Oitava: forma com oito versos;
poeta. Aprenda com Fernando Pessoa que:  Décima: forma com dez versos.
Alguns poemas apresentam FORMAS FIXAS,
O poeta é um fingidor. ou seja, obedecem a um número exato de
Finge tão completamente versos, de estrofes e de rimas. Dentre as
que chega a fingir que é dor principais formas fixas, temos:
a dor que deveras sente.  o acróstico, que é composto de uma só
estrofe cujas letras iniciais formam o nome
Ou seja, as emoções e sentimentos de uma pessoa ou de algo (um objeto, uma
expressos num poema lírico são filtrados pelo cidade, uma paisagem, por exemplo.);
poeta e recriados através da linguagem. Por isso,  a balada, que é composta de quatro
não é correto dizer: “O poeta fala de sua tristeza”. estrofes: três oitavas ou três décimas (oito
É mais exato e próprio afirmar: “O eu lírico fala de ou dez versos) e uma quadra ou quintilha
sua tristeza”. (quatro ou cinco versos);
Características do poema  o haicai, poema de origem japonesa,
O texto poético apresenta características que o composto por uma estrofe com três versos:
distinguem dos outros textos: o primeiro com cinco sílabas (redondilha
 A disposição gráfica, linhas desiguais, menor) e o segundo com sete sílabas
espaços brancos separando as estrofes; (redondilha maior);
 Cada linha do poema é um verso;  o soneto, forma composta por catorze
 Os versos aparecem agrupados em versos divididos em dois quartetos e dois
estrofes; tercetos ou, ainda, por uma estrofe com
 O ritmo que é obtido pela repetição do doze versos e outra com dois versos;
mesmo número de sílabas ou pela  a trova, que é composta de uma estrofe de
acentuação das mesmas sílabas; quatro versos com sete sílabas poéticas
 A rima e os recursos expressivos da língua. (redondilha maior).
Rimas
Versos, Estrofes e Rimas Estas coincidências de sons também
São elementos fundamentais na construção de um possuem suas classificações, podendo ser:
poema. De forma geral, um verso corresponde a  Emparelhadas
uma linha do poema, uma estrofe é uma unidade Conhecidas como tipo AABB. Confira um
com alguns destes versos e a rima é a repetição exemplo de rima emparelhada:
sonora de palavras que podem estar no final ou no Aos que me dão lugar no bonde (A)
interior dos versos. e que conheço não sei de onde, (A)
Versos aos que me dizem terno adeus (B)
Correspondente a uma linha do poema, o verso sem que lhes saiba os nomes seus (B)
(Carlos Drummond de Andrade)
é contado em números e sílabas. Pode ser
dividido em:  Opostas ou Interpoladas
 Verso alexandrino: de numeração par, com
Conhecidas como tipo ABBA. Veja um exemplo
doze sílabas; de rima interpolada:
 Verso decassílabo: de numeração par, com
Eu, filho do carbono e do amoníaco, (A)
dez sílabas; Monstro de escuridão e rutilância (B)
 Verso octossílabo: de numeração par, com
Sofro, desde a epigênese da infância, (B)
oito sílabas; A influência má dos signos do zodíaco (A)
(Augusto dos Anjos)

76
 Alternadas IMPORTANTE!
Conhecidas como tipo ABAB. Observe um NÃO CONFUNDA POEMA COM POESIA
exemplo de rima alternada: POESIA
Minha desgraça, não, não é ser poeta, (A) Do grego poiesis, poesia, no sentido
Nem na terra de amor não ter um eco, (B) etimológico, significa “produção artística” ou
É meu anjo de Deus, o meu planeta (A) ainda “criar” e “fazer”. Essa, portanto, está
Tratar-me como trata-se um boneco (B) presente não apenas em poemas, mas também
(Alvares de Azevedo) em objetos, paisagens e outras formas de
 Versos Brancos expressão.
Dentre os versos existem também os que não As poesias são caracterizadas pela utilização
possuem rima. Neste caso são chamados de versos de recursos para expressar a linguagem de
brancos (ou livres). forma especial e diferente do normal, e provoca
FIGURAS DE EFEITO SONORO diversos efeitos de sentido naqueles que
A-. Aliteração: é a repetição de uma consoante ao recebem a mensagem. É esta forma de escrita
longo de um verso ou ao longo do poema. Leia em que é responsável por dar sentimento ao
voz alta: conteúdo descrito pelas palavras em obras.
Roda mundo, roda vida, roda vento. Graças à ela, os textos possuem emoções e
Passa tudo, passa tanto, passa tempo. transpassam aos leitores.
POEMA
B-. Assonância: é nome dado à figura em que há Os poemas são também poesias, mas usam
repetição de vogais iguais ou semelhantes ao longo a palavra como matéria prima. Trata-se de obras
de um verso ou de um poema. em verso, composições poéticas, ou ainda
O tempo é um fio refere-se à arte de retratar no papel a poesia.
Bastante frágil. Estruturados em versos e estrofes, os
Um fio fino poemas existem por si mesmos. Entenda o que
Que à toa escapa. são estrofes e versos, parte da estruturação dos
C-. Repetição de palavras: recurso muito usado, poemas.
por isso é muito importante verificar que efeito ela BIBLIOGRAFIA
produz no poema.
Para além destas características, o poema CAMPEDELLI, Samira Yousseff. Literatura –
distingue-se, também, por não apresentar o mundo história & texto – vol 1. 8.ed. São Paulo:
exterior de forma objetiva e por, raramente, recorrer Saraiva, 2003.
a uma linguagem denotativa. MAIA, João Domingues. Português: noções
A linguagem poética é ambígua, uma conceituais. 11. ed. São Paulo: Ática, 2004.
linguagem figurada (conotativa) em que as palavras
usadas podem ter outro significado, significado
figurado, para além do mais imediato (significado
literal), valorizando-se a expressividade e a
musicalidade. Para isso contribui o uso de recursos
expressivos.

77
PROFESSOR RESPONDE O vento varria as mulheres...
1. Sobre as características do gênero textual poema, E a minha vida ficava
estão corretas as seguintes proposições: Cada vez mais cheia
I. O poema deve ser construído sob forma fixa, De afetos e de mulheres.
O vento varria os meses
sempre preservando elementos como a métrica e
E varria os teus sorrisos...
a musicalidade dos versos. O vento varria tudo!
II. O poema caracteriza-se por ser centrado em um E a minha vida ficava
trabalho peculiar com a linguagem. Em geral, Cada vez mais cheia
reflete o momento e o impacto dos fatos sobre o De tudo.
homem. BANDEIRA, M. Poesia completa e prosa. Rio de Janeiro: José
Aguilar, 1967.
III. O poema diferencia-se dos demais gêneros por
ser escrito em versos e por possuir um ritmo mais 3. Na estruturação do texto, destaca-se
marcado que o ritmo da prosa. a) a construção de oposições semânticas.
IV. A poesia não é exclusividade do poema: ela é b) a apresentação de ideias de forma objetiva.
uma atitude subjetiva que pode estar nas mais c) o emprego recorrente de figuras de
variadas manifestações artísticas. linguagem, como o eufemismo.
a) Todas estão corretas. d) a repetição de sons e de construções
b) Apenas I está correta. sintáticas semelhantes.
c) II, III e IV estão corretas. e) a inversão da ordem sintática das palavras.
d) I e III estão corretas. LEIA O POEMA ABAIXO:
e) I, III e IV estão corretas. Poética
LEIA O POEMA ABAIXO: Que é poesia?
Soneto de fidelidade uma ilha
(Vinicius de Moraes) cercada
De tudo, ao meu amor serei atento de palavras
Antes, e com tal zelo, e sempre, e tanto por todos os lados
Que mesmo em face do maior encanto Que é um poeta?
Dele se encante mais meu pensamento. um homem
Quero vivê-lo em cada vão momento que trabalha um poema
E em seu louvor hei de espalhar meu canto com o suor do seu rosto
E rir meu riso e derramar meu pranto Um homem
Ao seu pesar ou seu contentamento. que tem fome
E assim, quando mais tarde me procure como qualquer outro
Quem sabe a morte, angústia de quem vive homem.
(Cassiano Ricardo)
Quem sabe a solidão, fim de quem ama
Eu possa me dizer do amor (que tive): 4. No poema de Cassiano Ricardo é possível
Que não seja imortal, posto que é chama encontrar as seguintes funções da linguagem:
Mas que seja infinito enquanto dure. a) função fática e função poética.
2. Nos dois primeiros quartetos do soneto de Vinicius b) função emotiva e função referencial.
de Moraes, delineia-se a ideia de que o poeta c) função conativa e função metalinguística.
a) não acredita no amor como entrega total entre d) função poética e função metalinguística.
duas pessoas. e) função poética e função emotiva.
b) acredita que, mesmo amando muito uma pessoa, LEIA OS VERSOS DE CECÍLIA MEIRELES,
é possível apaixonar-se por outra e trocar de EXTRAÍDOS DO POEMA
amor. Epigrama n.º 8.
c) entende que somente a morte é capaz de findar Encostei-me a ti, sabendo bem que eras somente onda.
com o amor de duas pessoas. Sabendo bem que eras nuvem, depus a minha vida em
d) concebe o amor como um sentimento intenso a ti.
ser compartilhado, tanto na alegria quanto na Como sabia bem tudo isso, e dei-me ao teu destino
tristeza. frágil, fiquei sem poder chorar, quando caí.
e) vê, na angústia causada pela ideia da morte, o 5. O eu lírico reconhece que a pessoa em quem
impedimento para as pessoas se entregarem ao depôs sua vida representava
amor. a) uma relação incerta, por isso os desenganos
LEIA O POEMA ABAIXO vividos seriam inevitáveis.
Canção do vento e da minha vida b) um sentimento intenso, por isso tinha certeza
O vento varria as folhas,
de que não sofreria.
O vento varria os frutos,
O vento varria as flores... c) um caso de amor passageiro, por isso se
E a minha vida ficava sentia enganado.
Cada vez mais cheia d) uma angústia inevitável, por isso seria melhor
De frutos, de flores, de folhas. aquele amor.
[...] e) uma opção equivocada, por isso sempre teve
O vento varria os sonhos medo de amar.
E varria as amizades...
78
ALUNO RESPONDE 07. O poema “A montanha pulverizada” retrata a
LEIA O TEXTO I E RESPONDA À QUESTÃO. região onde Drummond nasceu. O Pico do
(IFMA 2017-INTEGRADO) Cauê faz parte dessa paisagem. No início do
TEXTO I século XX, descobriu-se no local uma das
A NAMORADA maiores jazidas de minério de ferro do mundo,
Havia um muro alto entre nossas casas. que em 1942 passou a ser explorada pela
Difícil de mandar recado para ela.
Companhia Vale do Rio Doce. Sobre o poema
Não havia e-mail.
O pai era uma onça. podemos afirmar que:
A gente amarrava o bilhete numa pedra presa por um cordão a) o eu-lírico se vê triste diante da possibilidade
E pinchava a pedra no quintal da casa dela. de perder as lembranças trazidas pela
Se a namorada respondesse pela mesma pedra montanha, já que a mesma desaparece para
Era uma glória! dar lugar ao progresso (trem-monstro).
Mas por vezes o bilhete enganchava nos galhos da goiabeira b) o eu-lírico mostra-se nacionalista, exaltando
E então era uma agonia.
No tempo da onça era assim.
as riquezas indígenas e naturais do Brasil
Manuel de Barros. Tratado geral das grandezas do ínfimo. Rio de Janeiro: Record, desde seu período colonial.
2001, p.71
c) o eu-lírico mostra-se feliz com a eminência
06. A ÚNICA opção que foge do entendimento no das novas tecnologias de transporte na região
texto acima é em que nasceu, ainda que isso significasse o
a) Do texto depreende-se o relato de uma história desaparecimento das belezas naturais.
vivida há bastante tempo, haja vista que o meio de d) o eu-lírico mostra-se indiferente em relação à
comunicação e o comportamento do pai da modernidade que se apresenta através das
namorada diferem-se dos atuais. locomotivas carregadas de pó de ferro, sendo
b) O eu-lírico considera o muro o obstáculo concreto, para ele natural a extinção dos bens naturais.
o qual também deveria ser ultrapassado para que e) o eu-lírico mostra-se esperançoso com a
houvesse comunicação entre ele e a sua chegada do progresso à sua terra,
namorada. descrevendo as vantagens e desvantagens
c) À época em que o eu-lírico narra a história a única ocorridas na cidade natal.
alternativa de comunicação com a namorada era 08. Sobre o poema abaixo “Adiado o tempo para
por meio do bilhete manuscrito. amar”, considere as afirmativas: (IFMA 2014-
d) O eu-lírico descreve a complicada trajetória do INTEGRADO)
bilhete até a namorada e, ainda assim, não Adiado o tempo para amar
garantiria a chegada desse bilhete ao destino Desculpa meu amor
pretendido pelo eu-lírico. não há tempo para o amor
e) O eu-lírico afirma que o pai da namorada, ao Quando melhor arfar o mar
longo dos tempos, seria uma onça fácil de ser O céu for mais azul
domada. A lua menos leviana
LEIA O TEXTO I E RESPONDA À QUESTÃO.
Desculpa meu amor
(IFMA 2016-INTEGRADO) ´inda é cedo para o amor
TEXTO I
A Montanha pulverizada Quando fenderem os ares
Chego à sacada e vejo a minha serra, Os pássaros da liberdade
a serra de meu pai e meu avô,
Desculpa meu amor
de todos os Andrades que passaram
Temos em breve o nosso amor
e passarão, a serra que não passa.
Era coisa de índios e a tomamos Quando soluçarem os tambores
para enfeitar e presidir a vida na Mãe -Terra distante
neste vale soturno onde a riqueza Quando endoideceram tinindo
maior é a sua vista a contemplá-la. os sinos todos de Cabo Verde
De longe nos revela o perfil grave.
A cada volta de caminho aponta (In: DÁSKALOS, Maria, A.; APA, Lívia; BARRETOS,
Arlindo, op. cit, p.156.)
uma forma de ser, em ferro, eterna,
e sopra eternidade na fluência I. Segundo o texto, o tempo abordado é o
Esta manhã acordo e tempo cronológico e este é utilizado como
Não a encontro.
contagem para amar.
Britada em bilhões de lascas
Deslizando em correria transportadora II. O poema traça uma breve cronologia a
Entupindo 150 vagões respeito do ato de amar.
No trem-monstro de 5 locomotivas III. No trecho, “desculpa meu amor ´inda é
- trem maior do mundo, tomem nota – cedo para o amor”, o fragmento destacado
foge minha serra, vai é característica da linguagem coloquial.
deixando no meu corpo a paisagem IV. No texto há a referência ao amor, o qual
mísero pó de ferro, e este não passa.
ANDRADE, Carlos Drummond de. Boitempo. São Paulo: Companhia das deve ser protelado devido aos
Letras acontecimentos referentes à pátria.

79
Estão corretas apenas: À Instabilidade das Cousas do Mundo
Gregório de Matos
“Nasce o Sol, e não dura mais que um dia,
a) I e III;
Depois da Luz se segue a noite escura,
b) I e II; Em tristes sombras morre a formosura,
c) III e IV; Em contínuas tristezas a alegria.
d) I, II e IV; Porém se acaba o Sol, por que nascia?
e) II, III e IV. Se é tão formosa a Luz, por que não dura?
Como a beleza assim se transfigura?
Como o gosto da pena assim se fia?
LEIA COM ATENÇÃO O TEXTO ABAIXO Mas no Sol, e na Luz falte a firmeza,
A Solidão e sua porta Na formosura não se dê constância,
Carlos Pena Filho E na alegria sinta-se tristeza.
Quando mais nada resistir que valha Começa o mundo enfim pela ignorância,
a pena de viver e a dor de amar E tem qualquer dos bens por natureza
E quando nada mais interessar A firmeza somente na inconstância.”
(nem o torpor do sono que se espalha)
Quando pelo desuso da navalha 11. (IFPI 2014- O poema acima trata:
A barba livremente caminhar a. Da duração efêmera de todas as
e até Deus em silêncio se afastar realidades do mundo;
deixando-te sozinho na batalha b. Da grandeza de Deus e a pequenez
Arquitetar na sombra a despedida humana;
Deste mundo que te foi contraditório c. Dos contrastes da vida;
Lembra-te que afinal te resta a vida d. Da falsidade das aparências;
Com tudo que é insolvente e provisório e. Da duração prolongada do sofrimento.
e de que ainda tens uma saída
Entrar no acaso e amar o transitório. Oh retrato da Morte, oh Noite amiga
09. (IFPI 2014) O poema traz como ideia central o Manuel Maria Barbosa Du Bocage
Oh retrato da Morte, oh Noite amiga,
que está explícito na alternativa:
Por cuja escuridão suspiro há tanto!
a) O valor da vida, mesmo transitória, está Calada testemunha de meu pranto,
acima de qualquer vicissitude. De meus desgostos secretária antiga!
b) Os princípios que regem o mundo são Pois manda Amor que a ti somente os diga,
contraditórios e efêmeros. Dá-lhes pio agasalho no teu manto;
c) A intervenção divina, mesmo em silêncio, Ouve-os, como costumas, ouve, enquanto
comparece para apaziguar a luta do Dorme a cruel, que a delirar me obriga.
homem frente à sua solidão. E vós, oh cortesãos da escuridade,
d) Nem mesmo as pequenas angústias Fantasmas vagos, mochos piadores,
podem ser suavizadas pela lembrança de Inimigos, como eu, da claridade!
Em bandos acudi aos meus clamores;
que a vida ainda perdura.
Quero a vossa medonha sociedade,
e) Os momentos de infelicidade são obra do Quero fartar meu coração de horrores.
acaso e da transitoriedade do amor.
12. (IFPI 2014) Qual(is) item(ns) caracteriza(m)
10. (IFPI 2014) Os textos literários podem aparecer elemento(s) neoclássico(s) no poema?
sob a forma de prosa ou poesia. A prosa tem I. A forma (soneto);
um tema que reflete a realidade exterior, que II. Referência à mitologia (Amor – Cupido –
mostra o mundo de fora, o que o autor observa. filho de Marte e de Vênus);
Ela é mais objetiva, mais concreta que a III. As antíteses;
poesia, e por essa razão se utilizam linhas
continuas. A poesia apresenta linguagem a) I, apenas;
conotativa, escrita em versos. Ela é b) II, apenas;
extremamente subjetiva, apresentando os c) I e II;
sentimentos, a realidade interior. Com base na d) II e III;
explicação, o poema acima caracteriza-se e) I, II e III.
como: TOME COMO REFERÊNCIA O TEXTO ABAIXO
a) Um poema escrito em prosa; PARA RESPONDER À QUESTÃO 13.
b) Um poema escrito em versos livres, sem
métrica; Cordel adolescente, ó xente!
c) Poesia, escrito em verso, com rima e Sou mocinha nordestina,
métrica; Meu nome é Doralice,
d) Um conto feito em prosa; tenho treze anos de idade,
e) Um conto, embora feito em verso. conto e reconto o que disse,
pois me chamo Doralice,
sou quem vende meu cordel
nas feiras lindas de longe
onde a poesia se esconde

80
nas sombras do meu chapéu! 14. (IFPI 2015.1- INTEGRADO) Quanto à
Eu falo tudo rimado estrutura do poema acima, é correto afirmar
no adoçado da palavra que:
do Nordeste feiticeiro; a) O poema está dividido em 4 estrofes e 14
no meu jeito brasileiro,
versos;
aqui vim dizer e digo
que escrevo muito livro b) O poema está dividido em 6 estrofes e 14
que penduro num cordel, versos;
todo fato acontecido c) O poema está dividido apenas em versos, no
eu coloco num papel! total de 10;
Vim pra feira, noutro dia, d) O poema está dividido apenas em versos,
armei a minha poesia no total de 14;
num cordel de horizonte. e) O poema possui 10 estrofes e 4 versos
Quem passava no defronte
daquilo que eu vendia, 15. (IFPI 2015.1- INTEGRADO) A linguagem da
parava e me escutava, poesia é plurissignificativa. Ou seja: tem vários
pois sou mocinha falante, sentidos. Com base nisso, assinale a
declamava o que escrevia! alternativa INCORRETA sobre o poema de
Contei de uma garota Mário Quintana.
que amava um cangaceiro, a) A palavra “medo”, no 1º verso, refere-se ao
era um tal cabra da peste, medo de “descobrir o novo”;
um valentão do Nordeste
b) O verso “Medo que ofusca: luz!” é uma
que montava a Ventania,
trazia susto e coragem metáfora para o medo que move os jovens
por cada canto que ia! para frente;
Virge Maria! c) Em “velho como o mundo”, há uma
O nome da tal mocinha? comparação que mostra a linguagem
Não digo... é um segredo, denotativa predominante no poema;
escrevo o que não devo, d) Em “jovem felino” que sai “da gruta” e segue
invento, pois tenho medo “para frente”, há uma referência ao medo
de contar que a tal menina que impulsiona o jovem;
era... toda fantasia! e) É o medo que impulsiona o jovem a
(...)
Sylvia Orthof. Cordel adolescente, ó xente!. São descobrir a vida.
Paulo, Quinteto, 1996.
TEXTOS 3 E 4 PARA RESPONDER À QUESTÃO
13. (IFPI 2015.1- INTEGRADO) Assinale o trecho em 16
que foram usadas palavras em sentido figurado.
TEXTO 3
a) Sou mocinha nordestina, Meu nome é Doralice “Leia a posteridade, ó pátrio rio,
b) Eu falo tudo rimado no adoçado da palavra Em meus versos teu nome celebrado;
c) Vim pra feira, noutro dia, armei a minha poesia Por que vejas uma hora despertado
d) Contei de uma garota que amava um O sono vil do esquecimento frio;
cangaceiro, Não vês nas tuas margens o sombrio,
e) O nome da tal mocinha? Não digo... é um Fresco acento de um álamo copado;
segredo, Não vês ninfa cantar, pastar o gado
Na tarde clara do calmoso estio.
LEIA O TEXTO ABAIXO PARA RESPONDER
Turvo banhando as pálidas areias
ÀS QUESTÕES 14 e 15.
Nas porções do riquíssimo tesouro
ADOLESCENTE
Mário Quintana O vasto campo da ambição recreias.
A vida é tão bela que chega a dar medo. Que de seus raios o planeta louro
Enriquecendo o influxo em tuas veias
Não o medo que paralisa e gela,
estátua súbita, Quanto em chamas fecunda, brota em ouro”
Claudio Manuel da Costa
mas esse medo fascinante e fremente de
curiosidade que faz TEXTO 4
o jovem felino seguir para frente
Eu, Marília, não sou algum vaqueiro,
varejando o vento
Que viva de guardar alheio gado;
ao sair, à primeira vez, da gruta.
De tosco trato, d'expressões grosseiro,
Medo que ofusca: luz!
Dos frios gelos, e dos sóis queimado.
Cumplicemente, Tenho próprio casal, e nele assisto;
as folhas contam-te um segredo Dá-me vinho, legume, fruta, azeite;
velho como o mundo: Das brancas ovelhinhas tiro o leite,
E mais as finas lãs, de que me visto.
Adolescente, olha! A vida é nova...
Graças, Marília bela,
A vida é nova e anda nua Graças à minha Estrela!
-Vestida apenas com o teu desejo! Tomás Antônio Gonzaga

81
16. (IFPI – INTEGRADO 2013) Podemos afirmar Está(ão) correto(s) o(s) item(ns):
que, em ambos os textos, o sentido a) I.
predominante das palavras é: b) II.
c) I e III.
a) Denotativo, pois apresenta sentido único,
d) II e IV.
literal;
e) II e III.
b) Conotativo, pois as palavras ganham um novo
contorno e levam à reflexão: Observe a imagem e responda:
c) Denotativo nos três primeiros versos e
conotativo nos demais;
d) Somente denotativo, porque se apresenta com
vários significados;
e) Somente conotativo, por apresentar palavras
em seu sentido real.
O problema ecológico
Se uma nave extraterrestre invadisse o espaço
aéreo da Terra, com certeza seus tripulantes diriam
que neste planeta não habita uma civilização
inteligente, tamanho é o grau de destruição dos
recursos naturais. Essas são palavras de um renomado 19. (IFPI 2013) O texto acima pertence ao
cientista americano. Apesar dos avanços obtidos, a gênero literário porque:
humanidade ainda não descobriu os valores a) Tem como objetivo o entretenimento, pois
fundamentais da existência. O que chamamos faz parte do universo imaginário, porém sem
orgulhosamente de civilização nada mais é do que uma perder sua interação com o mundo real.
agressão às coisas naturais. A grosso modo, a tal b) Traduz o mundo real de forma objetiva;
civilização significa a devastação das florestas, a c) Tem como função trazer uma informação
poluição dos rios, o envenenamento das terras e a sobre arte;
deterioração da qualidade do ar. O que chamamos de
progresso não passa de uma degradação deliberada e
d) Faz um protesto contra a arte da caricatura;
sistemática que o homem vem promovendo há muito e) Utiliza uma linguagem denotativa, típica dos
tempo, uma autêntica guerra contra a natureza. textos literários.
Afrânio Primo. Jornal Madhva (adaptado)
LEIA OS TEXTOS I E II ABAIXO E RESPONDA
17. (IFPI 2016- INTEGRADO) Ainda com relação ao A QUESTÃO 20.
texto acima, podemos afirmar que:
a) Trata-se de um texto literário, uma vez que as Texto I
palavras estão empregadas no sentido O tempo passa, e com ele os fatos que
conotativo. constituem as experiências humanas. Isto
b) Trata-se de um texto não literário, cujas não significa que sejam esquecidas: tudo
palavras estão empregadas no sentido pode se conservar enquanto memória afetiva
ou aprendizado.
denotativo.
c) Apesar de ser um texto não literário, apresenta
muitos trechos e expressões figuradas. Texto II
d) Trata-se de um texto poético. O vento varria os meses
e) É um texto informativo, porém com muitas E varria os teus sorrisos...
figuras de linguagem. O vento varria tudo!
LEIA O TEXTO ABAIXO: E a minha vida ficava
A menina tonta passa metade do dia a namorar quem Cada vez mais cheia
passa pela rua, De tudo.
que a outra metade fica pra namorar-se no espelho (Manuel Bandeira)
A menina tonta tem olhos de retrós preto, cabelos de 20. É correto afirmar que
linha de bordar,
e a boca é um pedaço de qualquer tecido vermelho.”
(Manuel de Fonseca) a) o texto I é mais objetivo que o II, além de ter
um ritmo mais marcado.
b) esses textos consideram a passagem do
18. (IFPI 2016- INTEGRADO) O texto acima: tempo de perspectivas opostas.
I. É escrito em prosa. c) o texto II vale-se de sonoridades que dotam
II. É escrito em verso. sua linguagem de intenção expressiva.
III. Emprega linguagem predominantemente d) o texto II perde, por sua subjetividade, o
conotativa. poder de expressar uma convicção sobre o
IV. Emprega linguagem predominantemente tempo que passou.
denotativa. e) esses textos adotam formas diferentes de
linguagem com o mesmo objetivo de
informação.

82
22. Quanto a sua forma, o poema de Luís Vaz de
21. Assinale a opção abaixo que apresenta
Camões é
versos com rimas interpoladas:
a) um soneto
a) Este fundo de hotel é um fim de mundo!
b) uma elegia
Aqui é o silêncio que tem voz. O encanto
c) um madrigal
Que deu nome a este morro, põe no fundo
d) uma écloga
De cada coisa o seu cativo canto.
Manuel Bandeira e) uma ode.

b) De tudo ao meu amor serei atento


Antes e com tal zelo, e sempre, e tanto
Que mesmo em face do meu maior encanto
Dele se encante mais meu pensamento.
Vinicius de Moraes

c) Atirei um limão doce


Na janela do meu bem:
Quando as mulheres não amam,
Que sono as mulheres têm.
Manuel Bandeira

d) Na messe que enloirece, estremece a


quermesse;
O sol, o celestial girassol, esmorece...
E as cantilenas de serenos sons amenos
Fogem fluidas, fluindo a fina flor dos fenos...
Eugênio de Castro

e) Eu sem você
Sou chama sem luz
Jardim sem luar
Luar sem amor.
Vinicius de Moraes

LEIA O POEMA A SEGUIR E RESPONDA À


QUESTÃO 22.

Amor é um fogo que arde sem se ver;

É ferida que dói, e não se sente;


É um contentamento descontente;
É dor que desatina sem doer.

É um não querer mais que bem querer;


É um andar solitário entre a gente;
É nunca contentar-se e contente;
É um cuidar que ganha em se perder;

É querer estar preso por vontade;


É servir a quem vence, o vencedor;
É ter com quem nos mata, lealdade.

Mas como causar pode seu favor


Nos corações humanos amizade,
Se tão contrário a si é o mesmo Amor?

Luís Vaz de Camões

83
GÊNEROS LITERÁRIOS f) O tempo é um dos elementos
fundamentais da narrativa e limita a
A literatura surgiu para atender à necessidade
duração dos fatos narrados.
humana de compreender melhor a realidade e de
transmitir experiências. Todas as culturas, em todo o OBSERVAÇÃO: Embora o gênero narrativo
mundo, desenvolveram sua própria literatura, que era seja quase sempre escrito em prosa, há
oral inicialmente. A literatura escrita, tal como a narrativas estruturadas em versos, como a
conhecemos hoje, só foi produzida posteriormente. epopeia ( narrativa de caráter sublime centrada
O conceito de gênero literário está ligado a nos feitos de um herói) e o cordel.
formas e funções dos textos escritos e tem sofrido 2-GÊNERO LÍRICO
inúmeras variações desde a Antiguidade clássica até
nossos dias. Etimologicamente, a palavra gênero (do Canção do exílio
latim gènus, èris) significa “nascimento, origem, Minha terra tem palmeiras
tronco...”. Para Massaud Moisés, “gêneros são Onde canta o sabiá
famílias de obras dotadas de atributos iguais ou As aves que aqui gorjeiam,
semelhantes”. Não gorjeiam como lá.
Segundo Aristóteles, há três gêneros básicos: Nosso céu tem mais estrelas,
épico, lírico e dramático. Porém, com as nossas várzeas têm mais flores
transformações ocorridas nas sociedades, novos Nossos bosques tem mais vida,
gêneros textuais foram surgindo. O que levou a uma nossa vida mais amores.
diversificação e ampliação dos gêneros literários. No Em cismar, sozinho, à noite,
final da Idade Média, surgiram as novelas de Mais prazer encontro eu lá;
cavalaria, precursoras do romance moderno (século Minha terra tem palmeiras,
XVIII). No século XIX, o conto e a crônica ganharam onde canta o sabiá.
novo fôlego com a acentuada expansão da imprensa. [...]
Assim, o romance, o conto, a crônica e até os
roteiros de cinema e as telenovelas são espécies do Não permita Deus que eu morra,
gênero narrativo, derivadas do antigo gênero épico. Sem que eu volte para lá;
Sem que desfrute os primores
1-GÊNERO NARRATIVO Que não encontro por cá;
Sem qu’inda aviste as palmeiras,
ESSAS MENINAS
Onde canta o sabiá.
As alegres meninas que passam na rua, com
(Gonçalves Dias)
suas pastas escolares, às vezes com seus namorados.
As alegres meninas que estão sempre rindo,
comentando o besouro que entrou na classe e pousou CARACTERÍSTICAS:
no vestido da professora; essas meninas; essas coisas
sem importância. a) O eu-poético (voz que fala) se concentra na
O uniforme as despersonaliza, mas o riso de sua realidade interior e procura expressá-
cada uma as diferencia. Riem alto, riem musical, riem la, geralmente em forma de poema.
desafinado; riem. b) Há o predomínio da subjetividade.
Hoje de manhã estavam sérias, era como se c) Aparece com evidência, além da função
nunca mais voltassem a rir e falar coisas sem poética, a função emotiva ou expressiva da
importância. Faltava uma delas. O jornal dera notícia linguagem.
do crime. O corpo da menina encontrado naquelas
d) Descrições e narrações, quando ocorrem,
condições, em lugar ermo. A selvageria de um tempo
que não deixa mais rir. funcionam sobretudo como pretexto para o
As alegres meninas, agora sérias, tornaram-se eu-poético tratar de seus estados
adultas de uma hora para outra; essas mulheres. interiores, de suas emoções.
(Carlos Drummond de Andrade)

CARACTERÍSTICAS: 2.1.PRINCIPAIS ESPÉCIES DO GÊNERO


LÍRICO
a) Há um enredo, situação de desequilíbrio e um a) Soneto – poema de forma fixa, composto
desfecho. de dois quartetos e dois tercetos.
b) O leitor toma conhecimento desse enredo pelas b) Ode – poema lírico dedicado a alguém.
palavras de um narrador. c) Canção – composição poética para ser
c) Existe uma ou várias personagens que vivem cantada.
os acontecimentos. d) Balada – poema popular antigo, com
d) A descrição, que quase sempre coexiste com a estrofes curtas e refrão.
narração, serve para caracterizar personagens e) Idílio – poemeto sobre a vida pastoril ou
e o espaço onde ocorrem os fatos. rural.
e) É comum que as personagens de uma f) Elegia – poema lírico, triste ou fúnebre.
narrativa conversem entre si. Nesse caso, o
texto pode apresentar diálogos.

84
3-GÊNERO DRAMÁTICO 3.1. PRINCIPAIS ESPÉCIES DO GÊNERO
O casamento de Maria Feia DRAMÁTICO
Personagens a) Tragédia – tem caráter sério, solene;
protagonista enfrenta uma desgraça; registro
1. Lamparina, cangaceiro. mais formal; situação inicial feliz e desfecho
2. Maria Feia, filha de Lamparina.
fatal; personagens são reis, príncipes que
3. Zé das Baratas, o grande matador.
4. Matilde, irmã de Zé das Baratas. sofrem nas mãos dos deuses.
(Cenário: tela ao fundo, com povoado nordestino. b)Comédia – tem caráter cômico, ridículo;
Lamparina e Maria Feia usam trajes de temática do cotidiano, centrada numa sátira
cangaceiro; Zé das Baratas, camisa social da sociedade e dos defeitos humanos;
abotoada no pulso; Matilde, vestido florido de chita registro mais coloquial; situação inicial
e um chapéu. Zé das Baratas e Matilde em cena.) complicada, mas com final feliz;
MATILDE – (para o público) Minha gente, chegue personagens são estereótipos das
mais. Chegue mais pra escutar. Essa é uma debilidades humanas ( o rabugento, o avaro,
história esquisita, que deu no que falar. É o o mesquinho, o apaixonado, etc.).
casamento de Maria Feia, a mulher que ninguém
c ) Farsa – peça teatral breve (século XIV),
queria namorar. E se você quer assistir, preste
atenção que agora vai começar. Porque eu com poucas personagens, explora situações
também fiz parte dela. Eu também estive lá. engraçadas, ridículas e grotescas do
(Matilde se coloca ao lado de Zé. Zé fica olhando cotidiano, de caráter extremamente cômico.
atentamente para o chão, procurando alguma d) Auto – breve peça, geralmente, religiosa.
coisa. Ele começa a usar um borrifador de veneno Tem conteúdo simbólico. As personagens
em volta. Matilde faz anotações numa caderneta.) são entidades ( o pecado, a luxúria, a
ZÉ – Vir pra esse sertão foi a pior coisa que me avareza, a bondade etc.).
aconteceu. Nessa terra não tem barata!
MATILDE – Isso é verdade, Zé. Eu também
(olhando ao redor) não vi nenhuma barata por PROFESSOR RESPONDE
aqui.
ZÉ – Pois eu digo uma coisa. Uma terra que não LEIA O TEXTO E RESPONDA ÀS QUESTÕES
tem barata, não é uma terra civilizada.
DE 01 A 03.
Todo lugar que se preza tem uma baratinha
subindo pelas paredes. Em Paris tem
barata, em Nova York tem barata. Será que só Inconstância dos bens do mundo
aqui é que não tem?! Nasce o Sol, e não dura mais que um dia,
MATILDE – Eu ainda acho que elas estão por aí, Depois da Luz se segue a noite escura,
só que bem escondidas. A culpa mesmo é das Em tristes sombras morre a formosura,
galinhas. Em contínuas tristezas a alegria.
ZÉ – Das galinhas?! Que é que tem as Porém se acaba o Sol, por que nascia?
galinhas?! Se formosa a Luz é, por que não dura?
MATILDE – O pessoal daqui deixa galinha Como a beleza assim se transfigura?
andando solta por tudo que é lugar. E, antes de Como o gosto da pena assim se fia?
você, elas dão conta do serviço. Mas no Sol, e na Luz, falte a firmeza,
ZÉ – Quer dizer que elas comem as baratas? Na formosura não se dê constância,
MATILDE – Barata, carrapato, percevejo e até E na alegria sinta-se tristeza.
piolho de cobra. Começa o mundo enfim pela ignorância,
ZÉ – Não brinca, Matilde! Então (bota a mão no E tem qualquer dos bens por natureza
estômago, enojado) acho que eu vou vomitar. A firmeza somente na inconstância.
[...]
(Gregório de Matos)
Rutinaldo Mirando Batista Júnior.
http://www.ciaateliedasartes.com.br/TEXTOS
TEATRAIS. (Acesso setembro/2011).
01. Com base no texto acima, analise os itens:

CARACTERÍSTICAS: I – Pela estrutura, esse texto é denominado


quarteto.
a) Texto escrito em forma de diálogo ou II – Essa composição poética pertence ao
monólogo. gênero lírico.
b) Destina-se a ser representado.
c) As indicações sobre cenário, movimentação III – Esse tipo de poema tem estrutura fixa:
em cena, falas e gestos do ator são feitas dois quartetos e dois tercetos.
pelas rubricas. Estão corretos os itens
d) Esse gênero só alcança sua realização
a) I e II.
quando transformado em peça, encenada
b) I e III.
num palco ou num espaço apropriado.
c) II e III.
e) Depende da linguagem verbal e da não
d) I, II e III.
verbal.
e) Nenhum deles.

85
02. (IFPI-adaptada) O poema retrata a seguinte
situação vivida pelo eu lírico: 02. (IFPI-2015) Sobre o texto acima é CORRETO
afirmar que
a) Inquietação diante da efemeridade e
instabilidade das coisas. a) narra com lirismo a história de uma
b) A esperança que o homem tem de construir um personagem que conseguiu esquecer um
futuro melhor. falso amor quando encontrou um grande
c) A desculpa por não se preparar para o futuro. amor que chorou.
d) O medo que o homem tem do futuro. b) o autor expõe seu ponto de vista sobre o
e) O medo de um futuro incerto. relacionamento amoroso, contando sua
própria história de amor.
03. (IFPI-adaptada) Por se tratar de um poema, o
c) o autor expressa a ideia, numa linguagem
texto apresenta palavras com várias
poético-argumentativa, de que o verdadeiro
possibilidades de interpretação. Marque a opção
amor sempre vence.
que melhor explica essa escolha.
d) só há elementos descritivos e esses foram
a) Há a predominância do sentido denotativo, uma transformados em poema, para acrescentar
característica desse tipo de texto. afetividade à poesia.
b) As palavras têm sentido conotativo, por e) só se esquece um amor com o perdão.
abordarem fatos de forma objetiva.
03. (IFPI-2015) Nos versos:
c) O emprego de figuras de linguagem, uma
característica desse tipo de texto. “Há de vencer o grande amor
d) São mescladas palavras em sentido conotativo e Que há de ser no coração
denotativo, com predominância do sentido Como um perdão para quem chorou.”
literal. O poeta expressa
e) O caráter subjetivo do texto é garantido pelo uso
da denotação. a) seu estranhamento com os percalços do
amor.
04. São exemplos de espécies de texto do gênero b) sua frustração por ter perdido um grande
lírico, EXCETO: amor.
a) soneto e balada. c) sua confiança no triunfo do amor.
b) haicai e elegia. d) seu estranhamento por ter amado em vão.
c) ode e idílio. e) a desilusão do amor.
d) epopeia e epístola.
e) canção e poema. LEIA O TEXTOABAIXO E RESPONDA ÀS
QUESTÕES 04 E 05.
ALUNO RESPONDE A menina tonta passa metade do dia
A namorar quem passa pela rua,
LEIA O TEXTO PARA RESPONDER AS Que a outra metade fica
QUESTÕES DA 01 A 03. Pra namorar-se no espelho
A menina tonta tem olhos de retrós preto,
O grande amor Cabelos de linhas de bordar,
(Tom Jobim)
Haja o que houver e a boca é um pedaço de qualquer tecido
Há sempre um homem para uma mulher vermelho.
(Manuel de Fonseca)
E há de sempre haver
Para esquecer um falso amor
E uma vontade de morrer 04. (IFPI) O texto acima:
Seja como for I - É escrito em prosa.
Há de vencer o grande amor II - É escrito em verso.
Que há de ser no coração III - Emprega linguagem predominantemente
Como um perdão para quem chorou. conotativa.
IV - Emprega linguagem predominantemente
01. (IFPI-2015) Quanto à estrutura do poema denotativa.
acima, é CORRETO afirmar que está (ão) correto(s) o(s) item (ns):
a) I.
a) o poema está dividido em 1 estrofe e 9 b) II.
versos. c) I e III.
b) o poema está dividido em 9 estrofes. d) II e IV.
c) o poema está dividido apenas em versos, no e) II e III.
total de 10.
d) o poema é uma narrativa, por isso não está
elaborado sob a forma de versos.
e) o poema é descritivo, por isso não está
elaborado sob a forma de versos.
86
05. Que palavra/expressão foi empregada no 08. (IFPI-adaptada) Com relação ao texto,
sentido conotativo? podemos afirmar que:

a) “A menina tonta passa metade do dia’. a) Trata-se de um texto literário, uma vez que
b) “... a namorar quem passa pela rua,...”. as palavras estão empregadas no sentido
c) “... pra namorar-se no espelho...”. conotativo.
d) “A menina tonta tem olhos de retrós preto, b) Trata-se de um texto não literário, cujas
...”. palavras estão empregadas no sentido
e) “... cabelos de linha de bordar...”. denotativo.
LEIA O TEXTO ABAIXO E RESPONDA ÀS c) Apesar de ser um texto não literário,
QUESTÕES 06 E 07 apresenta muitos trechos e expressões
figuradas.
O homem da Pré-História era nômade, ou d) É um texto informativo, porém com muitas
seja, não possuía moradia fixa. Por isso, vivia se figuras de linguagem.
deslocando a pé em busca de alimentos, e) Trata-se de um texto poético, não literário.
principalmente frutos, ervas e raízes. Os povos
nômades vivem em tribos e, ainda hoje, é
09. Pelas características apresentadas, esse
possível encontrá-los, principalmente nos
desertos. O camelo é muito utilizado pelos povos texto pertence ao gênero:
nômades, como meio de transporte.
( Recreio. Coleção De olho no mundo: Meios a) Épico.
de transporte. Vol. 18, p. 6-7. Fragmento.) b) Narrativo.
06. De acordo com o texto, nômade significa c) Dramático.
d) Lírico.
a) a pessoa que vive procurando alimentos. e) Epistolar.
b) a gente que vive em tribos no deserto.
c) o homem que muda sempre de moradia.
d) o povo que usa o camelo para transporte.
e) o ser que vive na África, no deserto.

07. (IFPI-adaptada)Trata-se de um texto,


predominantemente
a) Narrativo.
b) Descritivo.
c) Informativo.
d) Argumentativo.
e) Injuntivo.

LEIA O TEXTO ABAIXO E RESPONDA ÀS


QUESTÕES 08 E 09.
* Amigo
(Vinícius de Moraes)
Enfim, depois de tanto erro passado
Tantas retaliações, tanto perigo
Eis que ressurge noutro o velho amigo
Nunca perdido, sempre reencontrado.
É bom sentá-lo novamente ao lado
Com olhos que contêm o olhar antigo
Sempre comigo um pouco atribulado
E como sempre singular comigo.
Um bicho igual a mim, simples e humano
Sabendo se mover e comover
E a disfarçar com o meu próprio engano.
O amigo: um ser que a vida não explica
Que só se vai ao ver outro nascer
E o espelho de minha alma multiplica.

(* Título atribuído por razões didáticas)

87
2023

Matemática

R
INSTITUTO

É FEDERAL
PIAUÍ

Escola Municipal________________________________________________
Estudante:_______________________________________________________
SÚMARIO

1. Conjuntos Numéricos ......................................................................................................... 01


2. Potenciação e Radiciação .................................................................................................. 10
3. Unidades de Medidas ......................................................................................................... 18
4. Razão e Proporção ............................................................................................................. 23
5. Cálculo algébrico, Produto Notáveis e Fatoração .............................................................. 36
6. Equações, Inequações e sistemas de equações do 1º grau ............................................. 44
7. Equação do 2º grau ............................................................................................................ 52
8. Função do 1º grau ............................................................................................................ 57
9. Função quadrática ............................................................................................................ 67
10. Geometria plana ............................................................................................................... 76
11. Congruências e semelhança de polígonos ..................................................................... 87
12. Teorema de Talles ........................................................................................................... 92
13. Áreas de figuras planas .................................................................................................... 96
14. Segmentos e pontos notáveis do triângulo ..................................................................... 100
15. Relação trigonométricas nos triângulos retângulos ......................................................... 104
16. Relações métricas na circunferência e no circulo............................................................. 109
17. Bibliografia ........................................................................................................................ 113
18. Sites ................................................................................................................................. 113
CONJUNTO NÚMERICOS Z*- é conjunto dos números inteiros negativos
{ -1 ,- 2 , -3 , . . . }
CONJUNTO DOS NÚMEROS INTEIROS
No conjunto dos números naturais Reta numérica
N = { 0, 1, 2, 3, 4, ... } operações do tipo Podemos representar os elementos do
conjunto dos números inteiros na reta
9 - 5 = 4 é possível
numérica da seguinte forma: marcamos o
5 – 5 = 0 é possível ponto de origem que é o zero, para a esquerda
escrevemos os números negativos e para a
5 – 7 = ? não é possível e para tornar isso
possível foi criado o conjunto dos números direita os números positivos.
inteiros representados por Z.
Z = { . . . ,- 4 ,- 3 ,- 2 ,- 1 , 0, 1 , 2 , 3 , 4 , . . . }

Subconjuntos dos inteiros ( Z ) Número oposto ou simétrico

Observe o conjunto Z = {. . . , - 4 , - 3 , - 2 , - 1 São números que se encontram a uma


,0,1,2,3,4,...} mesma distância em relação ao ponto zero
na reta numérica.
Z* é conjunto dos números inteiros não-nulos
{ . . . , - 3 , - 2 ,- 1 , 1 , 2 , 3 , . . . } a) O oposto de +3 é - (+3) = -3
Z+ é conjunto dos números inteiros não- b) O oposto de -5 é - (-5) = 5
negativos { 0 , 1 , 2 , 3 ,. . . } Na prática, oposto de um número representa
Z- é conjunto dos números inteiros não- o mesmo número apenas com sinal contrário.
positivos { . . . , - 3 , - 2 , - 1 , 0 }
Z*+ é conjunto dos números inteiros positivos
{1,2,3,...} Módulo ou valor absoluto
Módulo ou valor absoluto de um número é a Exemplos:
ideia de quantidade que ele representa, ou
a) + 2 – 3 = – 1
seja, não consideramos o sinal de + ou de - .
b) - 5 + 3 = – 2
Exemplos:
c) + 8 – 3 = 5
a) + 3 valor absoluto 3.
b) - 5 valor absoluto 5. Multiplicação e divisão de números
inteiros
c) |+ 14| = 14.
Para multiplicar ou dividir dois ou mais
d) |- 24| = 24. números inteiros, multiplicamos ou dividimos
os algarismos e o sinal do resultado será:
Adição e subtração
Positivo se os dois termos tiverem o mesmo
Para somar ou subtrair números inteiros é
sinal.
necessário aprender duas regrinhas que
facilitarão muito o cálculo das operações. Exemplos:
1°) Quando dois ou mais números inteiros têm a) ( + 2 ) . ( + 3 ) = + 6
o mesmo sinal somamos e conservamos o
b) (– 5 ) . (– 2 ) = + 10
sinal.
c) ( - 15 ) : ( - 3 ) = + 5
Exemplos:
Negativo se os dois termos tiverem sinais
a) + 2 + 3 = + 5
diferentes.
b) – 5 – 4 = - 9 Exemplos:

2°) Quando dois números inteiros têm sinais a) ( + 3 ) . (– 4 ) = – 12


diferentes subtraímos (maior – menor) em b) (– 5 ) . ( + 4 ) = – 20
valor absoluto e conservamos o sinal do
c) ( + 20 ) : (– 5 ) = – 4
número de maior valor absoluto.

1
CONJUNTO DOS NÚMEROS RACIONAIS
(Q)
NÚMERO RACIONAL – É todo número que
𝑎
pode ser escrito na forma com b ≠ 0 em
𝑏
que a é numerador e b é denominador. Veja que 13 dividido por 5 é igual a 2 e resta
Representação fracionária 3, um número misto é formado pela seguinte
estrutura
Para se representar uma fração através de
figuras, devemos dividir a figura em partes
iguais, em que o numerador representa a
parte considerada (hachurada) e o
denominador representa a quantidade de 3
partes que a figura foi dividida. formando assim o número misto 2
5
Exemplos:
Dado um número misto podemos representá-
lo na forma fracionária.
2
Exemplo: 5
3

3 3 3 3 3 2
+ + + + + =
3 3 3 3 3 3
17
3
De modo prático, podemos fazer da seguinte
forma:

Fração Imprópria
Fração imprópria é a fração que representa
uma quantidade maior que o inteiro.
13
Exemplo:
5 Fração de quantidade
Exemplos:
3
5 5 3 a) Um candidato a prefeito foi eleito com dos
+ + =
4
5 5 5 1 260 votos. Com quantos votos ele foi eleito?
13 1 260 votos
5
13 315 315 315 315
Note que os estão representados por dois
5 1 1 1 1
inteiros e três quintos, que representamos por + + +
4 4 4 4
3
2 (número misto).
5
3
13
De modo prático, para representar em um 4
5
número misto, basta dividir 13 por 5. 3
Os correspondem a 3 x 315 = 945.
4
3 3
De modo prático: de 1 260 = x 1 260 = 945.
4 4

2
b) Em um concurso público, 10.650 OPERAÇÕES:
candidatos fizeram a prova. Essa quantidade
3 Adição e subtração
representa apenas dos candidatos inscritos.
5
Quantas pessoas se inscreveram no * Com mesmo denominador
concurso?
1 2 4 1+2−4 1
Representação da situação: Ex: ( ) - (− ) + (− ) = =
5 5 5 5 5
* Com denominadores diferentes
3 550 3 550 3 550 3 2 1 3 2 1
Ex: = ( ) + (− ) + (+ ) = - +
2 3 4 2 3 4
18 2 8 13
= - + =
10 650 12 3 12 12
1
Observa-se que cada corresponde a 3.550
5 Multiplicação
5
então, o inteiro que são correspondem a 5 Para efetuarmos a multiplicação fazemos o
5 jogo de sinal e multiplicamos os numeradores
x 3 550 = 17 750.
entre si e os denominadores entre si.
Frações equivalentes Exemplos:
São frações que representam a mesma parte 1 1 1
a) (+ ) . (− ) = −
de um inteiro. 2 3 6

1 1 5 5
b) (+ ) . (− ) . (+ ) = −
2 3 2 12

3 6
5 10 1 6
c) (− ) . (0,2) . (+ ) . (0,5)
Exemplos: 2 5
1 2 6 5
3 6 = (− 2) . (10) . (+ ) . (10)
5
a) =
5 10
60 6 3
= − 1000 = − 100 = −
50
2 6
b) =
3 9
Divisão
Pelo método prático, para dividirmos duas
Simplificação de fração frações conservamos a primeira e
Simplificar uma fração é encontrar a menor multiplicamos pelo inverso da segunda.
fração equivalente, que chamamos de fração
irredutível, fazemos isto dividindo o
numerador e o denominador por um ou mais Exemplos:
divisores até encontrar uma fração irredutível. 1 2
a) = (− 2) : (+ 3)
Exemplos:
1 2 3
= (− 2) . (+ 3) =
2
6:2 3
a) =
10:2 5
3 1
b) = (− 2) : (+ 6) =
6:3 2 3 6 18
b) = = (+ 2) . (− 1) = + 2 = 9
9:3 3

3
Representação decimal b) Calcular a fração geratriz de 0,151515...
Para representar uma fração na forma X = 0,151515... (I)
decimal devemos dividir o numerador pelo
100x = 15,151515... ( II )
denominador, assim podemos ou não obter
um número decimal exato. Fazendo ( II ) – ( I )
1 3 100𝑥 = 15,151515 … ( 𝐼 )
= 0,5 − 2 = - 1,5 𝑥 = 0,151515 … ( 𝐼𝐼 )
2
99𝑥 = 15,000
9 6 15
= 3 = 1,2 𝑥 = 99
3 5
Utilizamos essa representação para localizar
c) Calcular a fração geratriz de
o número racional na reta numérica. 0,2353535...

X = 0,2353535... ( I )
10x = 2,353535... ( II )
Podemos também representar um número
1000x = 235,353535... ( III )
decimal exato na forma de fração.
Fazendo ( III ) – ( II )
1000𝑥 = 235,353535 … ( 𝐼𝐼𝐼 )
10𝑥 = 2,353535 … ( 𝐼𝐼 )
990𝑥 = 233,000
233
𝑥 = 990
Dízima periódica
Potenciação
É o resultado de uma divisão não exata do
numerador pelo denominador em que a parte
decimal apresenta uma repetição de
algarismos o qual chamamos de período.
4
= 0,44444 ou podemos representar a O expoente indica quantas vezes a base será
9
dízima periódica substituindo as reticências multiplicada por ela mesma.
4 Uma consideração importante:
(...) por um traço acima do período. Assim
9 Todo número elevado a zero é sempre igual
= ̅̅̅̅
0,4
a 1.
124 124
Veja, = 1,252525... ou = ̅̅̅̅̅̅
1,25 30 = 1
99 99
(– 5 ) 0 = 1
Fração geratriz – 40 = – 1
A partir de uma dízima periódica podemos Radiciação
encontrar a fração que ao ser dividida temos
como resultado essa dízima. A essa fração Raiz quadrada é um número que multiplicado
damos o nome de fração geratriz. por ele mesmo seja igual ao radicando.
Exemplos:
a) Calcular a fração geratriz de 0,222...
X = 0,222... ( I )
10x = 2,222... ( II )

Fazendo ( II ) – ( I )
10𝑥 = 2,222 … ( 𝐼 ) Quando o radical não apresenta índice, ele é
𝑥 = 0,222 … ( 𝐼𝐼 ) 2 por isso chamamos de raiz quadrada.
9𝑥 = 2,000
O número 12 multiplicado por ele mesmo 12 x
2
𝑥= 12 é igual ao radicando 144.
9

4
PROFESSOR RESPONDE
P1) (IFAM – 2013) Na expressão 𝐴 = ALUNOS RESPONDE
1
1+3
+ 3 o valor de A é igual a: A1) (IFAM – 2015) Num prédio de
1
1−3 apartamentos há 20 andares acima do térreo
e 5 andares no subsolo. Um elevador
(A) 4 encontra-se parado no 3º andar do subsolo e
(B) 5 executa o seguinte itinerário: sobe 17
(C) 6 andares, sobe mais 4 andares, desce 5
(D) 7 andares, desce mais 14 andares e finalmente
(E) 8 sobe 2 andares, onde ficou estacionado no:
P2) (IFPI – 2015) Escrevendo na forma de (A) Térreo
fração a divisão 0,161616... ÷ 0,4777...
obtemos: (B) 1º andar
160 (C) 2º andar
(A)
473
(D) 1º andar do subsolo
16
(B) (E) 2º andar do subsolo
99
43
(C) A2) (IFMA – 2016) Dadas as alternativas:
90
I. A raiz quadrada de um número inteiro
16
(D) positivo sempre é exata
47 II. O número oposto do módulo de – 5 é raiz
34 quadrada de 25.
(E)
10 III. O número 250000 tem raiz quadra exata.
Das afirmações acima:
P3) (IFF – 2014) João tem 840 figurinhas e
2 (A) I e II são verdadeiras.
resolveu dividi-las com seus amigos. Deu de (B) II e III são corretas.
7
3 (C) I e III são falsas
suas figurinhas para André e das figurinhas (D) somente a III está correta
4
restantes para Carlos. Depois dessa (E) somente a II é verdadeira
distribuição, João ficou com
(A) 100 figurinhas. A3) (IFPI – 2015) Com relação aos conjuntos
(B) 120 figurinhas. numéricos, podemos afirmar que:
(C) 130 figurinhas.
(D) 140 figurinhas. (A) Toda dízima é número irracional;
(E) 150 figurinhas. (B) 5 ∉ Z (conjunto dos números inteiros);
P4) (IFMA – 2017) Os irmãos Pâmela, João e (C) Os números reais são somente aqueles
1 1 que podem ser representados pela razão
Roberto receberam, respectivamente, 4
, e entre dois números inteiros;
6
1 (D) Toda dízima periódica é número racional;
, de uma determinada herança. A fração (E) Todo número inteiro é natural.
18
desta herança que não foi distribuída entre
estes irmãos foi: A4) (IFAM – 2013) Considere o número x =
19 11
(A) .
36 12
17 É correto afirmar que, nesse caso, x é um
(B) número:
36
21 (A) natural
(C) (B) inteiro.
36
13 (C) racional.
(D)
36 (D) irracional.
11
(E) (E) real maior que 1.
36

5
A5) ( IFMA – 2016 ) Em qual das alternativas A9) (IFG – 2017) A família Sousa utiliza
17 69 botijões de gás de 30 kg. No primeiro
aparece um número que fica entre e semestre de 2016 o consumo desta família foi
5 7
na reta real? 4
o seguinte: Em janeiro, ela consumiu de um
5
5
(A) 11 botijão, fevereiro consumiu , março o
(B) 10 6
2 7
(C) 8 consumo foi de , em abril foi consumido
(D) 12 3 10
8 1
(E) 3 de um botijão, em maio e por fim, foi o
15 2
consumo em junho. Com estas informações,
A6) (IFPI – 2016) No número 3, 444..., o
podemos afirmar que:
algarismo 4 repete- se indefinidamente. Esse
número é chamado de dízima periódica de (A) O mês de maior consumo foi fevereiro com
parte inteira 3 e período 4. Qual a fração 25 kg de gás.
geratriz dessa dízima periódica? (B) A família Sousa consumiu exatamente 4
botijões de gás neste semestre.
34 (C) O mês de menor consumo foi maio.
(A) (D) A família Sousa consumiu menos de 4
10
botijões de gás no semestre.
31 (E) A família Sousa consumiu mais gás em
(B) janeiro do que em fevereiro.
10
A10) (IFMA– 2016) Sejam a= -13+7, b= -9 -
34 3 2 5
(C) 21, c=− + , e d= 4.(− ).Qual o valor da
9 4 3 8
𝑎 𝑐
30 expressão : ?
(D) 𝑏 𝑑
10 (A) – 4
31 (B) 6
(E)
9 6
(C)
5
3 8
A7) (IFAM – 2013 ) Sejam os números 𝑝
2 , (D)
1 2 5
𝑞 3 e 𝑟 3 , temos que:
1
(E)
150
(A) p < q
(B) p < q . r A11) (IFPI – 2016) Na reta numérica da figura
(C) p + q > p + r a seguir, temos o segmento PR dividido em
(D) p . q = p . r cinco segmentos congruentes, ou seja, PA
(E) p . r = q + r =AB= BC= CD= DR.

A8) (IFF – 2013) Suponha que numa viagem


Se os pontos A e D representam os números
à Europa uma pessoa gaste R$ 6.000,00.
1 7
3 e , respectivamente, então o ponto R
Sabendo que desta quantia destinou-se à 2 4
5 representa o número
hospedagem e a passeios; e o restante, à
12
passagem aérea. A quantia correspondente à (A)
passagem aérea é: 5
10
(B)
(A) R$ 1.200,00 3
(B) R$ 1.800,00 7
(C)
(C) R$ 2.400,00 3
(D) R$ 3.000,00 14
(E) R$ 3.600,00 (D)
5
13
(E)
6

6
A12) (IFPI – 2014 ) A soma da dízima 4
7 A15) (IFG–2015) O número racional tem
periódica 2,666... com a fração é: 7
9 como representação decimal a dízima
31 periódica 0, 571428571428571428.... O 101º
(A) algarismo dessa representação é
9
24 (A) 7
(B)
9 (B) 5
10 (C) 4
(C) (D) 2
9 (E) 1
10
(D) A16) (IFPI–2014) A alternativa que apresenta
3
5 8 2 36
25 ̅̅̅̅̅̅̅; √ ;
os números ; ( ) ; 0,6928; 0,619
(E) 8 10 81
3 em ordem decrescente é:
𝑚 5 8 2 36
A13) (IFPI–2016) Sendo a fração ̅̅̅̅̅̅̅; √
(A) ; ( ) ; 0,6928; 0,619
𝑛 8 10 81
1,8666…
irredutível equivalente a é correto 36 8 2 5
0,4444… (B) 0,6928; √ ̅̅̅̅̅̅̅
; (10) ; 8 ; 0,619
81
afirmar que é igual a:
5 8 2 36
(A) 22 (C) ; ( ) ; 0,6928; √ ̅̅̅̅̅̅̅
; 0,619
8 10 81
(B) 23 2
36 8
(D) 0,6928 ; √ ̅̅̅̅̅̅̅; 5
; (10) ; 0,619
(C) 24 81 8
(D) 25 2
5 8 36
̅̅̅̅̅̅̅; √
(E) ; 0,6928; ( ) ; 0,619
(E) 26 8 10 81

A14) (IFMA–2013) Um possível resultado, de A17) O valor de 1 − 5. (


32
) + 0,75 é:
acordo com o esquema abaixo, é: 45

(A) 0
33
(B) 44
(C) 1
1
(D) 20
(E) 2

2 1 2
A18) Calculando 3 ∶ (1 − 0,25) + 2 . (3 −
0,75), temos como resultado:
61
(A) 72
51
(A) 10 (B) 72
(B) 4 61
(C)
1 12
(C) 61
4 (D) 52
1 11
(D) (E)
5 12
(E) 1

7
4.(0,3) 4.(0,3) Assinale a igualdade que relaciona
A19) O valor de 2−1,4 2−1,4
é:
corretamente A, B e C.
a) A  B  C
1 b) A  C  B
(A) c) A  B  C
2
1 d) B : A  C
(B) 3
(C) 1 2. (G1 - cp2 2020) Em um campus do Colégio
Pedro II, foi realizada uma pesquisa para
(D) 2 saber quais eram as redes sociais preferidas
(E) 3 por 180 estudantes com idades entre 9 e 15
anos, sendo que cada estudante deveria citar
uma única rede. Os resultados apontaram que
7 2 7 1
A20) A expressão 0,333 … + − ( + 2) − desses estudantes preferia utilizar o
2 3 2 4
2
( + 2)representa o número: 1
3 Facebook, o Instagram, e o restante dos
3
entrevistados preferia utilizar o Twitter.
1
(A)
2 A fração que corresponde à quantidade de
1 estudantes que declararam preferência pelo
(B) Twitter é:
3
7 2
a)
(C) 7
6
5
3 b)
(D) 7
2 5
5 c)
(E) 12
6 7
d)
12
3. (G1 - cmrj 2019) Em um campeonato de
ATIVIDADE COMPLEMENTAR tiro ao alvo, Arthur, Bruno e César começaram
a atirar juntos, sempre efetuando disparos
1. (G1 - cftrj 2020) As frações podem ser simultaneamente. Arthur foi o primeiro a
representadas graficamente de diversas acertar um tiro no alvo, em sua segunda
formas. Observe um exemplo de tentativa. Em seguida, Bruno acertou o alvo
4 ao disparar pela terceira vez. Por fim, César
representação gráfica da fração :
7 consegue acertar no alvo no seu quarto tiro.
Após o primeiro tiro certo no alvo de cada
competidor, observou-se o seguinte padrão:
Considere as frações A, B e C, referentes à Arthur: 3 tiros errados, seguidos de um tiro
mesma unidade, representadas a seguir: certo no alvo.
A Bruno: 5 tiros errados, seguidos de um tiro
certo no alvo.
César: 7 tiros errados, seguidos de um tiro
certo no alvo.
B No campeonato, cada competidor disparou
420 tiros. O número de vezes em que os três
competidores acertaram, simultaneamente, o
alvo é igual a
C
a) 4
b) 3
c) 2
d) 1
e) 0

8
4. (G1 - ifpe 2018) Chamamos uma fração de a) apenas dois desses números, em sua
unitária se o numerador for igual a um e o forma decimal, são representados por
denominador for um inteiro positivo, por dízimas periódicas.
1 1 1 b) apenas um desses números, em sua forma
exemplo: , , . Os antigos egípcios
3 7 2 decimal, é representado por uma dízima
costumavam trabalhar com frações que periódica simples.
poderiam ser obtidas como soma de frações c) os três números, em sua forma decimal,
unitárias diferentes, por exemplo: podem ser representados por dízimas
periódicas tais que o período de cada uma
5 1 1 delas é um número primo.
  . d) os três números, em sua forma decimal,
8 2 8
podem ser representados por dízimas
Por esse motivo, esse tipo de fração, que periódicas tais que o período de cada uma
pode ser obtido por soma de frações unitárias delas é um número divisível por 3.
distintas, é conhecido por “frações egípcias”.
O uso das frações egípcias facilitava as 7. (G1 - ifsp 2016) Um pesquisador tem à
contas e comparações, especialmente num disposição quatro frascos com a mesma
mundo onde não havia calculadoras. substância. No frasco I, há um quarto de litro
dessa substância; no frasco II, há um quinto
Encontre uma fração, F, equivalente à soma de litro dessa substância; no III, há um oitavo
de litro dessa substância; e no frasco IV há um
décimo de litro da substância. Se ele utilizar
1 1 1 1
F    . os dois frascos que mais contêm dessa
3 4 6 7 substância, ele terá utilizado, ao todo:
a) 77 84. a) dois nonos de litro.
b) 51 56. b) dois dezoito avos de litro.
c) 25 28. c) nove vinte avos de litro.
d) 73 84. d) nove quarenta avos de litro.
e) um nono de litro.
e) 49 56.
8. (G1 - cftmg 2014) Um grupo de alunos cria
5. (G1 - ifpe 2018) Pedro, um aluno do curso um jogo de cartas, em que cada uma
de Almoxarife do IFPE – Cabo, em seu apresenta uma operação com números
estágio, se deparou com a seguinte situação: racionais. O ganhador é aquele que obtiver
no almoxarifado, encontravam-se 20 caixas um número inteiro como resultado da soma de
de lápis, cada caixa com 30 lápis. Ele suas cartas. Quatro jovens ao jogar
1 receberam as seguintes cartas:
precisava mandar dessas caixas para o
10
laboratório de matemática. Ao abrir as caixas 1ª carta 2ª carta
que chegaram ao laboratório, o professor de
5 4 7
matemática colocou dos lápis sobre as Maria 1,333...  1,2 
6 5 3
mesas, guardando o restante dos lápis no
armário. 1 1
Selton 0,222...  0,3 
Nessas condições, podemos afirmar que o 5 6
professor guardou, no armário do laboratório,
um total de 3 8
a) 10 lápis. Tadeu 1,111...  1,7 
10 9
b) 20 lápis.
c) 30 lápis. 7 1
d) 40 lápis. Valentina 0,666...  0,1 
2 2
e) 50 lápis.

6. (G1 - cftmg 2016) Sobre os números O vencedor do jogo foi


1 7 14 a) Maria.
racionais , e , é correto afirmar que
11 33 55 b) Selton.
c) Tadeu.
d) Valentina.

9
POTENCIAÇÃO E RADICIAÇÃO ● POTÊNCIA COM EXPOENTE 1

POTENCIAÇÃO 𝒂𝟏 = 𝒂
Exemplo: Exemplos:
Um condomínio é formado por 8 edifícios. 1001 = 100 (0,08)1=0,08
Cada edifício tem 8 andares. Cada andar tem (−3)1 = (=3) (¾)1 = ¾
8 apartamentos. Quantos apartamentos há no
condomínio? (√2)1 = √2 10241=1024
8 ∙ 8 ∙ 8 = 512 apartamentos
● POTÊNCIA COM BASE 10
Um produto de fatores iguais pode ser escrito
de forma simplificada através da potenciação. Exemplos:
Veja: 100= 1 104 = 10 000

EXPOENTE 101= 10 105 = 100 000


102= 10 ∙ 10 = 100 10 6
= 1 000 000
8 . 8 . 8 = 83 = 512 POTÊNCIA 10 = 10 ∙ 10 ∙ 10 = 1000
3

RESULTADO ● POTÊNCIA COM EXPOENTE INTEIRO


NEGATIVO
3 FATORES IGUAIS BASE

DEFINIÇÃO: 𝟏
𝒂−𝒏 = 𝒏 (n>1 e a≠0)
𝒂
Com a 𝟄 IR e n 𝟄 Z, temos an = a .a. a ... a.
a.a
Exemplos:
n FATORES
IGUAIS
1−3 23
Exemplos: = = 23 = 8
2 1
52 = 5 ∙ 5 = 25 1−2 102
43= 4 ∙ 4 ∙ 4= 64 = = 102 = 100
10 1
(-2)5=(-2) ∙ (-2) ∙ (-2) ∙ (-2) ∙ (-2) = -32 1
8−1
(-3) 4= (-3) ∙ (-3) ∙ (-3) ∙ (-3) = + 81 8
(-1,3)2=(-1,3) ∙ (-1.3) = 1,69 4−2 12
1 = 4
1 . 1 . 1 . 1 = 1_ = = 1
4 16
2 2 2 2 2 16
−2−2 32 9
= =
OBS: - 26 ≠ (- 2)6 3 2 4
- 26 = - (2 ∙ 2 ∙ 2 ∙ 2 ∙ 2 ∙ 2) = - (+64) = -64 ● NOTAÇÃO CIENTÍFICA
(- 2) = (-2) ∙ (-2) ∙ (-2) ∙ (-2) ∙ (-2) ∙ (-2)= + 64
6
A notação científica fornece uma ideia clara
da ordem de grandeza (bilhões, milhões,
CASOS PARTICULARES DE POTÊNCIA milésimos etc.), fundamental quando lidamos
com números “muito grandes” ou números
● POTÊNCIA COM EXPOENTE ZERO “muito pequenos”. A ordem de grandeza é
dada pela potência de base 10.
𝒂𝟎 = 𝟏
Exemplos:
Exemplos:
a) O diâmetro de uma bactéria, que é um
1000 = 1 (0,08)0=1 organismo unicelular, varia de 10-6 a 5 ∙ 10-6m.
(-3)0 = 1 (¾)0 =1 b) O raio do Sol tem aproximadamente 6,96
( √2 ) 0 = 1 10240 =1 ∙108 m.

10
Em geral, na notação científica os números 2. QUOCIENTE DE POTÊNCIAS DE
são escritos como produto de dois fatores, em BASES IGUAIS
que um deles é uma potência de base 10 com
Exemplo: 7 FATORES IGUAIS
expoente inteiro (positivo ou negativo) e o
outro é um número maior ou igual a 1 e menor 97 : 95 = 97 = 9.9.9.9.9.9.9 = 9. 9 = 92 =81
10. 95 9.9.9.9.9

Exemplos: : 5 FATORES IGUAIS

a) A distância da Terra ao Sol : DEFINIÇÃO: Um quociente de duas


150.000.000km = 1,5 ∙ 108km potências de mesma base diferente de zero,
pode ser reduzida a uma única potência,
b) A massa do átomo de hidrogênio: conservando-se a base e subtraindo os
0,00000000000000000000000166 g = 1,66 ∙ expoentes.
10-24
𝒂𝒎 . 𝒂𝒏 = 𝒂𝒎.𝒏
c) Distância média do planeta Saturno até o Exemplos:
Sol:
1.000.000.000.000 m = 1012 108 . 108 = 108−8 = 100 = 1
(−7)6 ∶ (−7)4 = (−7)6−4 = (−7)2
d) O diâmetro de um fio de cabelo é
aproximadamente: (0,5)12 ∶ (0,5)9 = (0,5)12−9 = (0,5)3
0,0001m = 10-4 m
OBS:
PROPRIEDADES DA POTENCIAÇÃO
3−6 ∶ 34 = 3(−6)−(+4) = 3−10
1. PRODUTO DE POTÊNCIAS DE BASES
IGUAIS (1,2)−3 ∶ (1,2)−7 = (1,2)(−3)−(−7) = (1,2)(−3+7)
Exemplos: = (1,2)4

6 FATORES IGUAIS 23 2−5 2(3)−(−5) 23+5 23


: = = =
3 3 3 3 3
a)52 . 54 = 5.5. 5.5.5.5 = 56
2 FATORES 4 FATORES 3. POTÊNCIA DE POTÊNCIA
2 6 4 12
b)2 .2 .2 = 2.2.2.2.2.2.2.2.2.2.2.2 = 2 Exemplo:
2 FATORES 6 FATORES 4 (45)2 = (45 .45) = 45+5 = 410
FATORES
(4 ∙ 4 ∙ 4 ∙ 4 ∙ 4 ∙ 4 ∙ 4 ∙ 4 ∙ 4 ∙ 4)
DEFINIÇÃO: Um produto de duas ou mais 10 FATORES IGUAIS
potências de mesma base diferente de zero,
pode ser reduzido a uma única potência,
conservando-se a base e somando os
expoentes. (45)2 = 45X2 = 410

am. an = am+n
DEFINIÇÃO: Uma potência de potência, com
Exemplos: base diferente de zero pode ser reduzida a
uma única potência, conservando-se a base e
102 . 10. 104 . 10. 105 = 102+1+4+1+5 = 1013
multiplicando os expoentes.
(0,3)6 . (0,3)4 = (0,3)6+4 = (0,3)10
(am)n = a mxn
3 2 3+1+4 8
(−4) . (−4). (−4) = (−4) = (−4)
Exemplos:
17 1 18 17+1+8 116
. . = = (74)2 = 7 4x2 = 78
2 2 2 2 2
(0,8 -3)-5 = (0,8)15
OBS:
(-52) -3= (-5)- 6
2-6 ∙ 2-3 ∙ 2-1 = 2 (-6-3-1) = 2-10
1–5 4
= 1 -20
(0,5)-4 ∙ (0,5)7 = (0,5) (-4+7) = (0,5)3 3 3
3 -6 . 3 2 . 3 4 = 3 ( -6 +2 +4) = 30 = 1 (63 )2 5
= 630

11
4. PRODUTO DE POTÊNCIAS DE MESMO
EXPOENTE
Exemplo:
32 .52 = (3.3) . (5.5) =(3.5)(3.5) = (3.5)2 =
2
15

15 . 15

32 .52 =(3.5)2 = 152

DEFINIÇÃO: Um produto de duas ou mais a) Determine a medida do comprimento


potências de mesmo expoente pode ser do lado do tabuleiro:
reduzido a uma única potência, multiplicando
as bases e conservando o expoente comum R: √576 = 24

am . bm = (a.b)m b) Determine a área e o comprimento do


lado de cada um dos quadradinhos
Exemplos: representados no tabuleiro:
R: 576 : 64 = 9cm2
34 ∙ 54 = (3 ∙ 5)4 = 154
√9= 3cm
23 ∙ 33 ∙ 53 = (2 ∙ 3 ∙ 5)3 = 30 3
(-10)5 ∙ 25 = (-10 ∙ 2 )5 = (-20)5 DEFINIÇÃO: De modo geral, sendo n um
(4 -1 ) ∙ (8 -1) = 32-1 número natural diferente de zero e a um
número real, dizemos que: 𝑛√𝑎 𝑛√𝑎 = b, se
e somente se n bⁿ = a
5. QUOCIENTE DE POTÊNCIAS DE
MESMO EXPOENTE ÍNDICE 𝑛
√𝑎 = b RAIZ
Exemplo: RADICANDO
2 2 2 2 2
30 : 5 = 30 = 30 = 6 = 36
52 5 RADICAL
DEFINIÇÃO: Um quociente de duas
potências de mesmo expoente pode ser Exemplos:
reduzido a uma única potência, dividindo as
√9 = 3; pois 3² = 9
bases e conservando o expoente comum
3
√𝑎8 = 2; pois 2³ =8
m m m 4
a :b = (a:b) √81 = 3; pois 3⁴ = 81
b≠0
9 3
√ =
Exemplo: 16 4
85 : 45 = (8 : 4)5 =25 = 32
64 8
103 : 23 = (10 : 2)3 =53 = 125 √0,64 = √ = = 0,8
100 10
(-12)4 : 44 = (-12 : 4)4 =(-3)4 = 81
57 : 47 = 5 7

4 ● POTÊNCIA COM EXPOENTE


FRACIONÁRIO
RADICIAÇÃO
DEFINIÇÃO: Se a é um número real positivo,
Exemplo: O tabuleiro de xadrez representado m é um número inteiro e n é um número
na figura tem 576 cm2 de área. natural não nulo, temos

12
As potências de base positiva e expoente
racional podem ser escritas na forma de
radical.
4ª PROPRIEDADE: A raiz de um quociente é
Exemplos: igual ao quociente das raízes.
1 2
362 = √361 = √36 = 6
1 33
83 = √81 = √8 = 2
3 2
22 = √23 = √8
De forma geral. Temos:
𝑏 𝑐
𝑎 𝑐 = √𝑎 𝑏 Exemplos:

PROPRIEDADES DOS RADICAIS 100 100 10


√ =√ = =2
25 25 5
1ª PROPRIEDADE: De modo geral, para os
radicais de índice n, de uma potência com 3 27 √27
3
3
expoente também igual a n, temos: √ = 3 = = 0,3
1000 √1000 10
● Se n é um número natural ímpar, então
𝑛 OPERAÇÕES COM RADICAIS
√𝑥 𝑛 = x, sendo x um número natural.
1) Potência de raízes
● Se n é um número natural par não nulo,
então
𝒏 𝒏
𝑛
√𝑥 𝑛 = |𝑥|, sendo x um número real ( √𝑎𝑘 )𝑚 = √𝑎𝑘.𝑚
Exemplos:
Exemplos:
6 4
√26 = 2 √𝑥 4 = X 4 4
3 7 (√2)(√2)4 = √24 √24 = 22 22 = 22 √2√2 = 4
√(−2)3 = -2 √57 =5
(√2)5 (√2)5 = √25 √25 = √24 . 2√24 . 2
2ª PROPRIEDADE: dividindo ou = 22 √2√2
multiplicando o índice e o expoente do
radicando por um mesmo número natural = 4√2√2
maior que zero o valor do radical não se altera:
2) Radiciação de radicais

𝒏 𝑚 𝑛.𝑚
√ √𝑎 = √𝑎

Exemplos:
Exemplos: 1 1 1
√ √16 = ( 2√16)2 = √162 = 162 =
2 2
20 20:5 4 I. 2
√715 = √715:5 = √73 4
√16 = 2
5 5.2 10
√23 = √23.2 = √26 OBS: basta multiplicar os índices.

3ª PROPRIEDADE: A raiz de um produto é √√81 = 4√81 = 3


igual ao produto das raízes.
3) Adição e subtração de raízes. (Índice e
radical iguais).
Exemplos:
Exemplos:
√2 + 3√2 = 4√2

√8√8 + 5√2 = √23 + 5√2 = 2√2 + 5√2 = 7√2

13
4) Multiplicação e divisão de raízes. (basta 3 3
P3) (PUC-RJ) O valor de √√729 − √ √64 é:
que tenham o mesmo índice). (A) -1
Exemplos: (B) 0
(C) 1
(D) 2
√3 . √12 = √3.12 = √36 = 6 (E) 3
√63 √63 63 P4)(PUC-RJ) O valor de √67 − √6 + √9 é
= √ = √9 = 3
√7 √7 7 igual a:
(A) -3
5) Redução ao mesmo índice (B) -9
(C) 8
Exemplos: (D) 4
3 6 6
6 6 (E) 2
√2 . √5 = √23 . √52 = √200 √400
2 2 6 6 P5) (UPF-RS) Simplifique a expressão =
√3 √3 27 √33 √27 6

3 3 = 6 = 6 = √ 5

317− 316
, obtém-se o valor:
√2 √2 √22 √4 4 6

5 1
(A) √2
6) Racionalização de denominadores.
3
Exemplos: (B)
2

5
(C) 27
5 5 √3 √3
I) = . = 3 5 3
√3 √3 √3
(D) √
2

4 4 (3−√5) 12−4√5 1
II) = . = = (E)
3+√5 3+√5 (3−√5) 32 −√52 6
12−4√5 12−4√5 6−2√5
= = ALUNO RESPONDE
9−5 4 2
1
3 3 3 1 4 1 2
5 5 √62 5 √36 5 √36 (Vunesp) O valor da expressão 4. ( ) +( )
III) 3 = 3 . 3 = 3 = 2 4
√6 √6 √62 √63 6
é:
3
(A) 4

PROFESSOR RESPONDE 4
(B)
3
.
P1) (Osec-SP) O produto 0,000015 1
0,000000002 é igual a: (C)
3
. -40
(A) 3 10 1
(D)
(B) 3 . 10-14 4
(C) 30 . 10-14 (E) 5
3
(D) 30 . 10-13 164
A2) (UFSM-RS) O valor da expressão 1 ∶
(E) 3 . 10-4 83
24
P2).(PUC-MG) O valor da expressão é:
. -24 . -22
82
A = 1,67 10 + 3,95 10 é: (A) 2−1
(A) 3,9667 . 10-22 (B) 20
(B) 3,9667 . 10-23 1
(C) 3,9667 . 10-24 (C) 22
(D) 3,9667 . 10-42
(E) 3,9667 . 10-46 (D) 24
(E) 26

14
3 2
A3) Sendo x = (22)3 , y = 22 e z = 23 , (D) 2√2+√3
𝑥.𝑦
determine o valor de :
𝑧 (E) 4
6
(A) 2 √7−2√10
(B) 210 A8) (IFPA) Em relação ao número ,
√5−√2
(C) 25 pode-se afirmar que é um número
(D) 22
(E) 23 (A) Racional, mais não inteiro.

A4) (Enem) Um dos grandes problemas da (B) Irracional negativo.


poluição dos mananciais (rios, córregos e (C) Inteiro negativo.
outros) ocorre pelo hábito de jogar óleo
(D) Irracional positivo.
utilizado em frituras nos encanamentos que
estão interligados com o sistema de esgoto. (E) Natural
Se isso ocorrer, cada 10 litros de óleo poderão
contaminar 10 milhões(107) de litros de água
A9) (IFPA) Sabendo-se que k=0,16, os
potável. Suponha que todas as famílias de
uma cidade descartem os óleos de frituras valores de√𝑘 , 6√𝑘 e 5k√𝑘 são,
através de encanamentos e consumam respectivamente:
1000litros de óleo em fritura por semana. Qual (A) 0,04; 2 e 1,6
seria, em litros, a quantidade de água
(B) 0,4; 2,4 e 0,32
contaminada por semana nessa cidade?
(A) 10-2 (C) 0,004; 2 e 0,16
(B) 103 (D) 0,4; 5 e 16
(C) 104
(E) 0,4; 10 e 16
(D) 106
(E) 109 10−3 . 105
A10) (PUC-SP) O valor da expressão 10 . 104
A5) (IFPI2014) A soma dos algarismos do é:
0
resultado da expressão 2013 2014 − 1 é: (A) 10
(A) 4
(B) 103
(B) 5
(C) 6 (C) 10-2
(D) 7 (D) 10-3
(E) 8
√2 (E) 100
A6) (IFPI) Simplificando a expressão −
√2−1
1 3
− , vamos obter: A11. (IFMA) Qual o valor da expressão
√2 1+√2
−5√2+10 √19 + 8√3 + √19 − 8√3?
(A)
2
(A) 8
(B) 7√2 + 2
(B) 38
(C) 5√2 − 15
(C) 2√19
√2+3
(D)
6 (D) 38√3
7√2+1
(E) 2
(E) 3√38

2+√3 A12) (IFMA) A expressão √50 − √8 − 3√2 é


A7) (IFIP) O valor da expressão √2−√3 + igual a:
2−√3 (A) 0
√ é:
2+√3
(B) 2
(A) 2
(C) √2
(B) 1
(D) 2√2
(C) √2
(E) 3√2

15
A13) (IFMA) A expressão √80 + 4√45 − √125 22003 .91001
A18) (OBM) O valor da soma +
é igual a: 41001 .32003
22002 .91001
(A) 7√5 é:
41001 .32003
(B) 8√5
1
(C) 9√5 (A) 3
(D) 10√5
2
(B)
(E) 11√5 3

A14) (PUC -RJ) O maior número abaixo é (C) 1


(A) 331 4
(B) 810 (D) 3
(C) 168
(D) 816 (E) 2
(E) 2434
A15) (OJSP) Classifique em verdadeiro ou A19) (IFMA) Considerando as afirmações:
falso:
● 82 : [32 – (20 – 33)] =4 3 4
I. 3√16 − 2 √27 + 3 √81 = 15
● 25 - ( -2)4 – (-2)3 – 22=28
● [(-2)2]5 : [(-2)3]2 . 20=16 II. 2√8 + √72 − √450 = −5√2
● (72)0=0 6
III. √ 3√1458 = 3 √2
(A) VFFF
√2.+√3
(B) VVVF IV. = 1 + √3
√2
(C) VFVF
(A) Todas são falsas
(D) VVFV
(B) Apenas uma é falsa
(E) FVFV
(C) Duas são falsas
16) (IFMA) O valor da expressão √√ √112 +
3

1
(D) III é falsa
3
(√√324 ) +é: (E) Todas são verdadeiras

(A) √11 + √3
A20) (Vunesp-SP) simplificando a expressão:
(B) 12
(29 .(22 . 2)-3 ). Obtém-se:
(C) √14
(D) 20
(A) 236
(E) √33
(B) 2-30
1030 +1040 (C) 2-6
A17) (IFMA) O valor da expressão √
1040 +1050
é: (D) 1
1
(A) 10⁻⁵ (E)
3
(B) 10⁻¹⁰
(C) 1/10
(D) √10−5
(E) 10⁵

16
ATIVIDADE COMPLEMENTAR 4. (Upf 2018) Considere as afirmações
abaixo, onde a e b são números reais.
1. (G1 - cp2 2020) Minecraft educacional: I. a2  a
jogos como ferramenta de aprendizagem II. a2  b 2  a  b
Muitos professores têm usado os jogos como III. a2  b2  a2  b2
ferramenta para estimular o aprendizado de a2 a2
seus estudantes. Suponha que um professor IV.  ,b  0
de Matemática, em uma de suas aulas sobre b2 b2
números naturais, tenha pedido à sua classe a) Apenas III e IV são verdadeiras.
que construa árvores usando o Minecraft. b) Apenas IV é verdadeira.
Como o jogo consiste em montar um cenário c) Apenas II é falsa.
utilizando blocos de construção do tipo Lego, d) Apenas I, II e IV são verdadeiras.
um dos estudantes decidiu manter, a cada e) Todas são verdadeiras.
etapa das construções, o mesmo padrão de
formação das árvores, colocando-as lado a 5. (G1 - cp2 2018) Um professor gosta de
lado e sempre acrescentando uma quantidade criar desafios para seus estudantes, com
de blocos em suas partes superiores. A figura expressões envolvendo um só número. Em
a seguir ilustra o início das construções: certa aula, apresentou o seguinte problema
dos quatro “quatros”:
44
x O valor de x é
4
4
a) 16.
b) 128.
Para a construção da árvore na Etapa 10, o c) 128 2.
estudante utilizou d) 256 2.
a) 10 blocos.
b) 21 blocos. 6. (G1 - cftrj 2017) Alex, Beatriz e Camila
c) 100 blocos. foram convidados a fazerem afirmações sobre
d) 121 blocos. o número N  250  420.
- Alex afirmou que N é múltiplo de 8;
2. (G1 - ifce 2019) Simplificando a expressão - Beatriz afirmou que metade de N é igual a
23 2 3 2 2 225  410 ;
, obtemos o número - Camila afirmou que N é par.
1
2 6 Quantas das afirmações feitas pelos
a) 4. participantes são verdadeiras?
b) 2. a) 0
c) 2. b) 1
c) 2
d) 3 2.
d) 3
e) 1.
7. (Unisinos 2016) Simplificando-se a
3. (G1 - ifsc 2018) Analise as afirmações
seguintes: 237
expressão , obtém-se o
I. 52  16  (10)  ( 5)2  17 235  238  239
número
II. 35  (3  81  23  1)  2  10
19
III. Efetuando-se (3  5)(3  5), obtém-se a)
4
um número múltiplo de 2. 19
b)
2
Assinale a alternativa CORRETA.
a) Todas são verdadeiras. c) 0,4
b) Apenas I e III são verdadeiras. d) 0,16
c) Todas são falsas. 2
e)
d) Apenas uma das afirmações é verdadeira. 37
2
e) Apenas II e III são verdadeiras.

17
8. (G1 - ifsul 2016) Em matemática, potências ● MEDIDA DE VOLUME
são valores que representam uma
A unidade fundamental de volume chama-
multiplicação sucessiva de um número.
se metro cúbico. O metro cúbico (m3) à
Usando as propriedades de potenciação, qual
medida correspondente ao espaço ocupado
dos números a seguir é o maior?
por um cubo com 1 m de aresta.
a) 345
O volume expressa quanto espaço ocupa uma
b) 921
figura espacial. As unidades-padrão de
c) 2438 volume são baseadas em cubos e são
d) 8112 chamadas de unidades cúbicas.

9. (G1 - ifsul 2016) O valor da expressão


2 2 V= 1m x 1m x 1m = 1m3
 1  1 3
 5    5   27 é Volume = 1m3
   
a) 3
b)  3
551
MEDIDA DO VOLUME DE UM
c) PARALELEPÍPEDO
25
701
d)
25

V=a.b.c
UNIDADES DE MEDIDAS
Medir é comparar duas grandezas de mesma
espécie, verificando quantas vezes uma
contém a outra (unidade de medida). Múltiplos e submúltiplos do metro cúbico
● MEDIDA DE COMPRIMENTO
De acordo com o SI (sistema internacional de
medidas), o metro é considerado a unidade
principal de medida de comprimento, seguido
de seus múltiplos e submúltiplos. Os múltiplos
do metro são o quilômetro (km), hectômetro
(hm) e decâmetro (dam) e os submúltiplos são Ex. Transformar
decímetro (dm), centímetro (cm) e milímetro a) 1,35 dm3 em cm3.
(mm).
1,35 dm3 = 1,35 x 1000 cm3 =1350 cm3.
São estabelecidos alguns critérios de
conversão, como mostraremos a seguir: b) 203 cm3 em m3
203 cm3 = 203 :1000000 = 0,000203 m3

● MEDIDA DE CAPACIDADE
A quantidade de líquido é igual ao volume
interno de um recipiente, afinal quando
enchemos este recipiente, o líquido assume a
À medida que as unidades seguem a
forma do mesmo. Capacidade é o volume
orientação da direita, os valores são
interno de um recipiente.
multiplicados por 10. E à medida que seguem
a orientação da esquerda, os valores são A unidade fundamental de capacidade
divididos por 10. chama-se litro.
Exemplos: Transformar Litro é a capacidade de um cubo que tem
1dm de aresta.
a) 10 km em metros → 10 * 10 * 10 * 10
= 10 000 metros 1L = 1dm3

b) 5 mm em m → 5 : 10 : 10 : 10 = 0,005
metros

18
Múltiplos e submúltiplos do litro O quilograma (kg) é a massa
de 1dm3 de água destilada à
Unida
temperatura de 4ºC.
de
Múltiplos Funda Submúltiplos Apesar de o quilograma ser a unidade
menta fundamental de massa, utilizamos na prática
l o grama como unidade principal de massa.
quilolitr hectolit decalitr
litro decilitro
centilitr
mililitro Múltiplos e Submúltiplos do grama
o ro o o
Unida
kl hl dal l dl cl ml de
Múltiplos Submúltiplos
0,001 princi
1000l 100l 10l 1l 0,1l 0,01l pal
l
deci
quilogr hectog decag centigra miligra
grama gra
Cada unidade é 10 vezes maior que a ama rama rama
ma
ma ma
unidade imediatamente inferior.
kg hg dag g dg cg mg
Relações
1.00 0,1 0,00
100g 10g 1g 0,01g
0g g 1g
1l = 1dm3
Observe que cada unidade de quilograma é
dez vezes maior que a unidade
1ml = 1cm3 imediatamente inferior. Exemplos:

1KL
1 dag = 10 g
= 1000l = 1m³

Transformar 1500 cm3 em litro. 1 g = 10 dg


1500 cm3 = 1500 : 1000 dm3= 1,5 dm3= 1,5 l 3,2 kg = 3 quilos e 200 gramas, isto é,
● MEDIDA DE MASSA
3,2 Kg = 3,2 x 1000 g = 3200 g.
Massa é a quantidade de matéria que
um corpo possui, sendo, portanto, constante 500 g = 500 : 1000 = 0,5 Kg.
em qualquer lugar da terra ou fora dela.
Peso de um corpo é a força com que
esse corpo é atraído (gravidade) para o centro ● MEDIDA DE TEMPO
da terra. Varia de acordo com o local em que
o corpo se encontra. Por exemplo:
A unidade de tempo escolhida como padrão
A massa do homem na Terra ou na no Sistema Internacional (SI) é o segundo.
Lua tem o mesmo valor. O peso, no entanto,
é seis vezes maior na terra do que na lua. Segundo
Explica-se esse fenômeno pelo fato da O Sol foi o primeiro relógio do homem: o
gravidade terrestre ser 6 vezes superior à intervalo de tempo natural decorrido entre as
gravidade lunar. sucessivas passagens do Sol sobre um dado
meridiano dá origem ao dia solar.
Obs: A palavra grama, empregada no sentido
de "unidade de medida de massa de um O segundo (s) é o tempo equivalente
corpo", é um substantivo masculino. Assim
200g, lê-se "duzentos gramas".
a do dia solar médio.
As medidas de tempo não pertencem ao
Quilograma Sistema Métrico Decimal.
A unidade fundamental de massa chama-
se quilograma.

19
Múltiplos e Submúltiplos do Segundo A resposta correta então é 4 min e 20 s.
Como subtrair:
Quadro de unidades
3 h 35 min – 1 h 25 min 20 s.
Múltiplos
Sabemos que 3 h 35 min = 3h 34 min 60 s,
minutos hora dia então
min h d 3 h 34 min 60 s
-
60 min = 24 h = 1.440 min = 1 h 25 min 20 s
60 s
3.600 s 86.400s 2 h 09 min 40 s
São submúltiplos do segundo: PROFESSOR RESPONDE
● décimo de segundo P1) (IFAM-2015) Um reservatório de água
● centésimo de segundo possui instaladas duas torneiras e um ralo.
Uma das torneiras enche-o em 2 horas; a
● milésimo de segundo outra torneira enche-o em 3 horas e o ralo o
Cuidado: Nunca escreva 2,40h como forma esvazia em 6 horas. Estando cheio até a
de representar 2 h 40 min. Pois o sistema de metade da sua capacidade, com as torneiras
medidas de tempo não é decimal. e o ralo totalmente abertos, o reservatório
ficará completamente cheio em:
Observe:
(A) 2 horas e 15 minutos
(B) 1 hora e 30 minutos
(C)1 hora e 18 minutos
(D) 45 minutos
(E) 30 minutos

P2) (IFAM-2015) É possível obter 60 litros de


uma substância composta, misturando duas
substâncias simples A e B de dois modos
diferentes: a metade da substância A, um
ADIÇÃO E SUBTRAÇÃO
terço da substância B e 26 litros de água, ou,
a terça parte da substância A, a metade da
Na soma e na subtração basta que se armem substância B e 24 litros de água. Misturando
as contas colocando na mesma coluna toda a quantidade das duas substâncias para
unidades iguais. Observando-se as seguintes obter 100 litros da composição, a quantidade
regras: de água que devemos acrescentar é:
Quando o total de unidade do segundo (A) 10 litros
exceder 59 deve ser transformado em minuto, (B) 15 litros
e quando os minutos excederem 59 devem (C) 16 litros
ser transformados em horas. (D) 20 litros
(E) Nada
Exemplo:
P3) (OBM-2000) Uma fábrica embala latas de
2 min 35 s palmito em caixas de papelão cúbicas de 20
+ 1 min 45 s cm de lado, de modo que cada caixa contém
8 latas. Para poderem ser melhor
3 min 80 s
transportadas, essas caixas são colocadas,
A resposta final seria 3min e 80s, mas não da melhor maneira possível, em caixotes de
falamos assim no nosso cotidiano, então madeira de 80 cm de largura por 120 cm de
devemos transformar esses segundos comprimento por 60 cm de altura. O número
de latas de palmito em cada caixote é:
excedentes em minuto.
80 segundos = 60 segundos + 20 segundos (A) 576
= 1 minuto e 20 segundos (B) 4.608
(C) 2.304
3 min (D) 720
+ 1 min 20s
(E) 144
4 min 20s

20
P4) Um veículo de um hospital transporta
diariamente as mesmas quatro caixas de A4) (IFMA -2013) Mariana estudou durante a
remédios: A, B, C e D e, a cada dia, é incluída semana e registrou os horários de início e
uma caixa extra que pode ser qualquer uma término na tabela a seguir:
dessas quatro. A tabela a seguir mostra o DIA Seg Ter Qua Qui Sex
peso total em kg das cinco caixas
8h25 9h30 8h15
transportadas em cada caso da caixa extra: Início 8h 8h
min min min
Térmi 10h35 11h20 10h50 10h30
11h
no min min min min
Quantas horas ela estudou durante essa
semana?
(A) 12 h 05 min
(B) 11 h 35 min
(C) 12 h 55 min
(D) 11 h 55 min
O peso, em kg, de uma caixa C é (E) 13 h 10 min
(A) 8
(B) 12 A5) (IFMA-2014) Maria consegue limpar sua
(C) 13 casa em 3 horas de serviço e Eline, sua neta,
(D) 15 consegue limpar a mesma casa em 2 horas.
(E) 18 Se as duas trabalharem juntas, em quanto
tempo conseguirão limpar a mesma casa?
ALUNO RESPONDE (A) 1 h e 20 min
A1) (IFAM-2012) Um balde, de capacidade (B) 1 h e 48 min
7,95 litros, está completamente cheio de leite. (C) 1 h e 12 min
Uma xícara cheia comporta 150 ml de leite. (D) 2 h e 10 min
Com a quantidade de leite contida no balde é (E) 2 h e 30 min
possível encher quantas xícaras? A6) (IFPI-INTEG) Em setembro de 2013, a
(A) 50 leitura de um hidrômetro (aparelho usado em
(B) 51 residências para medir o consumo de água)
(C) 52 registrou 1321 m³. No mês seguinte, a nova
(D) 53 leitura registrou 1346 m³. Qual o consumo,
(E) 54 em litros, nesse período?
(A) 25
A2) (IFAM-2012) O tanque de (B) 250
armazenamento de água de um avião de (C) 2 500
combate a incêndio tem capacidade para (D) 25 000
24000 galões. Sabendo que um galão (E) 250 000
corresponde a 3,8 litros, pode-se dizer que o A7) (IFPI-INTG) Duas torneiras de um tanque,
tanque cheio de água desse avião é capaz de de vazões distintas, uma de entrada e outra
abastecer, aproximadamente, quantas casas de saída, são ligadas simultaneamente
com caixa d’água de 2 m3? quando o tanque está vazio. Sabe-se que a
(A) 46 torneira de entrada o enche em 2 horas, e a
(B) 92 de saída, o esvazia em 7 horas. Então, em
(C) 120 quanto tempo o tanque ficará totalmente
(D) 248 cheio?
(E) 456 (A) 2 h 30 min
A3) (IFMA-2013) O resultado da operação (B) 2 h 36 min
(0,03 km + 200 mm + 100 cm + 1 hm) é:
(C) 2 h 42 min
(A) 13,12 km
(D) 2 h 48 min
(B) 131,2 m
(C) 1,3 hm (E) 2 h 54 min
(D) 1312 cm
(E) 1.3120 mm

21
A8) (IFPI-2011) Pedro precisa construir um (C) 31
muro nos fundos de sua casa e dispõe apenas (D) 32
de uma trena graduada, cuja unidade de (E) 33
medida é o côvado. De acordo com esta trena,
o muro terá um comprimento de 25 côvados. A12) (OBM-2017) Um cachorro avista um gato
Sabendo-se que o comprimento deste muro é que está a 30 m de distância e começa a
completamente coberto por 20 lajotas em persegui-lo. Ambos começam a correr em
formato de cubo, colocadas lado a lado em linha reta, no mesmo sentido e com passadas
linha reta, e supondo que um côvado sincronizadas. O cachorro se desloca 50 cm a
corresponde a 0,52 m, cada lajota utilizada cada passada enquanto o gato se desloca
nesta construção possui dimensões de: apenas 30 cm. Depois de quantas passadas o
cachorro alcançará o gato?
(A) 65 mm.
(B) 6,5 dm. (A) 100
(C) 6,5 cm. (B) 110
(D) 6,5 mm. (C) 120
(E) 650 dm. (D) 150
(E) 160
A9) (IFPI-2012) Os norte-americanos utilizam
um sistema de medida chamado de A13) Na casa de Manoel há uma caixa d’água
consuetudinário. Esse sistema foi baseado vazia com capacidade de 2 metros cúbicos.
nos primeiros padrões de medidas, onde Manoel vai encher a caixa trazendo água de
antigamente os costumes e tradições um rio próximo, em uma lata cuja base é um
utilizavam partes do corpo humano e quadrado de lado 30 cm e cuja altura é 40 cm,
utensílios de uso cotidiano para unidades de como na figura. No mínimo, quantas vezes
medidas. Nesse sistema, a milha equivale Manoel precisará ir ao rio até encher
aproximadamente a 1600 metros e o galão completamente a caixa d’água?
equivale a aproximadamente 3,8 dm³.
(A) 53
Considere um automóvel norte-americano
que consome 24 milhas por galão. (B) 54
Transformando essas medidas para o sistema (C) 55
decimal, esse carro gastará aproximadamente
quantos litros para percorrer 100 km? (D) 56
(A) 1 litro. (E) 57
(B) 10 litros.
(C) 25 litros. A14) Para assar um frango são necessários
(D) 50 litros. 15 minutos para aquecer o forno e mais 12
(E) 100 litros. minutos para assar cada meio quilo de frango.
Paula comprou um frango de 2,5 kg. A que
A10) (Ferreira de Moura - 2012 - Prefeitura de horas ela deve ligar o forno para que o frango
Arapoti - PR) Um trem saiu de uma estação A fique pronto às 20 horas?
às 15 h 32 min 40 s e chegou na estação B às (A) 18 h
18 h 23 min 12 s, do dia seguinte. Qual foi a (B) 18 h 15 min
duração total dessa viagem do trem? (C) 18 h 30 min
(A) 2 h 50 min 32 s. (D) 18 h 45 min
(B) 3 h 09 min 28 s. (E) 19 h
(C) 26 h 50 min 32 s. A15) Atualmente, estima-se que cada
(D) 27 h 09 min 28 s. brasileiro produza 378 quilos de resíduos
(E) 27 h 10 min. urbanos (lixo) por ano. De acordo com essa
informação, no mínimo quantos brasileiros
A11) Você possui muitos palitos com 6 cm e 7 são necessários para produzir mais de 10
cm de comprimento. Para fazer uma fila de toneladas de resíduos urbanos em um ano?
palitos com comprimento total de 2 metros, o (A) 3
número mínimo de palitos que você precisa (B) 27
utilizar é: (C) 60
(A) 29 (D) 124
(B) 30 (E) 265

22
A16) Um município colheu uma produção de RAZÃO
9.000 toneladas de milho em grãos em uma
A razão entre dois número a e b, com b ≠ 0,
área plantada de 2.500 hectares. A 𝑎
produtividade média do município em termos nessa ordem, é o quociente 𝑏 .
3(𝑎𝑛𝑡𝑒𝑐𝑒𝑑𝑒𝑛𝑡𝑒)
de sacas de 60 kg colhidas por hectare foi? ✔ Ex: A razão de 3 para 4 é 4 (𝑐𝑜𝑛𝑠𝑒𝑞𝑢𝑒𝑛𝑡𝑒)
(o
(A) 50 1º número é o antecedente e o 2º número é o
(B) 60 consequente) . Lemos 3 está para 4
6 3
(C) 72 ✔ Ex: A razão de 6 para 8 é 8 = 4
(D) 90 5
(E) 100 ✔ Ex: A razão de 5 meses para um ano é 12
3 2 5
✔ Ex: A razão de 4 para 5 é 12.
A17) Muitos remédios são tomados em doses
menores que o mg. Um comprimido de certo
PROPORÇÃO
remédio tem 0,025 mg de uma certa
substância. Com 1 kg desta substância, Quatro números não-nulos a, b, c e d
quantos comprimidos podem ser feitos? formam, nessa ordem, uma proporção
𝑎 𝑐
(A) menos de um quando 𝑏 = 𝑑 .
(B) 4 Propriedade fundamental
(C) 40 Em toda proporção, o produto dos extremos
(D) 40.000 é igual ao produto dos meios.
2 3
(E) 40.000.000 = 6 é exemplo de uma proporção
4
1° 𝑡𝑒𝑟𝑚𝑜 3° 𝑡𝑒𝑟𝑚𝑜
é 2° 𝑡𝑒𝑟𝑚𝑜 = 4° 𝑡𝑒𝑟𝑚𝑜
A18) Um copo de 200 ml de refrigerante
O termo abaixo é uma proporção.
possui 28 mg de sódio, ou seja, 0,2 L desse 6 3
refrigerante possui 0,028 g de sódio. Qual a 8
= 4 escrevendo os termos na forma 2 : 4 =
quantidade de sódio, em gramas, contida em 3 : 6 temos:
uma garrafa com 1 L desse refrigerante?
(A) 0,100
(B) 0,140
(C) 0,080
(D) 0,056
Aplicando a propriedade temos:
(E) 0,120
3x 4 = 2 x 6
12 = 12
A19) Qual o peso de 200 ml de certo tipo de ✔ Ex: Verifique se os números 2, 3, 5 e
óleo isolante para transformador, cujo litro 15 formam nesta ordem uma proporção.
pesa 840 g?
(A) 168 g
(B) 210 g
(C) 240 g
(D) 288 g
3 x 5 e 2 x 15
(E) 420 g
15 30 como são diferentes, sendo assim
não forma uma proporção.
A20) Comprei um aquário para meu filho com ✔ Ex: Calcule o valor de x para que os
capacidade de 72.000 cm³. Quantos números 3, x, 21 e 28 formem nessa ordem
quilogramas o aquário pesará depois que uma proporção.
estiver cheio d’água se, vazio, ele pesa 3
kg? (Considere 1L d’água = 1 kg)
(A) 7,2 kg 21.x = 3 . 28
(B) 7,5 kg 21x = 84
(C) 72 kg
(D) 75 kg
(E) 76 kg
X=4
Além da propriedade fundamental ainda
temos na proporção outras propriedades.

23
1ª Propriedade 2+y=5
⮚ A soma dos dois primeiros termos está y=5–2
para o 1º ou 2º termos, assim como a soma
y=3
dos dois últimos termos está para o 3º ou 4º
termo. Portanto x é 2 e y é 3.
𝑎 𝑐 2ª Propriedade
Veja: 𝑏 = 𝑑
⮚ A soma dos antecedentes está para a
soma dos consequentes, assim como o
antecedente está para o consequente da
mesma razão.

⮚ A diferença dos antecedentes está para a


diferença dos consequentes, assim como o
antecedente está para o consequente da
Essa propriedade também pode ser aplicada mesma razão.
usando a diferença.

Ex: Calcule os números a e c na proporção


𝑎 𝑐
= , sabendo que a soma de a e c é 48.
5 15

Ex: Calcule o valor de x e y em cada caso.

a)

20 . a = 48 . 5
20a = 240

20.x = 5 . 8
20x = 40 a = 12
a + c = 48
12 + c = 48

x=2 C = 48 – 12

x+y=5 C = 36

24
NÚMEROS DIRETAMENTE 1
PROPORCIONAIS b)
2
Os números não-nulos a, b, c, ... são 3
c)
diretamente proporcionais aos números x, y e 4
w..., nessa ordem, quando: 4
d)
3

2. (G1 - ifce 2019) Em duas piscinas há 2200


litros de água. O volume da piscina maior,
NÚMEROS INVERSAMENTE sabendo que suas capacidades estão na
PROPORCIONAIS 4
proporção de , em litros, é
Os números não-nulos a, b, e c são 7
inversamente proporcionais aos números
não-nulos x, y e w, nessa ordem quando: a) 1.400.
b) 1.500.
c) 1.600.
d) 1.700.
e) 1.800.

de forma simplificada temos a.x = b.y = c.w 3. (G1 - ifpe 2019) O Homem-Escorpião, o
Menino-Vespa e a Garota-Abelha já
derrotaram, juntos, 600 vilões na proporção
PROFESSOR RESPONDE 13, 2 e 5, respectivamente. Quantos vilões o
Homem-Escorpião derrotou a mais que o
P1) (IFPI 2014) O Sr. Francisco precisa pagar Menino-Vespa?
uma dívida de R$ 4.000,00 a Paulo, R$
5.000,00 a João e R$ 6.000,00 a Maria. Como a) 240 vilões.
só tem R$ 10.000,00, resolve pagar quantias b) 330 vilões.
diretamente proporcionais a cada débito. c) 90 vilões.
Dessa forma quanto receberá Maria? d) 360 vilões.
(A) R$ 2.800,00 e) 210 vilões.
(B) R$ 3.000,00
(C) R$ 3.800,00 4. (G1 - cftrj 2019) O gráfico a seguir mostra
(D) R$ 4.000,00 as quantidades de unidades vendidas por
(E) R$ 4.800,00 cada tipo de docinho que Cecília faz para
vender.
P2) (IFMA 2017) Um determinado mapa do
Maranhão foi feito na escala de 1 : 2 500
000. Sabendo-se que a distância real entre
duas cidades maranhenses, Santa Inês e
São Luís, é de, aproximadamente, 250 km,
determine essa distância no referido mapa.
(A) 5 cm
(B) 22 cm
(C) 10 cm
(D) 15 cm O docinho mais vendido responde por qual
(E) 25 cm fração do total de docinhos vendidos?
3
a)
ATIVIDADE COMPLEMENTAR 13
4
b)
1. (Uerj 2020) Admita que, em dezembro de 13
2014, uma filha tinha 20 anos e seu pai, 50. 5
c)
13
Em dezembro de 2024, a razão entre as 6
idades da filha e do pai será de: d)
13
1
a)
5

25
5. (G1 - cotil 2019) Previsões indicam que, no 11
d)
ano de 2050, a população mundial será de 9,6 30
bilhões de habitantes. Destes, 2/3 estarão 2
vivendo nas cidades. Sendo assim, qual e)
3
alternativa nos dá o número de habitantes que
estarão nas cidades? 8. (G1 - ifpe 2019) A super-heroína Garota-
a) 3,2 milhões de pessoas Abelha tem o poder de diminuir seu tamanho
b) 64 bilhões de pessoas na escala de 1:40 Se, ao utilizar seu poder, ela
c) 6 milhões de pessoas fica com apenas 12mm de altura, qual a altura
d) 6,4 bilhões de pessoas normal da heroína?
a) 1,65 m
6. (G1 - cftmg 2019) Uma pessoa foi ao b) 1,68 m
supermercado comprar o creme de leite de c) 1,70 m
sua preferência e percebeu que o produto é d) 1,52 m
vendido em quatro embalagens distintas. Os e) 1,62 m
volumes e preços dessas embalagens estão
representados no quadro abaixo: 9. (G1 - ifpe 2019) Ao realizar um experimento
no laboratório de Química do IFPE, o
Creme de leite professor Clécio formou uma solução de água
Embalagem Volume (mL) Valor (R$) e sal com 10kg de massa, sendo 95% dessa
massa constituída por água. Após um
I 200 3,80
processo de aquecimento da massa, os
II 300 5,20 estudantes verificaram que apenas água foi
III 500 7,80 eliminada e que a sua participação na massa
foi reduzida a 75% Determine a massa total da
IV 800 11,20
solução, após o processo de aquecimento.
a) 2,0 kg
De acordo com esse quadro, a embalagem de
b) 7,5 kg
creme de leite que proporciona o menor custo,
por mL, é a c) 9,5 kg
a) I. d) 8,0 kg
b) II. e) 5,0 kg
c) III.
d) IV. 10. (G1 - cp2 2019) Na entrada do Colégio
Pedro II existe um painel luminoso com as
7. (G1 - cmrj 2019) Nunca se olhou tanto para letras C, P, I e I, formado por lâmpadas
baixo. Na fila, no parque, na escola, no incandescentes, conforme a figura a seguir:
trabalho, no museu, no ônibus e,
perigosamente, no carro, as pessoas parecem
só ter um interesse: a tela do smartphone. A
ponto de, nos Estados Unidos, um estudo do
Pew Research Center ter apontado que
aproximadamente 50% da população diz não
conseguir viver sem seu celular com acesso à
internet. Certo dia, ao se ligar o painel, percebeu-se
que 18% das lâmpadas estavam queimadas
Enzo, aluno do 6º ano do CMRJ, passa cerca e deveriam ser substituídas por lâmpadas de
de 10h 24min por dia, olhando para a tela do LED. A razão entre a quantidade de lâmpadas
seu celular. Sabendo que, dentro das 24 de LED e a quantidade de lâmpadas
horas do seu dia, ele dorme durante 8 horas, incandescentes que não queimaram é
a fração referente ao tempo gasto por Enzo no a)
7
celular enquanto está acordado é igual a 32
9
13 b)
a) 41
30 9
13 c)
b) 50
20
50
11 d)
c) 9
20

26
11. (G1 - ifpe 2019) Elvis planejou uma REGRA DE TRÊS
viagem com sua esposa a Campos do Jordão, É o processo destinado a resolver problemas
a fim de curtir o feriadão da Semana Santa e, que envolvam grandezas diretamente ou
para isso, juntou uma certa quantidade de inversamente proporcionais. Assim, se em um
1 dado problema temos grandezas diretamente
dinheiro. Sabendo que do dinheiro que ele
4 ou inversamente proporcionais, podemos
1 utilizar regra de três simples ou composta
juntou foi gasto com hospedagem, foi gasto
5 para resolver o problema dado. Se temos três
3
com alimentação, com transporte e os R$ valores e queremos encontrar um deles,
8 usamos a regra de três simples para
560,00 restantes, com turismo, é CORRETO encontrar esse valor desconhecido. Se temos
afirmar que Elvis juntou um total de mais de três valores, usamos a regra de três
a) R$ 3.200,00. composta para encontrar o valor
b) R$ 3.000,00. desconhecido do problema.
c) R$ 3.600,00.
Passos de uma regra de três.
d) R$ 4.000,00.
1º) Agrupar as grandezas da mesma espécie
e) R$ 4.400,00. em colunas e mantendo, na mesma linha, as
grandezas de espécies diferentes em
12. (G1 - ifpe 2019) Um reservatório está com correspondência.
1,5 metros cúbicos de água mineral. 2º) Verificar se as grandezas são diretamente
Pretende-se encher botijões de água com ou inversamente proporcionais.
capacidade de 20 litros cada um. Supondo 3º) Montar a proporção e resolver a equação.
que não haja desperdício de água no Exemplos:
enchimento desses botijões, é CORRETO a) Três escavadeiras multiuso transportam
afirmar que, com toda a água contida no 220m³ de areia. Para transportar 1 600 m³ de
reservatório, encheremos a seguinte areia, quantas escavadeiras iguais a essa
quantidade de botijões seriam necessária?
a) 7,5. Resolução:
b) 75. Escavadeiras Areia (m³)
c) 750. 3 200
d) 7.500. X 1 600
e) 30. Como uma grandeza aumenta, a outra
também aumenta, logo são grandezas
13. (Uerj 2018) Duas latas contêm 250 mL e diretamente proporcionais.
350 mL de um mesmo suco e são vendidas,
3  1600
respectivamente, por R$ 3,00 e R$ 4,90. x  24
200
b) Em 8 horas, 20 caminhões descarregam
160m3 de areia. Em 5 horas, quantos
caminhões serão necessários para
descarregar 125m3?
Tomando por base o preço por mililitro do Resolução:
suco, calcule quantos por cento a lata maior é Tempo (h) Transporte Areia (m³)
mais cara do que a lata menor. 8 20 160m³
5 X 125m³
14. (G1 - ifal 2018) Uma herança de Observe que a grandeza transporte
R$ 320.000,00 foi dividida entre 3 filhos na aumentam enquanto o tempo diminui, temos,
seguinte proporção: O mais novo recebeu 1 8 então, grandezas inversamente
proporcionais. E, quanto mais aumenta a
da herança e o mais velho recebeu 1 2 da quantidade de areia, mais aumenta a
herança. Qual foi o valor recebido pelo filho do quantidade de transporte, tendo, assim
meio? grandezas diretamente proporcionais. As
a) R$ 40.000,00 setas servem para auxiliar na comparação.
b) R$ 80.000,00
c) R$ 120.000,00 20 5 160 20 4 20  5
d) R$ 160.000,00     x  x  25
x 8 125 x 5 4
e) R$ 200.000,00

27
PROFESSOR RESPONDE a) 150
P1) (IFPI 2014) Vinte homens, trabalhando 8 b) 120
horas diárias, fazem um poço de 400 m de c) 90
profundidade em 15 dias. Marque a alternativa d) 180
que indica quantos homens devem ser e) 240
acrescentados para que em 25 dias,
trabalhando 6 horas diárias, façam um poço A5)(IFMA 2014)Um muro é construído por 4
de 600 m de profundidade. pedreiros em 8 dias. Quantos pedreiros a
a) 38 mais serão necessários para se construir
b) 37 outro muro, igual ao anterior, em apenas 4
c) 24 dias?
d) 9
e) 4 a) 4 pedreiros
b) 2 pedreiros
ALUNO RESPONDE
c) 6 pedreiros
A1)(IFAM 2013) Trinta operários deveriam
d) 8 pedreiros
fazer um serviço em 40 dias. Treze dias após
o início das obras, 15 operários deixaram o e) 10 pedreiros
serviço. Trabalhando no mesmo ritmo, em
quantos dias ficará pronto o restante da obra? A6) (IFPI 2013) Se seis impressoras iguais
imprimem 500 cartazes em 1,5 hora, em
a) 54 quanto tempo três dessas máquinas
b) 55 imprimirão o triplo de cartazes?
c) 56
d) 58 a) 4 horas
e) 60 b) 5 horas
c) 5 horas e 30 minutos
A2)( IFAM 2013) Numa fábrica, 8 operários d) 7 horas e 30 minutos
montam 20 produtos em 5 dias. Quantos e) 9 horas
produtos serão montados por 4 homens em
16 dias? A7) (IFSP 2016)Uma fábrica produz peças de
a) 28 automóveis. Um lote de peças é feito, em 10
b) 32 dias, por 18 operários, que trabalham 8 horas
c) 38 por dia. Se fossem disponibilizados apenas 12
d) 40 operários, com uma carga diária de 6 horas,
e) 42 quantos dias eles levariam para produzir o
mesmo lote de peças?
A3)(IFGOIANO 2016)Um automóvel se
desloca de uma cidade A até uma cidade B (a) 15 dias.
em 15 minutos, a uma velocidade média de 80 (b) 9 dias.
km/h. Se no caminho de volta ele reduz a (c) 13 dias.
velocidade para 60 km/h, o tempo gasto, em (d) 20 dias.
minutos, é de (e) 17 dias.

a) 18. A8) (IFSP 2016)Uma indústria produz 2.940


b) 20. blocos de concreto em 7 dias, em um período
c) 22. de 6 horas diárias. Assinale a alternativa que
d) 25. apresenta quantos blocos essa indústria
e) 11. produziria em 15 dias se o período de trabalho
fosse de 12 horas diárias, considerando o
A4)(IFMA 2017) Numa corrida de mesmo ritmo de trabalho.
motocicletas, 20 motos de mesma potência e
consumo gastam 600 litros de gasolina em (a) 18.500 blocos.
30 minutos de corrida. Quantos litros de (b) 9.200 blocos.
gasolina uma dessas motos consome em 2 (c) 17.300 blocos.
horas de corrida? (d) 10.800 blocos.
(e) 12.600 blocos.

28
A9) (IFTM 2011) Artistas gráficos produzem De acordo com a projeção apresentada na
maior muro de grafite com temas de paz do tabela, no período de 2011 a 2020, o país com
mundo. maior aumento percentual na produção de
“Sob um sol arrasador, 700 artistas, na petróleo seria o Iraque. O segundo país com
maioria jovens grafiteiros e pintores maior aumento percentual seria:
muralistas, dedicaram-se a refletir em a) EUA
imagens suas ideias sobre a paz, o b) Brasil
desarmamento e o repúdio às diferentes c) Canadá
formas de violência. A atividade impulsionada d) Arábia Saudita
pelo “Mundo sem Guerras” levou 3 dias para
cobrir os 1800 metros de um muro localizado 2. (G1 - ifpe 2019) A Petrobras anunciou,
em um bairro da capital chilena... O desejo de nesta 6ª feira (3 de maio de 2019), o reajuste
Daniela Hirsch, uma das organizadoras do do valor do Gás Liquefeito de Petróleo,
Muro pela Paz – “criar uma consciência sobre conhecido como gás de cozinha, em 3,5%
a necessidade da paz e mudar esse mundo”- para o botijão de 13 kg. Se o aumento
conta desde hoje com uma tela pública anunciado pela Petrobras for totalmente
exposta à visão e à reflexão de milhares de repassado para o consumidor que comprava
transeuntes.” um botijão de 13 kg de gás por R$ 60,00, o
Pergunta-se: Quantos dias 525 artistas consumidor passará a pagar
levariam para cobrir 50% desse muro? a) R$ 64,20.
a) 1 dia.
b) R$ 63,50.
b) 1,25 dias.
c) 1,5 dias. c) R$ 81,00.
d) 2.25 dias. d) R$ 65,30.
e) 2 dias. e) R$ 62,10.

A10) (IFPI 2014) Um trabalho é executado por 3. (G1 - ifpe 2019) João Pedro, formado pelo
6 máquinas iguais, em 10 dias, trabalhando 8 IFPE campus Barreiros no curso de Operador
horas diárias. Quantos dias levariam 5 de Processamento de Frutas e Hortaliças,
máquinas do mesmo tipo, trabalhando 6 horas conseguiu um emprego 15 dias após
diárias para concluir o mesmo trabalho? formado, com um salário inicial de
a) 10 dias R$ 1.800,00. Dois meses após sua
b) 12 dias contratação, o seu chefe resolveu conceder-
c) 14 dias lhe um aumento de 8% devido ao seu
d) 16 dias excelente desempenho no cargo. Após
e) 18 dias receber esse aumento, João Pedro passou a
receber um salário de
ATIVIDADE COMPLEMENTAR a) R$ 1.956,52.
1. (Uerj 2019) b) R$ 1.808,00.
c) R$ 2.600,00.
d) R$ 1,944,00.
e) R$ 2.160,00.

4. (G1 - ifpe 2019) Em uma loja de roupas,


uma determinada camisa custa R$ 80,00.
Essa loja resolveu aumentar o preço dessa
camisa em 20%. Após esse aumento,
Carolina foi à loja para comprar a camisa e,
por pagá-la à vista, a vendedora aplicou um
desconto de 10%. Carolina aceitou o desconto
e pagou à vendedora a quantia de
a) R$ 88,00.
b) R$ 86,40.
c) R$ 90,00.
d) R$ 86,00.
e) R$ 88,60.

29
5. (Enem PPL 2019) Deseja-se comprar 8. (G1 - cp2 2018) Marcelo foi a uma pizzaria
determinado produto e, após uma pesquisa de e consumiu uma pizza do menu mostrado a
preços, o produto foi encontrado em 5 lojas seguir:
diferentes, a preços variados.
Brotinh
- Loja 1: 20% de desconto, que equivale a Pizza Média Grande
o
R$ 720,00, mais R$ 70,00 de frete;
Muçarela R$ 15,00 R$ 25,00 R$ 30,00
- Loja 2: 20% de desconto, que equivale a
R$ 740,00, mais R$ 50,00 de frete; Calabresa R$ 16,00 R$ 27,00 R$ 33,00
- Loja 3: 20% de desconto, que equivale a
R$ 760,00, mais R$ 80,00 de frete; Presunto R$ 17,00 R$ 29,00 R$ 36,00
- Loja 4: 15% de desconto, que equivale a Portugues
R$ 19,00 R$ 31,00 R$ 39,00
R$ 710,00, mais R$ 10,00 de frete; a
- Loja 5: 15% de desconto, que equivale a
R$ 690,00, sem custo de frete. Quando veio a conta, o garçom avisou que foi
acrescentada uma taxa de serviço de 10%
O produto foi comprado na loja que sobre os valores do cardápio, perfazendo um
apresentou o menor preço total. O produto foi total de R$ 31,90 a pagar. A pizza que Marcelo
adquirido na loja consumiu foi
a) 1. a) presunto média.
b) 2. b) calabresa média.
c) 3. c) muçarela grande.
d) 4. d) portuguesa brotinho
e) 5.
6. (G1 - ifal 2018) No exame de seleção para 9. (G1 - ifpe 2018) Daiana é aluna do curso
o ano de 2017, o IFAL ofereceu 504 vagas de Informática para Internet no campus
para seus cursos Integrados e, no exame de Igarassu e está estagiando no setor de testes
seleção para o ano de 2018, está oferecendo em uma empresa que desenvolve aplicativos
630 vagas. Qual é o percentual de aumento (apps) para celulares. No primeiro semestre
do número de vagas para o ano de 2018? do estágio ela já testou 44 apps para o
a) 12,6%. sistema Android, 36 apps para o sistema IOS
b) 20,0%. e 30 que foram feitos para ambos os
c) 25,0%. sistemas. Considerando que Daiana
d) 30,0%. encontrou bugs (erros) em 20% dos apps que
e) 33,0%. testou, quantos estavam funcionando
corretamente?
a) 110.
7. (Uerj 2018) No mapa mensal de um
b) 50.
hospital, foi registrado o total de 800 cirurgias
c) 30.
ortopédicas, sendo 440 em homens,
d) 88.
conforme os gráficos abaixo.
e) 40.

10. (Famerp 2018) Em 2016, um determinado


país teve T casos de cânceres em homens,
dos quais 64% correspondiam aos dez tipos
mais frequentes. Sabe-se que 30% dos dez
tipos mais frequentes correspondiam ao
câncer de próstata, que totalizaram, naquele
De acordo com esses dados, o número total ano, 60.000 casos. Nessas condições, T é
de cirurgias de fêmur realizadas em mulheres igual a
foi: a) 312.500.
a) 144 b) 292.500.
b) 162
c) 296.500.
c) 184
d) 298.000.
d) 190
e) 305.000.

30
𝑥
PORCENTAGEM X% de 400 = 80 → 100
.• 400 = 80 → 4x = 80
80
Você já viu, no seu dia a dia, imagens como →x= . → x = 20%.
4
mostra a figura a seguir.
Veterinária
𝑥
X% de 500 = 60 → .• 500 = 60 → 5x = 60
100
60
→x= 5
. → x = 12%.

Nesse comercial o valor apresentado, 50%


(lê-se cinquenta porcento) é o que chamamos PROFESSOR RESPONDE
de porcentagem. Que pode ser representado
50
na forma fracionária 100.
P1) O preço de um litro de gasolina comum
Porcentagem (ou percentagem) é qualquer passou de R$ 3,20 para R$ 3,40. Significa
𝑎 dizer que o aumento percentual foi de:
razão ., na qual o número b é igual a 100.
𝑏
(A) 5,45
Agora vamos ver diferentes maneiras de
resolver problemas que envolvam (B) 5,80
porcentagens. (C) 6,25
1ª situação: (D) 6,50
Um amigo está me devendo R$ 80,00. Esse (E) 7,35
mês ele vai me pagar 20% do que me deve.
Quando irei receber?
Resolução P2) Dois quintos de 30% de 80 vale:
(A) 8,4
(B) 9,6
20% de 80 = x 80 = = 16
(C) 10,2
Logo, irei receber R$ 16,00.
(D) 11,5
2ª situação:
(E) 12
Mauro trabalha como empacotar numa
fábrica. Ele já empacou 30 produtos que P3) Apesar de a principal pauta das
correspondem a 15% do total. Qual o total de manifestações em São Paulo ser a redução
produtos que ele empacotou nesse dia? das tarifas do transporte público, o principal
motivo de participação foi a luta contra a
Resolução corrupção, de acordo com pesquisa Datafolha
15% de x = 30 realizada durante o protesto da última quinta-
15 30𝑋100 feira (20/06/2013). Metade dos entrevistados
100
.x = 30 → x = 15
.= 200 produtos. citou a corrupção como a principal bandeira.
Em seguida, aparece queda na tarifa (32%),
3ª situação:
contra os políticos (27%), melhora na
Observe, na tabela, o número de inscritos e o qualidade do transporte (19%) e contra a PEC
número de aprovados para os cursos de 37 (16%).
Arquitetura e Veterinária de certa Fonte:
universidade. Qual a taxa percentual de http://www.tribunahoje.com/noticia/67488/politica/2013/
aprovação em cada um desses cursos. 06/21/pesquisa-aponta-corrupco-como-principal-
motivo-de-protestos.html
Arquitetura Veterinária
Inscritos 400 500
Sabendo que a pesquisa entrevistou 550
Aprovados 80 60 manifestantes durante a manifestação na
avenida Paulista, pode-se afirmar que o
Arquitetura número de manifestantes que citou mais de
um motivo é:

31
(A) 33 T = 4 meses.
(B) 242 J=?
(C) 275 J = C.i.t → J = 800 • 0,02 • 4 = 64
(D) 308 Logo o juro (J) pago por Mário foi de R$ 64,00.
(E) 517 2. Determine quanto redeu de juros simples
um capital de R$ 1 200,00, durante três anos,
à taxa de 2% ao mês.
JUROS SIMPLES
Resolução:
C = 1 200,00
O termo “juros” é amplamente utilizado no dia
I = 2% = 2 • 12 (ao ano) = 24% a.a. = 0,24 a.a.
a dia. Quando tomamos emprestado uma
quantia, pagamos um “aluguel” ou quando T = 3 anos
emprestamos, por exemplo, cobramos um
J=?
“aluguel”. Esse “aluguel” denomina-se juros.
J = C.i.t → J = 1 200 • 0,24 • 3 = 864
Os juros simples são sempre calculados em
relação ao capital inicial, período a período. Logo a aplicação rendeu R$ 864,00 de juros.
Assim, o valor dos juros é constante em cada
período de tempo.
3. Calcule o montante recebido por uma
O valor dos juros depende: pessoa que aplicou R$ 15 000,00, durante 15
● do capital (C) – dinheiro que se meses, a juros simples de 3% ao ano.
empresta ou toma emprestado. Resolução
● da taxa (i) – percentual que representa C = 15 000,00
os juros recebidos ou pagos, ao final de um
período. I = 3% a.a. = 3 : 12 (ao mês) = 0,25% a.m.
● do tempo (t) – período utilizado na T = 15 meses
transação. M=?
O total de juros é dado pela fórmula: J = C . i
.t
J = C.i.t → J = 15 000 • 0,25 • 15 = 562,50
O total a ser pago ao final do empréstimo
denomina-se montante (M).
E é dado pela fórmula M = C + J. O montante será:
OBS.: M = C + J → M = 15 000 + 562,50 = 15 562,50
1. Na determinação dos juros, a taxa e o Logo o montante será R$ 71 250,00.
tempo devem estar relacionados na mesma
unidade.
PROFESSOR RESPONDE
2. A taxa é normalmente calculada ao ano ou
ao mês.
3. Por convenção, o mês comercial tem 30 P1) Uma televisão é vendida à vista por
dias e o ano comercial tem 360 dias. R$ 1 600,00 ou a prazo com 25% de entrada
Exemplos: mais uma parcela de R$ 1 332,00 após 5
meses. Qual a taxa mensal de juros simples
1. Mário pediu um empréstimo de R$ 800,00 do financiamento?
a um amigo à taxa de juro simples de 2% ao
mês, durante 4 meses. Qual o juro pago por (A) 2,0%
Mário? (B) 1,9%
Resolução: (C) 2,1%
C = 800,00 (D) 2,3%
I = 2% = 0,02 a.m. (E) 2,2%

32
P2) Certa quantia foi aplicada a uma taxa de A3) SP e Rio recuam no aumento das
2% ao mês durante 9 meses no regime de passagens; em BH queda depende de projeto
capitalização simples. Se os juros obtidos de lei
foram de R$ 135,00, a quantia aplicada
No sexto e maior protesto ocorrido em São
durante esse período foi:
Paulo, mais de 65 mil pessoas, a maioria
(A) R$ 620,00 jovens, saíram às ruas para pedir, entre outras
coisas, o cancelamento do reajuste de R$
(B) R$ 750,00
0,20 nas tarifas do transporte público.
(C) R$ 700,00
As manifestações pelo país que começaram
(D) R$ 780,00 com a reivindicação de redução das tarifas
(E) R$ 800,00 dos transportes públicos tiveram três
importantes vitórias. Governantes de São
Paulo e Rio de Janeiro recuaram e reduziram
P3) Os juros simples produzidos por um as passagens de ônibus, metrô e trem. Em
capital de R$ 3600,00 aplicado durante 10 Belo Horizonte, o prefeito Marcio Lacerda
meses, à taxa de 25% ao ano é: (PSB) anunciou que vai enviar à Câmara
Municipal um projeto de lei para viabilizar
(A) R$ 900,00 também a queda nos valores.
(B) R$ 850,00 Fonte: http://www.em.com.br/app/noticia/politica/2013/06/20/
interna_politica,409073/sp-e-rio-recuam-no-aumento-das-
(C) R$ 800,00 passagens-em-bh-queda-depende-de-projeto-de-lei.shtml,
acesso em 20/06/2013.
(D) R$ 750,00
O reajuste da tarifa de ônibus em São Paulo
(E) R$ 700,00 seria de R$ 3 para R$ 3,20 e no Rio de Janeiro
seria de R$ 2,75 para R$ 2,95, representando
um aumento percentual aproximado,
ALUNO RESPONDE respectivamente:
A1) Um determinado produto é vendido na loja (A) 6,3% e 6,8%
X por um preço de R$ 80,00 a mais do que é
vendido na loja Y. Sabendo-se que os preços (B) 6,7% e 6,8%
nas duas lojas tornam-se iguais quando da (C) 6,7% e 7,3%
aplicação de um desconto de 20% no preço
da loja X, qual é o preço desse produto na loja (D) 7,3% e 6,3%
Y? (E) 7,3% e 6,8%
(A) R$ 280,00 A4) Sabe-se que a comissão de um corretor
(B) R$ 290,00 de imóveis é 5% do valor da venda efetuada.
Se um proprietário recebe, pela venda de um
(C) R$ 300,00 imóvel, R$ 380 000,00, já descontada a
(D) R$ 310,00 comissão do corretor, qual o valor da
comissão?
(E) R$ 320,00
(a) R$ 5 000,00
A2) Um comerciante pagou 20% de uma (b) R$ 8 000,00
dívida. Marque a alternativa que determina a (c) R$ 12 000,00
dívida inicial, sabendo que com R$ 32 000,00 (d) R$ 15 000,00
ele pagou 20% do restante. (e) R$ 20 000,00
(A) R$ 160 000,00 A5) Um determinado calçado era vendido a
R$ 320,00 o par. Durante uma promoção, o
(B) R$ 180 000,00 preço do referido produto passou para R$
(C) R$ 200 000,00 240,00. O percentual dessa queda foi de:
(D) R$ 220 000,00 (a) 8%
(b) 10%
(E) R$ 240 000,00 (c) 15%
(d) 25%
(e) 75%

33
A6) O salário mensal líquido de Elias é R$ A11) (IFPI – 2015) Em uma prova de
8.610,00. Esse valor é obtido após serem português, sabe-se que a razão entre o
descontados do salário bruto: 11% para o número de questões que Evelina acertou e o
plano de seguridade social e 27,5% para o número de questões que errou, nessa ordem,
imposto de renda. Não havendo outros foi de 7 para 3. Se Evelina errou 12 questões,
descontos, afirmamos que o salário bruto de então, de quantas questões a prova era
Elias é: composta?
(a) R$ 13.550,00
(b) R$ 13.700,00
(a) 48
(c) R$ 13.850,00
(b) 46
(d) R$ 14.000,00
(c) 45
(e) R$ 14.150,00
(d) 40
(e) 32
A7) Um capital, diminuído dos seus juros
simples de 8 meses, reduz-se a R$ 7 640,00.
O mesmo capital acrescido de seus juros A12) (IFPI 2015) Na compra de um imóvel em
simples de 10 meses soma R$ 8 000,00. sociedade, Esmeraldina investiu R$
Marque a alternativa que indica o valor desse 120.000,00 e Ponciano, R$ 180.000,00. No
capital. final de um ano, venderam o imóvel por R$
450.000,00. Tal valor foi dividido entre eles em
(a) R$ 7950,00
partes diretamente proporcionais aos seus
(b) R$ 7850,00
capitais de participação na sociedade. Dessa
(c) R$ 7800,00
forma, pode-se concluir que a quantia que
(d) R$ 7750,00
Esmeraldina recebeu após a venda foi de:
(e) R$ 7700,00

A8) Um capital de R$ 9.000,00 é aplicado a (a) R$ 178.000,00


juros simples, à taxa de 2% a.m., durante 5 (b) R$ 178.500,00
meses. Qual o montante dessa aplicação? (c) R$ 179.000,00
(a) R$ 900,00 (d) R$ 179.500,00
(b) R$ 4500,00 (e) R$ 180.000,00
(c) R$ 9300,00
(d) R$ 9900,00
A13) (IFPI 2014) Em um mapa da Região
(e) R$ 13500,00 Nordeste, de escala 1:12000000, a distância
em linha reta, entre a cidade de Nova Viçosa-
A9) O juro simples correspondente a um BA e a cidade de Carutapera-MA mede
capital de R$ 20 000,00, aplicado durante, 5 aproximadamente 16,6cm. Em linha reta, a
meses, à taxa de 36% ao ano é: distância real em quilômetros, segundo essa
escala é aproximadamente:
(a) R$ 5200,00
(b) R$ 4100,00
(c) R$ 3300,00 (a) 1 250 km
(d) R$ 3000,00 (b) 1 585 km
(e) R$ 2400,00 (c) 1 730 km
(d) 1 992 km
A10) Marque a alternativa que indica a que (e) 2 210 km
taxa mensal deve estar aplicada a quantia de
R$ 75 000,00 para que em, 3 meses e 10 dias, A14) (IFPI 2016) Numa classe de 32 alunos, a
renda um juro simples de R$ 5 000,00. razão entre o número de meninos e o de
meninas é 3/5 . Quantos são os meninos?
(a) 3,5 %
(b) 3,0 % (a) 24
(c) 2,5 % (b) 20
(d) 2,0 % (c) 12
(e) 1,5 % (d) 10
(e) 8

34
A15) (IFAM 2013) Um quadrado foi dividido (d) 5,4 m
em quadrados menores e estes pintados em (e) 6,0 m
duas cores. Em qual figura a razão entre as A19) (IF/Goiano 2014) A tabela a seguir
áreas pintadas é de 1 para 2? representa o preço (em reais) de cada pacote
de determinado produto em função de sua
massa, dada em quilogramas (kg). Esse
produto esta disponível em pacotes com
quatro pesos diferentes e o preço por
quilograma independe do tamanho do pacote.
Acidentalmente, a tabela foi danificada e
alguns valores ficaram ilegíveis.

Nessas condições, o preço por quilo do


produto, o preço do pacote menor, o preço do
pacote equivalente ao quádruplo do pacote
menor e a massa do pacote maior são,
respectivamente,
(a) R$ 0,50, R$ 1,00, R$ 4,00 e 10,0 kg.
(b) R$ 0,50, R$ 1,00, R$ 4,00 e R$ 10,0.
(c) R$ 2,00, R$ 4,00, R$ 16,00 e R$ 10,0.
(d) R$ 2,00, R$ 1,00, R$ 4,00 e 10,0 kg.
(e) R$ 2,00, R$ 4,00, R$ 16,00 e 10,0 kg.
A20) (IFMA 2016) Uma caixa de chocolate
A16) (IFF 2014) Uma dimensão de uma foto custa R$ 12,00. Outra caixa de mesma
retangular mede 7 cm a mais do que a outra qualidade, valores proporcionais ao número
dimensão e o perímetro mede 78 cm. Essa de chocolates e com mais 4 chocolates custa
foto deve ser ampliada de modo que a R$ 18,00. Quantos chocolates há em cada
ampliação seja semelhante à foto. A menor caixa?
dimensão da ampliação é 32 cm. Então a sua (a) 9 e 13
maior dimensão será de (b) 7 e 11
(a) 36 cm. (c) 5 e 9
(b) 40 cm. (d) 6 e 10
(c) 45 cm. (e) 8 e 12
(d) 46 cm.
(e) 64 cm. ATIVIDADE COMPLEMENTAR
A17) (IFGoiano 2014) Em um colégio local
existem 1 400 estudantes dos quais 600 são 1. (G1 - ifsc 2017) Analise as seguintes
do sexo masculino. Qual e a proporção de situações:
estudantes do sexo masculino em relação a Seu João fez um empréstimo de R$ 1.000,00
do sexo feminino? no Banco A, a uma taxa de juros simples; após
4 meses, pagou um montante de R$ 1.320,00
(a) 3/4 e quitou sua dívida.
(b) 3/7 Dona Maria fez um empréstimo de R$
(c) 4/7 1.200,00 no Banco B, a uma taxa de juros
(d) 7/3 simples; após 5 meses, pagou um montante
(e) 4/3 de R$ 1.800,00 e quitou a dívida.
A18) (IFGoiano 2014) Um homem de 1,80 m
de altura projeta uma sombra de 3 m de A taxa mensal de juros simples cobrada pelo
comprimento no mesmo instante em que um Banco A e pelo Banco B, respectivamente, é:
poste projeta uma sombra de 10 m. Nessas a) 8% a.m. e 10% a.m.
condições, a altura do poste e
b) 18% a.m. e 13% a.m.
(a) 3,0 m c) 6,4% a.m. e 12,5% a.m.
(b) 4,8 m
d) 13% a.m. e 18% a.m.
(c) 5,0 m
e) 10% a.m. e 8% a.m.

35
2. (Ufsm 2015) A chegada da televisão no CÁLCULO ALGÉBRICO, PRODUTOS NOTÁVEIS E
Brasil facilitou o acesso à informação. Com o FATORAÇÃO
avanço da tecnologia, os aparelhos estão
cada dia mais modernos e consequentemente EXPRESSÕES ALGÉBRICAS
mais caros. Uma expressão matemática que apresenta
Um consumidor deseja adquirir uma televisão números e letras é uma expressão algébrica
com tecnologia de última geração. Enquanto ou literal.
aguarda o preço da televisão baixar, ele aplica
o capital disponível de R$3.000,00 a juros CLASSIFICAÇÃO:
simples de 0,8% ao mês em uma instituição EXPRESSÃO ALGÉBRICA IRRACIONAL:
financeira, por um período de 18 meses. O ✔ Apresenta letras no radicando.
montante, ao final desse período, é igual a
a) R$7.320,00. Exemplos:
b) R$5.400,00.
c) R$4.320,00.
d) R$3.432,00.
e) R$3.240,00.
EXPRESSÃO ALGÉBRICA RACIONAL:
3. (Unicamp 2015) Uma compra no valor de ✔ Não apresenta letras no radicando.
1.000 reais será paga com uma entrada de
Exemplos:
600 reais e uma mensalidade de 420 reais. A
taxa de juros aplicada na mensalidade é igual
a
a) 2%.
b) 5%.
c) 8%. ✔ As expressões algébricas racionais podem
d) 10%. ser inteiras ou fracionárias.
EXPRESSÃO ALGÉBRICA RACIONAL
4. (Uepa 2015) Um agricultor financiou junto INTEIRA:
a uma cooperativa os insumos utilizados na
lavoura em 2014. Pagou 20% do valor dos ✔ Não apresentam letras no denominador.
insumos no ato da compra, utilizando parte do Exemplos:
lucro obtido no ano anterior, e financiou o
restante em 10 meses a uma taxa de 2% ao
mês a juros simples. Observou que havia
gastado o montante de R$ 208.800,00 com a
EXPRESSÃO ALGÉBRICA RACIONAL
parte financiada. Neste caso, o valor
FRACIONÁRIA:
financiado dos insumos pelo agricultor foi de:
a) R$ 217.500,00 Apresentam letras no denominador.
b) R$ 174.000,00 Exemplos:
c) R$ 164.000,00
d) R$ 144.500,00
e) R$ 136.000,00
VALOR NUMÉRICO DE UMA EXPRESSÃO
5. (G1 - cftmg 2014) Uma concessionária ALGÉBRICA
anunciou um veículo no valor de R$30.000,00
à vista. Após negociação, um cliente adquiriu Exemplos:
o veículo pagando R$20.000,00 de entrada e 𝑛(𝑛−3)
a) A expressão algébrica 𝑑 = 2
R$11.200,00 após 30 dias. A taxa mensal de
juros cobrada nessa venda foi de permite calcular o número de diagonais de um
a) 4%. polígono convexo. Calcule quantas diagonais
b) 6,6%. possui um polígono de 15 lados.
c) 11,2%. 𝑛(𝑛−3) 15(15−3) 15.12
d) 12%. R: 𝑑 = = = = 90
2 2 2
90 Diagonais.

36
b) Sendo a = 2, b = -1 e c = 3, determine OBS: Todo número real não-nulo é um monômio
o valor numérico da expressão sem parte literal.

Exemplo: 5; -10; ;
22 −2(−1) 2 4+2 6 2 GRAU DE UM MONÔMIO
R: + + 3(−1) = = + −3
3(3) 6 9 9 6
É dado pela soma dos expoentes da parte
literal.
12 6 54 12 + 6 − 54 −36 Exemplo:
= + − = = = −2
18 18 18 18 18
A) 1+1=2 é um monômio do 2º
grau
OBS: Existem expressões algébricas
fracionárias que não representam números
reais quando determinados valores são B) 1+2=3 é um monômio do 3º
atribuídos às letras (variáveis). Isso acontece grau
quando esses valores anulam o denominador
da expressão, pois não existe divisão por C) 2+5=7 é um monômio do 7º
zero. grau
Assim:
𝑎+2
A expressão 𝑎−1não representa número real MONÔMIOS SEMELHANTES
quando a = 1.

Possuem a mesma parte literal ou não


Exemplo: Para qual valor de x a expressão possuem parte literal(são apenas números).
𝑥−3
algébrica 2𝑥−1não representa número real? Exemplo:
São monômios semelhantes:

R:
A expressão não representa número real A)

para . B)
MONÔMIO OU TERMO ALGÉBRICO Não são monômios semelhantes:
✔ É toda expressão algébrica inteira
representada apenas por um número, ou apenas
uma variável, ou por apenas um produto de A) e não possuem a mesma
números reais. parte literal

Exemplo:
ADIÇÃO ALGÉBRICA E MONÔMIOS
PARTE
MONÔMIO COEFICIENTE
LITERAL Na prática a adição algébrica é obtida
somando-se algebricamente os coeficientes
5 e conservando-se a parte literal.
Exemplos:

A)
B)
1
=

37
MULTIPLICAÇÃO DE MONÔMIOS OPERAÇÕES COM POLINÔMIOS
Na prática o produto de dois monômios é ⮚ ADIÇÃO ALGÉBRICA DE POLINÔMIOS
obtido da seguinte forma:
Exemplo: Qual polinômio representa o
I.Multiplicam-se os coeficientes perímetro da figura
II. Multiplicam-se as partes literais X–3 x+1

Exemplos: 2x+1
P = x – 3 + 2x + 1 + x + 10 = 4x + 8
A)
Exemplo: Dados os polinômios A= x² - 3x + 5,
B) B = x² + 2x – 4 e C = x² + 6x – 1, calcule A +
B – C:
C)
R: A + B – C =
DIVISÃO DE MONÔMIOS
(x² - 3x + 5)+( x² + 2x – 4) – (x² + 6x – 1)=
Na prática o quociente de dois monômios é
obtido da seguinte forma: x² - 3x + 5 + x² + 2x – 4 - x² - 6x + 1=
I.Dividem-se os coeficientes x² - 7x + 2
II.Dividem-se as partes literais ⮚ MULTIPLICAÇÃO DE POLINÔMIOS
Exemplos: Exemplo: escreva o polinômio que
representa a área do retângulo abaixo

A) X+1

B) 2x + 1
Área: (2x+1)(x+1) = 2x² + 3x + 1
POTENCIAÇÃO DE MONÔMIOS
Na prática a potência de um monômio é
obtida da seguinte forma: ⮚ DIVISÃO DE POLINÔMIOS
I.Eleva-se o coeficiente à potência indicada ✔ Divisão de polinômio por monômio
II.Eleva-se a parte à potência indicada Exemplo: Determine o polinômio que
Exemplos: multiplicado por -7x dá como resultado o
polinômio .
A) R:

B) =

POLINÔMIOS
Representam uma adição algébrica de ✔ Divisão de polinômio por polinômio
monômios Exemplo: Determine o polinômio M que
MONÔMIO POLINÔMIO multiplicado por 2x -1 dá 6x² -7x + 2
DE UM SÓ TERMO R: 6x² - 7x + 2 2x - 1
BINÔMIO POLINÔMIO - 6x² +3x 3x - 2
DE DOIS TERMOS - 4x + 2
TRINÔMIO POLINÔMIO 4x - 2
DE TRÊS TERMOS
(0)
POLINÔMIO M = 3x - 2

38
⮚ PRODUTOS NOTÁVEIS ✔ Fatoração da diferença de dois
quadrados
I.QUADRADO DA SOMA DE DOIS
TERMOS Exemplos:
(x + y)² = x² + 2xy + y² x² - 9 =x² - 3² =(x + 3)(x - 3)
II.QUADRADO DA DIFERENÇA DE DOIS 49x² -a²b² =(7x)² - (ab)²=(7x+ab) (7x-ab)
TERMOS
(x - y)² = x² - 2xy + y²
✔ Fatoração do trinômio quadrado
III.PRODUTO DA SOMA PELA perfeito
DIFERENÇA DE DOIS TERMOS
Exemplos:
(x + y)(x – y) = x² - y²
x² + 6x + 9=(x+3)²
IV.CUBO DA SOMA DE DOISTERMOS
(x + y )³ = x³ + 3x²y + 3xy² + y³
x² 2.x.3 3²
V.CUBO DA DIFERENÇA DE DOIS
TERMOS ✔ Fatoração da soma ou da diferença de
dois cubos
( x – y )³ = x³ - 3x²y + 3xy² - y³
Exemplos:
Exemplo: a³+b³ = (a+b)(a²+2ab +b²)
1. A expressão (x+1)(x-1) + 1 é igual a: a³- b³ = (a- b)(a²+2ab +b²)
a)x² b)x² -1 c)x² + 3 d)2x
R: x² - 1² +1= x² -1+1= x² PROFESSOR RESPONDE
2. A expressão (a + b)² - 2ab é igual a: P1) (Mack-SP) Se (x-y)² - (x+y)² = -20 então x
. y é igual a:
a) a²-b²
(A) 0
b) a² -4ab +b²
(B) -1
c) a² +4ab +b²
(C) 5
d) a² +b²
(D) 10
R: a² +2ab +b² -2ab = a² + b²

⮚ FATORAÇÃO DE POLINÔMIOS P2) O valor exato de 88888² -11112² é


Fatorar um polinômio, quando possível, (A) 77776 x10²
significa escrevê-lo na forma de um produto
de polinômios mais simples. (B) 77776 x10³
(C) 77776 x104
✔ Colocando o fator comum em
evidência (D) 77776 x105

Exemplo: (E) 77776 x106

2x + 2y = 2(x +y) P3) Dividindo o polinômio P por 2x +3,


obtemos como quociente x + 5 e como resto,
6ax +8ay = 2a(3x + 4y) x – 2. Qual é o polinômio P?
2x + 2y +2z =2(x + y + z)
(A)
✔ Fatoração por agrupamento
(B)
Exemplos
A) ax+ay +bx+by= B) xy +2x +4y +8 = (C)

a(x+y) +b(x+y) = x(y+2) +4(y+2)= (D)


(x+y)(a+b) (y+2)(x+4)

39
P4) (Prova Brasil) Dada a expressão: ALUNO RESPONDE

A1) (IFAM-2012) Observe as seguintes


afirmações:

Sendo a = 1, b = -7 e c = 10, o valor I.


numérico de x é:

II.
(A) -5
(B) -2
(C) 2 III.
(D) 5
IV.

P5) (UECE) Considere a expressão


(A) Todas são verdadeiras
(B) Apenas uma é falsa
(C) Duas são falsas
(D) Três são falsas

, x≠0 e x≠1. Seu valor numérico (E) Todas são falsas

para é: A2) Partindo de um retângulo de dimensões


8 e 12, um garoto recorta, de cada canto, um
quadrado de lado x, conforme a figura:
(A)
(B) negativo
(C) 2,5
(D) 5,2 x

P6) Sendo o valor das


x

expressões e são
respectivamente:
Dobrando nas linhas tracejadas, o garoto
obtém uma caixa. A expressão que melhor
(A) 1 e 3
representa o volume máximo dessa caixa é:
(B) 3 e 5
(C) 7 e 5
(A) 24 -3x.
(D) 7 e 3
(B) 8x – 2x².
(E) 2 e 1
(C) 12x – 2x².
(D) 4x³ – 40x² + 96x.
(E) 8x + 20.

40
A3) (IFAM-2015) Dividindo os polinômios A7) (IFPI-2014) Podemos afirmar que na
A = x3 – 2x2 – 8x + 17 e B = x3 – 3x2 – 8x + 28 divisão do polinômio
por um polinômio P, os restos são os pelo polinômio x2 + 2x + 1:
binômios
(A) o resto é
x - 1 e x + 1, respectivamente. Nessas
condições, o polinômio P de maior grau (B) o resto é
possível é:
(A) x - 3 (C) o quociente é
(B) x - 2 (D) o quociente é
(C) x² + 9 (E) a soma do quociente com o resto é x² + 2x -7
(D) x² - 9
A8) (IFPI-2014) Sendo e
(E) x² + 8
, temos que:

A4) (IFAM-2012) Fatorando a expressão (A) x. y = 21


(B) x. y = -37

obtemos: (C) x. y = 37

(A) a - b (D) x. y = -24

(B) a + b (E) x. y = -21

(C) 1 A9) (IFPI-2015) Sabendo-se que (x + y)² = 18


e x² + y² = 10, o valor de xy é:
(D) 2ab
(A) 2
(E) a² - b²
(B) 3
A5) (IFAM-2012) A expressão é (C) 4
igual a: (D) 6

(A) (E) 9
A10) (IFPI-2015) A expressão (2x + 1).(1 - x
(B) + x²) + (x - 1)² é equivalente a:
(A) 2x³ - x + 2
(C)
(B) x³ + x² + 1
(D) (C) 2x³ - 1

(E) (D) 2x³ - x² + x - 1

A6) (IFPI-2014) Qual é o valor da expressão (E) x³ + x + 2

xa – xb + ya - yb, sabendo-seque x + y = 7 e A11) Dividindo o polinômio


a - b = 12? pelo polinômio
(A) 5 obtemos o polinômio:
(B) 19
(A)
(C) 45
(D) 84 (B)
(E) 90 (C)

(D)

(E)

41
A12) (Unifor-CE) Nas sentenças abaixo a,b,c (D) x² +2x
são números reais.
(E) 2x² +x

I.
A18) (UCSal-BA) Sejam os polinômios
II.
p = x³ - 2x² + x, q = 2x – 1 e r = x + 1
III.
Efetuando-se p + q.r, obtém – se:

É correto afirmar que somente: (A)

(A) I é verdadeira (B)


(B) II é verdadeira
(C)
(C) III é verdadeira
(D)
(D) I e II é verdadeira
(E) I e III é verdadeira (E)

A13) (Saresp) Considerando A = a³ - 2a² +3 e A19) (Cesgranrio-RJ) Simplificando a


B= a³ - 2a² - a + 5.Temos que A – B é igual a: expressão a³(a² +a³) : a5 , encontramos:
(A) a -2 (A) 1+a
(B) -a+8 (B) a +a²
(C) -4a² - a + 8 (C) 1 + 5a
(D) 2³ - 4a² - a + 8 (D) 1 - a
(E) a³
A14) (PUC-MG) Se x² + y² =17 e xy = 16 o
valor de (x+y)² é:
(A) 32 A20) Sendo A =
(B) 41
B= e
(C) 49
(D) 53 C=

(E) 54
A16) (OBM) Se x + y = 8 e xy =15, qual é o Determine C – (A + B):
valor de x² +6xy + y² ?
(A)
(A) 64
(B) 109 (B)

(C) 120 (C)


(D) 124
(D)
(E) 154
A17) (Uespi-PI)Qual o resto da divisão do ATIVIDADE COMPLEMENTAR
polinômio pelo
1. (Uece 2019) Considerando o polinômio
polinômio ? 3 2
P(x)  4x  8x  x  1, é correto afirmar que o
(A) x² +3x  1
valor da soma P( 1)  P    é um número
(B) 2x² +3x  3
localizado entre
(C) 3x² +2x
a) 5,0 e 5,5.

42
b) 4,0 e 4,5. e) 04
c) 4,5 e 5,0.
d) 5,5 e 6,0. 7. (G1 - ifsc 2017) Após analisar as
afirmações a seguir sobre produtos notáveis e
2. (G1 - cotuca 2019) Ao considerar x  2.020 fatoração, marque com (V) o que for
e y  2.019, o valor da expressão verdadeiro e, com (F), o que for falso.

x8  y8 ) (3a2  2b)2  9a4  12a2b  4b2


E é: (
6 4 2 2 4 6
x x y x y y
( ) (a  b)3  a3  b3
a) 1.
b) 2019. ( ) 64a2  49b2  (8a  7b)(8a  7b)
c) 2020. ( ) 4a2  16b2  (2a  4b)2
d) 4039.
e) 4040.
( ) a3  b3  (a  b)(a2  ab  b2 )

3. (Espm 2019) O número que se deve somar Assinale a alternativa que contém a ordem
a 456.7882 para se obter 456.7892 é: CORRETA de preenchimento dos parênteses,
de cima para baixo.
a) 456.789
a) V – F – V – F – V.
b) 1
b) V – V – F – F – F.
c) 456.788
c) V – F – V – V – F.
d) 913.579 d) F – F – V – V – V.
e) 913.577 e) F – V – F – V – V.
4. (G1 - ifce 2019) Simplificando a expressão
8. (Ufrgs 2017) Se x y  2 e x2  y2  8,
a6b6  4x2
, com a3b3  2x  0, encontramos o então x3  y3 é igual a
a3b3  2x
resultado a) 12.
b) 14.
a) a4b6  2x.
c) 16.
b) a4b3  2x. d) 18.
c) a3b4  2x. e) 20.
d) a3b3  2x.
e) a3b6  2x. 9. (G1 - cftmg 2017) Simplificando a
a  b  ab  a3b  ab2  a2b
4 4 3
expressão ,
5. (Espm 2018) O valor numérico da a2  b2
x3  y3 a  b, obtém-se:
expressão para x  0,8 e
3 2 2
x  x y  xy a)
a
y  0,3 é igual a: b
a) 0,325 ab
b)
b) 0,125 ab
c) 0,415 a3  ab  b3
c)
d) 0,625 ab
e) 0,275 3(a ab  b)
d)
ab

6. (G1 - ifsc 2018) Considere x o resultado


da operação 5252  5232. 10. (G1 -ifpe 2017) Efetuando-se
(2.341)  (2.340)2 , obtém-se:
2

Assinale a alternativa CORRETA, que a) 6.489


representa a soma dos algarismos de x. b) 1
a) 18 c) 4.681
b) 13 d) 2.681
c) 02 e) 8.689
d) 17

43
EQUAÇÕES, INEQUAÇÕES E SISTEMA DE ● Utilizamos o princípio multiplicativo
EQUAÇÕES DO 1º GRAU. 1 1
. 2𝑥 .
EQUAÇÕES DO 1º GRAU 2 2
X = -1
Equação é toda sentença matemática aberta
que exprime uma relação de igualdade. A
palavra equação tem o prefixo equa, que em INEQUAÇÕES DO 1º GRAU
latim quer dizer "igual". Exemplos: Inequação é toda sentença matemática
aberta que exprime uma relação de
a) 2x + 8 = 0
desigualdade.
b) 5x - 4 = 6x + 8
c) 3a - b - c = 0 Exemplos:
Não são equações: a) 2x + 8 > 0
b) 5x - 4 < 6x + 8
a) 4 + 8 = 7 + 5 (não é uma sentença aberta)
c) 3a - b - c ≥ 0
b) x - 5 < 3 (não é igualdade)
PRINCÍPIOS DE EQUIVALÊNCIA
c) 5≠-2 (não é sentença aberta, nem
igualdade) Observe: 2x – 8 > 3x – 10
Considera a equação 2x – 8 = 3x – 10 ● PRINCÍPIO ADITIVO
A letra é a incógnita da equação. A palavra Adicionando o mesmo valor aos dois
incógnita significa “desconhecida”. membros da inequação encontraremos uma
inequação equivalente a inicial.
Na equação acima a incógnita é x; tudo que
antecede o sinal da igualdade denomina-se 1º 2x – 8 + 5 > 3x – 10 + 5
membro, e o que sucede, 2º membro. ● PRINCÍPIO MULTIPLICATIVO
Multiplicando o mesmo valor positivo aos
dois membros da inequação encontraremos
uma inequação equivalente a inicial. Mas se
multiplicarmos o mesmo valor negativo aos
PRINCÍPIOS DE EQUIVALÊNCIA
dois membros de uma inequação,
● PRINCÍPIO ADITIVO encontraremos uma inequação equivalente
com alteração do sinal de desigualdade.
Adicionando o mesmo valor aos dois
membros da equação encontraremos uma 5 (2x – 8) > (3x – 10) 5
equação equivalente a inicial.
(–5) (2x – 8) < (3x – 10) (– 5)
2x – 8 + 5 = 3x – 10 + 5
Resolução de uma Inequação do 1º grau
● PRINCÍPIO MULTIPLICATIVO
Vamos resolver a inequação 3x – 8 < 5x – 10
Multiplicando o mesmo valor aos dois
membros da equação encontraremos uma ● Semelhante ao processo de equação do 1º
equação equivalente a inicial. grau, agrupamos os termos semelhantes em
cada membro da inequação, geralmente
5 (2x – 8) = (3x – 10) 5
usamos o 1º membro para os termos com a
incógnita, utilizando o princípio aditivo.
Resolução de uma Equação do 1º grau
Vamos resolver a equação 5x – 8 = 3x – 10 3x – 8 + 8 – 5x < 5x – 10 + 8 – 5x
● Agrupamos os termos semelhantes 3x – 5x < – 10 + 8
em cada membro da equação, geralmente
● Adicionamos os termos semelhantes.
usamos o 1º membro para os termos com a
incógnita, utilizando o princípio aditivo. – 2x < – 2
5x – 8 + 8 – 3x = 3x – 10 + 8 – 3x ● Utilizamos o princípio multiplicativo
5x – 3x = – 10 + 8 1 1
(− ) . −2𝑥 > −2 . ( )
● Adicionamos os termos semelhantes. 2 2
2x = – 2 X = -1

44
SISTEMA DE EQUAÇÕES DO 1º GRAU 7 − 2𝑦 = 1
3º passo: Resolvemos a equação obtida no
Introdução 2º passo:
Alguns problemas de matemática são 7 − 2𝑦 = 1
resolvidos a partir de soluções comuns a duas
equações do 1º grau com duas variáveis. −2𝑦 = 1 − 7

Nesse caso, diz-se que as equações formam −2𝑦 = −6


um sistema de equação do 1º grau com −6
duas variáveis, que indicamos escrevendo 𝑦=
−2
as equações abrigadas por uma chave. Veja
os exemplos: 7=3

𝑥+𝑦 =5 3𝑥 − 𝑦 = 10 obtendo, assim, o valor de y.


a) { b) {
2𝑥 − 𝑦 = 9 𝑥 − 𝑦 = 18
4º passo: (Para encontrarmos o valor de x)
O par ordenado que verifica, ao mesmo Substitui-se o valor encontrado no 3º passo
tempo, as duas equações é chamado em qualquer uma das equação iniciais.
solução do sistema. Indicamos pela letra S,
de solução. 𝑥+𝑦 =7
Por exemplo, o par (7,3) é solução do sistema 𝑥 + (3) = 7

𝑥 + 𝑦 = 10 𝑥 =7−3
{
𝑥 − 3𝑦 = −2 𝑥=4
Pois verifica as duas equações.
5º passo: Por último, escrevemos a solução
7 + 3 = 10 do sistema: S = {(4,3)}.
{ 𝑥 = 2𝑦
7 − 3. (3) = −2
2º exemplo: Resolva o sistema {
2𝑥 − 5𝑦 = 3
Resolução de sistemas de equações do 1° 𝑃𝑎𝑠𝑠𝑜 1: 𝑥 = 2𝑦
grau ( 2 x 2)
𝑃𝑎𝑠𝑠𝑜 2:
Os processos ou métodos mais comuns são:
2𝑥 − 5𝑦 = 3 ⇒ 2(2𝑦) − 5𝑦 = 3 ⇒ 4𝑦 − 5𝑦 = 3
o método da substituição, método da adição,
⇒ −1𝑦 = 3
método da comparação, além do método
gráfico. 𝑃𝑎𝑠𝑠𝑜 3: − 𝑦 = 3 ⇒ 𝑦 = −3
Método da substituição 𝑃𝑎𝑠𝑠𝑜 4: 𝑥 = 2𝑦
Para aprender a trabalhar com esse método, : 𝑥 = 2 . (−3)
você deve acompanhar os passos indicados
nos exemplos a seguir: : 𝑥 = −6
𝑥+𝑦 =7
1º exemplo: Resolver o sistema { A solução do sistema é: 𝑆 = {(−6, −3)}
𝑥−𝑦 =1
2º passo: Isola-se uma das variáveis em uma Método da comparação
das equações. Vamos isolar x na 1ª equação: Este método consiste, basicamente, em isolar
𝑥+𝑦 =7⟹ 𝑥 =7−𝑦 a mesma variável nas duas equações.
𝑥+𝑦 =7
a) {
3º passo: Substitui-se a expressão 𝑥−𝑦 =1
encontrada no passo 1 na outra equação. 𝑥−𝑦 =1
Obtemos então uma equação do 1º grau com 1º exemplo: Resolver o sistema {
𝑥 − 3𝑦 = −3
apenas uma incógnita
1° passo: Isolando x na 1ª equação:
𝑥−𝑦 =1
𝑥−𝑦 =1⇒𝑥 =1+𝑦 1
(7 − 𝑦) − 𝑦 = 1
2º passo: Isolando x na 2ª equação:
7−𝑦−𝑦 =1
𝑥 − 3𝑦 = −3 ⇒ 𝑥 = −3 + 3𝑦 2

45
3º passo: Comparando 1 e 2, vem: 5𝑥 − 3𝑦 = 15 Como -3y+3y=0, o y
{ ⊕ ⟶
2𝑥 + 3𝑦 = 6 desaparece.
𝑥=𝑥 Ai, fica tudo mais fácil!
7𝑥 + 0 = 21
1 = 𝑦 = −3 = 3𝑦 7𝑥 = 21
𝑦 − 3𝑦 = −3 − 1 𝑥=3
−2𝑦 = −4
−4 Agora, é só substituir o valor de x em uma das
𝑦=
−2 equações do sistema:
𝑦=2
5𝑥 − 3𝑦 = 15
5. (3) − 3𝑦 = 15
4º passo: Como x = 1+y, temos: 15 − 3𝑦 = 15
𝑥 = 1 + (2) −3𝑦 = 15 − 15
−3𝑦 = 0
𝑥=3
𝑦=0
Conjunto-Solução: S = {(3,4)} A única solução do sistema é o par (3,0)
𝑥 = 5𝑦 Exemplo 2: Vamos resolver o sistema
2º exemplo: Resolver o sistema {
𝑥 + 3𝑦 = 16
1º passo: x = 5y 1
2º passo: Isola-se x na 2ª equação
Aqui, seria inútil somar imediatamente as
𝑥 + 3𝑦 = 16 equações. Como não observamos termos
𝑥 = 16 − 3𝑦 2 opostos (que somados resulta 0), nenhuma
letra desaparece. Mas, podemos obter termos
3º passo: Comparando 1 e 2, vem opostos.
5y = 16 – 3y Veja que o MMC entre 5 e 2 (coeficientes de x
5y + 3y =16 nas duas equações) é 10. Daí, multiplicamos
8y = 16 a 1ª equação por 2 e a 2ª equação por -5:
y=2 2𝑥 − 5𝑦 = 16 . (2) 4𝑥 − 10𝑦 = 32
{ ⇒ {
4º passo: Como x = 5y, temos: 3𝑥 + 2𝑦 = 2 . (−5) 15𝑥 − 10𝑦 = −10
x = 5.(2) Você viu bem?!!! Com isso, conseguimos
x = 10 termos opostos neste último sistema.

A solução é S = {(10,2)} E como +10y –10y = 0, vem:


4𝑥 + 10𝑦 = 32 ⊕
Método da Adição {
−15𝑥 − 10𝑦 = −10
Adicionando ou subtraindo membro a membro −11𝑥 + 0 = 22
duas igualdades, obtemos uma nova −11𝑥 = 22
igualdade. 22
O método consiste em somar as duas 𝑥 = −11
equações, mas isso deve ser feito sempre de 𝑥 = −2
modo a eliminar uma das variáveis na nova
equação obtida. Ou seja, é preciso chegar a Agora, levamos x = -2 na 2ª equação para
uma só equação, com uma só incógnita. Para encontrar o valor de y:
que isso ocorra, é necessário existam termos 3𝑥 + 2𝑦 = 2
opostos nas duas equações (em relação a
uma mesma letra...). 3(−2) + 2𝑦 = 2
Exemplo 1: Considere o sistema −6 + 2𝑦 = 2
5𝑥 − 3𝑦 = 15
{ 2𝑦 = 2 + 6
2𝑥 + 3𝑦 = 66
Observe que a equação 1 tem o termo -3y, e 𝑦=4
a equação 2 tem o termo +3y (oposto de -3y). A solução é o par (-2,4).
Esse fato nos permite obter uma só equação
sem a incógnita y, somando as duas
equações membro a membro.

46
PROFESSOR RESPONDE A2) (IFPI – 2104) Uma loja verificou que
quanto mais anunciava na televisão, mais ela
P1) O perímetro de um retângulo de vendia. Logo, a venda era dada em função
dimensões 4 + x e 4x+1é igual a 30 m. Então, dos anúncios feitos na televisão. Verificou-se
quanto mede a sua área? que essa função era definida pela fórmula 𝑦 =
3
(A) 36 2
𝑥 + 187 em que y representa a quantidade
(B) 42 de mercadoria vendida na semana e x
(C) 45 representa o número de comerciais de
(D) 48 televisão durante essa semana. Quantas
(E) 54 vezes o comercial da loja apareceu na
televisão durante a semana em que a loja
P2) A soma de três números inteiros vendeu 256 mercadorias?
consecutivos é igual a 150. Qual é o maior
destes três números? (A) 62
(B) 58
(A) 42 (C) 46
(B) 43 (D) 42
(C) 48 (E) 38
(D) 51
(E) 54
A3) A soma de três números inteiros
P3) Pipoca, em sua última partida, acertou x consecutivos é igual a 225. Qual é o maior
arremessos de 2 pontos e y arremessos de 3 destes três números?
pontos. Ele acertou 25 arremessos e marcou (A) 74
55 pontos. Quantos arremessos de 3 pontos (B) 75
ele acertou? (C) 76
(A) 1 (D) 77
(B) 2 (E) 78
(C) 3
(D) 4 A4) Para qual valor de x a igualdade é
(E) 5 verdadeira? 2
4+ =0
(A) 1 7
3−
(B) 2 1+𝑥
ALUNO RESPONDE (C) 3
A1) O quadrado mágico indicado a seguir é (D) 4
uma tabela quadrada formada por nove (E) 5
quadrados menores. Ela é composta por
números inteiros positivos de modo que a A5) (IFPI 2011) Qual é o número que somado
soma dos números de cada linha, de cada com o seu antecessor é igual à diferença entre
coluna e de cada uma das duas diagonais a quarta parte de seu triplo e 6 unidades?
maiores é sempre a mesma.
(A) 0
(B) 4
(C) -4
(D) 2/5
(E) -2/5

A6) (IFAM 2012) A raiz real da equação


3(𝑥 − 2) 𝑥 + 3
+ = 2 é um número:
2 4
(A) inteiro positivo.
Baseado nisso, o valor de X+Y+Z é: (B) inteiro negativo.
(A) 25 (C) menor que 2.
(B) 26
(D) maior que 3.
(C) 27
5
(D) 28 (E) racional próximo de 2
(E) 29

47
4(2𝑥+3) 4 Assinale a alternativa que apresenta quantos
A7) A expressão 7(2𝑥+3) é equivalente a 7 para
litros de água há nesse reservatório.
todo x real diferente de:
(A) 250 litros.
3
(A) − (B) 251 litros.
2 (C) 252 litros.
3 (D) 253 litros.
(B)
2 (E) 255 litros.
3
(C) − A12) Uma pilha de livros com 132 cm de altura
7
é formada por 27 livros, alguns com 4 cm e
3
(D) outros com 6cm de espessura. Quantos livros
7 de 4 cm foram colocados nessa pilha?
7
(E) − (A) 18
2
(B) 16
𝑥−2) 𝑥
A8) Para tornar a igualdade 2
=9−3 (C) 15
verdadeira, o valor de x deve ser: (D) 12
(E) 10
(A) primo e negativo.
(B) natural e múltiplo de 3. A13) A diferença entre o preço de uma
coxinha de frango e o preço de um pastel de
(C) ímpar e múltiplo de 5.
carne, é R$ 2,00. Pedro comeu uma coxinha
(D) par e negativo. e três pasteis, gastando ao todo R$ 14,00 com
(E) natural e múltiplo de 7. salgados. Qual é o preço do pastel de carne?
(A) R$ 2,50
A9) Antônio pretende usar 120 m de tela de (B) R$ 3,00
arame para fazer três cercados. Dois dos (C) R$ 3,50
cercados são quadrados de perímetros iguais, (D) R$ 3,80
e o lado de cada um desses dois cercados (E) R$ 4,00
mede 8,5 m. O terceiro cercado é retangular e
tem 16 m de comprimento. Com base nessas A14) Deseja-se dividir uma certa quantia, em
informações, qual a medida da largura do reais, entre 12 pessoas em partes iguais. Se
terceiro cercado? a divisão fosse realizada apenas entre 8
dessas pessoas, cada uma delas receberia
(A) 52 m R$ 900,00 a mais do que previra receber
(B) 32 m originalmente. Qual é o valor dessa quantia?
(C) 20 m
(D) 10 m (A) R$ 18.600,00
(E) 5 m (B) R$ 19.712,00
(C) R$ 19.840,00
A10) (IFAM 2012) O conjunto solução da
inequação (D) R$ 20.016,00
𝑥 𝑥+1 1−𝑥
3
− 2 ≤ 2 é: (E) R$ 21.600,00

(A) {𝑥 ∈ ℛ/𝑥 ≥ 0} A15) (IFPI-2014) Em Mandacaru, propriedade


dos pais de Isabela, há bodes e carneiros.
(B) {𝑥 ∈ ℛ/𝑥 ≤ 0}
Sabe-se que a quantidade de animais, dentre
(C) {𝑥 ∈ ℛ/𝑥 ≤ 2} bodes e carneiros, é 975. Assim, se a
quantidade de carneiros é o dobro da
(D) {𝑥 ∈ ℛ/𝑥 ≤ 3}
quantidade de bodes, podemos afirmar que a
(E) {𝑥 ∈ ℛ/𝑥 ≥ 3} quantidade de bodes é igual a:
(A) 310
A11) (IFSP 2016) A capacidade de um
(B) 315
reservatório de água é maior que 250 litros e
(C) 320
menor que 300 litros. O número x de litros que
(D) 325
há nesse e reservatório satisfaz à inequação
𝑥 (E) 330
+ 1 < 127.
2

48
A16) Em um escritório de advocacia, a) 550
trabalham apenas dois advogados e uma b) 570
secretária. Como Dr. Pedro e o Dr. Carlos c) 590
sempre advogam em causas diferentes, a d) 610
secretária Juliana coloca 01 grampo em cada e) 630
processo do Dr. Pedro e 02 grampos em cada
processo do Dr. Carlos, para diferenciá-los 3. (Ufjf-pism 3 2019) Em um edifício de
facilmente no arquivo. Sabendo-se que ao 20andares, há alguns andares com somente
todo são 78 processos nos quais foram dois apartamentos, e os demais andares
usados 110 grampos. Podemos concluir que possuem três apartamentos cada. No total
o número de processos do Dr. Carlos é: são 54 apartamentos. Nesse edifício, a
quantidade de andares que possuem três
apartamentos é
(A) 32
(B) 46 a) 8
(C) 40 b) 10
(D) 64 c) 12
(E) 28 d) 14
e) 27
A17) Num determinado campeonato, um time
ganha 3 pontos por vitória, 1 ponto por empate
e nenhum ponto em caso de derrota. Até hoje 4. (G1 - ifpe 2019) Wagner tenta economizar
cada time já disputou 20 jogos. Se um desses dinheiro, mas a verdade é que ele gasta
times perdeu 4 jogos e até agora tem 34 quase tudo que tem em lanches. Certa vez,
pontos, quantos jogos ele ganhou? ele comprou 2 hambúrgueres, 5 coxinhas e 3
sucos, tudo no mesmo dia, gastando R$ 29,40
Se cada hambúrguer custou R$ 4,50 e cada
(A) 6 suco custou R$ 2,80, qual era o preço de cada
(B) 7 coxinha comprada?
(C) 8
(D) 9 a) R$ 2,40
(E) 10 b) R$ 2,20
c) R$ 2,10
d) R$ 2,80
ATIVIDADE COMPLEMENTAR e) R$ 3,50
1. (G1 - cp2 2020) Duas irmãs viajaram juntas
nas férias de julho. Ao retornarem, elas
selecionaram 12 dezenas de fotos para 5. (G1 - cps 2019) A Mata Atlântica é uma
postar, durante alguns dias, em uma rede série de ecossistemas de florestas tropicais
social. Considere que a quantidade de fotos da América do Sul que abriga uma
postadas a cada dia correspondeu ao dobro diversidade de espécies endêmicas. Estudos
da quantidade do dia anterior, e que o tempo estimam que haja um total de 8.732 espécies
gasto para postar todas as fotos foi de 4 dias. entre plantas e vertebrados endêmicos nesse
Foram postadas, no último dia. bioma, e que a diferença entre a quantidade
daquelas plantas e a quantidade destes
a) 64fotos. vertebrados, nessa ordem, seja de 7.268
b) 32fotos. espécies. Nessas condições, a quantidade de
c) 16fotos. plantas endêmicas nesse bioma é
d) 8 fotos.
a) 723
2. (G1 - cotuca 2019) Ana coleciona figurinhas b) 1.464
para colar no álbum da Copa do Mundo. Se c) 5.813
ela ganhasse mais 24 não repetidas, ficariam d) 8.000
4 e) 16.000
faltando 31 do álbum para completar a
coleção. Sabendo que um álbum completo
tem 682 figurinhas, calcule quantas Ana
possui agora.

49
6. (Fatec 2019) Entre as tarefas de um 9. (G1 - ifpe 2019) Jeison Orlando Rodríguez
professor, está a elaboração de exercícios. Hernández, um venezuelano de 20 anos, foi
Professores de Matemática ainda hoje se reconhecido pela organização Guinness de
inspiram em Diofanto, matemático grego do recordes mundiais como a pessoa viva com o
século III, para criar desafios para seus maior pé do mundo. O pé direito dele mede
alunos. Um exemplo de problema diofantino é: 41,8 centímetros. O esquerdo tem 36,8
“Para o nascimento do primeiro filho, o pai centímetros. Rodríguez se deu conta de que o
esperou um sexto de sua vida; para o tamanho de seus pés "destoava" quando
nascimento do segundo, a espera foi de um ainda era muito jovem, ao compará-lo com os
terço de sua vida. Quando o pai morreu, a de seus amigos. Disponível em:
soma das idades do pai e dos dois filhos era https://www.bbc.com/portuguese/noticias/2015/09/150918_mai
or_pe_do_mundo_rm>. Acesso em: 05 maio 2019 (adaptado).
de 240 anos. Com quantos anos o pai
morreu?” Considerando que, quando o pai Se o sistema de numeração dos calçados no
morreu, ele tinha x anos, assinale a equação Brasil tem uma relação com o comprimento
matemática que permite resolver esse dos pés de acordo com a fórmula 𝑁 =
problema. 5𝑝+28
5𝑥 2𝑥 4
com N representando o número do
a) 𝑥 + 6 + 3 = 240
𝑥 𝑥
calçado e p representando o comprimento do
b) 𝑥 + 6 + 3 = 240 pé, em centímetros, qual é a numeração do pé
4𝑥 3𝑥 esquerdo de Jeison Orlando, no Brasil,
c) 𝑥 + 5
+ 4 = 240
𝑥 3𝑥 segundo o texto?
d) 𝑥 + + = 240 a) 59
6 2
6𝑥 3𝑥 b) 53
e) 𝑥 + + = 240
6 4 c) 52
7. (Ueg 2019) Para a inauguração da
d) 57
Sorveteria “Picolé Gelado”, foi feita a seguinte
e) 50
promoção: 10. (G1 - ifpe 2018) Um pai percebeu que a
PICOLÉ GELADO soma da sua idade com a idade de seu filho
PROMOÇÃO DE INAUGURAÇÃO totalizava 52 anos. Sabendo que a idade do
Dia: 12/12/18 pai é 12 vezes a idade do filho, assinale a
Moças R$ 5,00 e Rapazes R$ 7,00 alternativa que indica quantos anos o pai é
Válido até às 15 horas mais velho do que o filho.
a) 36 anos.
Após o encerramento da promoção, verificou- b) 40 anos.
se que 312 pessoas haviam comprado os c) 34 anos.
ingressos e a arrecadação total foi de R$ d) 44 anos.
1.880,00 O número de moças e de rapazes e) 24 anos.
que compraram os ingressos nesse dia foi,
respectivamente, igual a ATIVIDADE COMPLEMENTAR
a) 148 e 150
b) 152 e 200 1. (G1 - ifpe 2019) Adriano ganhou um pote
c) 160 e 182 de bombons. Ele quer separá-los em sacos
d) 152 e 160 com a mesma quantidade de bombons em
e) 160 e 148 cada um. Se Adriano colocar quatro bombons
em cada saco, ele usará uma certa
8. (G1 - ifpe 2019) Estudantes do IFPE quantidade de sacos e sobrará um bombom.
campus Olinda juntaram-se para comprar tinta Se Adriano colocar cinco bombons em cada
e pincéis. Compraram 8 potinhos de tinta, saco, ele usará quatro sacos a menos e
todos pelo mesmo valor, e 5 pincéis iguais, sobrarão três bombons. O pote que Adriano
gastando um total de R$ 37,00 Sabendo que ganhou tem, exatamente, a seguinte
o valor de cada potinho de tinta excede o valor quantidade de bombons
de cada pincel em R$ 1,70 é CORRETO a) 73.
afirmar que cada potinho custou b) 13.
a) R$ 3,50 c) 53.
b) R$ 3,40 d) 33.
c) R$ 5,10
e) 93.
d) R$ 4,80
e) R$ 4,20

50
2. (G1 - cftmg 2019) Uma coleção de doze 7. (G1 - cps 2018) No lanche da tarde, João
livros foi distribuída entre Augusto e Bárbara. comeu um pão com queijo, de massa total de
Se Augusto tivesse recebido três livros a mais 200g Curioso como sempre, determinou que,
do que recebeu dessa coleção, então a considerando só a produção dos dois
quantidade de livros recebida por ele seria ingredientes desse lanche (o pão e o queijo),
igual ao dobro da quantidade de livros o consumo de água foi de 830 litros. Sabendo
recebida por Bárbara. O número de livros que que, em média, a pegada hídrica do pão é de
Bárbara recebeu é igual a 1,6 L/g e a do queijo é de 5,0 L/g pode-se
a) 8. concluir corretamente que, em relação a esse
b) 7. consumo,
c) 5. a) a quantidade de pão é igual à quantidade
d) 4. de queijo.
b) a quantidade de pão é o dobro da
3. (G1 - ifal 2018) Em uma certa turma de quantidade de queijo.
49 alunos, o número de homens c) a quantidade de pão é o triplo da
3 quantidade de queijo.
corresponde a do número de mulheres. d) a quantidade de queijo é o dobro da
4
Quantos homens tem essa turma? quantidade de pão.
a) 14. e) a quantidade de queijo é o triplo da
b) 21. quantidade de pão.
c) 28.
d) 35. 8. (G1 - ifsc 2017) Um cliente foi ao caixa do
e) 42. banco do qual é correntista e sacou
R$ 580,00. Sabendo-se que a pessoa recebeu
4. (G1 - cftmg 2018) Uma senhora resolveu toda a quantia em 47 notas e que eram
vender bombons e trufas na porta de uma apenas notas de R$ 5,00 e de R$ 20,00, é
escola para complementar a renda familiar. CORRETO afirmar que a pessoa recebeu
No primeiro dia, ela faturou R$ 107,50 com a a) 27 notas de R$ 5,00 e 22 notas de R$
venda de 25 bombons e 15 trufas. No dia 20,00
seguinte, seu faturamento foi igual a b) 20 notas de R$ 5,00 e 27 notas de R$
R$ 185,00 e foram vendidos 20 bombons e 20,00
c) 23 notas de R$ 5,00 e 24 notas de R$
45 trufas. Um aluno que comprou, dessa
20,00
senhora, 4 bombons e 3 trufas, pagou a
d) 27 notas de R$ 5,00 e 20 notas de R$
quantia de
20,00
a) R$ 19,00.
e) 24 notas de R$ 5,00 e 23 notas de R$
b) R$ 19,50. 20,00
c) R$ 22,50.
d) R$ 23,00. 9. (G1 - ifal 2017) Resolva o sistema de
equações abaixo para x e y Reais e
5. (G1 - ifal 2018) Resolva o sistema de determine o valor do produto xy.
equações abaixo para x e y Reais e a) 74
determine o valor do produto xy. b) 80
a) 5. c) 91
b) 9. d) 94
e) 108
c) 25.
d) 45.
10. (G1 - ifpe 2017) Karina foi à feira e
e) 81. comprou 15 frutas (maçãs e abacaxis).
Karina pagou R$ 0,80 por cada maçã e
6. (G1 - ifal 2018) A soma de dois números
naturais é 13 e a diferença entre eles é 3. R$ 4,50 por cada abacaxi, totalizando
Qual o produto entre esses números? R$ 34,20. Karina comprou
a) 30. a) 6 maçãs.
b) 36. b) 9 abacaxis.
c) 39. c) 9 maçãs.
d) 40. d) 8 abacaxis.
e) 42. e) 8 maçãs.

51
Equação do 2º Graus (Segundo Grau) forma o conjunto solução ou o conjunto
verdade da equação.
São expressões que satisfazem a condição
ax² + bx + c = 0, com a, b e c números reais ● Fórmula de Bhaskara
e a ≠ 0 se enquadram na condição de
equações do 2º grau. −𝒃 ± √∆
𝒙=
Exemplo: 𝟐. 𝒂
onde: Δ = b – 4ac (Δ é delta )
2
2x² + 5x + 3 = 0 (essa é uma equação do
segundo grau, veja o grau 2 na primeira Exercício resolvido de uma equação do 2°
incógnita) grau
Chamamos a, b e c de coeficientes, a é A) Encontre a solução para a seguinte
sempre coeficiente de x², b é sempre equação:
coeficiente de x e c é sempre coeficiente do 2x2 + 8x – 24 = 0
termo independente.
Resposta: (Em três passos)
Exemplo:
● Primeiro passo: Escreva os valores
A) 3x² + 4x + 1 = 0 é uma equação do numéricos dos coeficientes a, b e c.
segundo grau, com a = 3, b = 4, c = 1.
B) x² – x – 1 = 0 é uma equação do segundo a = 2, b = 8 e c = – 24
grau, com a = 1, b = -1, c = -1. ● Segundo passo: Calcule o valor de
C) 9x² – 5x = 0 é uma equação do segundo delta.
grau, com a = 9, b = -5, c = 0.
D) 5x² -4 = 0 é uma equação do segundo Δ = b2 – 4ac
grau, com a = 5, b = 0, c = -4. Δ = 82 – 4·2·(– 24)
Δ = 64 + 192
Equação do 2° grau completa e
incompleta Δ = 256
● Terceiro passo: calcule os valores de x
Uma equação do 2° é chamada completa da equação.
quando os coeficientes b e c diferentes de −𝒃 ± √∆
zero. 𝒙=
𝟐. 𝒂
Exemplos: Observe que nessa expressão aparece o sinal
A) 2x² + 3x + 3 = 0 ±. Isso indica que x possui dois valores: o
B) x² + x + 1 = 0 primeiro para a √Δ (raiz de delta) negativa e o
segundo para √Δ positiva.
São equações completas. −8 ± √256
𝑥=
2 .2
Uma equação do 2° grau é chamada −8 ± 16
incompleta quando os coeficientes b ou c é 𝑥=
igual a zero, basta um deles ser igual a zero, 4
ou ambos serem iguais a zero. ● Para √Δ negativa, teremos:
x' = – 8 – 16 = –24 = –6
Exemplos:
4 4
A) x² – 3 = 0 (b = 0)
● Para √Δ positiva, teremos:
B) 2x² + x = 0 (c = 0)
C) 5x² = 0 (b = 0 e c = 0) x'' = – 8 + 16 = 8 = 2
4 4
São equações incompletas.
*OBS.: O valor de Δ pode ser utilizado como
Raízes de uma equação do 2° grau parâmetro para decidir como serão as raízes
da equação. Uma equação em que Δ > 0
Para resolvermos uma equação do 2° grau é possui duas raízes reais distintas, uma
necessário que encontremos as raízes da equação em que Δ = 0 possui duas raízes
equação. As raízes são valores que quando reais iguais ou uma raiz real dupla, isto é, x' =
substituímos nas incógnitas torna a sentença x'', e uma equação em que Δ < 0 não possui
verdadeira. Assim, as raízes da equação raízes reais.

52
RELAÇÕES DE GIRARD (A) 0
(B) 5
(OUTRA MANEIRA DE RESOLVER UMA
(C) 6
EQUÇÃO DO 2º GRAU)
(D) –5
Dada a equação ax² + bx + c = 0, temos: (E) –6
𝑏−𝑏
Soma das Raízes(S) = 𝑥1 + 𝑥2 = 𝑎
𝑐𝑐 P3) (G1 - ifal 2016) A equação
Produto das Raízes(P) =𝑥1 . 𝑥2 = 𝑎𝑎
tem como raízes os números
Exemplos: (A) -2 e -6
(B) -2 e 6
A) Quais são os valores da soma e do produto (C) 2 e -6
das raízes da equação 2x2 + 7x + 6 = 0? (D) 2e6
Solução: Chamemos de S a soma e P o (E) -4 e 4
produto, então temos S= −b/a = −7/2 e P
= c/a = 6/2 = 3.
B) Quais são os valores da soma e do produto P4) (G1 - ifal 2012) A soma dos quadrados de
das raízes da equação x2 + 4x – 9 = 0? dois números inteiros “a” e “b” (a < b) é igual
Solução: Sendo S a soma e P o produto, a 125. Aumentando-se 5 unidades no número
então temos S = −b/a = −4/1 = − 4 e P menor e diminuindo-se 5 unidades no número
= c/a = −9/1 = −9. maior, o valor da soma supracitada diminui em
100 unidades. Assinale a alternativa
*OBS.: Uma outra aplicação para as relações verdadeira.
de Girard, é obter uma equação do segundo
grau que possua raízes cuja soma e produto (A) “a” e “b” são números positivos.
sejam dois números S e P, previamente (B) a – b = 15.
escolhidos. Para isto basta tomarmos a
equação x2 – Sx + P = 0, pois teremos como (C) b – a = – 15.
soma das raízes −(−S)/1 = S e como produto (D) “a” e “b” são números pares.
das raízes P/1 = P.
(E) Existem dois valores para “a” e dois para
Exemplo: “b” que satisfazem essas condições.

Encontre uma equação do segundo grau que


possua −3 e 2 como suas raízes. ALUNO RESPONDE
Solução: A soma dessas raízes é S = −3 + 2 A1) (G1 - ifal 2012) Assinale a alternativa que
= −1e o produto é P = (−3) . 2 = − 6, então, a complete a frase: A equação do 2º grau 2x2 –
equação pode ser da forma x2 – Sx + P = 0, ou 5x = 3...
seja, uma equação do segundo grau que
(A) admite duas raízes inteiras.
possui como raízes os números − 3 e 2 é a
(B) admite uma raiz natural.
equação x2 + x – 6 = 0.
(C) não admite raízes reais.
PROFESSOR RESPONDE
(D) admite duas raízes naturais.
P1) (G1 - ifal 2017) Determine o valor de (E) admite duas raízes negativas.
na equação de modo que A2) (G1 - ifal 2011) Sejam w e z dois números
uma raiz seja o dobro da outra: reais tais que a soma é 21 e o produto é -7.
1 1
(A) 12 Calcule o valor da expressão + 𝑧2 .
𝑤2
(B) 18 445
(A) 59
(C) 24 445
(B)
(D) 28 49
455
(E) 32 (C) 59
P2) (G1 - ifal 2017) Determine o valor de 455
(D) 49
para que a equação tendo
435
como raízes os valores e (E) 49

53
A3) (G1 - ifba 2012) Considere a equação do (A) 545 M
2º grau, em x, dada por 5x2+bx+c=0. Se as (B) 225 M
raízes dessa equação são r1=-1 e r2=2/5, (C) 200 M
então o produto b . c é igual a: (D) 500 M
(E) 450 M
(A) 1
(B) 5
A9) (G1 - ifsc 2016) Considere que a equação
(C) - 5
(D) 6 do segundo grau 3𝑥 2 + 𝑎𝑥 + 𝑑 = 0 tem como
raízes os números 4 e -3.
(E) – 6
A4) (G1 - ifce 2016) A soma de dois números Assim sendo, é CORRETO afirmar que os
reais vale e o produto dos mesmos vale valores de (a+d) e (a.d) são,
Então esses dois números são, respectivamente,
respectivamente, (A) -1 e -12
1+√3 1−√7
(A) , (B) -39 e 108
2 2
1+√7 1−√3
(C) 33 e -108
(B) , (D) -3 e -36
2 2
1+√5 1−√5 (E) 1 e 12
(C) ,
2 2
(D) 2, 1 A10) (G1 - ifsc 2016) Considerando-se a
2
(E) 1, -1 equação 𝐸 = ( √𝑥 2 − 7𝑥 + 12 = 2√3), sendo
A5) (G1 - ifce 2014) Determinando-se, na ∪= ℝ é CORRETO afirmar que o seu
equação 2x2 – 6x + 12 + 0, a soma das conjunto solução será:
raízes, obtém-se
(A) 5 (A) S={7}
(B) 4 (B) S={0, -7}
(C) 3 (C) S={0}
(D) 2. (D) S={0, 7}
(E) 1 (E) S={2, 3}
A6) (G1 - ifce 2012) Se x1 e x2 são as raízes
da equação 3x2 – 5x + p – 2 = 0, onde p é A11) (G1-Ifsc 2011) Quanto à equação 𝑥 2 −
1 1
um número real, e sabendo-se que + = 4𝑥 + 3 = 0 é correto afirmar que:
𝑥1 𝑥2
5 (A) a soma de suas raízes é igual a – 4.
, pode-se concluir corretamente que
2 (B) tem duas raízes reais e iguais.
(A) p = –2. (C) tem duas raízes reais e distintas.
(B) p = –8/5. (D) não tem raízes reais.
(C) p = 0. (E) o produto de suas raízes é nulo.
(D) p = 2. A12) (G1 - ifsp 2014) A soma das soluções
(E) p = 4. inteiras da equação (𝑥 2 + 1). (𝑥 2 − 25). (𝑥 2 −
A7) (G1 - ifsc 2017) Dada a equação 5𝑥 + 6) = 0 é
quadrática 3X2+9X-120=0 determine suas
raízes. Assinale a alternativa que contém a (A) 1
resposta CORRETA. (B) 3
(C) 5
(A) 16 e 10 (D) 7
(B) -5 e 8 (E) 11
(C) -8 e 5
(D) -10 e 16
(E) -9 e 15 A13) (G1 - ifsp 2011) Considere a equação do
A8) (G1 - ifsc 2017) Pedro é pecuarista e, com 2º grau, em x, dada por 2x2 + bx + c = 0. Se
o aumento da criação, ele terá que fazer um as raízes dessa equação são r1 = 2 e r2 = –3,
novo cercado para acomodar seus animais. então a diferença b – c é igual a
Sabendo-se que ele terá que utilizar 5 voltas (A) 8
de arame farpado e que o cercado tem forma (B) 14
retangular cujas dimensões são as raízes da (C) 19
equação 𝑥 2 − 45𝑥 + 500 = 0 qual a (D) 23
quantidade mínima de arame que Pedro terá (E) 27
que comprar para fazer esse cercado?

54
A14) (G1 - ifsul 2017) As raízes das equações Determine o valor de para que uma das
5x-2=3x+6 e (y-1).(y+4)=y2+5 representam as raízes seja o dobro da outra.
medidas dos comprimentos dos catetos do
triângulo retângulo da figura, representada a (A) 25
seguir. (B) 30
(C) 32
(D) 35
(E) 38
A18) (G1 - ifsul 2016) Equações biquadradas
é uma equação escrita da seguinte forma
geral:

Assim, o comprimento da hipotenusa z desse Para encontrarmos as suas raízes, é preciso


triângulo retângulo é transformá-las em uma equação do segundo
(A) 4 grau, que pode ser resolvida pela fórmula de
(B) 5 Bhaskara Akaria (matemático que viveu na
(C) 6 Índia meados do século XII).
(D) 7 Portanto a soma das raízes da equação
A15) (ifsul -2017) As medidas do comprimento
e da altura (em metros) do outdoor retangular, é
representado na figura abaixo, são (A) 0
exatamente as soluções da equação x² - 10x (B) -10
+ 21= 0 (C) 2
(D) 9
(E) 11
A19) (G1 - ifsul 2015) Um móvel de R$ 360,00
deveria ser comprado por um grupo de
rapazes que contribuíram em partes iguais.
Como deles desistiram, os outros
precisaram aumentar a sua participação em
R$ 15,00 cada um.
Dessa forma, é correto afirmar que a área Qual era a quantidade inicial de rapazes?
desse outdoor é
(A) 8
(A) 10 m2 (B) 12
(B) 20 m2 (C) 15
(C) 21 m2 (D) 20
(D) 24 m2 (E) 22
(E) 3 e 18
A16) (G1 - Ifsul 2016) Os valores da soma e
do produto das raízes da função quadrática ATIVIDADE COMPLEMENTAR
𝑓(𝑥) = −𝑥 2 + 9𝑥 − 18, são, respectivamente, 1. (G1 - cps 2019) Suponha que um terreno
(A) -1 e 3 retangular de área 4.225 km2 será delimitado
(B) 3 e 6 para se tornar uma nova Reserva Extrativista.
Se o comprimento do terreno excede em
(C) -3 e -6 100 km sua largura (x), uma equação que
(D) 9 e 18 permite determinar essa largura (x) e
a) x2  100x  4.225  0
A17) (G1 - Ifsul 2016) Problemas que recaem
b) x2  100x  4.225  0
numa equação do segundo grau já apareciam
em textos escritos pelos babilônios, nas c) x2  100x  4.225  0
tábuas cuneiformes. Observe a equação d) x2  4.225x  100  0
abaixo: e) x2  4.225x  100  0

55
2. (Efomm 2019) Numa equação, 5. (G1 - ifal 2018) Sendo x1 e x 2 as raízes da
encontramos o valor de 884. Para chegar a equação x2  x  12  0, o resultado da soma
esse resultado, somamos os quadrados de
x1  x2 é
dois números pares, consecutivos e positivos.
Determine o quociente da divisão do maior a) 1.
pelo menor b) 3.
a) 0,87. c) 4.
b) 0,95. d) 7.
e) 12.
c) 1,03.
6. (G1 - utfpr 2018) Dada a equação do
d) 1,07.
segundo grau: 3x2  20x  12  0
e) 1,10.
3. (G1 - cp2 2019) Nas salas de aula do Assinale a alternativa que apresenta o
Colégio Pedro II serão colocados pisos conjunto solução da equação dada.
conforme a figura a seguir:  2
a) 6, .
 3
 1
b) 3, .
 3
 1
c) 6, .
 3
 1
d) 3, .
 2
Cada piso é formado por quatro retângulos  3
iguais de lados 10 cm e (x  10) cm,
e) 2,  .
 2
respectivamente, e um quadrado de lado igual 7. (Mackenzie 2018) O número inteiro
a x cm. Sabendo-se que a área de cada piso positivo, cujo produto de seu antecessor com
equivale a 900 cm2 , o valor de x, em seu sucessor é igual a 8, é
a) 5
centímetros, é
a) 10. b) 4
c) 3
b) 23.
d) 3
c) 24.
d) 50. e) 2
8. (G1 - cmrj 2018) A figura ilustra uma chapa
metálica retangular bem fina cuja superfície
4. (G1 - cp2 2019) Luíza estava brincando
com seu joguinho no celular, no qual uma vale 204 cm2 .
serpente deve comer os insetos que
aparecem na tela. No início do jogo, a
serpente é formada por um retângulo de
dimensões x mm por (5x  12) mm e, a cada
inseto que come, ela aumenta o seu tamanho
em um quadrilátero de área 10 mm2 . Após Devido à dilatação térmica, a maior das
comer 8 insetos, a serpente, totalmente dimensões (comprimento) foi aumentada de
esticada, representa um retângulo de área 3 cm e a largura, de 2 cm, fazendo com que
112 mm2 . As dimensões da serpente, em essa superfície seja aumentada de 76 cm2 .
milímetros, no início do jogo são, “Observe que a área de um retângulo
respectivamente, iguais a corresponde ao produto do comprimento pela
a) 1,6 e 20,0. largura.”
b) 2,0 e 22,0. Nessas condições, o comprimento pode ter
c) 3,6 e 30,0. dois valores, ambos contidos no intervalo
a) [11,0; 12,5].
d) 4,0 e 32,0.
b) [13,5; 15,5].
c) [14,5; 16,5].
d) [16,5; 18,5].
e) [17,5; 19,5].

56
9. (G1 - ifsc 2017) Dada a equação quadrática Podemos expressar a relação de função entre
3x2  9x  120  0, determine suas raízes. duas grandezas através de uma expressão
Assinale a alternativa que contém a resposta que chamamos de lei de formação da função.
CORRETA. Nos exemplos anteriores, teremos as
a) 16 e 10 seguintes leis de formação:
b) 5 e 8
a) y = 0,30 . x + 0,50
c) 8 e 5
d) 10 e 16 b) d = 20 . t
e) 9 e 15 c) p = 0,5 . x
10. (Acafe 2017) Uma biblioteca possui 300 Nesses casos, as variáveis y, d e p são
livros, todos do mesmo tamanho. Um chamadas de variáveis dependentes, pois
funcionário pretende dividi-los igualmente dependem do valor da outra grandeza. E as
entre as prateleiras da loja. Sabendo que, se variáveis x e t são chamadas variáveis
os livros forem igualmente divididos entre 3 independentes, pois podemos estabelecer
prateleiras a menos, cada prateleira receberá valores para essas variáveis.
5 livros a mais do que o previsto inicialmente. OBSERVAÇÃO: Podemos usar a notação f(x)
Assim, o número de prateleiras para colocar para representar a lei de uma função que
todos os livros é: depende do valor de x.
a) Múltiplo de 4.
Exemplo:
b) Múltiplo de 3.
c) Entre 10 e 12. f(x) = 0,30.X + 0,50
d) Maior que 20.

FUNÇÃO AFIM
É toda função que a lei de formação pode ser
escrita na forma f(x) = ax + b, em que a e b
FUNÇÃO são números reais e x pode ser qualquer
IDEIA DE FUNÇÃO número real. Exemplos:
Quando existe a relação entre duas a) f(x) = 3x + 4, em que a = 3 e b = 4
grandezas de tal forma que para cada medida b) f(x) = -3x, em que a = -3 e b = 0
da primeira corresponde uma única medida da
segunda, chamamos essa relação de função. Quando a função for da forma f(x) = ax,
E dizemos que a segunda grandeza é função dizemos que é uma função linear.
da primeira. Exemplos: c) f(x) = -4, em que a = 0 e b = -4
a) O valor total a pagar pela compra de uma Quando a função for da forma f(x) = b,
certa quantidade de um mesmo tipo de pão dizemos que é uma função constante.
que custa R$ 0,30 a unidade. E que tem um
acréscimo de R$ 0,50 pelo uso de uma ● GRÁFICO DA FUNÇÃO AFIM
embalagem para viagem. Nesse caso, o valor O gráfico de uma função afim é sempre uma
a pagar é função do número de pães reta não perpendicular ao eixo x.
comprados.
Sabendo que x é qualquer número real,
b) A distância percorrida em uma maratona, vamos construir o gráfico das seguintes
por um atleta durante um certo tempo funções:
(Considerando que ele manteve o ritmo
constante de 20 km/h). Nesse caso, a a) f(x) = 3x + 2
distância percorrida é função do tempo de 1º passo: atribuir valores para x e calcular os
prova desse atleta. valores correspondentes para y.
c) A nota de um aluno que responde uma Para x = -2, temos f(-2) = 3.(-2) + 2 = -6 + 2 =
prova objetiva, onde cada questão vale meio -4,
ponto. Nesse caso, a nota da prova é função
do número de questões certas desse aluno. para x = 0, temos f(0) = 3.0 + 2 = 0 + 2 = 2 e
● LEI DE FORMAÇÃO DE UMA FUNÇÃO para x = 1, temos f(1) = 3.1 + 2 = 3 + 2 = 5

57
2º passo: representar no plano cartesiano os Exemplos:
pares ordenados encontrados e unir os pontos
a) f(x) = x – 1 tem como zero da função x = 1.
para formar a reta que representa o gráfico da
função.

b) y = 3x + 9 tem como zero da função x = -3.

b) f(x) = - 2
1º passo: atribuir valores para x e calcular os
valores correspondentes para y. ● VARIAÇÃO DE UMA FUNÇÃO AFIM
Uma função afim é crescente quando o
coeficiente a é maior que zero. E é
Para x = -3, temos que f(-3) = -2,
decrescente quando o coeficiente a é menor
para x = 0, temos que f(0) = -2 e que zero. E quando a for igual a zero temos
uma função constante.
para x = 2, temos que f(2) = -2
Exemplos:

2º passo: representar no plano cartesiano os a) y = 3x + 7 é crescente pois a = 3 > 0.


pares ordenados encontrados e unir os pontos
para formar a reta que representa o gráfico da
função.

Observe que a medida que o valor de x


aumenta, temos que o valor do y também
aumenta.
b) y = -x + 12 é decrescente pois a = -1 < 0.

ZERO DE UMA FUNÇÃO AFIM


Zero da função é o valor de x para que f(x) =
0. É o ponto onde o gráfico da função
intercepta o eixo x.

Observe que a medida que o valor do x


aumenta, temos que o valor do y diminui.

58
ESTUDO DO SINAL DA FUNÇÃO AFIM
Em uma função afim, podemos determinar
para que valor de x a função é nula, para quais
valores de x a função é negativa e para que
valores de x a função tem valores positivos.
Vamos estudar o sinal das seguintes funções:
a) y = 2x + 1 Observando o gráfico, vemos que para:
1º passo: determinamos o zero da função e ✔ x = 2, temos y = 0, ou seja a função é
se a função é crescente ou decrescente. nula.
y = 2x + 1 fazendo y = 0, temos ✔ x < 2, temos y > 0, ou seja a função é
y = 2.0 + 1 = 0 + 1 = 1 positiva.
O zero da função é 1. ✔ x > 2, temos y < 0, ou seja a função é
negativa.
Como a = 2 > 0, então a função é crescente.

PROFESSOR RESPONDE
2º passo: construímos um esboço do gráfico
e fazemos o estudo do sinal da função. P1) (IFPI-2014) Uma loja verificou que quanto
mais anunciava na televisão, mais ela vendia.
Logo, a venda era dada em função dos
anúncios feitos na televisão. Verificou-se que
essa função era definida pela fórmula y = 3x/2
+ 187 em que y representa a quantidade de
mercadoria vendida na semana e x representa
o número de comerciais de televisão durante
essa semana. Quantas vezes o comercial da
loja apareceu na televisão durante a semana
em que a loja vendeu 256 mercadorias?
(A) 62
Observando o gráfico, vemos que para: (B) 58
✔ x = ½, temos y = 0, ou seja a função é nula. (C) 46
(D) 42
✔ x > ½, temos y > 0, ou seja a função é (E) 38
positiva.
✔ x < ½, temos y < 0, ou seja a função é P2) (IFPI-2014) Na figura abaixo, está
negativa. representado o gráfico de uma função
polinomial do 1º grau
f: R → R
b) Y = -3x + 6
1º passo: determinamos o zero da função e
se a função é crescente ou decrescente.
y = -3x + 2 fazendo y = 0, temos
y = -3.0 + 2 = 0 + 2 = 2
O zero da função é 2. A expressão algébrica que define a função f é:
Como a = -3 < 0, então a função é (A) y=x+1
decrescente. (B) y = x/2 + 1
(C) y=x+2
(D) y = –x/2 + 1
2º passo: construímos um esboço do gráfico
(E) y = –x + 1
e fazemos o estudo do sinal da função.

59
P3) (IFPI-2016) O departamento de promoção ALUNO RESPONDE
de uma loja verificou que a venda de suas
A1) (IFPI-2016) A figura a seguir mostra o
mercadorias é dada em função dos anúncios
gráfico da função f definida por f(x) = ax + b no
feitos na televisão. Em estudos constatou-se
conjunto dos números reais.
que a função é definida pela fórmula y =
3x/2 + 180, em que y representa a quantidade
de mercadorias vendidas no mês e x
representa o número de comerciais de
televisão durante o mês. Com base nessa
informação, quantas vezes o comercial da loja
apareceu na televisão durante o mês em que
a loja vendeu 315 mercadorias?
(A) 90 vezes
(B) 85 vezes
(C) 60 vezes
(D) 45 vezes Com base nos dados do gráfico, qual a lei que
define a função f ?
(E) 30 vezes
(A) y = 2x + 1/2
(B) y = 2x/3 + 1
P4) (IFMA-2013) Ane alugou um carro em
uma locadora que cobra R$ 22,10 por dia pelo (C) y = -x + 1
aluguel do mesmo, além disso, cobra (D) y = -x/2 - 2
também, uma taxa fixa de R$ 30,00. Quanto
Ana gastou, tendo alugado o carro por 21 (E) y = 3x/2 + 1
dias?
(A) R$ 494,00 A2) (IFPI-2015) Antônio é vendedor em uma
(B) R$ 494,10 loja e seu salário mensal é calculado da
seguinte maneira: uma quantia fixa de R$
(C) R$ 424,00 788,00 mais 5% do valor das vendas que ele
(D) R$ 400,00 efetuar no mês. A função que permite calcular
o salário S em função do valor das vendas v
(E) R$ 250,00 efetuadas por Antônio é:
(A) S = 788 + 5v
P5) (G1 - ifal 2017) A quantidade de (B) S = 5 + 788v
pessoas que assistem a um espetáculo teatral (C) S = 7,88 + 5v
(D) S = 5 + 7,88v
varia de acordo com o preço em reais, (E) S = 788 + 0,05v
cobrado na entrada, conforme a expressão
Nessas condições, qual preço deve- A3) (IFPI-2015) Margarida pretende vender
se cobrar no espetáculo para que a renda seja bombons para ajudar nas despesas de casa e
máxima? para isso comprou bombons que serão
(A) 30 revendidos. Pelos bombons que comprou,
pagou R$ 60,00 e pretende vender cada
(B) 40 bombom por R$ 2,40. Sabe-se que a função
(C) 50 que expressa o lucro L obtido por Margarida
com a venda da quantidade q de bombons é
(D) 60 L = 2,4q - 60. Com base nessas informações,
(E) 70 quantos bombons Margarida precisa vender,
no mínimo, para ter lucro (positivo)?
(A) 30 bombons
(B) 26 bombons
(C) 25 bombons
(D) 24 bombons
(E) 12 bombons

60
A4) (IFMA-2017) Para uma função real, que
satisfaz as condições:
I. f(a + b) = f(a) + f(b)
II. f(1) = 5
O valor de f (4) é igual a:
(A) 25
(B) 20
(C) 18
(D) 16
(E) 10

A5) (IFMA-2012) Um grupo de estudantes do


curso de Física do IFMA Campus-Santa Inês
decidiu viajar de ônibus para participar de um
Seminário de Física em Imperatriz. Ao
fazerem uma pesquisa de preço, os
estudantes receberam de uma empresa a
seguinte proposta, na qual o preço de cada
passagem dependia do total de alunos: cada
aluno pagará R$ 90,00 acrescido de R$ 5,00
por poltrona que ficar vaga no ônibus.
Sabendo que o ônibus tem 52 poltronas, é
correto afirmar:
(A) Se o total de alunos for x, o valor em reais
de cada passagem será calculado pela
expressão 90 + 5.(52 – x ).
(B) Se viajarem 30 alunos, cada um deles
pagará R$ 110,00.
(C) Se viajarem 50 alunos, a empresa deverá
receber um total de R$ 6.500,00 referente ao
total de pagamentos das passagens. A7) (IFMA-2012) O gráfico mostra o salário
mensal dos vendedores de plano de saúde
(D) Se viajarem x alunos, o valor total (em
em função da quantidade de contratos
reais) que a empresa deverá receber,
realizados. A função que relaciona o salário
referente ao pagamento das passagens, é
em reais (y) a contratos realizados (x) é dada
calculado pela expressão 300 – 5x2.
por:
(E) O valor total máximo que a empresa
poderá receber pelo pagamento das
passagens ocorrerá quando o total de alunos
for igual a 20.

A6) (IFMA-2017) A companhia de telefonia


móvel “Ligue Mais” dispõe de um plano que
consiste em um valor fixo de R$ 25,80 por até
40 minutos de ligações para qualquer
operadora. A partir daí, cobra mais R$1,20 por
minuto excedente. Com base nessas (A) y = 400 + 200x/3
informações, qual a representação gráfica (B) y = 400x – 200/3
mais apropriada para o valor cobrado nesse (C) y = 600 + 80x
plano em função de minutos de ligação para (D) y = 600x – 70
qualquer operadora? (E) y= 400x + 600

61
A8) (IFMA-2016) Seja a função f, definida de (A) Q(t) = 200 000 + 2000t
R em R, dada por f(x) = kx + t, em que k e t (B) Q(t) = 200 000 + 6000t
são constantes reais não nulas e os pontos (- (C) Q(t) = 200 000 - 2000t
1, 3) e (0, - 1) pertencem ao gráfico de f. (D) Q(t) = 200 000 - 6000t
Sejam as afirmativas: (E) Q(t) = - 200 000 + 2000t

I. f é crescente para todo x real. A11) (IFPI-2013) Seja a função f: R → R dada


por f(x) = (2m – 10)x. Marque a alternativa que
II. 3/4 é zero da função.
determina m para que f seja decrescente em
III. O ponto (-10; 39) pertence ao gráfico de f. R.
IV. f(x) < 0, se x < 1/4. (A) m>2
V. f(x) ≤ 0, se x ≥ -1/4. (B) m<5
(C) m>3
(D) m<6
Das afirmativas acima, as verdadeiras são: (E) m > 10
(A) II e IV
(B) I e II A12) (IFGO-2014) O gráfico abaixo foi
(C) II e V elaborado por um economista e representa a
(D) III e V renda obtida por um vendedor de determinado
(E) IV e V produto, em função da quantidade de
A9) (IFPI-2013) A empresa “Estrada e unidades vendidas na ultima semana de 2013.
Companhia”, para construir uma estrada, Analisando o gráfico o filho do vendedor
cobra uma taxa fixa e uma taxa que varia de concluiu que:
acordo com o número de quilômetros de
estrada construída. O gráfico abaixo descreve
o custo da obra, em milhões de reais, em
função do número de quilômetros construídos.

I) O custo dos produtos adquiridos na semana


foi R$ 500,00.
II) Se vender 15 unidades do produto, o
vendedor terá um pequeno lucro.
III) Para obter um lucro de R$ 250,00, o
Marque a alternativa que indica a expressão vendedor deve vender 30 unidades do
algébrica da função f, para x ≥ 0, que produto.
representa esse gráfico. IV) Para recuperar o investimento o vendedor
deve vender exatamente 20 unidades do
(A) y = x/5 + 5 produto.
(B) y = x + 5 V) Se vender entre 20 e 30 unidades o
(C) y = x/10 + 5 vendedor terá prejuízo.
(D) y = x + 10 Sobre as considerações acima, é correto
(E) y = x + 7 afirmar que
A10) (IFPI-2013) Três bombas d'água, cada
uma com vazão de 2000 litros por hora,
retiram água de um reservatório que contém (A) todas estão corretas.
200 mil litros. Marque a alternativa que indica (B) apenas as alternativas I e III estão
a lei da função que representa a quantidade Q corretas.
(em litros) de água no reservatório em função (C) alternativas I, III e IV estão corretas.
do tempo t(em horas) em que as três bombas (D) apenas a alternativa III esta correta.
permanecem ligadas. (E) todas as alternativas são incorretas.

62
A13) (G1- ifsul) Numa serigrafia, o preço y de
cada camiseta relaciona-se com a quantidade
x de camisetas encomendadas, através da
fórmula y = -0,4x + 60 Se foram
encomendadas 50 camisetas, qual é o
custo de cada camiseta?
(A) R$ 40,00 Em que temperatura, aproximadamente,
(B) R$ 50,00 ocorre a solidificação da água na escala E?
(C) R$ 70,00 (A) – 16° E
(D) R$ 80,00 (B) – 32° E
(E) R$ 90,00 (C) – 38° E
(D) – 51° E
A14) (G1-ifsp) O gráfico abaixo apresenta (E) – 58° E
informações sobre a relação entre a
quantidade comprada x e o valor total pago y A17) (Ucs) O custo total C em reais, de
para um determinado produto que é produção de x kg de certo produto é dado pela
comercializado para revendedores. expressão C(x) = 900x + 50. O gráfico abaixo
é o da receita R, em reais, obtida pelo
fabricante, com a venda de x kg desse
produto.

Um comerciante que pretende comprar 2.350


unidades desse produto para revender
pagará, nessa compra, o valor total de:
(A) R$ 4.700,00
(B) R$ 2.700,00
(C) R$ 3.175,00 Qual porcentagem da receita obtida com a
(D) R$ 8.000,00 venda de 1 kg do produto é lucro?
(E) R$ 1.175,00
(A) 5%
A15) (Eear) Na função f(x) = mx – 2(m – n), (B) 10%
com m e n números reais. Sabendo que f(3) (C) 12,5%
= 4 e f(2) = -2, os valores de m e n são,
respectivamente (D) 25%

(A) 1e–1 (E) 50%


(B) –2e3
(C) 6e–1
(D) 6e3 A18) (G1 - ifal) Os pontos de um plano
(E) –1e6 cartesiano de coordenadas (2,2) e (4, - 2)
pertencem ao gráfico de uma função f R →
R, definida por f (x) = ax + b. Qual o valor de
A16) (Upe-ssa 2) Everton criou uma escala E a + b?
de temperatura, com base na temperatura
máxima e mínima de sua cidade durante (A) 0
determinado período. A correspondência (B) 2
entre a escala E e a escala Celsius (C) é a
seguinte: (C) 4
(D) 6
(E) 8

63
A19) (Upe-ssa 1) Na fabricação de 25 mesas, 2. (G1 - ifpe 2019) A equivalência entre as
um empresário verificou que o custo total de escalas de temperatura geralmente é obtida
material foi obtido por meio de uma taxa fixa por meio de uma função polinomial do 1º grau,
de R$ 2.000,00, adicionada ao custo de ou seja, uma função da forma y  a  x  b. Um
produção que é de R$ 60,00 por unidade. grupo de estudantes do curso de Química do
Qual é o custo para fabricação dessas IFPE desenvolveu uma nova unidade de
mesas? medida para temperaturas: o grau Otavius. A
(A) R$ 1.500,00 correspondência entre a escala Otavius (O) e
a escala Celsius (C) é a seguinte:
(B) R$ 2.900,00
(C) R$ 3.500,00 O C
(D) R$ 4.200,00
6 18
(E) R$ 4.550,00
60 36
A20) (Pucpr) Seja a uma função afim f(x), cuja
forma é f(x) = ax + b, com a e b números reais.
Se f(- 3) = 3 e f(3) = -1, os valores de a e b Sabendo que a temperatura de ebulição da
são respectivamente: água ao nível do mar (pressão atmosférica
igual a 1atm) é 100 C, então, na unidade
Otavius, a água ferverá a
(A) 2e9 a) 112.
(B) 1e–4 b) 192.
c) 252.
(C) 1/3 e 3/5 d) 72.
(D) 2e–7 e) 273.
(E) – 2/3 e 1 3. (Enem 2019) Uma empresa tem diversos
funcionários. Um deles é o gerente, que
recebe R$ 1.000,00 por semana. Os outros
ATIVIDADE COMPLEMENTAR
funcionários são diaristas. Cada um trabalha
1. (Enem PPL 2019) Em um município foi 2 dias por semana, recebendo R$ 80,00 por
realizado um levantamento relativo ao número dia trabalhado. Chamando de X a quantidade
de médicos, obtendo-se os dados: total de funcionários da empresa, a quantia Y,
em reais, que esta empresa gasta
Ano Médicos
semanalmente para pagar seus funcionários é
1980 137 expressa por
a) Y  80X  920.
1985 162 b) Y  80X  1.000.
1995 c) Y  80X  1.080.
212
d) Y  160X  840.
2010 287 e) Y  160X  1.000.

4. (G1 - ifce 2019) Rafael chamou um Uber


Tendo em vista a crescente demanda por para ir ao cinema com sua namorada, mas a
atendimento médico na rede de saúde atendente informou que o valor final a ser
pública, pretende-se promover a expansão, a pago é compreendido por uma parcela fixa de
longo prazo, do número de médicos desse R$ 3,00, mais R$ 1,50 cobrado por quilômetro
município, seguindo o comportamento de
crescimento linear no período observado no rodado. Sabendo que Rafael pagou R$ 48,00,
quadro. Qual a previsão do número de a distância da casa de Rafael para o cinema,
médicos nesse município para o ano 2040? em km, é
a) 387 a) 40.
b) 424 b) 50.
c) 437 c) 30.
d) 574 d) 60.
e) 711 e) 70.

64
5. (Eear 2019) A função que corresponde ao Pedro alugou na Locadora Bháskara, que
gráfico a seguir é f(x)  ax  b, em que o valor cobra R$ 50,00 a diária, mais R$ 0,90 por
de a é quilômetro percorrido. Ao final do dia, João e
Pedro pagaram o mesmo valor total pela
locação.
Quantos quilômetros cada um percorreu e
quanto pagaram?
a) 150 km e R$ 185,00
b) 160 km e R$ 192,00
c) 170 km e R$ 199,00
a) 3
d) 180 km e R$ 206,00
b) 2
c) 2 e) 190 km e R$ 213,00
d) 1
9. (Enem PPL 2017) Em um mês, uma loja
6. (Ueg 2018) No centro de uma cidade, há de eletrônicos começa a obter lucro já na
três estacionamentos que cobram da seguinte primeira semana. O gráfico representa o lucro
maneira: (L) dessa loja desde o início do mês até o dia
20. Mas esse comportamento se estende até
Estacionam Estaciona Estacionament o último dia, o dia 30.
ento A mento B oC
R$ 5,00 pela R$ 6,00 pela
primeira hora primeira hora
R$ 4,00
R$ 3,00 por R$ 2,00 por
por hora
cada hora cada hora
subsequente subsequente

Será mais vantajoso, financeiramente, parar


a) no estacionamento A, desde que o A representação algébrica do lucro (L) em
automóvel fique estacionado por quatro
horas. função do tempo (t) é
b) no estacionamento B, desde que o a) L(t)  20t  3.000
automóvel fique estacionado por três horas. b) L(t)  20t  4.000
c) em qualquer um, desde que o automóvel c) L(t)  200t
fique estacionado por uma hora. d) L(t)  200t  1.000
d) em qualquer um, desde que o automóvel
fique estacionado por duas horas. e) L(t)  200t  3.000
e) no estacionamento C, desde que o
automóvel fique estacionado por uma hora. 10. (G1 - ifsul 2017) Uma função do 1º grau
f:  possui o gráfico abaixo.
7. (G1 - ifsul 2017) Numa serigrafia, o preço
y de cada camiseta relaciona-se com a
quantidade x de camisetas encomendadas,
através da fórmula y  0,4x  60. Se foram
encomendadas 50 camisetas, qual é o custo
de cada camiseta?
a) R$ 40,00
b) R$ 50,00
c) R$ 70,00 A lei da função f é
x 3
d) R$ 80,00 a) f(x)  
2 2
b) f(x)  x  1
8. (Unisinos 2017) João e Pedro alugaram o
mesmo modelo de carro, por um dia, em duas 1
c) f(x)  2x 
locadoras distintas. João alugou o carro na 2
locadora Arquimedes, que cobra R$ 80,00 a x 1
d) f(x)  
diária, mais R$ 0,70 por quilômetro percorrido. 2 2

65
11. (G1 - epcar (Cpcar) 2017) João, ao a) R$ 1.500,00
perceber que seu carro apresentara um b) R$ 2.900,00
defeito, optou por alugar um veículo para c) R$ 3.500,00
cumprir seus compromissos de trabalho. A
locadora, então, lhe apresentou duas d) R$ 4.200,00
propostas: e) R$ 4.550,00
14. (G1 - ifsc 2016) De acordo com Agilar e
- plano A, no qual é cobrado um valor fixo de Fioreze (2011), o modelo que melhor
R$ 50,00 e mais R$ 1,60 por quilômetro representa a concentração de álcool para
rodado. indivíduos do sexo masculino que ingerem
- plano B, no qual é cobrado um valor fixo de uma lata de cerveja por hora, durante 5 horas,
R$ 64,00 mais R$ 1,20 por quilômetro é:
rodado. C(t)  0,022  0,007  (t  1), para 1  t  5
C(t )  0,050  0,016  (t  5), para 5  t  8,125)
João observou que, para certo deslocamento t  tempo decorrido após a ingestão da
que totalizava k quilômetros, era indiferente primeira lata de cerveja.
optar pelo plano A ou pelo plano B, pois o
valor final a ser pago seria o mesmo. É correto Suponha que um indivíduo tenha chegado à
afirmar que k é um número racional entre Oktoberfest às 20 horas, permanecido na
a) 14,5 e 20 festa por 5 horas e que tenha bebido uma
b) 20 e 25,5 cerveja por hora.
c) 25,5 e 31 Sabendo-se que a Lei Seca não permite que
d) 31 e 36,5 o indivíduo apresente um valor positivo de
concentração de álcool ao dirigir, é
12. (G1 - ifsc 2017) Durante a colheita em um CORRETO afirmar que esse motorista poderá
pomar de uvas, o proprietário verificou que às começar a dirigir novamente
9 horas haviam sido colhidos 730 kg de uva. a) antes das 4h do dia seguinte.
Considerando que a quantidade de uvas b) somente depois das 8h15min e 30s do dia
colhidas é linear durante o dia e que às 14 seguinte.
c) às 4h12min e 5s do dia seguinte.
horas haviam sido colhidos 3.650 kg de uva,
d) somente depois das 6h do dia seguinte.
analise as afirmativas: e) às 4h7min e 30s do dia seguinte.
I. A equação que permite calcular o número de 15. (G1 - ifsp 2016) O gráfico abaixo
quilogramas (y) em função do tempo (x) é apresenta informações sobre a relação entre
dada pela expressão y  584x  4.526. a quantidade comprada (x) e o valor total
II. Às 18 horas haviam sido colhidos 5.986 kg. pago (y) para um determinado produto que é
III. A colheita teve início às 8 horas. comercializado para revendedores.

Assinale a alternativa CORRETA.


a) Apenas as afirmativas I e II são
verdadeiras.
b) Todas as afirmativas são verdadeiras.
c) Apenas as afirmativas I e IIII são
verdadeiras.
d) Apenas as afirmativas II e III são
verdadeiras.
e) Todas as afirmativas são falsas.
Um comerciante que pretende comprar 2.350
13. (Upe-ssa 1 2016) Na fabricação de 25 unidades desse produto para revender
mesas, um empresário verificou que o custo pagará, nessa compra, o valor total de:
total de material foi obtido por meio de uma a) R$ 4.700,00.
taxa fixa de R$ 2.000,00, adicionada ao custo b) R$ 2.700,00.
de produção que é de R$ 60,00 por unidade. c) R$ 3.175,00.
Qual é o custo para fabricação dessas d) R$ 8.000,00.
mesas?
e) R$ 1.175,00.

66
16. (G1 - cftmg 2015) Um estudante de Indicando esse volume por y temos:y = 2x² +
engenharia faz trabalhos de digitação para 6x + 4
complementar seu ganho mensal. Ele
As funções definidas pelas leis y = x² - 3x + 6
estabelece que a relação entre o preço P e a
e
quantidade q de páginas de cada trabalho é
dada pela função P(q)  aq  b, sendo a e b y = 2x² + 6x + 4 são exemplos de Funções
Quadráticas ou Funções do 2º grau.
números reais positivos, e q pertencente ao
intervalo 1  q  100. Sabendo-se que o DEFINIÇÃO: Uma função polinomial do 2º
conjunto imagem dessa função é o intervalo grau é toda função do tipo y = ax² + bx + c,
6  P(q)  105, o estudante calcula os valores com a,b e c números reais e a ≠ 0, e é definida
de a e b. Desse modo, a média aritmética para todo x real.
entre a e b é igual a Exemplos:
a) 1,5
b) 2,0 y = x² - 5x + 4 ( a = 1, b = -5, c =
4)
c) 2,5
d) 3,0 y = 2x² + 5x – 2 ( a = 2, b = 5, c =
-2)

FUNÇÃO QUADRÁTICA (FUNÇÃO y = x² - 2x – 3 ( a = 1, b = -2, c =


POLINOMIAL DO 2º GRAU) -3)

Exemplo 1: Considere a figura abaixo. Vamos y = - x² + 4x – 3 ( a = -1, b = 4, c =


calcular a área (y) da parte pintada em função -3)
de x. y = -x² + 9 ( a = -1, b = 0, c =
9)

X 3 y = x² + 2 ( a = 1, b = 0, c =
2)
y = 2x² + 6x ( a = 2, b = 6, c =
X–2 0)
X REPRESESENTAÇÃO GRÁFICA DE UMA
Calculamos da seguinte forma: FUNÇÃO DO 2º GRAU

● A área do quadrado é: x² O gráfico de uma função polinomial do 2°


grau é representado por uma curva chamada
● A área do retângulo é: 3 . (x – 2) PARÁBOLA. Para construir o gráfico de uma
Então, a área representada pela parte pintada função 2º grau atribuímos valores a x e
é: calculamos os valores de y, determinando os
pares ordenados correspondentes. Em
(Área do quadrado) – (Área do retângulo) = seguida localizamos no plano cartesiano
x² - 3.(x - 2) = x² - 3x + 6 todos os pontos determinados por esses
pares ordenados, ligamos os pontos e
Indicando essa área pintada por y, temos: construímos o gráfico da função.
y = x² - 3x + 6
Exemplo 1: Construir o gráfico a função f
Exemplo 2: Escreva uma expressão que dada pela lei y = x² - 2x – 3 onde x representa
permite calcular o volume (y) do bloco um real qualquer.
retangular em função de x.
x y = x² - 2x – 3 (x , y)
x+2
-2 y = (-2)² - 2.(2) – 3 = 5 (-2,5)
-1 y = (-1)² - 2(-1) – 3 = 0 (-1,0)
x+ 1 0 y = (0)² - 2 .(0) – 3 = -3 (0,-3)
2 1 y = 1² - 2 . (1) – 3 = -4 (1,-4)
2 y= (2)² - 2.(2) – 3 = - 3 (2,-3)
Volume = 2 . (x + 1) . (x + 2) = 2 . ( x² + 3x +
3 y = (3)² - 2.(3) – 3 = 0 (3,0)
2) =
4 y = (4)² - 2. (4) – 3 = 5 (4,5)
2x² + 6x + 4

67
Gráfico da função y = x² - 2x – 3. ● A parábola tem concavidade voltada para
baixo (a < 0 )
● O vértice da parábola é ponto de
coordenadas (0 , 4 ).
● A função y = -x² + 4 possui dois zeros :
-2 e 2. Os pontos de intersecção da parábola
com o eixo x são (-2,0) e (2,0).
● O ponto de intersecção da parábola com o
eixo y é o ponto (0,4)
OBS: O Ponto de intersecção do gráfico
com o eixo y é o ponto onde x é igual a
zero ou seja o ponto de coordenadas (0,c)
Conforme observamos nos exemplos
anteriores a parábola pode ter concavidade
voltada para cima ou para baixo.
De modo geral temos:
Podemos observar que: ● Se a > 0, a parábola tem a concavidade
● A parábola tem concavidade voltada para voltada para cima.
cima. (a > 0) ● Se a < 0, a parábola tem a concavidade
● O vértice da parábola é ponto de voltada para baixo.
coordenadas (1, - 4).
● A função y = x² - 2x – 3 possui dois zeros RAÍZES OU ZEROS DA FUNÇÃO
: POLINOMIAL DO 2° GRAU
-1 e 3. Os pontos de intersecção da parábola Dada a função definida por y = ax² + bx + c,
com o eixo x são (-1,0) e (3,0). os zeros dessa função são os valores de x
● O ponto de intersecção da parábola com o para os quais temos y = 0 ou seja ax² + bx +
eixo y é o ponto (0,-3) c = 0.
Exemplo 2: Construir o gráfico a função f Algebricamente, podemos obter os zeros de
dada pela lei y = -x² + 4 onde x representa uma função quadrática resolvendo a equação
um real qualquer. do 2° grau ax² + bx +c = 0. O discriminante (∆)
da equação é também o discriminante da
x y = -x² + 4 (x,y) função, assim temos:
-2 y = - (-2)² + 4 = 0 (-2, 0)
-1 y = - (-1)² + 4 = 3 (-1,3) ● (∆> 0) A função y = ax² + bx + c
tem dois zeros diferentes. O gráfico cruza o
0 y = 0² + 4 = 4 (0,4)
eixo x em dois pontos distintos.
- 1 y = -1² + 4 = 3 (1,3) ● (∆= 0) A função y = ax² + bx + c
2 y = -2² + 4 = 0 (2,0) tem um único zero real. O gráfico cruza o eixo
x em apenas um ponto. ou seja, a parábola
tangencia o eixo x.
● (∆< 0) A função y = ax² + bx + c
não possui zeros reais. Não existe intersecção
do gráfico com o eixo x.
Veja o resumo no quadro abaixo:

Podemos observar que:

68
Exemplo 1: Determinar os zeros das funções Portanto as raízes(zeros) da função são: 0 e
dadas e em seguida fazer o esboço do gráfico. 4. Agora podemos fazer o esboço do gráfico
da função y = - x² + 4x
A) y = x² - 3x – 10

Para determinar os zeros da função vamos


resolver a equação x² - 3x – 10 = 0.

x² - 3x – 10 = 0 ( a = 1, b= -3 e c = -10) 0 4 x
∆ = b² - 4.a.c C) y = x² - 6x + 9
∆ = (-3)² - 4. 1 .(-10)
∆ = 9 + 40 Para determinar os zeros da função vamos
resolver a equação x² - 6x + 9 = 0.
∆ = 49
−𝑏±√∆ 3+7 x² - 6x + 9 = 0 ( a = 1, b= - 6 e c = 9)
𝑥 =
2.𝑎 2 ∆ = b² - 4.a.c
10
=5 ∆ = (-6)² - 4. 1 .(9)
2
−(−3)±√49 3±7 ∆ =36 – 36
3−7
𝑥 = = ∆=0
2.1 2 2
−4
= −2
2
−𝑏±√∆ 6+0 6
Portanto as raízes (zeros) da função são: -2 e 𝑥 𝑥′ = =
2.𝑎 2 2
5. 3
Agora podemos fazer o esboço do gráfico da
−(−6)±√0 6±0 6−0 6
função y = x² - 3x – 10 𝑥 = 𝑥′′ = =
2.1 −2 2 2
3

-2 5 x
Portanto a raiz (zero) da função é: 3. Agora
B) y = -x² + 4x podemos fazer o esboço do gráfico da função

Para determinar os zeros da função vamos y = x² - 6x + 9


resolver a equação - x² + 4x = 0.

x² + 4x = 0 ( a = -1, b= 4 e c = 0)
∆ = b² - 4.a.c
∆ = (4)² - 4.(- 1) .(0)
3 x
∆ =16
D) y = -3x² + 2x – 1

−𝑏±√∆ −4+4 Para determinar os zeros da função vamos


𝑥 𝑥′ = resolver a equação -3x² + 2x – 1= 0.
2.𝑎 −2
0
=0 -3x² + 2x – 1= 0 (a = -3, b= 2 e c = -1)
−2
−4±√16 −4±4 −4−4 ∆ = b² - 4.a.c
𝑥 = 𝑥′′ = ∆ = (2)² - 4.(-3) .(-1)
2.(−1) −2 −2
−8
=4 ∆ = 4 - 12
−2
∆ = -8

69
A equação não tem raízes reais. Portanto a y = 4 e isso ocorre para x = 1.
parábola não intercepta o eixo x. Agora Exemplo 2: Observe o gráfico da função
podemos fazer o esboço do gráfico da função
y = x² - 2x – 3 em que a>0.
y = -3x² + 2x – 1

VÉRTICE DA PARÁBOLA, VALOR MÍNIMO


E VALOR MÁXIMO DE UMA FUNÇÃO DO
2° GRAU
Identificar as coordenadas do vértice de uma
parábola nos ajuda na construção do gráfico
de uma função do 2º grau, nos permite
determinar a imagem da função, bem como o
seu valor máximo ou mínimo.

Exemplo 1: Observe o gráfico da função

y = -x² + 2x + 3 em que a<0. Os zeros da função y = x² - 2x – 3 são -1 e 3.


O vértice da parábola pertence ao eixo de
simetria que passa exatamente no meio dos
zeros da função y = x² - 2x – 3. Dessa forma
para calcular a abscissa do vértice basta
determinar a média aritmética dos valores -1
e 3.
Xv = -1 + 3 = 2 = 1
2 2

A ordenada do vértice é calculada


substituindo o valor Xv=1 em y = x² - 2x – 3 .
Yv = x² - 2x – 3 = 1² -2.1 – 3 = 1- 2 -3 = 1 -5
=-4
Os zeros da função y = -x² + 2x + 3 são -1 e Yv=-4
3. O vértice da parábola é o ponto de
coordenadas (1,-4).Nesse caso o vértice da
O vértice da parábola pertence ao eixo de parábola é o ponto de mínimo da função. O
simetria que passa exatamente no meio dos valor mínimo da função é
zeros da função y = -x² + 2x + 3. Dessa forma
para calcular a abscissa do vértice basta y = -4 e isso ocorre para x =1.
determinar a média aritmética dos valores -1 De modo geral temos:
e 3.

Xv = -1 + 3 = 2 = 1
2 2

A ordenada do vértice é calculada


substituindo o valor Xv=1 em y = -x² + 2x + 3.
Yv = -x² + 2x + 3 = - 1² + 2.1 + 3= -1 +2 +3= 4
Yv=4

O vértice da parábola é o ponto de


coordenadas (1,4) Nesse o caso vértice da
parábola é o ponto de máximo da função. O
valor máximo da função é

70
OBS: O vértice de uma parábola dada por (a > 0 e ∆ > 0) (a < 0 e ∆ > 0)
f(x)=ax² +bx +c, a≠0 também pode ser
calculado assim: + + x +
x
Xv= - b x’ _ x” _ x’
V(Xv, Yv) 2a x” _
f(x)=0 para x=x’ e x=x” f(x)=0 para x=x’
Yv= -∆ e x=x”
4a
f(x)>0 para x<x’ e x>x” f(x)>0 para
x’<x<x”
Exemplo 3: A função y = -x² + 4 tem ponto de
mínimo ou de máximo? Determine as f(x)<0 para x’<x<x” f(x)<0 para x<x’
coordenadas desse ponto. e x>x”
(a > 0 e ∆ = 0) (a < 0 e ∆ = 0)
Resposta: Pela função dada temos a<0
portanto essa função tem ponto de máximo. x’= x”
Coordenadas do vértice:
x
Xv= - b = - 0 = 0 _ _ _
2a 2.1 + + +
Yv = -∆ = - (b² - 4.a.c) = - (0² - 4. (-1) . 4) = x’= x” x
16 = 4
f(x)=0 para x=x’=x” f(x)=0 para
4.a 4.a 4.1 x=x’=x”
4 f(x)>0 para x≠x” f(x)<0 para x≠x”

V(Xv, Yv) = (0 ,4) . Nesse caso valor máximo (a > 0 e ∆ < 0 ) (a < 0 e ∆ < 0)
é 4 e isso ocorre para x = 0.
_ _ _ x

+ + + x
f(x)>0 para todo x real f(x)<0 para todo x
real
Exemplo 1: Estude o sinal das seguintes
funções quadráticas
a) f(x)=x² -4x +4
b) f(x) = x² -7x + 6
c) f(x) = -2x² +3x - 4
a) Resposta: f(x)=x² -4x +4
(a>0) então a concavidade da parábola está
voltada para cima.
∆=b² - 4.a.c = (-4)² - 4.1 .4 =16 -16 = 0
ESTUDO DO SINAL DE UMA FUNÇÃO (∆=0) a parábola intercepta o eixo x em
POLINOMIAL DO 2° GRAU apenas um ponto.
Coordenadas do vértice: V(Xv,Yv)
Estudar o sinal da função do 2º grau significa Xv = -b = -(-4) = 4 = 2
determinar os valores reais de x para os quais
2.a 2.1 2
a função se anula ( f(x)=0), é positiva( f(x)>0),
e é negativa( f(x)<0).
Esboço do gráfico: + + +
O estudo do sinal da função do 2º grau
x
depende do valor do discriminante ∆ =b² -4a.c
2
e do valor do coeficiente a. Observe o quadro
f(x) >0 para x ≠ 2
resumo:
f(x) =0 para x =2
a função f(x) nunca será negativa

71
b) Resposta: f(x) = x² -7x + 6 (a>0) então a P2) Para quais valores de x a função
concavidade da parábola está voltada para
f(x) = - 5x² + 4x + 1 é positiva?
cima.
(A) f(x) > 0 para { x ϵ IR/ -1 < x < 1}
∆= b² - 4.a.c = (-7)² - 4.1 .6 = 49 - 24 = 25
5
(∆>0) a parábola intercepta o eixo x em dois
pontos diferentes. (B) f(x) > 0 para{ x ϵ IR/ x< -1 ou x > 1}
−𝑏±√∆ 7+5 12 5
𝑥 𝑥′ = =
2.𝑎 2 2 (C) f(x) > 0 para{ x ϵ IR x = -1 ou x = 1}
6 5
7±√25 7±5 7−5 2 (D) f(x) > 0 para{ x ϵ IR/ x < -1 }
𝑥 = 𝑥′′ = =
2.1 −2 2 2
5
1

Esboço do gráfico: P3) Uma função quadrática é dada pela


lei
+ 1 6 + f(x)=0 para x=1 ou x=6 y = (k - 3) x² + x . Para que valores de k o
_ f(x)>0 para x<1 ou x gráfico dessa função é uma parábola com a
>6 concavidade voltada para cima?
f(x)<0 para 1<x<6 (A) k > 0
c) Resposta : f(x) = -2x² +3x - 4 (B) k = 3
(a<0) então a concavidade da parábola está (C) k > 3
voltada para baixo. (D) k < 0
∆=b² - 4.a.c = 3² - 4.(-2).(-4) = 9 – 32 = -23 (E) k >-3
(∆< 0) a parábola não intercepta o eixo x.
Esboço do gráfico: P4) (UFRGS – RS) Uma bola colocada no
chão é chutada para o alto, percorrendo uma
X f(x) < 0 para todo x trajetória descrita por y = - 2x² + 12x, em que
real, ou y é a altura dada em metro. A altura máxima
_ _ _ seja, f(x) é sempre atingida pela bola é
negativa (A) 10m
(B) 12m
PROFESSOR RESPONDE (C) 16m
P1) As coordenadas do vértice do gráfico da
função quadrática h(x) =ax² +3 x + c são (D) 18m
(6,0). Qual o valor dos coeficientes a e c (E) 36m
dessa função?
(A) a = -1 e c = 9
ALUNO RESPONDE
2
A1) (IFPI2016) Maria dispõe de uma tela de
(B) a = -1 e c = - 9 arame de 56 metros de comprimento para
4 cercar um terreno. Quais devem ser as
medidas dos lados do terreno para que a área
(C) a = -1 e c = 9 seja máxima?
4 (A)18 m e 10 m
(D) a = -1 e c = 6 (B)17 m e 11 m
3 (C)16 m e 12 m
(E) a = -1 e c = -6 (D)15 m e 13 m
3 (E)14 m e 14 m

72
A2) (IFPI2016) Qual o valor de m real sabendo (D) y é positivo se x < 2 ou x > 3.
que o gráfico da função quadrática f(x) = −mx2 (E) y é zero se x < 2
+ 3m2 tem concavidade voltada para baixo e
que o ponto de intersecção desse gráfico A7) (IFPI2014) Dada a função f(x)=3x² +6x-
com o eixo y é (0, 27). m, marque a alternativa que determina o valor
da constante m para que a função tenha valor
(A) −3 mínimo igual a 4.
(B) 3
(C) −4 (A) 7
(D) 4 (B) 5
(E) 5 (C) 1
(D) -1
A3) (IFPI2015.1) O Sr. Paulo tem 24m de tela (E) -7
e quer construir um cercado retangular
aproveitando um muro já existente. Qual a A8) (IFPI2014) A potência elétrica lançada em
maior área que o Sr. Paulo poderá obter com um circuito por um gerador é expressa por P
essa tela? = 10i - 5i2 (SI), onde i é a corrente elétrica.
(A) 72m² Marque a alternativa que indica a potência
(B) 76m² máxima, em watts, do gerador.
(C) 78m² (A) 5
(D) 80m² (B) 4
(E) 82m² (C) 3
(D) 2
A4) (IFPI2015.1)
(E) 1 O par ordenado (2,5)
pertence à função y = x² + bx + 1. Com A9) (IFPI 2013) O gráfico a seguir é a
base nessa informação, o valor mínimo de y representação da função R (x) = – x2 +
nessa função é: 100 x Analisando o gráfico descrito pode-se
(A) -3 afirmar que:
(B) -2
(C) 1
(D) 2
(E) 3

A5) (IFPI2015.1) Considere a função cujos


valores são dados por h (x) = (3x - 1)2 e cujo
domínio é o conjunto dos números primos.
Determine o elemento do domínio associado
ao número 400 da imagem.
(A) 5
(B) 7 (A) A função atinge o máximo de 3000
(C) 11 (B) f(40) = 2f(20)
(D) 13 (C) a função atinge o máximo de 2500
(E) 17 (D) a função atinge o máximo para x = 60
A6. (IFPI2014) Observe o gráfico da função (E) a função atinge um mínimo para x = 20
quadrática.
A10) (IFPI 2013) Custo obtido por uma
empresa quando a mesma produz n unidades
de um produto segue a seguinte lei: C = 15000
– 250x + x 2, onde C representa o custo, em
reais, para se produzirem x unidades do
determinado produto. Quantas unidades
deverão ser produzidas para se obter o custo
mínimo?
Desse gráfico conclui-se que: (A) 135.
(B) 125.
(A) y é negativo se x > 3. (C) 124.
(B) y é zero se x > 3. (D) 225.
(C) y é positivo se x está entre 2 e 3. (E) 250

73
A11) (IFMA2017) Um criador de galinhas (B) 5
dispõe de 80 metros lineares de uma tela para (C) 7
a confecção de um cercado retangular para a (D) 10
criação de aves. Uma parede plana será (E)15
utilizada como um dos lados do cercado.
Dessa forma, utilizando-se toda a tela para os A16) (ESPM – SP) A estrutura de lucro de
outros 3 lados do retângulo, qual a área uma empresa pode ser estudada através da
máxima obtida para o cercado? função y = -x² + 120x – 2000 sendo y o lucro
em reais quando a empresa vende x
(A) 800m² unidades. Com base nisso pode-se afirmar
(B) 600m² que:
(C) 400m² (A) O lucro é máximo quando x = 60
(D) 1200m² (B) O lucro é máximo quando x = 1600
(E) 1600m² (C) O lucro é máximo quando x = 20 ou x
A12) (IFMA2016.2) Uma função quadrática é =100
dada pela relação y = (m – 5)x2 - x. Para que (D) O lucro é máximo quando x < 20 ou x >
valores de m o gráfico desta função é uma 100
parábola com concavidade voltada para
cima? A17) A função definida por y = x² - 8x +16 :
(A) m>-5 (A) É positiva para todo x real.
(B) m=5 (B) É positiva para todo x real e diferente de
(C) m = -5 4.
(D) m<5 (C) É negativa para todo x real
(E) m>5 (D) É negativa para todo x real e diferente de
A13) Determine o valor de n, na função 4
quadrática g(x) = nx² + (n+1).x + 5, sabendo (E) É sempre negativa
que o vértice da parábola correspondente tem
A18) (PUC -MG) O valor máximo da função
coordenadas (-1, 4)
f(x) = - x² + 2x + 2 é:
(A) n=-1
(A) 2
(B) n =0
(B) 3
(C) n=1 (C) 4
(D) n=2 (D) 5
(E) 6
(E) n=3
A14) Determine os valores de p na função 1. (Efomm 2019) Examine a função real
quadrática y = (2p + 1)x² - 2x + 1 para que a 𝑓(𝑥) = 2𝑥 − 3𝑥 2 quanto à existência de
parábola tenha a concavidade voltada para valores e pontos de máximos e mínimos.
baixo. Analise o problema e assinale a alternativa
CORRETA.
(A) p > -1 2
2 a) A função atinge o valor máximo de no
3
(B) p = -1 1
ponto 𝑥 3.
2
1
(C) p < -1 b) A função atinge o valor mínimo de no
2 3
1
(D) p < 1 ponto 𝑥
3
2 1
(E) p > 1 c) A função atinge o valor máximo de no
3
2 2
ponto 𝑥 3.
A15) Uma bola é largada do alto de um edifício
2
e cai em direção ao solo. Sua altura h em d) A função atinge o valor mínimo de no
relação ao solo, t segundos após o 3
1
lançamento, é dada pela expressão h = - 25t² ponto 𝑥 3
+ 625. Após quantos segundos do lançamento 1
a bola atingirá o solo? e) A função atinge o valor máximo de no
3
1
(A) 3 ponto 𝑥 3

74
2. (Ueg 2019) Em um jogo de futebol, um função 𝒑(𝒙) = 𝟒𝟎𝟎 − 𝒙 onde x é a quantidade
jogador chuta uma bola parada, que descreve de produtos comprados, considerando uma
uma parábola até cair novamente no compra de, no máximo, 300 produtos.
gramado. Sabendo-se que a parábola é Sabendo-se que a receita de uma empresa é
descrita pela função𝑦 = 20𝑥 − 𝑥 2 , a altura o valor arrecadado com a venda de uma certa
máxima atingida pela bola é quantidade de produtos, qual a receita
a) 100 m máxima que essa empresa pode ter quando
b) 80 m fechar uma venda com um determinado
c) 60 m cliente, na moeda corrente no Brasil?
d) 40 m a) R$ 200,00
e) 20 m b) R$ 400,00
3. (Efomm 2019) Considere a função real c) R$ 20.000,00
𝑓(𝑥) = 1 + 4𝑥 + 2𝑥 2 Determine o ponto 𝑥 ∗ d) R$ 40.000,00
que define o valor mínimo dessa função. e) R$ 80.000,00
a) 𝑥 ∗ = −2
b) 𝑥 ∗ = −1 7. (G1 - ifpe 2017) Um técnico em
1 administração, formado pelo IFPE Campus
c) 𝑥 ∗ = − 2
Paulista, trabalha numa empresa e que o
d) 𝑥 ∗ = 𝑧𝑒𝑟𝑜 faturamento e o custo dependem da
e) 𝑥 ∗ = 1 quantidade x de peças produzidas. Sabendo
4. (G1 - ifpe 2019) Em um laboratório do que o lucro de uma empresa é dado pelo
IFPE, alunos do curso subsequente em faturamento menos o custo e que, nessa
Zootecnia observaram que a concentração C empresa, o faturamento e o custo obedecem
de certa medicação, em mg/L no sangue de respectivamente às funções 𝑓(𝑥) = −𝑥 2 +
animais de uma certa espécie, varia de acordo 3.800𝑥 e c(x)  200x  3.200, o número de
1
com a função 𝐶 = 6𝑡 − 𝑡 2 em que t é o peças que devem ser produzidas para que a
4
tempo decorrido, em horas, após a ingestão empresa obtenha o lucro máximo é
da medicação, durante um período de a) 3.200
observação de 24 horas. Determine o tempo b) 1.600
necessário, após o início do experimento, c) 3.600
para que o medicamento atinja nível máximo d) 2.000
de concentração no sangue desses animais. e) 1.800
a) 4 horas.
b) 16 horas. 8. (G1 - ifal 2017) Determine o valor de k na
c) 6 horas. equação x2  12x  k  0, de modo que uma
d) 12 horas. raiz seja o dobro da outra:
e) 2 horas. a) 12
5. (G1 - ifpe 2018) Quando estudamos b) 18
Cinemática, em Física, aprendemos que c) 24
podemos calcular a altura de uma bala atirada d) 28
para cima pela fórmula: h  200t  5t2, onde h é e) 32
a altura, em metros, atingida após t segundos
do lançamento. Qual o menor intervalo de 9. (G1 - ifal 2017) Em uma partida de futebol,
tempo para a bala atingir 1.875 metros de um dos jogadores lança a bola e sua trajetória
altura? passa a obedecer à função h(t)  8t  2t 2, onde
a) 20 s h é a altura da bola em relação ao solo
b) 15 s medida em metros e t é o intervalo de tempo,
c) 5 s em segundos, decorrido desde o instante em
d) 11 s que o jogador chuta a bola. Nessas
e) 17 s condições, podemos dizer que a altura
máxima atingida pela bola é
6. (G1 - ifal 2018) Certo fabricante, segundo a) 2 m
levantamentos estatísticos, percebe que seus b) 4 m
clientes não têm comprado mais de 10 de c) 6 m
seus produtos por compras. Para incentivar as d) 8 m
compras em maior quantidade, ele estabelece e) 10 m
um preço unitário p por produto dado pela

75
10. (Pucrs 2017) O morro onde estão situadas
as emissoras de TV em Porto Alegre pode ser
representado graficamente, com algum
prejuízo, em um sistema cartesiano, através
de uma função polinomial de grau 2 da forma
y  ax2  bx  c, com a base da montanha no
eixo das abscissas. Ângulo Ô ou AÔB
O ponto O é o vértice do ângulo e as
semirretas OA e OB são os lados.
Ângulo reto: é todo ângulo que med 90°.

Para que fique mais adequada essa


representação, devemos ter
a) a  0 e b2  4ac  0
b) a  0 e b2  4ac  0 Ângulo agudo: é um ângulo menor que o
c) a  0 e b2  4ac  0 reto.
d) a  0 e b2  4ac  0
e) a  0 e b2  4ac  0
11. (Ueg 2017) A temperatura, em graus
Celsius, de um objeto armazenado em um
determinado local é modelada pela função
x2
f(x)    2x  10, com x dado em horas. A
12 Ângulo obtuso: é um ângulo maior que o
temperatura máxima atingida por esse objeto reto.
nesse local de armazenamento é de
a) 0 ºC
b) 10 ºC
c) 12 ºC
d) 22 ºC
e) 24 ºC
12. (G1 - ifal 2017) No Laboratório de Química
Um ângulo cuja medida é de 0° é chamado de
do IFAL, após várias medidas, um estudante
ângulo nulo. E aquele cuja a medida é de
concluiu que a concentração de certa
180° é chamado de ângulo raso ou de meia
substância em uma amostra variava em
volta.
função do tempo, medido em horas, segundo
a função quadrática f(t)  5t  t2. Determine
em que momento, após iniciadas as medidas,
a concentração dessa substância foi máxima
nessa amostra.
a) 1 hora.
b) 1,5 hora. Ângulos congruentes: são ângulos que
c) 2 horas. possuem a mesma medida.
d) 2,5 horas.
e) 3horas.

GEOMETRIA PLANA: ÂNGULOS,


POLÍGONOS E CIRCUNFERÊNCIA
Ângulo: é toda região convexa do plano
determinada por duas semirretas de mesma
Os ângulos BÂC e DÊF são congruentes, pois
origem.
med (BÂC) = med (DÊF) = 50°.

76
Ângulos consecutivos: são ângulos que tem Dizemos que dois ângulos são
um um lado em comum. suplementares quando a soma de suas
medidas é igual a 180.

Ângulos opostos pelo vértice (OPV): são


ângulos formados por semirretas opostas e
que tem o vértice em comum.

Na figura acima, temos os seguintes pares de


ângulos consecutivos, AÔC e AÔB (com parte
Ângulos OPV são congruentes.
interna comum); AÔC e CÔB (sem parte
interna comum); AÔB e CÔB (com parte
interna comum). Ângulos formados por duas retas paralelas
cortadas por uma transversal
Ângulos adjacentes: são ângulos
consecutivos que não tem pontos internos
comum.

As retas r e s são paralelas e a reta t é


Na figura acima, temos apenas os ângulos
transversal as retas r e s. Elas determinam
AÔC e CÔB adjacentes.
oito ângulos que são divididos em
Bissetriz de um ângulo: é a semirreta interna a) ângulos correspondentes: a e e; b e f; c e g;
ao ângulo, de mesma origem e que o divide d e h.
em dois ângulos de mesma medida.
b) ângulos alternos externos: a e g; b e h;
c) ângulos alternos internos: c e e; d e f;
d) ângulo colaterais externos: a e h; b e g;
e) ângulos colaterais internos: d e e; c e f;

Ângulos correspondentes são congruentes,


ângulos alternos externos também são
congruentes e ângulos alternos internos são
suplementares. Ângulos colaterais externos
AD é bissetriz e med (CÂD) = med (BÂD) são suplementares e ângulos colaterais
internos são congruentes.
Dizemos que dois ângulos são
complementares quando a soma de suas
medidas é igual a 90°.

77
Unidade de medida de ângulos: para medir Diagonais: AC e BD
ângulos usamos o grau (°) e seus
OBSERVAÇÃO: Em qualquer polígono
submúltiplos, o minuto (‘) e o segundo (“).
convexo, o número de vértices, de lados e de
1° tem 60‘ (Em 1 grau tem 60 minutos) ângulos internos e ângulos externo é o
mesmo.
1’ tem 60” (Em 1 minuto tem 60 segundos)
Nomes dos polígonos
Para fazer operações com graus podemos
recorrer aos submúltiplos do grau. Número de lados Nome do polígono
Exemplos: 3 Triângulo
a) 43° 10’ 50” + 2° 50’ 60” 4 Quadrilátero
5 Pentágono
43° 10’ 50” + 2° 50’ 60” = 6 Hexágono
45° 60’ 110” = 7 Heptágono
46° 01’ 50” 8 Octogono
b) 90° - 3° 10’ 9 Eneágono
10 Decágono
Temos que 90° é igual a 89° e 60’. Dai, 11 Undecágono
90° - 3° 10’ = 12 Dodecágono
89° 60’ – 3° 10’ =
15 Pentadecágono
86° 50’
20 Icoságono
c) 127° - 2° 10’ 45”
Temos que 127° é igual a 126° e 60’, que é
Polígono regular: é um polígono convexo
igual a 126° 59’ e 60”. Dai,
que tem todos os lados com a mesma medida
127° - 2° 10’ 45” = e também todos os ângulos com a mesma
126° 59’ 60” – 2° 10’ 45” = medida.

124° 49’ 15” Exemplos:


a) triângulo equilátero - poligono regular de 3
lados
Polígono: é uma linha fechada formada
apenas por segmentos de reta do memo plano b) quadrado – polígono regular de 4 lados
que não se cruzam. c) pentágono regular – polígono regular de 5
lados
Soma das medidas dos ângulos internos
de um polígono convexo
Para determinar a soma das medidas dos
ângulos internos de um polígono convexo de
n lados usamos a fórmula
Si = (n – 2).180
Exemplos:
a) Vamos determinar a soma das medidas
Elementos do polígono ABCD dos ângulos internos de um pentágono.
Vértices: A, B, C e D Temos n = 5 lados, logo
Lados: AB, BC, CD e AD Si = (5 – 2).180
Ângulos internos: â, b, c, d Si = 3.180
Ângulos externos: ê, f, g, h Si = 540°

78
b) Vamos determinar qual polígono tem a Ângulos internos de polígonos regulares
soma das medidas dos ângulos internos igual
A medida de cada ângulo interno de um
a 360°.
polígono regular é dada pela fórmula

Temos Si = 360°, logo ai = Si/n = (n – 2).180/n


Si = (n -2).180 Ângulos externos de polígonos regulares
360 = (n-2).180 A medida de cada ângulo externo de um
360/180 = n – 2 polígono regular é dada pela fórmula
2=n–2 ae = Se/n = 360/n
n=2+2 Exemplo:
n = 4 (Quadrilátero) Para o octógono (n = 8), temos
Soma das medidas dos ângulos externos ai = (8 – 2).180/8
de um polígono convexo ai = 6.180/8
Em todo polígono convexo a soma das ai = 1080/8
medidas dos ângulos externos é igual a 360°.
ai = 135°
e
Número de diagonais de um polígono ae = 360/8
convexo ae = 45°
Para determinar o número de diagonaiss de OBSERVAÇÃO: ai + ae = 180°
um polígono de n lados usamos a fórmula
Estudo dos triângulos

d = n.(n – 3)/2
Condição de existência de um triângulo
(Desigualdade triangular)
Exemplo: Em todo triângulo, a medida de um lado é
a) Vamos determinar o número de diagonais sempre menor do que a soma das medidas
de um octógono. dos outros dois lados.
Temos n = 8 lados, logo Exemplos:
d = 8.(8 – 3)/2 a) Verifique se é possível construir um
triângulo com lados medindo 3 cm, 5 cm e 7
d = 8.5/2 cm.
d = 40/2 Observamos que
d = 20 3 < 5 + 7 (verdadeira)
b) Vamos determinar qual polígono tem um 5 < 1 + 7 (verdadeira)
total de 14 diagonais.
7 < 3 + 5 (verdadeira)
Temos d = 14, logo
d = n.(n -3)/2 Logo, é possível a construção pedida.
14 = n.(n-3)/2 b) Verifique se é possível construir um
triângulo com lados medindo 8 cm, 4 cm e 3
14.2 = n.(n – 3) cm.
28 = n2 – 3n Observamos que
n – 3n – 28 = 0
2 3 < 4 + 8 (verdadeira)
n = – 4 ou n = 7 4 < 3 + 8 (verdadeira)
8 < 3 + 4 (falsa)
O valor negativo não convém, logo temos um
polígono de 7 lados (heptágono). Logo, NÃO é possível a construção pedida.

79
Classificação de triângulos Teorema do ângulo externo de um
triângulo

Quanto as medidas dos lados


Em todo triângulo, a medida de um ângulo
Equilátero – três lados com medidas iguais e externo é igual à soma das medidas dos
três ângulos com medidas iguais a 60°. ângulos internos não adjacentes a ele.

Exemplo: Determine a medida de x no


triângulo abaixo.

Isósceles – dois lados com medidas iguais e


os ângulos opostos a esses lados
congruentes.
Pelo Teorema do ângulo externo, temos que
140° = 75° + x
x = 140° – 75°
x = 65°

Escaleno – todos os lados e ângulos com Quadriláteros Notáveis


medidas diferentes.
Paralelogramos: são quadriláteros que tem
dois pares de lados paralelos.

Propriedades dos paralelogramos

Quanto as medidas dos ângulos internos a) Ângulos opostos são congruentes.


b) Ângulos não opostos são suplementares.

Acutângulo – todos os ângulos com medida c) Lados opostos congruentes.


menor que 90°. d) Diagonais que cortam-se ao meio.

Tipos de paralelogramo
a) Paralelogramo qualquer
Retângulo – um ângulo com medida igual a
90°.

b) Retângulo

Obtusângulo – um ângulo com medida maior


que 90°.

No retângulo os ângulos são todos retos e as


diagonais são congruentes.

80
c) Quadrado
c) Trapézio retângulo

No trapézio retângulo dois ângulos internos


No quadrado todos os lados são congruentes são retos e um dos lados que não é base é
e todos os ângulos são retos. perpendicular às duas bases.

d) Losango Base média de um trapézio

O segmento que liga os pontos médios dos


lados não paralelos de um trapézio é
chamado de base média do trapézio.

No losango todos os lados são congruentes e Na figura acima M é o ponto médio de AD e N


as diagonais são perpendiculares entre si é o ponto médio de BC. Logo, MN é a base
(formam 90° entre si). E as diagonais são média do trapézio ABCD.
bissetrizes dos ângulos internos. Em todo trapézio, a medida da base media é
igual à média aritmética das medidas das
Trapézios: são quadriláteros que tem apenas bases maior e menor do trapézio.
um par de lados paralelos. Med (MN) = [med (AB) + med (CD)] / 2
Tipos de trapézio
a) Trapézio qualquer Exemplo: Sabendo que MN é base média do
trapézio abaixo, determine o valor de x.

b) Trapézio isósceles

20 = (3x + 2x + 15) / 2
2.20 = 5x + 15
5x = 40 – 15

No trapézio isósceles os lados não paralelos 5x = 25


são congruentes, os ângulos das bases são X=5
conguentes e as diagonais também são
congruentes.

81
Circunferência: é o conjunto dos pontos do PROFESSOR RESPONDE
plano que estão a uma distância (dada) de um
outro ponto desse plano. O ponto dado é o
centro e a distância dada é o raio da P1) (IFPI – 2014) O retângulo da figura abaixo
circunferência. é composto pelos quadrados P, Q e R e pelo
polígono sombreado S.

Se os lados dos quadrados medem,


respectivamente, 3, 5 e x, em metros, qual
deve ser o perímetro do polígono sombreado
Raio é o segmento que liga o centro S?
Corda é o segmento que liga dois pontos da (A) 18 m
circunferência. (B) 20 m
(C) 22 m
Diâmetro é uma corda que passa pelo centro (D) 24 m
da circunferência. (E) 28 m
Podemos determinar o comprimento (C) de
uma circunferência através da fórmula P2) (IFPI – 2014) Emanuel fez um protótipo de
um carro com rodas de 40 cm de raio.
C = 2.π.r Considerando π = 3,1, quantos quilômetros
o carro percorreu depois que as rodas deram
5000 voltas?
onde r é o raio da circunferência e π (pi) é (A) 12,40 km
aproximadamente 3,14. (B) 12,56 km
(C) 12,60 km
Exemplos: (D) 12,72 km
(E) 12,88 km
a) Determinar o comprimento de uma
circunferência de raio 5 cm. Considere π = 3. P3) (IFPI – 2016) O complemento do ângulo
62º45’37’’ é:
(A) 28º44’23’’
C = 2. π.r (B) 28º35’23’’
C = 2.3.5 (C) 27º14’53’’
(D) 27º14’23’’
C = 30 cm
(E) 27º45’37’’
b) Determinar o raio de uma circunferência de
comprimento igual a 8π cm. P4) (IFPI – 2015) Na figura, P e R são os
centros das semicircunferências de diâmetro
BC e AD respectivamente. O quadrilátero
C = 2. π.r ABCD é um quadrado de lado 4m. Qual é o
8π = 2.π.r comprimento (perímetro) da figura?

r = 8π/2π
r = 4cm
(A) 8 + 4π
(B) 8 + 8π
Perímetros de figuras planas
(C) 12 + 4π
O valor da medida do contorno de uma figura
(D) 16 + 8π
plana é chamado de perímetro dessa figura.
(E) 12 + 8π

82
ALUNO RESPONDE (A) 239
(B) 236
A1) (IFPI – 2016) Carl Bolt, corredor de longas (C) 328
distâncias, decidiu fazer corridas numa pista
(D) 200
circular de raio 60 metros num total de 20
voltas por dia, pelo menos duas vezes por (E) 150
semana. Nesse caso, qual será a distância
diária percorrida por Carl Bolt? Considere π ≅
3,14. A6) (IFMA – 2013) O valor de x na figura
corresponde a:
(A) 7.159 metros
(B) 7.536 metros
(C) 7.912 metros (A) 19º
(D) 8.289 metros (B) 32°
(E) 8.666 metros (C) 12º
(D) 25º
A2) (IFMA – 2017) Na figura seguinte, o (E) 21º
triângulo ABC é isósceles, com AB
congruente a BC. Sabendo-se que B = 50º, A7) (IFM – 2014) Associe “V” para verdadeiro
que CM é congruente a DM, determine a e “F” para falso nas afirmações:
medida do ângulo x.

I. ( ) As diagonais de qualquer retângulo


(A) 120° são congruentes.
(B) 130°
(C) 110° II. ( ) As diagonais de qualquer losango
(D) 100° são congruentes.
(E) 90° III. ( ) As diagonais de um quadrado são
congruentes.
A3) (IFMA – 2017) Uma pessoa caminha em IV. ( ) As diagonais de qualquer retângulo
volta de uma praça de forma circular com raio são perpendiculares entre si.
equivalente a 20m. Após determinado tempo,
essa pessoa constatou que tinha dado 20 V. ( ) As diagonais de qualquer losango
voltas ao redor da praça. Essa pessoa são perpendiculares entre si.
caminhou: (considere π = 3,14) VI. ( ) As diagonais de um quadrado são
(A) 2.512 m perpendiculares entre si.
(B) 2.199 m A sequência das associações nas afirmações
(C) 1.256 m anteriores é:
(D) 3.199 m
(E) 4.000 m (A) I –V; II – F; III- F; IV – V; V – V; VI – V
(B) I –V; II – F; III- F; IV – F; V – V; VI – V
A4) (IFMA – 2017) O valor do ângulo x no (C) I –V; II – F; III- V; IV – F; V – V; VI – V
triângulo ABC abaixo é:
(D) I –F; II – F; III- F; IV – V; V – V; VI – F
(A) 105°
(E) I –V; II – V; III- V; IV – V; V – V; VI – V
(B) 120°
A8) (IFPI – 2015) Considere um polígono
(C) 90° regular que possui a partir de cada um de seus
(D) 110° vértices 21 diagonais. Então, quanto mede
cada ângulo interno desse polígono?
(E) 80°
(A) 150º
A5) (IFMA – 2012) Um atleta de uma prova de (B) 160º
resistência deve percorrer 60 Km sobre uma (C) 165º
pista circular de raio 40m. O número (D) 170º
aproximado de voltas que ele deve dar é: (E) 175°
(considerar π = 3,14).

83
A9) O valor de x na figura abaixo é: A(s) afirmação(ões) correta(s) é (são):
(A) I e II
(B) III
(A) 60° (C) II e III
(B) 45° (D) I
(C) 50° (E) I, II e III
(D) 30°
A12) (IFAM – 2013) Dados a, b e c, assinale
(E) 70°
a alternativa verdadeira a respeito do tipo do
triângulo:
A10) (IFMA – 2016) Leia as afirmativas. (A) Se a= 10, b= 8 e c=3, é triângulo.
I. O polígono que possui 9 lados é (B) Se a= 9, b= 4 e c=5, não é triângulo.
denominado decágono.
(C) Se a= 8, b= 5 e c=4, o triângulo é
II. A soma dos ângulos internos de qualquer isósceles.
quadrilátero é 360º.
(D) Se a= 7, b= 7 e c=7, o triângulo é
III. Um polígono convexo regular tem todos os escaleno.
ângulos internos com a mesma medida.
(E) Se a= 8, b= 5 e c=4 o triângulo é
IV. Se dois polígonos têm quantidades equilátero.
diferentes de lados, necessariamente eles
têm a soma dos ângulos internos também
diferentes. A13) (IFAM – 2013) Sendo OB a bissetriz do
ângulo AÔC, a medida do ângulo AÔB é:
A(s) afirmação(ões) correta(s) são:
(A) IV
(B) I
(A) 20°
(C) I, II, III e IV
(B) 30°
(D) II, III e IV
(C) 40°
(E) II e IV
(D) 50°
(E) 60°
A11) (IFMA – 2016) Claudia é uma estudante
do IFMA e fez uma prova de geometria onde A14) (IFAM – 2012) Classifique as afirmações
uma das questões relacionava–se com a em verdadeira (V) ou falsa (F) e depois
figura abaixo. A questão solicitava que a aluna assinale a alternativa correta:
apresentasse algumas características do I. Duas retas em um plano são paralelas
trapézio. Ela, então descreve as seguintes quando não tem ponto em comum
características:
II. Duas retas no plano são concorrentes
quando se cruzam formando um ângulo de
90º
III. Dois segmentos de reta são consecutivos
se estão em uma mesma reta
IV. Um figura geométrica é plana quando
todos os seus pontos pertencem a um mesmo
I- Trata-se de um trapézio escaleno plano
I- Os ângulos medem, cada um, 120,75º D ˆ C (A) V F F V
ˆ (B) V F V F
(C) V F V V
III- Os lados e são congruentes AD e BC
(D) F V F F
(E) V V V V

84
A15) (IFAM – 2012) Observe as seguintes A19) (IFPI – 2013) No polígono abaixo,
afirmações: os lados consecutivos são perpendiculares
I. Um ângulo é obtuso quando sua medida é entre si. Qual é o perímetro desse
maior que 90º.
polígono?
II. Um polígono convexo que possui 11 lados
é denominado onzedecágono. (A) 28
III. Um triângulo é isósceles quando dois lados (B) 34
têm a mesma medida.
(C) 42
IV. Trapézios são quadriláteros com apenas
um par de lados paralelos. (D) 50
V. Losangos são paralelogramos cujos lados (E) 56
têm medidas diferentes. A20) No trapézio ABCD da figura, RS e a base
(A) Todas são verdadeiras média. O valor de x é
(B) Apenas uma é falsa
(C) Duas são falsas (A) 8 cm
(D) Três são falsas (B) 28 cm
(E) Todas são falsas (C) 48 cm
(D) 76 cm
A16) (IFAM – 2012) A razão entre o número (E) 96 cm
de diagonais de um eneágono e o número de
diagonais de um pentadecágono é:
(A) 3/10
(B) 1/5
(C) 1/2
ATIVIDADE COMPLEMENTAR
(D) 3/8
1. (G1 - ifpe 2019) Em uma olimpíada de
(E) 2/5 robótica, o robô BESOURO caminha de fora
do círculo de manobras e, após se apresentar,
retorna ao ponto inicial conforme a figura a
A17) A medida do ângulo x na figura abaixo, seguir.
onde r//s é:
(A) 22°
(B) 49°
(C) 65°
(D) 71°
(E) 82°

A18) (IFAM – 2015) A soma dos ângulos


internos de um polígono vale 2340°. O número
de diagonais desse polígono é: Considerando que o caminho percorrido pelo
(A) 160 robô está indicado pelas setas, qual o ângulo
x formado entre o caminho de saída e o
(B) 145 caminho de retorno do robô ao ponto inicial?
(C) 138
a) 28
(D) 123 b) 22
(E) 90 c) 21
d) 49
e) 56

85
2. (G1 - ifpe 2018) Para encontrar quais os 4. (Eear 2016) Duas cordas se cruzam num
assentos em um teatro possibilitam que um ponto distinto do centro da circunferência,
espectador veja todo o palco sob um ângulo conforme esboço.
de visão determinado, utilizamos o conceito
de “arco capaz”. A esse respeito, analise a
figura abaixo:

A partir do conceito de ângulo excêntrico


interior, a medida do arco x é
O “arco capaz do ângulo θ(θ  90) sobre o
a) 40
segmento AB" corresponde ao arco maior da b) 70
circunferência representada na figura acima, c) 110
que possui centro em O, e tem AB como d) 120
corda. Como os ângulos APB e AMB são
ângulos inscritos nessa circunferência e 5. (Fgv 2016) As cordas AB e CD de uma
determinam o mesmo arco, eles têm a mesma circunferência de centro O são,
medida. Esses ângulos são conhecidos como respectivamente, lados de polígonos
“inscritos”. Considere o arco capaz de 60 regulares de 6 e 10 lados inscritos nessa
sobre o segmento AB representado abaixo. circunferência. Na mesma circunferência, as
cordas AD e BC se intersectam no ponto P,
conforme indica a figura a seguir.

Qual é o valor do ângulo α  OAB, sabendo


que O é o centro da circunferência?
a) 30.
A medida do ângulo BPD, indicado na figura
b) 36.
por , é igual a
c) 20.
d) 60. a) 120.
e) 45. b) 124.
c) 128.
3. (G1 - cftmg 2017) A figura a seguir mostra d) 130.
uma circunferência, em que os arcos ADC e e) 132.
AEB são congruentes e medem 160 cada
um. 6. (G1 - ifsc 2014) Um estudante ao chegar
ao prédio do câmpus Florianópolis do IFSC
percebeu que no seu relógio os ponteiros
estavam marcando exatamente duas horas.
Considerando o ângulo agudo formado pelos
ponteiros das horas e dos minutos, é
CORRETO afirmar que esse ângulo agudo é
de:

A medida, em graus, do ângulo x, é a) 20


a) 10. b) 120
b) 20. c) 60
c) 30. d) 300
d) 40. e) 30

86
7. (Fgv 2013) Na figura, AB e AE são CONGRUÊNCIAS E SEMELHANÇA DE
tangentes à circunferência nos pontos B e E, POLÍGONOS
ˆ  60.
respectivamente, e BAE CONGRUÊNCIAS DE POLÍGONOS
Polígonos congruentes são aqueles que
possuem lados e ângulos correspondentes
com medidas iguais.
Exemplo:

Se os arcos BPC, CQD e DRE têm medidas


ˆ
iguais, a medida do ângulo BEC, indicada na
figura por α, é igual a
a) 20°
b) 40° Observe que os ângulos correspondentes
c) 45° agora ocupam posições não tão óbvias
d) 60° quanto anteriormente. Observe as relações de
e) 80° congruência: a = i, d = j, c = k e b = l.
As relações de congruência entre os lados
8. (Mackenzie 2012) Na figura, se a são as seguintes: AB = IL, BC = LK, CD =
circunferência tem centro O e BC = OA, então KJ e DA = IJ.
a razão entre as medidas dos ângulos AÔD e
CONGRUÊNCIAS DE TRIÂNGULOS
CÔB é
No caso de congruência de triângulos é
5 possível descobrir se um triângulo é
a) congruente ao outro apenas comparando os
2
seus elementos.
3
b)
2 Os casos de congruência comparam
c) 2 elementos de um triângulo com outro
4 triângulo, veja quais são estes casos:
d) 1º Caso: LAL: neste caso teremos dois
3
e) 3 lados congruentes e o ângulo formado
por eles também será congruente.

9. (G1 - ifsp 2011) Na figura, a reta t é


tangente, no ponto P, ao círculo de centro O.
A medida do arco é 100º e a do arco é 194º.
O valor de x, em graus, é

2º Caso: LLL: aqui os três lados são


congruentes.

a) 53.
b) 57.
c) 61.
d) 64.
e) 66.

87
3º Caso: ALA: temos dois ângulos Em polígonos semelhantes a razão entre as
congruentes e o lado compreendido entre áreas é proporcional ao quadrado da razão
eles é congruente. entre dois lados correspondentes e a razão
entre os perímetros é proporcional à razão
entre dois lados correspondentes.

Perímetro=34 Perímetro=68
4º Caso: LAAo: um lado congruente, um Área=60 Área=240
ângulo adjacente e o ângulo oposto a esse Observe que:
lado é congruente. e = 22

SEMELHANÇA DE TRIÂNGULOS
É possível verificar a semelhança nos
triângulos de uma forma mais simples. Basta
observar se eles se enquadram em um
dos casos de semelhança de triângulos a
seguir:
1º Caso: Ângulo, Ângulo (AA): Dois
triângulos são semelhantes se possuírem dois
● SEMELHANÇA DE POLÍGONOS ângulos correspondentes congruentes.
Dois polígonos A e B são semelhantes se
os seus ângulos respectivos forem
congruentes e se os seus lados
correspondentes forem iguais.

2º Caso: Lado, Lado, Lado (LLL): Se dois


triângulos possuem três lados proporcionais,
então esses dois triângulos são semelhantes.

Multiplicando o mesmo valor aos dois


Observe que os polígonos A e B, na imagem,
são parecidos e possuem ângulos iguais.
Como os ângulos são iguais, para que eles
sejam considerados semelhantes, falta
apenas garantir a proporcionalidade entre
lados correspondentes. Observe:
FG = GH = HI = IJ = JF =2
DE CD CB AB EA
Nesse caso dizemos que os A e B são AB = BC = CA = 1
semelhantes, pois possuem os mesmos DE EF FD 2
ângulos e os lados correspondentes são
proporcionais.

88
3º Caso: Lado, Ângulo, Lado (LAL): Dois
triângulos que possuem dois lados
proporcionais e o (C)
ângulo entre eles
congruente são
semelhantes. (D)

AB = CA = 1 (E)
DE FD 2
ALUNO RESPONDE
PROFESSOR RESPONDE A1) Seja ABC um triângulo tal que AB = 24m
P1) Na figura o ABC é congruente ao EDC. e BC =16m. Traçando-se MN paralelo a BC,
Determine, o valor de x + y. com o ponto M a 9 metros de A, qual deve ser
o valor de MN?
(A) 6
(A) 4,5m
(B) 15
(B) 6m
(C) 12
(C) 7m
(D) 18
(D) 7,2m
(E) 21
(E) 8m
P2) Na figura abaixo, note que os triângulos
AEC e ADB são semelhantes. Sendo assim, A2) Um retângulo é dividido em seis
determine o número x, que está na figura: retângulos menores congruentes cujas
diagonais definem uma região pintada
(sombreada). Qual é a fração correspondente
(A) 9 a todas as regiões pintadas?
(B) 9,5 5
(A) 12
(C) 10
1
(B)
(D) 10,5 3
5
(E) 14 (C) 9
3
P3) Os lados correspondentes de dois (D) 10
pentágonos semelhantes estão na razão 1:2. 1
Qual é a razão entre as suas áreas? (E) 2
(A) 1:2 A3) Dois polígonos são semelhantes, sendo
(B) 2:4 que os lados do polígono maior medem o
(C) 1:8 dobro dos lados do polígono menor. Nesse
(D) 2:8 caso, os ângulos internos do polígono maior:
(E) 1:4
(A) São congruentes aos ângulos do
P4) Dois hexágonos semelhantes possuem polígono menor
áreas iguais a 36 cm² e 64 cm², (B) Medem a metade dos ângulos do
respectivamente. Qual é a razão entre as polígono menor.
medidas de um par de lados correspondentes
(C) Medem o dobro dos ângulos do
do menor para o maior (um em cada
polígono menor.
hexágono)?
(D) Medem o quádruplo dos ângulos do
polígono menor.
(A) (E) Medem o triplo dos ângulos do
polígono menor.
(B)

89
A4) Na ilustração temos `CE=1`, `EB=2`, lado, a sombra projetada de um poste mede 2
`DB=2` e `AD=x`. Determine a medida de `x= m. Se, mais tarde, a sombra do poste diminui
AD. 50 cm, a sombra da pessoa passou a medir:
(A) 30cm
(A) `1` (B) 40cm
(B) `2` (C) 45cm
(C) `3` (D) 50cm
(D) `4` (E) 60cm
(E) nenhuma das anteriores. A9) Uma estaca tem 150 cm e projeta uma
sua sombra 2,20 m ao mesmo tempo em que
um poste projeta uma sombra de 440 cm.
A5) Um prédio tem sombra, pela luz solar, Qual é a altura aproximada do poste em
projetada no solo horizontal com 70 m. metros?
Simultaneamente um poste de 8m de altura
(A) 3m
localizado nas proximidades deste prédio tem
sombra do mesmo tipo com 14 m. Calcule a (B) 3,50m
altura do prédio.
(C) 4,50m
(D) 5m
(A) 10 m
(E) 6m
(B) 20 m
(C) 35 m
A10) Determine x.
(D) 40 m
(A) 4m
(E) 80 m
(B) 4,4m
(C) 5,6m
A6) Dado os triângulos retângulos ARE e
(D) 6,8m
OTE:
(E) 8m
Se AR = OE = AE/2, então:

(A) AR = 4TO
(B) AR = TO
11) (Unesp) Um obelisco de 12 m de altura
(C) AR = 2TO
projeta, num certo momento, uma sombra de
(D) AR = TO/2 4,8 m de extensão. Calcule a distância
máxima que uma pessoa de 1,80 m de altura
(E) AR = TO/4
poderá se afastar do centro da base do
obelisco, ao longo da sombra, para, em pé,
A7) A sombra de uma pessoa que tem 1,80 m continuar totalmente na sombra.
de altura mede 60 cm. No momento, a seu (A) 0,72m
lado, a sombra projetada de um poste mede 2
(B) 1,08m
m. A altura desse poste é:
(C) 1,72m
(A) 3,60m
(D) 4,08m
(B) 4m
(E) 4,72m
(C) 5,80m
A12) Uma rampa de inclinação constante,
(D) 6m
como a que dá acesso ao Palácio do Planalto
(E) 10m em Brasília, tem 4 metros de altura na sua
A8) A sombra de uma pessoa que tem 1,80 m parte mais alta. Uma pessoa, tendo começado
de altura mede 60 cm. No momento, a seu a subi-la, nota que após caminhar 12,3 metros

90
sobre a rampa está a 1,5 metros de altura em A16) Determine o comprimento do segmento
relação ao solo. Quantos metros a pessoa OQ sabendo que MN é paralela a PQ.
ainda deve caminhar para atingir o ponto mais
alto da rampa.
(A) 5
(A) 7,7m
(B) 4,5
(B) 11,7m
(C) 3,9
(C) 12,3m
(D) 3
(D) 20m
(E) 2,8
(E) 20,5m
A13) Nos triângulos ABC e AED,
representados na figura a seguir, temos o A17- Em um triângulo retângulo foi traçada a
ângulo ADE congruente ao ângulo ACB. Se altura relativa à hipotenusa formando dois
BC = 16 cm, AC = 20 cm, AD = 10 cm e AE = triângulos retângulos menores, conforme
10,4 cm, o perímetro do quadrilátero BCED, mostra a figura abaixo. Determine a medida
em centímetros, é: do ângulo β:

(A) 28,4 m (A) 75º


(B) 44,4 m (B) 50º
(C) 56,4 m (C) 45º
(D) 56,8 m (D) 40º
(E) 85,2 m (E) 30º

A14) Na figura, o triângulo ABC é retângulo A18) Considere dois triângulos OMN e NPQ
em A, ADEF é um quadrado, AB=2cm e semelhantes, em que a área de NPQ
AC=6cm. Quanto mede o lado do quadrado? corresponde a 25% da área de OMN. A razão
entre lados correspondentes dos triângulos
NPQ e OMN, nessa ordem é de:
(A) 1,8 m
(B) 1,6 m
(A)
(C) 1,5 m
(D) 1,3 m
(B)
(E) 1,2 m

(C)
A15) Na figura, o triângulo ABC é retângulo
em A, ADEF é um retângulo, AB=2cm e
AC=6cm. Qual é a área máxima do quadrado? (D)

(A) 5m2 (E)


2
(B) 3m
(C) 2,5m2
(D) 2,25m2
(E) 2m2

91
TEOREMA DE TALES
FEIXE DE PARALELAS: é o conjunto de três ou
mais retas coplanares paralelas.
TRANSVERSAL: é toda reta que intercepta o
feixe.

c) Determine os valores de x e y no seguinte


feixe de paralelas:

As retas r, s e u são paralelas e a reta t é a


transversal.
ENUNCIADO DO TEORAM DE TALE

“Um feixe de paralelas determina sobre duas


transversais segmentos proporcionais”.
CONSEQUENCIA DO TEOREMA DE TALES
A partir da figura, podemos Sempre que houver uma reta paralela a um dos
escrever: lados de um triângulo, interceptando os outros
𝐴𝐵 𝑅𝑆 𝐴𝐵 𝑅𝑆 𝐵𝐶 𝑆𝑇 dois lados, essa reta estabelecerá sobre eles
𝐵𝐶
= 𝑆𝑇 𝐴𝐶
= 𝑅𝑇 𝐴𝐶
= 𝑅𝑇 pares de segmentos correspondentes e
proporcionais.
Veja na figura que ∆ABC ~ ∆AMN
Geralmente, em situações problema,
estruturamos as resoluções a partir da figura, do
seguinte modo:

𝑥 𝑎
=
𝑦 𝑏

EXEMPLOS DE APLICAÇÃO
Na figura seguinte, as retas a, b e c são
paralelas, determine o valor de x: Exercícios de aplicação

a) 01) No triângulo a seguir DE // BC. Nessas


condições, determine:

A) A medida X.

b) A figura ao lado indica três lotes de terreno


com frente para a rua A e para rua B. as divisas
dos lotes são perpendiculares à rua A. As
frentes dos lotes 1, 2 e 3 para a rua A, medem,
respectivamente, 15 m, 20 m e 25 m. A frente do
lote 2 para a rua B mede 28 m. Quais são as
medidas da frente para a rua B, dos lotes 1 e 3?
B) O perímetro do ∆ABC.

92
02) No triângulo ABC da figura, sabe – se que h Altura relativa à hipotenusa
DE // BC . Calcule as medidas dos lados AB e men Projeções dos catetos sobre a
AC do triângulo. hipotenusa

03) Num triângulo ABC, o lado AB mede 24 cm. EXEMPLO DE APLICAÇÃO


Por um ponto D, sobre o lado AB, distante 10 cm 1) Aplicando as relações métricas nos
do vértice A, traça–se a paralela ao lado BC, que triângulos retângulos abaixo, determine o valor
corta AC no ponto E distante 15 cm do vértice A. de x:
Determine a medida do lado AC.
TEOREMA DA BISSETRIZ INTERNA (TBI)
Em todo triângulo, a bissetriz interna relativa a
um dos lados determina sobre ele segmentos
proporcionais aos outros dois.

PROFESSOR RESPONDE
TEOREMA DA BISSETRIZ EXTERNA (TBE) P1) As retas r1, r2 e r3 são paralelas e os
comprimentos dos segmentos de transversais
são indicados na figura. Então x é igual a:
Em todo triângulo, com exceção do triângulo
isósceles, a bissetriz externa a qualquer ângulo
divide o lado oposto, prolongado, em dois (A) 58
segmentos proporcionais aos outros lados.
(B) 7,5
(C) 6
(D) 5,21
(E) 1

P2) A sombra de uma pessoa que tem 1,80 m


de altura mede 60 cm. No mesmo momento, a
RELAÇÕES MÉTRICAS NO TRIÂNGULO seu lado, a sombra projetada de um poste mede
RETÂNGULO 2 m. Se, mais tarde, a sombra do poste diminui
50 cm, a sombra da pessoa passou a medir:
Elementos de um Triângulo Retângulo
A Ângulo Reto (A) 30 cm
BeC Ângulo Agudos (B) 45 cm
(C) 50 cm
A Hipotenusa (D) 80 cm
bec Catetos (E) 90 cm

93
P3) A figura mostra um edifício que tem 15 m de A3) Na figura abaixo, a // b // c // d.
altura, com uma escada colocada a 8 m de sua
base ligada ao topo do edifício.

15

8
O comprimento dessa escada é de
(A) 12 m
Os valores respectivos de x e y são:
(B) 15 m
(C) 17 m (A) 2 e 10.
(D) 20 m (B) 10 e 2.
(E) 20 m
(C) 4 e 8.
ALUNO RESPONDE (D) 5 e 10.
A1) A soma dos números correspondentes às (E) 2 e 8.
medidas a, b, c e h no triângulo da figura abaixo
formam uma senha que abre o cofre do senhor
A4) Para determinar a altura de um edifício, seu
zelador usou um artifício. Mediu a sombra do
prédio, que 6 metros, e mediu sua sombra, que
deu 0,20 metros. Como sua altura é de 1,60
metros, ele obteve, para altura do prédio, o valor
de
Adamastor. (A) 24 m
Qual a senha que abre o cofre do Adamastor? (B) 36 m
(A) 174 (C) 42 m
(B) 164
(C) 144 (D) 48 m
(D) 134 (E) 50 m
(E) 124
A2) A figura abaixo mostra r // s // t.
A5) A figura abaixo mostra r // s // t.

Sendo a e b
duas transversais, então x é igual a:
(A) 6 Sendo a e b duas transversais, então x é igual
a:
(B) 5
(A) 5
(C) 4
(B) 4
(D) 3
(C) 3
(E) 2
(D) 2
(E) 1

94
A6) Duas transversais partem de um mesmo A11) Os números 3; 4 e 5 são conhecidos como
ponto A e encontram duas paralelas. A primeira trio Pitagórico pois 32 +42 = 52. Qual das
corta as paralelas em B e C, enquanto a alternativas seguintes também representa um
segunda corta em D e E. Se AB=6cm, BC=9cm trio Pitagórico?
e AD=4cm, então AE mede:
(A) 1, 7 e 9.
(A) 6 cm (B) 2, 5 e 6
(C) 7, 8 e 11.
(B) 8 cm
(D) 6, 8 e 10.
(C) 9 cm (E) 5, 7 e 9.
(D) 10 cm
A12) A soma das projeções dos catetos de um
(E) 12 cm triângulo retângulo é igual a 13 cm e um dos
catetos mede 5 cm. Nessas condições, o
perímetro desse triângulo é igual a:
A7) No triângulo da figura abaixo, AD // BC e EF
é a bissetriz do ângulo. Então, y é igual a: (A) 24 cm
(B) 28 cm
(A) 5 (C) 30 cm
(B) 6 (D) 32 cm
(E) 36 cm
(C) 7
(D) 8 A13) Tales desenhou um triângulo retângulo
com as seguintes medidas, todas dadas em
(E) 9 centímetros.
Qual é o perímetro deste triângulo?
A8) O perímetro de um triângulo é de 100m. A (A) 6 cm
bissetriz do ângulo interno  divide o lado oposto
em dois segmentos de 16 m e 24 m. Então, os (B) 9 cm
lados desse triângulo medem, em metros: (C) 12 cm
(A) 24, 40 e 36 (D) 15 cm
(B) 20, 40 e 40 (E) 20 cm
(C) 25, 30 e 45
(D) 40, 30 e 30 A14) A hipotenusa de um triângulo retângulo é
igual a 13 cm e seu maior cateto é igual a 12 cm.
(E) 20, 30 e 45 Qual a medida da área do triângulo?
A9) No triângulo FGH (figura), sendo DE // GH, (A) 28 cm2
x é igual a: (B) 30 cm2
(A) 4,0. (C) 32 cm2
(D) 34 cm2
(B) 4,5. (E) 36 cm2
(C) 5,0.
A15) No triângulo retângulo ABC mostrado na
(D) 5,54. figura, a soma dos valores de c e a é 36, e a
(E) 6,0. 4
razão entre eles é
A10) O perímetro de um triângulo é de 100 m. A 5
4
bissetriz do ângulo interno  divide o lado oposto , nessa ordem.
em dois segmentos de 16m e 24m. Então, os 5
lados desse triângulo medem, em metros: Nesse caso, b vale
(A) 24, 40 e 36. (A) 12
(B) 20, 40 e 40. (B) 14
(C) 25, 30 e 45. (C) 15
(D) 40, 30 e 30. (D) 16
(E) 25, 40 e 45. (E) 20

95
A16) Em um triângulo retângulo, um cateto tem ÁREA DO RETÂNGULO
27 cm e a hipotenusa tem 45 cm, qual a medida
do outro cateto?
(A) 15 cm
(B) 18 cm
(C) 36 cm
(D) 38 cm
(E) 45 cm ÁREA DO QUADRADO
A17) Em um triângulo retângulo as projeções
dos catetos sobre a hipotenusa medem 6 cm e
8 cm. A altura relativa à hipotenusa desse A=lxl
triângulo mede, em cm:

(A) 2 cm
(B) 3 cm
ÁREA DO TRAPÉZIO
(C) 4 cm
(D) 5 cm
(E) 16 cm

A18) As medidas dos catetos de um triângulo


retângulo são (x + 5) cm e (x + 1) cm e a
hipotenusa (x + 9) cm. O perímetro desse ÁREA DO LOSANGO
triângulo vale:
(A) 33 cm
(B) 38 cm
(C) 48 cm
(D) 54 cm
(E) 58 cm

A19) Num triângulo retângulo, um cateto é o


ÁREA DO PARALELOGRAMO
dobro do outro, e a hipotenusa mede 10 cm. A
soma dos catetos mede:
(A) 4 cm
(B) 6 cm
A=bxh
(C) 8 cm
(D) 10 cm
(E) 12 cm
ÁREA DO TRIÂNGULO
ÁREAS DE FIGURAS PLANAS
A plana baixa é de um terreno plano usando as
A=bxa
medidas dadas.

ÁREA DA REGIÃO TRIANGULAR


EQUILÁTERA

Para determinar a área do terreno, é preciso


calcular as áreas de várias figuras que o
compõem. Para isso, veremos, a seguir,
algumas fórmulas correspondentes as áreas de
figuras planas.

96
ÁREA DO POLÍGONO REGULAR P2) (IFPI 2016) O terreno do Sr. Matias é
retangular e tem 104 m de perímetro. O
comprimento tem 12 m a mais que a largura.
A=pxa Qual a área desse terreno?
P – semiperímetro (A) 384 m2
a – apótema (B) 420 m2
(C) 640 m2
(D) 730 m2
(E) 920 m2
ÁREA DO CÍRCULO
P3) (IPI 2016) Qual é a área de um retângulo
que possui diagonal igual a 15 cm e um dos
lados igual a 12 cm?

A = πr² (A) 108 cm2


(B) 54 cm2
(C) cm2
ÁREA DO SETOR CIRCULAR (D) cm2
(E) cm2
A=
P4) O salão de entrada de um prédio tem a
forma de um triângulo retângulo com sua
hipotenusa paralela ao solo, como mostra a
figura a seguir.
r – raio
A – área do setor
A área desse salão é
α- ângulo do setor (A) 12 m2
(B) 20 m2
ÁREA DA COROA (C) 24 m2
(D) 30 m2
(E) 36 m2
A = π (R² - r²)
P5) A parede externa de um túnel, cujas
dimensões são indicadas na figura a seguir, terá
R – raio externo de ser pintada.
r – raio interno

PROFESSOR RESPONDE

P1) O Estádio Governador Alberto Tavares


Silva, ou simplesmente Albertão, foi inaugurado
em 1973. Localiza-se na cidade de Teresina,
capital do Estado do Piauí. Sabe-se que o
campo de futebol do Albertão tem 110m de Sabe-se que o túnel tem formato semicircular.
comprimento por 75m de largura. Supondo que Sendo assim, a área a ser pintada é igual a
para cobrir esse campo foram adquiridas placas (A) 110 m2
de grama, sendo que cada uma pode cobrir uma
área de 25000cm². Quantas placas foram
usadas para cobrir o campo todo? (B) 5(25 − )m2
(A) 3500
(B) 3300
(C) 2800 (C) 5(22 − )m2
(D) 2450
(E) 2230 (D) 95 m2
(E) n.d.a

97
ALUNO RESPONDE A5) (IFAM-2015) A figura abaixo ilustra uma
praça quadrada circunscrita a um lago circular.
A1) (IFPI-2015) As raízes da função quadrática A área gramada da praça em destaque na figura
f (x) = x² - 25x +136 representam as medidas da (parte escura) mede:
hipotenusa e de um dos catetos de um triângulo
(A) 150 m²
retângulo. Então, a área desse triângulo
(B) 100 m²
retângulo é igual a:
(C) 60 m²
(A) 54 (D) 43 m²
(B) 60 (E) 15 m²

(C) 65 A6) (IFAM-2013) Um terreno retangular de área


700 m2 tem o comprimento que excede em 15
(D) 68
metros a largura. A equação que representa o
(E) 70 problema é:
(A) x2 + 15x + 700 = 0
A2) (IFAM-2012) A área do triângulo abaixo, em (B) x2 + 15x - 700 = 0
cm2, é (obs.: assuma que as medidas dos lados (C) x2 + 700x + 15 = 0
estão expressas em cm): (D) x2 + 700x - 15 = 0
(E) 2x2 - 15x - 700 = 0
(A) 14
A7) (IFC-2012) Num triângulo retângulo as
(B) 27
medidas dos lados estão expressas em
(C) 36 centímetros. A medida da hipotenusa vale x, um
(D) 48 cateto mede 6 e outro cateto tem medida x – 2.
É CORRETO afirmar a respeito deste triângulo:
(E) 54
(A) Um cateto mede 4 cm.
(B) Os lados tem medidas iguais a 6 cm, 10 cm
e 12 cm.
(C) A área é igual a 24 cm.
A3) (IFAM-2012) A área do quadrado inscrito (D) A medida da hipotenusa vale 8 cm.
num círculo de √ cm de raio é, em cm2: (E) O perímetro mede 24 cm.

A8) (IFF-2014) Na figura a seguir, ABCD é um


retângulo, N é ponto médio do segmento CD e
OB =1/4 AB. Se AD = 3 cm e CD = 14 cm, a área
do quadrilátero AOND é:
(A) 21/4 cm2
(B) 17/3 cm2
A4) (IFAM-2015) Numa casa pretende-se
(C) 15/2 cm2
construir dez paredes retangulares em alvenaria
de 3,00 m de altura, sendo que, em quatro delas (D) 21/2 cm2
o comprimento terá 2 m a mais do que nas (E) 105/4 cm2
demais, perfazendo uma área construída de no
mínimo 159 m² e no máximo 189 m². Então o
comprimento das paredes varia de: A9) (IFGOIANO-2014) A medida fixa da base de
um triangulo retângulo e 40 cm. Para que esse
(A) 4,5 m a 5,5 m triangulo tenha área igual a 300 cm2, a altura
(B) 4,5 m a 6,5 m relativa a base deve medir
(C) 4,5 m a 7,5 m
(D) 5,0 m a 7,0 m (A) 15 cm
(E) 5,5 m a 7,5 m (B) 20 cm
(C) 25 cm
(D) 30 cm
(E) 35 cm

98
A10) (IFGOIANO-2015) Uma folha de papel (A) 36 m2
quadrada de lados medindo 30 cm é dobrada, (B) 42 m2
unindo-se dois cantos não consecutivos, como (C) 45 m2
mostra a figura abaixo: (D) 48 m2
(E) 54 m2
A área do triângulo formado por essa dobradura
é igual a: A15) (IFPI-2013) A área de um determinado
(A) 450 m2. terreno com o formato de um quadrilátero, mede
324 m2. Se o quadrilátero for um quadrado, o
(B) 450 cm2.
perímetro dele será:
(C) 600 cm2.
(D) 900 m2.
(A) 68 m
(E) 900 cm2. (B) 72 m
(C) 70 m
(D) 74 m
A11) (IMA-2106) Dois retângulos e são tais que: (E) 66 m
A medida da base de R1 é o triplo da medida da
base de R2; A medida da altura de R1 é a terça A16) Os participantes de um grupo de alunos do
parte da medida da altura de R2. IFTM devem organizar um painel sobre o tema
“PAZ E MÍDIA”. Para a construção do painel os
Nessas condições é verdade que: alunos dispõem de uma região retangular
(A) a medida da área de R1 igual a medida da medindo 2 m de altura por 90 cm de largura.
área de R2. Esta área deve ser coberta com papel cartolina
de cor branca cujas dimensões são 21cm de
(B) o perímetro de R1é igual ao perímetro de R2. altura e 30cm de largura. Sabendo que cada
(C) a perímetro de R1 é o dobro do perímetro de cartolina tem um custo de R$0,50 pode–se
R2. afirmar que o valor gasto com as cartolinas será
de:
(D) a medida da área de R2 é o dobro da medida
da área de R1.
(E) a medida da área de R1 é o triplo da medida (A) 15,0 reais.
da área de R2. (B) 13,5 reais.
(C) 14,5 reais.
A12) (IFMA -2017) No Brasil, é comum utilizar o (D) 13,0 reais.
hectare para medir superfícies de grandes (E) 12,5 reais.
propriedades de terra. Um fazendeiro possui um
terreno retangular de 2,6 km de comprimento e A17) (IFTM 2014) A figura abaixo é constituída
1,3 km de largura. Qual é a área desse terreno? por dois retângulos. O retângulo ABCD tem 4 cm
(A) 339 hectares de largura e 10 cm de comprimento, e o
(B) 3380 hectares retângulo EFGH tem 8 cm de largura e 18 cm de
(C) 338 hectares comprimento.
(D) 3390 hectares
(E) 3480 hectares

A13) (IFMA-2012) Marque a alternativa que


representa a área da figura hachurada (pintada).
(A) 20 cm²
(B) 15 cm²
(C) 10 cm² É correto afirmar que a razão da área do
retângulo ABCD para a do retângulo EFGH é:
(D) 5 cm²
(A) 5/18
(E) 40 cm² (B) 7/9
(C) 7/13
A14) (IFPI-2013) O perímetro de um retângulo (D) 13/7
de dimensões 4 + x e 4x + 1 é igual a 30 m. (E) 18/5
Então, quanto mede a sua área?

99
A18) (IFTM 2015) O desenho a seguir SEGMENTOS E PONTOS NOTÁVEIS DO
representa uma piscina retangular que será TRIÂNGULO
construída em uma escola. Ela terá 4 metros de
1) Altura: segmento que une um vértice
largura e 6 metros de comprimento. Em seu
com um ponto do suporte do lado oposto, sendo
contorno, será construída uma calçada de
este segmento perpendicular ao suporte.
lajotas, representada pela área sombreada da
figura a seguir. A medida x para que a calçada
de lajotas tenha área de 76 m2 é:

1.1) AI, BJ, CK são alturas;


𝐴𝐼 ⊥ 𝐵𝐶, 𝐵𝐽 ⊥ 𝐴𝐶, 𝐶𝐾 ⊥ 𝐴𝐵
1.2) O ponto de encontro das alturas chama-se
ORTOCENTRO. No triangulo ao lado é o ponto
O.
(A) 1,5 m. 2) Mediana: segmento que une o vértice ao
(B) 2,5 m. ponto médio do lado oposto.
(C) 2 m.
(D) 2,75 m.
(E) 3 m.

A19) (ITM 2015) Para fazer trabalhos de


artesanato Dona Maria recortará círculos de um
tecido de algodão, conforme o modelo de corte 2.1) AI, BJ, CK são medianas;
da figura abaixo. O tecido tem dimensões 60 cm 2.2) O ponto de encontro das medianas
x 54 cm e todos os círculos têm o mesmo raio. chama-se BARICENTRO. No triângulo ao lado
A medida da área desperdiçada de tecido, é o ponto G.
representada pelo sombreado na figura é: 2.3) A distância do vértice ao baricentro vale
(Considere =3,14) 2
da mediana e a distância do baricentro ao pé
3
1
da mediana vale da mediana, ou seja, o
(A) 1 205,28 cm2 3
baricentro é o ponto que divide cada mediana
(B) 1 096,28 cm2 numa razão de 2 para 1.
(C) 840 cm2 2.4)
2𝐴𝐼 𝐴𝐼 𝐴𝐺 2
(D) 728 cm2 Ex: 𝐴𝐺 = 𝑒 𝐺𝐼 = 𝑒 =
3 3 𝐺𝐼 1
(E) 602,64 cm2 2.5) Num triângulo retângulo, a mediana relativa
A20) (IFPI 205) Nas notações da figura, a área à hipotenusa é igual a metade da hipotenusa.
do triângulo ABC mede 120m2 e BC = 16m. 3) Bissetriz Interna: segmento que liga o
Então, quanto mede a altura AH? vértice a um ponto do lado oposto, dividindo o
ângulo interno em duas partes congruentes.

(A) 10m
(B) 12m
(C) 13m
(D) 14m
(E) 15m

3.1) AM, BN, CP são bissetrizes internas;


3.2) O ponto de encontro das bissetriz chama-se
INCENTRO. NO triângulo ao lado é o ponto I.
3.3) O incentro é o centro do círculo inscrito no
triângulo.

100
4) Mediatriz: é a reta que divide o (C) 60º
segmento em duas partes congruentes, sendo (D) 90º
perpendicular ao segmento. (E) 120º

P4) (CESESP) Dentre os quatro centros


principais do triângulo qualquer, há dois deles
que podem se situar no seu exterior, conforme o
tipo de triângulo. Assinale a alternativa em que
os mesmos são citados.
4.1) 𝐴𝑀 = 𝑀𝐵 𝑒 𝑟 ⊥ 𝐴𝐵 , r é mediatriz. (A) O baricentro e o ortocentro.
(B) O baricentro e o incentro.
4.2) O ponto de encontro das mediatrizes dos (C) O circuncentro e o incentro.
lados do triângulo chama-se CIRCUNCENTRO. (D) O circuncentro e o ortocentro.
4.3) O circuncentro representa o centro da (E) O incentro e o ortocentro.
circunferência circunscrita ao triângulo. Na
figura abaixo o circuncentro é o ponto P. P5) (CFTCE) A altura e a mediana traçadas do
vértice do ângulo reto de um triângulo retângulo
formam um ângulo de 24°. Sendo assim, os
ângulos agudos do triângulo são:
(A) 33° e 57°
(B) 34° e 56°
(C) 35° e 55°
(D) 36° e 54°
(E) 37° e 53°

PROFESSOR RESPONDE ALUNO RESPONDE


P1) Na figura, N e P são os pontos médios dos A1) Em um triângulo ABC, M é o ponto médio do
lados AC e BC, respectivamente. Se G é o lado BC e G é o baricentro. Se AG=5x – 18 e
baricentro do triângulo ABC, AP = 6cm e GN = GM = x, então:
1,5 cm, a medida de AG e BN respectivamente,
(A) AM =18
em cm, é
(B) AM =16
(A) 4 e 4,5
(C) AM =12
(B) 4,5 e 4 (D) AM = 8
(C) 5 e 5,5 (E) AM = 6
(D) 5,5 e 5 A2) (Uff - 2001) Um pedaço de papel tem a
forma do triângulo equilátero PQR com 7cm de
(E) 3 e 3 lado, sendo M o ponto médio do lado PR:
P2) Na figura, o triângulo ABC é retângulo em A
e M é o ponto médio do lado BC. Então a medida
de α , em graus, é:
(A) 80º
(B) 90º
(C) 100º
(D) 110º Dobra-se o papel de modo que os pontos Q e M
(E) 120º coincidam, conforme ilustrado acima. O
perímetro do trapézio PSTR, em cm, é igual a:
P3) (FUVEST-SP) Um triângulo ABC têm
(A) 9
ângulos A= 40º e B = 50º. Qual é o ângulo
formado pelas alturas relativas aos vértices A e
(B) 17,5
B desse triângulo? (C) 24,5
(D) 28
(A) 30º (E) 49
(B) 45º

101
A3) (Ufc 2002) Na figura a seguir, temos dois A7) Um ponto Q pertence à região interna de um
triângulos equiláteros ABC e A'B'C' que triângulo DEF, equidista dos lados desse
possuem o mesmo baricentro, tais que AB//A'B', triângulo. O ponto Q é:
AC//A'C' e BC//B'C'. Se a medida dos lados de
(A) O baricentro do triângulo DEF.
ABC é igual a cm e a distância entre os (B) O incentro do triângulo DEF.
lados paralelos mede 2 cm, então a medida das
alturas de A'B'C' é igual a: (C) O circuncentro do triângulo DEF.
(D) O ortocentro do triângulo DEF.

(A) 11,5 cm (E) Um ex-incentro do triângulo DEF.

(B) 10,5 cm
A8) No triângulo ABC da figura AH é altura e BS
(C) 9,5 cm
é a bissetriz do ângulo 𝐴𝐵̂𝐶 , determine 𝐵𝑆̂𝐶,
(D) 8,5 cm sendo dados 𝐵𝐴̂𝐻 = 30° e o 𝐴𝐶̂ 𝐵 = 40°.
(E) 7,5 cm
(A) 110º
A4) No triângulo ABC, da figura, AM e CN são
medianas que se interceptam em G. Sendo AG (B) 120º
= 10 cm e CN = 18 cm, os valores de x, y e z são (C) 130º
respectivamente:
(D) 140º
(E) 150º
(A) 5 cm, 9 cm, 10 cm
(B) 9cm, 10 cm, 18 cm
A9) Da figura sabemos que AH é altura e AS é
(C) 5cm, 6 cm, 12 cm bissetriz do ângulo 𝐵𝐴̂𝐶, do triângulo ABC. Se
𝐶𝐵̂𝐴 = 70° e 𝐻𝐴̂𝑆 = 15°a medida de 𝐴𝐶̂ 𝐵 é
(D) 10cm, 18cm, 12cm
(E) 5cm, 12cm, 6cm
(A) 80 graus
(B) 70 graus

A5) Na figura, M é o ponto médio do lado BC e (C) 60 graus


CN é a bissetriz interna. Então a medida α, em (D) 50 graus
graus, é:
(E) 40 graus
(A) 80º
A10) No triângulo ABC da figura, 𝐶𝐵̂𝐴 = 60° e
(B) 75º
𝐴𝐶̂ 𝐵 = 20°. Qual o valor do ângulo HÂS formado
(C) 70º pela altura e a bissetriz ?
(D) 65º
(E) 60º (A) 70 graus
(B) 50 graus
A6) O triângulo ABC da figura é retângulo em A,
AS é a bissetriz interna e AM é mediana. (C) 40 graus
Então, a medida de α , em graus, é (D) 20 graus
(A) 10º (E) 15 graus
(B) 15º
(C) 20º
(D) 25º
(E) 30º

102
A11) Dada a figura: A15) Um ponto P eqüidista dos vértices de um
triângulo ABC. O ponto P é:

(A) O baricentro do triângulo ABC.


(B) O incentro do triângulo ABC.
(C) O circuncentro do triângulo ABC.
Sobre as sentenças
(D) O ortocentro do triângulo ABC.
I. O triângulo CDE é isósceles.
(E) Um ex-incentro do triângulo ABC.
II. O triângulo ABE é equilátero.
III. AE é bissetriz do ângulo BÂD.
A16) A área de um triângulo equilátero inscrito
É verdade que numa circunferência de 8 cm de raio é
(A) somente a I é falsa.
(B) somente a II é falsa. (A) 12√3𝑐𝑚2
(C) somente a III é falsa. (B) 36√3𝑐𝑚2
(D) são todas falsas.
(C) 48√3𝑐𝑚2
(E) são todas verdadeiras.
(D) 60√3𝑐𝑚2
A12) (UFF) O triângulo MNP é tal que ângulo M (E) 72√3𝑐𝑚2
= 80° e ângulo P=60°. A medida do ângulo
formado pela bissetriz do ângulo interno N com A17) Um triângulo equilátero com lados
a bissetriz do ângulo externo P é: medindo 10 cm está inscrito numa
(A) 20° circunferência de raio r. Calcule a área do círculo
delimitado por essa circunferência.
(B) 30°
(C) 40°
100𝜋
(D) 50° (A) 𝑐𝑚2
3
(E) 60° (B) 100𝜋𝑐𝑚2
(C) 100√3𝑐𝑚2
A13) (FGV) AN e BM são medianas do triângulo
ABC. G é o ponto de encontro de AN e BM. Se (D) 200𝜋𝑐𝑚2
BM é igual a 12cm, a medida do segmento GM 200𝜋
é igual a: (E) 𝑐𝑚2
3
(A) 10
(B) 9
(C) 8 18) Na figura AB = AC, BD é bissetriz de 𝐴𝐵̂𝐶, ,
(D) 6 CE é bissetriz de 𝐵𝐶̂ 𝐷, e 𝐴𝐸̂ 𝐹 = 140°. As
(E) 4 medidas de 𝐷𝐸̂ 𝐶, e 𝐴𝐵̂ 𝐷, respectivamente, em
A14) (UFES) Um dos ângulos internos de um graus, são
triângulo isósceles mede 100°. Qual é a medida
do ângulo agudo formado pelas bissetrizes dos
outros ângulos internos? (A) 30 e 40
(A) 20° (B) 40 e 50

(B) 40° (C) 40 e 60

(C) 60° (D) 50 e 6

(D) 80° (E) 30 e 60

(E) 140°

103
RELAÇÕES TRIGONOMÉTRICAS NO
TRIÂNGULO RETÂNGULO
Começaremos o nosso estudo com a exposição
de uma situação problema que será resolvida ao
final da apresentação do conteúdo. O problema
é o seguinte:
Queremos saber a largura L de um rio. Para
isso, marcamos com estaca dois pontos, A e B,
um em cada margem, de tal modo que o ângulo Podemos perceber que para ângulos
no ponto A seja reto. Depois, marcamos um complementares α e β, temos que
ponto C, distante oito metros de A, onde fixamos
o Teodoito. Medimos, então, o ângulo de 60° no
ponto C. Nessas condições, indique a largura L
do rio. (√3 = 1,7√3 = 1,7)

3. ÂNGULOS NOTÁVEIS
3.1. ÂNGULO DE 30°
1. INTRODUÇÃO Para determinar as razões trigonométricas de
um ângulo de 30° recorremos a um triângulo
Nem sempre é possível medir diretamente um equilátero. Vejamos
ângulo ou a distância entre dois pontos. Para
resolver esse tipo de problema, os matemáticos
estudaram a relação entre ângulos e medidas
dos lados de um triângulo. Desses estudos
surgiu a trigonometria. E nessa apresentação
veremos as razões trigonométricas no triângulo
retângulo.
2. RAZÕES TRIGONOMÉTRICAS
Estudaremos as razões trigonométricas seno
(sen), cosseno (cos) e tangente (tg). Para um
ângulo agudo α elas são determinadas por
𝐶𝑎𝑡𝑒𝑡𝑜 𝑜𝑝𝑜𝑠𝑡𝑜 𝐶𝑂
𝑆𝑒𝑛 𝛼 = = No triângulo retângulo HBC, temos que
ℎ𝑖𝑝𝑜𝑡𝑒𝑛𝑢𝑠𝑎 𝐻

𝐶𝑎𝑡𝑒𝑡𝑜 𝑎𝑑𝑗𝑎𝑐𝑒𝑛𝑡𝑒 𝐶𝐴
𝐶𝑜𝑠 𝛼 = =
ℎ𝑖𝑝𝑜𝑡𝑒𝑛𝑢𝑠𝑎 𝐻

𝐶𝑎𝑡𝑒𝑡𝑜 𝑜𝑝𝑜𝑠𝑡𝑜 𝐶𝑂
𝑇𝑎𝑛 𝛼 = =
𝐶𝑎𝑡𝑒𝑡𝑜 𝑎𝑑𝑗𝑎𝑐𝑒𝑛𝑡𝑒 𝐻𝐶𝐴
Para os ângulos A e C do triângulo a seguir,
temos que

104
3.2. ÂNGULO DE 45° 3.4. RESOLUÇÃO DO PROBLEMA INICIAL
Para determinar as razões trigonométricas de Com as informações apresentadas acima,
um ângulo de 45° recorremos a um quadrado. podemos resolver o problema proposto no início
Vejamos da apresentação. Nele temos um ângulo de 60°
e a medida de 8 m do lado adjacente ao ângulo
dado, então podemos usar a razão tangente
para descobrir a medida L do lado oposto ao
ângulo de 60°. Segue que

No triângulo retângulo ABC, temos que

Portanto, a largura L do rio é de 13,6 metros.


PROFESSOR RESPONDE

P1) (IFPI – 2015) Para calcular a quantidade de


cabos de aço necessários para unir os topos A
e B de dois morros, mediram-se as alturas dos
morros em relação a um mesmo plano
horizontal, obtendo-se 103 m e 156 m. A seguir
mediu-se o ângulo que a reta AB forma com a
3.3. ÂNGULO DE 60°
horizontal, obtendo 32°. Encontre a distância
Como os ângulos de 60° e de 30° são entre os pontos A e B, sabendo que sen 32° =
complementares, então 0;53; cos 32° = 0;85; tg 32° = 0;62.

(A) 110 m
(B) 100 m
(C) 85 m
(D) 80 m
(E) 62 m

Dessa forma, temos que


P2) (IFPI – 2015) Observa-se que no momento
em que os raios solares formam com o solo um
ângulo de 32°, uma árvore projeta uma sombra
de 7m. Com base nessas informações, qual a
altura da árvore? (Use: sem 32° = 0;53; cos 32°
= 0;85; tg 32° = 0;62).

(A) 5,95 m
(B) 4,34 m

OBSERVAÇÃO: Em algumas questões (C) 4,12 m


aparecem ângulos diferentes dos ângulos (D) 3,98 m
notáveis. Nesses casos, a questão informará os
valores das razões trigonométricas necessárias. (E) 3,71 m

105
P3) (IFMA – 2012) Os alunos Tadeu e Pedro ALUNO RESPONDE
observaram a uma distância de 15 m e sob um
ângulo de 45° com a horizontal a árvore de natal
que foi colocada no pátio do IFMA. Então Pedro A1) (IFMA – 2013) Patrícia olha uma pedra na
desafiou Tadeu a descobrir a altura somente da outra margem de um rio exatamente a sua
árvore. Considerando que sua ponteira (estrela) frente. Percorrendo 200 m o ângulo de visão da
mede 100 cm. Tadeu acertou e deu como pedra fica de 45° como na figura. Assim Patrícia
resposta: conclui que a largura do rio é de:

(A) 15,62 m
(B) 16,00 m
(C) 16,62 m
(D) 15,00 m
(E) 14,62 m

P4) (IFAL– 2017) Considere um triângulo


retângulo, cujos ângulos α e β satisfazem à
condição cos α = 0;8 e cos β = 0;6. Determine a
área desse triângulo, em cm2, sabendo que o 100√3
comprimento da hipotenusa é 5 cm. (A) 𝑚
3
(A) 4,5 (B) 200√3𝑚
(B) 6,0 (C) 200 m
(C) 7,5 (D) 100 m
(D) 8,0 (E) 100√3𝑚
(E) 10,0

A2) (IFPI – 2014) Em certa hora do dia, um poste


P5) (IFMA – 2013) Um observador vê o topo de de 5 metros de altura projeta uma sombra de 1,7
um prédio segundo um ângulo visual de 45°. metros. De acordo com a tabela, qual é,
Aproximando-se 30 metros do prédio, observa- aproximadamente, o ângulo de inclinação do sol
o sob um ângulo de 60°. A altura desse prédio é em relação ao chão, nesse momento.
de aproximadamente: (admita , )

(A) 65,72 m (A) 67°


(B) 71,09 m (B) 68°
(C) 85,13 m (C) 69°
(D) 69,11 m (D) 70°
(E) 96,07 m (E) 71°

106
A3) (IFPI – 2014) Na figura abaixo, verificou-se
que a árvore menor mede 1,68 m de altura e AB
= BC. Consultando a tabela, indique a
alternativa que determina aproximadamente a
altura da árvore maior.

Desconsiderando a altura do engenheiro, a


altura, em metros, do edifício é:
(A) 40√3𝑚
(B) 36 m
(C) 30√3𝑚
(D) 28 m
(D) 20√3𝑚

A6) (IFPI – 2016) Um avião decolou segundo um


(A) 2,00 m ângulo de 30° com o solo.
(B) 2,52 m
(C) 3,10 m
(D) 3,25 m
(E) 3,50 m

A4) (IFPI – 2014) A hipotenusa de um triângulo


retângulo mede H, e um de seus ângulos Com base nessa informação, qual a altura
internos, 30°. Marque a alternativa que indica a atingida pelo avião após percorrer 1,8 km?
medida do cateto adjacente a esse ângulo
1 √3 √3
interno. Use: 𝑆𝑒𝑛 30° = 2 ; 𝐶𝑜𝑠 30° = 2
; 𝑡𝑔 30° = 3

𝐻 √3 a) 0,5 km
a)
2 b) 0,6 km
𝐻 √2 c) 0,8 km
b)
2 d) 0,9 km
𝐻 e) 1,9 km
c)
2
d) 𝐻√3
A7) (IFPI – 2016) Uma pista de skate é
𝐻 √3 construída por uma rampa de de
e)
3 comprimento e 45° de inclinação. Com essas
informações, qual a altura dessa rampa?
√2 √2
Use: 𝑆𝑒𝑛 45° = ; 𝐶𝑜𝑠 45° = ; 𝑡𝑔 45° = 1.
A5) (IFPI – 2015) Para medir a altura de um 2 2
edifício, um engenheiro fixou um ponto (C) no (A) 3 m
terreno e mirou o topo do prédio sob um ângulo
de 30° com o solo. Depois, andou 40 metros em (B) 3√2𝑚
direção ao prédio e mirou novamente seu topo,
(C) 4√2𝑚
mas, agora sob um ângulo de 60°, conforme
figura. (D) 4,5 m
(E) 5 m

107
A8) (IFPI – 2015) O acesso a um edifício é feito Um fotógrafo que ficará no canto A da plateia
por uma escada de 6 degraus, sendo que cada deseja fotografar o palco inteiro e, para isso,
degrau tem 20 cm de altura. Para atender deve conhecer o ângulo da figura para escolher
portadores de necessidades especiais, foi a lente de abertura adequada.
construída uma rampa que forma com o solo um
ângulo de 3°. Com base nessa informação, qual
o comprimento da rampa em metros? Use: sen O cosseno do ângulo da figura acima é:
3° = 0,05; cos 3° = 0,99. (A) 0,5
(A) 26 m (B) 0,6
(B) 24 m (C) 0,75
(C) 20 m (D) 0,8
(D) 12 m (E) 1,33
(E) 10 m

A9) (IFPI – 2013) Um avião decolou segundo um A12) Uma esfera foi liberada no ponto A de uma
ângulo de 30°. Após percorrer 0,6 km, conforme rampa e percorreu 8 m até atingir o solo no
indicado na figura, sua altura em relação ao ponto B. Sabendo-se que o caminho percorrido
chão, em metros será igual a: pela esfera é exatamente a hipotenusa do
triângulo retângulo da figura abaixo, determine a
(A) 360 m
distância do ponto A ao solo.
(B) 300 m
(C) 280 m
(D) 270 m
(E) 250 m

A10) Uma escada de 10 metros de comprimento


forma um ângulo de 60° com a horizontal
quando encostada ao edifício de um dos lados
da rua, e ângulo de 45° ser for encostada ao
edifício do outro lado da rua, apoiada no mesmo
(A) 5 metros
ponto do chão. A largura da rua (em metros) é:
(B) 3 metros
(A) 10√2 𝑚
(B) 10 + 3√2 𝑚 (C) 4 metros
(C) (10√5) − 5 𝑚 (D) 6 metros
(D) 5 + 5√2 𝑚 (E) 8 metros
(E) 5 + 10√2 𝑚
A13) Um projétil é lançado com um ângulo de
A11) A plateia de um teatro, vista de cima para 30° em relação a um plano horizontal.
baixo, ocupa o retângulo ABCD da figura a Considerando que a sua trajetória inicial pode
seguir, e o palco é adjacente ao lado BC. As ser aproximada por uma linha reta e que sua
medidas do retângulo são AB = 15 cm e BC = 20 velocidade média, nos cinco primeiros
cm. segundos, é de 900 km/h, a que altura em
relação ao ponto de lançamento este projétil
estará exatamente cinco segundos após o
lançamento?
(A) 0,333 km
(B) 0,625 km
(C) 0,5 km
(D) 1,3 km
(E) 1 km

108
A14) A figura a seguir representa uma escada. Dois segmentos secantes saindo do mesmo
ponto (externo à circunferência)
Se você traçar dois segmentos secantes a uma
circunferência que se interceptam em um ponto
externo, você criará uma relação de semelhança
entre a medida dos segmentos externos à
circunferência e a medida do segmento inteiro.
Isso ocorre independente do tamanho da
circunferência. Observe:
Considere
A altura dessa escada é de: O segmento
AR multiplicado
(A) 2,36 m pelo segmento
(B) 2,52 m BR é
(C) 2,74 m proporcional ao
(D) 2,88 m segmento CR
(E) 2,47 m multiplicado pelo segmento DR, assim:
A15) Um avião está voando paralelo ao solo. Ao
avistar o aeroporto ele é inclinado 30° e percorre
AR . BR = CR . DR
6 km até tocar na pista de pouso. A que altura
ele está voando?
Segmento secante e segmento tangente
partindo de um mesmo ponto
Ao se traçar um segmento secante e um
segmento tangente a uma circunferência, sendo
que estes segmentos de reta se encontram em
um ponto externo a circunferência, estes
segmentos irão se relacionar da seguinte forma:

(A) 5 km
(B) 6 km
(C) 4 km
(D) 10 km
(E) 3 km

RELAÇÕES MÉTRICAS NA O quadrado da medida do segmento tangente é


CIRCUNFERÊNCIA E NO CIRCULO igual a multiplicação da medida do segmento
secante pela medida de sua parte externa,
Cruzamento entre duas cordas sendo assim:
Quando duas cordas se cruzam no interior de (AQ)² = CQ . BQ
uma circunferência, o ponto de cruzamento
determina segmentos de reta que são Ordenada de um ponto e diâmetro
proporcionais entre si. Essa proporcionalidade
pode ser dada através de uma multiplicação. A ordenada de um ponto P da circunferência em
Veja o esquema: relação a um diâmetro é a média geométrica
entre os segmentos AM e BM determinados por
este diâmetro.

O segmento AP multiplicado pelo segmento PB


é proporcional ao segmento CP multiplicado
pelo segmento PD. Veja a representação a A relação entre as retas se dá por:
seguir para entender melhor:
AP . PB = CP . PD (PM)² = AM * BM

109
Diâmetro e projeção da corda ALUNO RESPONDE
A corda PB traçada da extremidade de um
diâmetro é a média geométrica entre o diâmetro 1) (IFPI-2020) Na figura, os lados do triângulo
AB e a projeção desta corda sobre ele BM. ABC tangenciam uma circunferência,
respectivamente, nos pontos P, M e N. Se AP =
k, BM = 2k, CN = 3k e o perímetro de ABC é 96
cm, quanto mede o lado AB?

a) 22 cm
b) 23 cm
A relação entre essas retas se dá da seguinte c) 24 cm
maneira: d) 25 cm
(PB)² = AB . BM e) 26 cm

2) (IFPI-2019) Observe o hexágono regular


inscrito em uma circunferência de raio 2x-3.
PROFESSOR RESPONDE

1) Observe a figura

Qual o valor de x para que o perímetro desse


hexágono seja 78 cm?

a) 6

Nessa figura, contém os centros O e O’ das b) 7


circunferências que se tangenciam no ponto T . c) 8
Sendo AB = 44, O‘B = 16 e AC = 6, a medida TD
é: d) 9
a) 8 e) 10
b) 15
c) 6 3) (IFPI-2018) Na circunferência de centro O,
d) 20 conforme figura, o ângulo inscrito MKN mede
e) 16 40°.

2)Na figura abaixo têm–se as circunstâncias de


centros O1 e O2, tangentes entre si e tangentes
à reta r nos pontos A e B, respectivamente.

Qual a medida do ângulo KPNˆ inscrito nessa


circunferência?

Se os raios das circunferências medem 18cm e a) 260°


8cm, então o segmento mede, em centímetros:
b) 180°
a) 20
b) 22 c) 150°
c) 23
d) 130°
d) 24
e) 26 e) 80°

110
4) (IFMA-2017) Na figura ao lado, o segmento cordas P1P2 e Q1Q2. Se P1M = 4 cm, MP2 = (k
AB é tangente à circunferência de raio r e centro + 1) cm, Q1M = 3 cm e MQ2 = (3k – 7) cm, então
O, no ponto A. Sabendo-se que BC = 2cm, a corda Q1Q2, em cm, mede:
determine a medida de AB.

a) 2√𝑟 + 1
a)5
b) √𝑟 + 2
c) √2𝑟 + 1 b)8
d) √4𝑟 + 1 c)11
e) √𝑟 2 + 4 d)14
e)17
5) (IFMA-2017) A área do quadrilátero ABCD
inscrito na circunferência de centro O e raio 4
cm, conforme a figura, é igual a: 9) (Unifor-CE) Na figura abaixo, tem-se uma
circunferência e duas de suas cordas
a) 8(2+√3) concorrentes no ponto M.

b) 8(1-√3) Se as medidas dos segmentos CM, MD, AM e


c) 2(2+√3) MB, indicados na figura, são dadas em
centímetros, a corda AB mede, em centímetros:
d) 8(2+√2)
a) 36
e) 16(1+√3)
b) 18
6) (IFMA-2918) Das três circunferências abaixo, c) 15
as duas menores têm raio igual a 4 cm e a d) 14
circunferência maior tem raio igual a 9 cm.
e) 13
Quanto mede a distância entre os centros das
duas circunferências menores?
10 )Na figura abaixo tem-se a reta r tangente à
a) 24
circunferência de centro O no ponto T. Se o raio
b) 12 da circunferência mede 3 cm e AT = 4 cm, a
c) 17 medida de AQ, em centímetro, é:
d) 13
e) 11
a) 0,2
b) 0,4
7) (IFMA-2018) Seja o triângulo equilátero ABC c) 1
inscrito em uma circunferência de raio 6 dm e
d) 1,8
centro em O. Qual a medida da altura do
triângulo ABC? e) 2

a) 3√6 𝑚 11) Se AC = 12 cm e AB = 6 cm, então, na


circunferência abaixo o raio mede:
b) 9 m
c) 3√3 a) 6 cm
b) 9 cm
d) 12 𝑚
c) 18 cm
e) 6√2 𝑚
d) 12 cm
8) (UE-CE) A circunferência da figura tem centro e) 24 cm
no ponto O e M é o ponto de interseção das

111
12) Se o segmento da perpendicular traçado do b) 19
ponto médio de uma corda de 24 cm até a c) 21
circunferência mede 9 cm, o raio da
d) 30
circunferência mede:
e) 15
a) 12,5 cm
b) 25 cm 17) Uma circunferência tem 7,5 cm de raio. Por
um ponto situado a 12,5 cm do centro traça-se
c) 50 cm uma tangente que por medida:
d) 6 cm
a)5√3 cm
e) 8 cm
b)2√5 cm
13) Na circunferência abaixo, de centro O e raio
de medida r = 4, a corda CD corta o diâmetro AB c) 2√3
no ponto P, de tal forma que P é o ponto médio
do raio OA e PC = 2.PD. Então: c)√93,75 cm
d) 10 cm
a) CD = 6√6

b) CD = 3√6 18) De um ponto exterior a uma circunferência


c) CD = 8√6 traçam-se uma tangente de 9 cm e uma secante
que passa pelo centro e mede 27 cm. O raio da
d) CD = √6
e) CD = 6 circunferência mede:

14) O valor de x na figura é: a) 12 cm

a) 3 b) 3 cm

b)4,8 c) 6 cm
c) 6,6
d) 24 cm
d) 7,5
1 e) 36 cm
e) 3
3

15) Em um círculo, a corda CD é perpendicular


ao diâmetro AB no ponto E. Se AE.EB = 3, a 19) Por um ponto P a uma circunferência
medida de CD é: traçamos uma secante que encontra a
circunferência nos pontos A e B. Se PA = 12 cm
a) 3
e PB = 8 cm, então a potência do ponto P em
b) √3 relação a essa circunferência é:

c) 2√3 a) 96

d) 3√3 b) 160
e) 4√3 c) 80

16) Duas cordas cortam-se no interior de um d) 32


círculo. As medidas dos segmentos da primeira
são expressas por 3x e X + 1, e as dos e) 18
segmentos da segunda, por x e 4x -1. A medida
do comprimento da maior corda, qualquer que
seja a unidade, é expressa pelo número:

a) 17

112
Bibliografia

DANTE, Luiz Roberto. Projeto Teláris, Matemática. 9º ano. 1 ed. São Paulo
Ática 2012.
CHAVANTE, Eduardo Rodrigues. Convergências: Matemática, 9º ano. 1 ed.
São Paulo. Edições SM, 2015.
SOUZA, Joaquim Roberto de. Vontade de Saber Matemática, 9º ano,
Patrícia Rosana Moreno Pataro. 3. Ed- São Paulo: FTD, 2015.
Sites:
https://libra.ifpi.edu.br/teresinacentral
https://timon.ifma.edu.br
https://www.ifsp.edu.br https://www.ifrj.edu.br

Você também pode gostar